Combo 1 (11/5 to 29/9)

Réussis tes devoirs et examens dès maintenant avec Quizwiz!

What do patients with insulinoma's present with? How would you confirm the diagnosis?

1) Episodes of hypoglycemia - Syncope - Adrenergic sx - Neurological: AMS, seizures, personality changes - Symptoms relieved by food. 2) Confirmation of diagnosis: Via Prolonged fasting test 72 hrs - Remove all glucose influencing factors and medication - At the time of hypoglycemia during a 72-h fasting test: o 5 mIU/L (36 pmol/L) insulin threshold o 0.6 ng/mL (0.2 nmol/L) C-peptide threshold o Insulin/C-peptide ratio < 1.0 o 20 pmol/L proinsulin cut-off level o Absence of sulfonylurea (metabolites) in the plasma or urine

What is panniculitis and what are some forms?

1) Erythema nodosum 2) Lipodermatosclerosis 3) Erythema induratum 4) Pancreatic panniculitis 5) Lupus erythematosus panniculitis

What is the natural history of acute irritant contact dermatitis?

1) Erythema w dull non glistening surface 2) Blistering/vesiculation 3) Erosions 4) Crusting 5) Shedding of crusts 6) Necrosis 7) Shedding of necrotic tissue 8) Ulceration 9) Healing

What are the possible presentations and pathogens for corynebacterium infections? How would you treat it?

1) Erythrasma - C.minutissimum - Brown-red macular patches in groin and axilla - Fluoresces coral pink with wood's light 2) Pitted keratolysis - C.minitissimum - Carteriform pitting of pressure zones of feet - a/w hyperhydrosis 3) Trichomoniasis axillaris - C.tenius - Yellow, black or red franular conrections that stick the hair shaft - Sweat may also be colored according to the conretions Treat with: Topical or oral erythromycin (pictures to be added)

What are the indications for surgical fixation of humeral shaft fractures?

1) Etiology: - Polytrauma - Pathological fracture 2) Failure of conservative treatment - Inadequate closed reduction - Unacceptable deformity - N&V after reduction 3) Severity of injury - Associated neurovascular injury - Bilateral humeral fractures - Open fracture - Floating elbow - Intra-articular fracture extension

What are the nerves that may be injured in thyroidectomy and what do they innervate?

1) External branch of superior laryngeal nerve: Voice projection via tensing of vocal cords 2) Recurrent laryngeal nerve: breathing and voice production via aBduction and aDduction of vocal cords, also provides protective function from aspiration (ask about coughing when drinking, can provide thickened fluid)

What are the possible types of calcaneal fractures and how might they be graded?

1) Extra-articular - Tuberosity fractures - Medial process fractures - Anterior process fractures - Sustenaculumn fractures (a shelf that arises from anteromedial portion of the calcaneum) - Body fractures not involving the subtalar joint 2) Intra-articular - According to Sander's classification (on CT) - 4 types 1: Undisplaced, irregardless of number of fracture lines 2: 2 fracture lines, with minimal displacement 3: 3 fractures lines with more displacement and depression fractures 4: Severe comminution

What are the possible causes of cellulitis?

1) Extrinsic o Sources of infection - Salt water, fresh water - Insect bites - Traumatic injury 2) Intrinsic o b/g immunocompromisation - DM - HIV - Undergoing chemo - Extremes of age

What to look out for in face in Parkinsonism. (5)

1) Eye movements (Doll's eyes if vertical gaze impaired) 2) Blepharospam when eyes closed (seen in SNP) 3) KF rings in eyes (wilson's disease) 4) Feel for seborrhea (dryness of skin around nose and mouth, dandruff from scalp, due to autonomic dysfunction) 5) Ask patient to count from 1-20 (rough cognitive function assessment)

What are the signs of cerebellar dysfunction?

1) Eye signs: - Nystagmus signs - Jerky tracking? 2) Upper limb signs: - Intention tremor/dysmetria - Disdiadokinesia 3) Lower limb signs: - Heel shin test - Toe to finger test 4) Sitting: - Pendulum knee jerk 5) Standing: - Rombergism - Frontal disequilibrium 6) Gait: - Frontal disqeuilibrium - Magnetic gait 7) Speech - Scanning, explosive speech

What are the Ocular manifestations of rheumatoid arthritis?

1) Eyes: - Conjunctiva: keratoconjunctivitis sicca (dry eyes), pallor - Sclera: episcleritis, scleritis, necrotizing scleritis - Iris/Uvea: Iritis, uveitis - Lens: cataract from chronic steroid use - Retina: associated vasculitis, drug-induced vasculitis (gold, hydroxychloroquine) - Extra ocular muscles: mononeuritis multiplex, myasthenia 2' penicillamine

What is the cut off for intubation in GBS/Mg?

1) FVC <18ml/kg (N 60-80ml/kg) 2) NIF <20cmH2O 3) >30% change in NIF

When is MRI indicated in spinal stenosis?

1) Failure of conservative treatment 2) Pre-surgical planning 3) Equivocal symptoms

What are the complications of hemostatic treatment of UBGIT?

1) Failure of hemostasis 2) Aspiration 3) Altered hemodynamics: affects heart, lungs, renal, brain etc 4)Recurrent bleeding

What are the causes of hypertriglyceridemia?

1) Familial 2) Metabolic - Metabolic syndrome - Cushing's - Cholestasis - Hypothyroidism - Alcohol abuse - DM - Pregnancy 3)Drug induced - Steroids - BBlockers - Diuretics - OCP - Retinoids 4) Renal - CKD - Nephrotic syndrome

What are the risk factors for breast cancer?

1) Family history - Breast cancer (either 1st degree @ < 40y/o or >/= 2 relatives @ ~ 50y/o) - ALSO ovarian, prostate and colorectal - BRCA (80% lifetime risk) 3) Previous breast disease - Treated cancer - Previous biopsy showing atypical ductal hyperplasia or lobular carcinoma in situ - Exposure to ionizing radiation (esp in RT of previous breast CA) 2) Ostreogen exposure history - Early menarche ( < 12 y/o) - Late menopause (>55y/o) - Use of HRT/oestrogen OCP (> 1yr) - Nulliparity (oestrogen falls during pregnancy) - Late pregnancy (> 30 y/o when first child) - Not breast feeding (or < 6/12) 4) Lifestyle - Daily alcohol intake (esp before 30y/o)

What are the risk factors of thyroid carcinoma?

1) Family history - MEN syndrome - FAP - Cowden (multiple hamartoma, breast and thyroid Ca) 2) Radiation exposure - Previous scans - Occupation

What are the causes of Hallux valgus?

1) Family history/genetic predisposition 2) Constricting foot wear 3) Ligamentous laxity 4) Pes planus 5) Rheumatoid arthritis 6) Convex metatarsal head

What do you need to check before and during use of isotretinoin?

1) Fasting lipids (hypertriglyceridemia) 2) Liver function test (transaminitis) 3) Urine pregnancy test (if female)

What are the possible radiographic findings?

1) Features of kyphosis - Anterior wedgeing of 3 consecutive vertebrar - Kyphosis angle of >45 - Disc narrowing - Endplate irregularities - Schmorl's nodes (herniation of nucleus pulposus into adjacent endplate -> defects on endplate) 2) Associated conditions - Scoliosis - Compensatory lumbar hyperlordosis - Spondylolisthesis 2'spondylolysis

What are the various presentations of dengue according to the disease course?

1) Febrile phase (Day 1-3, up to 7) - Rapid onset high grade fever +/- saddlebacking - Headache, eye pain - Arthralgia, myalgia - Flu - GIT (N&V, diarrhoea, pain, anorexia) - Conjunctival injection - Pharyngeal edema, facial puffiness - Lymphadenopathy, hepatomegaly - +/- Rash 2) Critical phase (24-48hrs) - Vascular leak o Persistent vomiting o Worsening abdominal pain o Tender hepatomegaly o Easy bruising, tourniquet test o Pleural effusion/ascites o Mucosal bleeding - Hematocrit >/= 20%, moderate to severe thrombocytopenia 3) Recovery phase (2-4 days) - Recovery of symptoms - Fatigue may progress - Rash

What are the possible causes of intoeing gait in a paediatric patient?

1) Femoral anteversion - Child sits in W position - Awkward running style, tripping - +/- activity limitation - **Internal rotation > 70 degrees, < 20 degrees of external rotation 2) Internal tibial torsion - Increased tripping/falling - Usually asysmptomatic - **Thigh-foot angle >10 degrees internal 3) Metatarsus Adductus - **Medial deviation of the forefoot, normal hind foot 4) Club foot

What is the expected blood loss for femoral, tibial/fibular, humeral, radius/ulnar and rib fractures?

1) Femoral: 1-2L 2) Tibial/fibular: 500ml-1L 3) Humerus: 750ml 4) Radius/ulna: 250-500ml 5) Rib (1): 125ml

What are the differentials for a single breast lump?

1) Fibroadenoma 2) Breast cyst 3) Fat necrosis 4) Cancer

What are the risk factors of gastric cancer?

1) Modifiable - Diet: preserved foods (nitrosamines in salted/smoked/ pickled foods) - Smoking - Alcohol - Occupational: - Asbestos - Heavy metals - Rubber - Low SES 2) Non-modifiable - Age (>50) - Blood type A - Genetic - HNPCC (lynch syndrome II) - P53 mutation (Li Fraumeni syndrome) - Germline mutation of E-cadherin - FmHx of gastric cancer 3) Hx of precursor conditions: - Partial gastrectomy for benign disease with Bilroth II reconstruction (~ 15years after surgery) - Gastric polyps - Chronic atrophic gastritis (hypertrophic gastritis/ pernicious anemia) - Peptic ulcer disease (<1% risk of malignant change) - H.pylori infection (3-6x increased risk)

What is the Monteggia fracture and what is a Galeeazi fracture?

1) Monteggia fracture - Proximal 1/3 ulnar shaft fracture - a/w radial head dislocation 2) Galeazzi fracture - Distal third of radius - a/w distal radioulnar joint dislocation

What is the screening algorithm for breast cancer?

1) Monthly self breast exam 2) 40-49 y/o -> yearly mamogram 3) >/= 50y/o -> every 2 yearly 4) In patients with 10 years at least of life expectancy -> will benefit from continued screening hence ~ 70 y/o can stop

What is the life cycle of malaria?

1) Mosquito - Specifically, the anopheles mosquito - It injects the damn sporozoites into the blood - They go to the liver 2) Hepatic - Sporozoites infect hepatocytes, forming hypnozoites (latent forms of parasite) - The hypnozoites eventually mature to form hepatic schizonts - Hepatic schizonts rupture to release merozoites into the blood stream 3) Blood stream (erythrocytic stage) - Merozoites infect RBCs - Once inside, they evolve (LOL) into trophozoites (ring shaped) and then EITHER replicate for form blood-stage schizonts (that rupture to release more merozoites) OR - Form gametocytes that can be picked up from blood when another damn mosquito comes by

What are the available types of biopsy methods for breast disease?

1) Fine Needle Aspiration Cytology - less invasive - less pain - smaller 23G needle wound - LA not required - cheap/repeatable - BUT no architecture for in situ vs invasion - hence f/u with frozen section intraop before proceding 2) Core Biopsy - Via trucut/ mamotome (has vaccum assistance) - vice versa of FNAC w 14G needle wound - Higher risk of complications (spring loaded blade may puncture heart/lung) - More expensive due to tissue processing - BUT CAN see architecture/invsion - CAN stain for ER/PR status - Do w U/S or stereotactic (2 view XR) guidance to improve accuracy 3) Incisional Biopsy - Remove part of lump + intraop frozen section - Less common now 4) Excisional Biopsy - Remove entire lump - GA and larger wound left (incisional as well)

What are the signs of dermatomyositis?

1) Fingers - Periungal erythema - Mechanic's hands - Gottron's papules 2) Face/neck - Heliotrope rash - Shawl's sign 3) Muscles - Proximal myopathy

What are the special tests for Dequervain TS?

1) Finklestein (think of the bastard) - examiner grabs thumb and forces ulnar deviation, pain = positive 2) Eichhoff test - ulnar deviation w thumb in fist leads to pain + relief when thumb extended

What are the possible femoral deformities in Perthe's disease?

1) Flattening: Coxa plana 2) Enlarging: Coxa magna 3) Fragmentation/collapse 4) Lateral displacement (via lateral centre-edge angle)

How would you test the range of motion of the spine?

1) Flexion-extension - Get patient to bend forward + measure - Get patient to bend back + measure - Measure schoeber's test: spinal excursion should exceed 5cm, measured from 2 bony prominence 2) Lateral flexion - Should be able to touch ipsilateral knee 3) Rotation - Measure angle between line joining shoulders and line joining ASIS - Should be ~ 30deg

What are the biochemical signs of re-feeding syndrome?

1) Fluid imbalance 2) Electrolyte imbalance - hypophosphatemia - hypomagnesemia - hypokalemia 3) Metabolic - Vitamin deficiency (thiamine) - hypo/hyperglycemia Hence start VERY SLOW and replace deficient electrolytes along the way

What is Light's criteria for pleural effusion?

1) Fluid protein/serum ratio > 0.5 2) Fluid LDH/serum ratio > 0.6 3) Fluid LDH > 2/3 of upper limit of laboratory range.

What are the 2nd lines of TB drugs and their side effects?

1) Fluoroquinolones - Prolonged qt - Tendinitis/rupture - Hypoglycemia 2) Linezolid - thrombocytopenia - optic neuritis - MOAI effect can precipitate serotonin syndrome 3) Amikacin/streptomycin - renal impairment (interstitial nephritis) - vestibular and auditory injury Treat for 18-24 months

What are the types of thyroid carcinoma?

1) Follicular 2) Papillary 3) Medullary 4) Anaplastic 5) Mucosa associated lymphoid tissue (MALT) lymphoma 2' Hashiomotos

What are the features of neuropathic pain and how is it treated?

1) Follows a dermatomal/myotomal distribution or that of a peripheral nerve 2) Negative symptoms e.g weakness/sensory loss 3) Allodynia/hyperalgesia: - Allodynia: pain in response to a normally non-painful stimulus - Hyperalgesia: excessive pain response to a painful stimuli Treatment: 1) Conservative therapy: - Physiotherapy/occupational therapy - Splinting/immobilisation 2) Pharmacological: Gabapentinoids - Gabapentin - Pregabalin 3) Surgical/invasive: - Neurostimulation/electrotherapy (deep brain stimulation in periventricular grey areas) - Regional nerve blocks

What should be avoided in G6PD deficiency?

1) Food - Fava bean (found in mixed nuts) - Red wine (like a Chianti/amarone hahah) - Soya products - Tonic water - Camphor/ menthol - Henna - Vitamin C supplements 2) Drugs - Naphthalene - NSAIDs aspirin - Sulfur based drugs (sulfamethoxazole) - Fluoroquinolines - Carbonic anhydrase inhibitors - Quinine anti-malarials

What are the available allergy tests?

1) Food allergy - Skin prick test (any and all foods tested w needle prick to skin, recheck in 48hrs (acute) and 96 hrs (delayed) - Serum IgE (applied to panel of known allergens) 2) Allergic contact dermatitis - Patch test used (agent placed on aluminium disc and applied to skin, recheck in 3 days and 7 days (remove after 2 days), vesicles = positive

How do you treat typhoid and parathyphoid infections?

1) Food and water safety 2) Antimicrobial therapy o Empiric therapy - IF outside pakistan: IV ceftriaxone or cefotaxime - IF in pakistan: IV carbapenem e.g meropenem o Directed therapy: - If fluoroquinolone S: ciprofloxacin or ofloxacin etc - If fluoroquinolone R: azithromycin or bactrim +/- adjuctive steroids 3) Vaccination - conjugated typhoid vaccine for children - Polysaccharide vaccine - Oral vaccine (avoid in immunocompromised and pregnant, since live)

How would you manage a hiatal hernia?

1) GERD symptoms - Diet - PPI, H2RA 2) Surgery - In symptomatic hernia refractory to medical treatment: o Nissen/partial fundoplication - Complications (gastric volvulus, uncrontrolled bleeding, obstruction, strangulation, perforation and respiratory compromise): o Paraesophageal hernia repair (PEHR) with fundoplication and anterior gastropexy

What are the side effects of Bisphosphonates?

1) GIT - Esophagitis - Gastritis, ulceration, structure - Diarrhoea, vomiting 2) Bone - Osteonecrosis of jaw - Atypical fractures

In a patient with hypokalemic metabolic acidosis, what do you need to think of?

1) GOO (due to vomitting) 2) Conn's syndrome (comes with hypertension, due to too high aldosterone)

What is the most common cause of viral meningitis and what are the other pathogens?

1) Most common cause is enteroviruses 2) Other causes - HIV meningoencephalitis - HSV - 1 - Recurrent (mollaret's) meningitis from HSV 2 - Arenavirus (lymphocytic choriomeningitis virus) - Mumps (asceptic meningitis) - Others: arthropod borne virus, west nilte virus etc

What are the pathogen's responsible for urinary tract infections?

1) Most common: - Ecoli 2) Others: - S.saprophyticus (females) - Proteus - Klebsiella - Enterococcus

What are the clinical features of Charcot Marie Tooth?

1) Motor 2) Sensory 3) Associated features 4) Etiologies to rule out

What are the risk factors for malcongruence?

1) Muscular - Weak vastus medialis/quadriceps 2) Bone - Larger pelvis - Lateral patellar tilt - Genu valgus - Laterally positioned tibial tuberosity

What are the clinical signs of myotonic dystrophy?

1) Muscular - Weakness (SCM, distal muscles, CN e.g dysphagia) - Myotonia: contraction persists with delayed relaxation/percussion myotonia - Hypotonia 2) Face - Frontal balding - Ptosis w smooth forehead - Cataracts/evidence of cataract surgery - Expressionless face - Difficulty in opening eyes after firm closure - Wasting of muscles ( temporalis, masseters, SCM) - Swan neck from thinning of neck muscles 3) Respiratory - Infections - Dyspnoea - Most common cause of death

What are the signs of cervical myelopathy?

1) Gait - Gait unsteadiness 2) Neck - +ve Spurlings - +ve Davidsons (relief on arm abduction) - +ve Lhermitte sign (extreme cervical flexion leads to electric shock like sensations that radiate down the spine into the extremities) 3) Hands - Hoffman's test - Grab and release test - Functional impairment - Thenar atrophy - Finger escape sign - Inverted supinator reflex 4) Sensory (Depends on part of spine affected) - Loss of propioception - Decreased pain sensation 5) Motor - Weakness - Hyperreflexia

What is the classification of supracondylar fractures and how does it guide management?

1) Gartland's classification 3 types I: Non displaced - Immobilisation in long arm cast at 60-90deg of flexion for 2-3 weeks II: Displaced but cortical contact present - M&R with casting - KIV K wire if unstable/excessive flexion required for reduction (risk of N&V compromise) III: Completely displaced with no cortical contact - Attempt M&R - ORIF with K-wire

What antimicrobials have good penetration through the blood brain barrier? (FAT PAM)

1) Good penetration (FAT PAM) - Fluroquinolones - Ampicillins - Trimethoprim - Penicillins - Acyclovir - Metronidazole

What are the complications of Psoriatic Arthritis?

1) Gout: associated disease due to high cell turnover rate 2) Arthritis mutilans: bone resorbtion and destruction 3) Treatment associated complications: Cushings e.g striae, truncal obesity, moon facies etc

What are the causes of high T4 and what would you have to ask in terms of history?

1) Graves disease 2) Thyroiditis - recent viral illness - Trauma - Radiation - Recent gestation (post partum thyroiditis) 3) Drugs - Amiodarone (drug induced thyroiditis) - Exogenous thyroxine - TCM - Interferon (viral hep etc) 4) Previous/existing thyroid disease - Toxic adenoma - Toxic MNG 5) Cancers - Sturma ovarii - Metastatic follicular thyroid cancer

What are the physical signs of retroperitoneal haemorrhage?

1) Gray turner's 2) Cullen's 3) Kehr's

What are some common gas-forming skin infecting organisms?

1) Group A strep (pneumoniae and viridans in Nec Fash) 2) Bacteroides fragillis 3) Peptostreptococci 4) Clostridium Welchii

How might dermatologic lesions be arranged?

1) Grouped - Possible due to exogenous causes e.g herpes simplex, bites, warts 2) Annular/arciform - Fungal: tinea, mycosis fungoides - Bacterial: leprosy - Idiopathic: pityriasis rosea, granuloma annulare, erythema multiforme 3) Linear

What investigations do you do for newly diagnosed HIV patients? (INCOMPLETE)

1) HIV severity - Ag-AB tests (4th gen) - Viral load - CD4 counts 2) Treatment considerations - HLA B5701 (abacavir) - G6PD (bactrim) - Lipid panel, HbA1c, fasting glucose (metabolic syndrome) - LFT (nevaripine), RFT (tenofovir) - Genotype resistance testing 3) Opportunistic infections - CMV IgG - EBV Vca IgG - VZV IgG - Toxo IgG - Serum cryptococcal Ag 4) Co-infections - Anti- HAV - HbsAg, anti Hbc, anti Hbs, HBV viral load - Anti HCV - TB (active TB or TB screening depending on sx) 5) Other STIs - GC/CT urine, anus, throat swab - Syphilis screen (RPR, TPPA)

What are the possible modes of spread of spondylodiscitis?

1) Haematogenous seeding 2) Direct innoculation (penetrating trauma, open fractures, surgery) 3) Contiguous spread from local infections (retorpharyngeal abscess, retroperitoneal abscess)

What investigations would you order to work up a patient with secondary hypertension?

1) Nephritic syndrome, RAS - UFEME - UPCR - Renal artery doppler 2) Endocrine o 1' Hyperaldosteronism - serum aldosterone - serum potassium - renin aldosterone ratio - salt loading test o Cushing's - 8am cortisol - Dexa suppression test o Pheochromocytoma - 24 hr urinary metanephrine/catecholamine o Acromegaly - IGF-1, - OGTT GH test 3) Cardiac o Aortic stenosis/Co-arctation of aorta - TTE 4) OSA - polysomnography

What are the possible clinical manifestations kidney disease?

1) Nephrotic syndrome 2) Nephritic syndrome 3) Acute kidney injury 4) Chronic kidney disease and hypertension 5) End stage kidney disease, hypertensive crisis and fluid overload

What are the possible post-operative anesthesia complications?

1) Haemodynamic instability - Hypertension - Hypotension - Tachycardia - Bradycardia 2) Respiratory - Airway obstruction (tongue, edema, laryngospasm and bronchospasm) - Hypoventilation (residual anesthesia, diaphragmatic splinting, tight dressing) - Hypoxemia (atelectasis, pnuemothorax) 3) Neurological: - Residual anesthesia, hyper-sedation - Stroke - AMI - Metabolic derangements - Emergence delirium - Spinal cord injury during intubation 4) Nausea and vomitting - PONV 5) Pain 6) Hypothermia

What medicinal vehicle would you use for which areas?

1) Hairy areas, large areas, nail folds - Lotion 2) Flexures, other areas - Cream 3) For night use, only on palms and soles - Ointments

What are the possible forefoot toe deformities and their pathophysiologies? How would you manage them?

1) Hammer - Longer than mallet: @PIPJ - Fixed flexion deformity - 2' RA, imbalance between intrinsic and extrinsic muscles - IF flexible: Treat with flexor to extensor transfer - IF rigid: Resection of phalangeal head and neck +/- tendon release -> fix in straight position w K wire +/- arthrodesis 2) Mallet - Flexion at DIPJ - IF flexible: FDL tenotomy - IF rigid: Resection of middle phalangeal head and neck +/- flexor tenotomy +/- arthrodesis OR just amputate 3) Claw - Flexion and both DIPJ and PIPJ - w Hyper-extension at MTPJ - IF flexible: flexor to extensor transfer - IF rigid: Resection +/- tendon release OR arthrodesis OR extensor tenotomy OR just amputate 4) Overlapping - Valgus of great toe -> 2nd toe riding above great toe, treat by correcting Hallux valgus - Congenital 5th toe overlap, treat by holding w tape/K wire for 6 weeks.

What are the skeletal dysplasias associated with neurofibromatosis?

1) Head: - Sphenoid wing dysplasia - Macrocephaly 2) Thorso: - Scoliosis - Dural ectasia (ballooning of dural sac around spinal cord - pain in lowerback/abdomen/headaches/legs w perineal numbness) - Short stature - Pectus excavatum 3) Peripheries: - Pseudarthrosis (false joint due to traumatic/non-traumatic fracture) - Tibial bowing

In what other conditions may RF be raised?

1) Healthy people 2) People with chornic inflammation e.g hepatitis, TB 3) Other rheumatic diseases e.g SLE, Sjogrens

What are the causes of post-thyroidectomy desaturation?

1) Hematoma 2) Recurrent laryngeal nerve injury 3) Hypocalcemia (from unplanned parathyroid excision) 4) Tracheomalacia 5) Laryngeal edema

What are the tumors associated with NF1?

1) Neurofibroma (cutaneous, subcutaneous and plexiform, a/w pain and neurological deficit in large numbers) 2) Neurofibrosarcoma (5% lifetime risk) 3) Optic glioma (astrocytoma along optic tract; 10% lifetime risk, possible diplopia/disconjugate gaze, non-pulsatile proptosis) 4) Lisch nodules (hamartomas on iris, seen in 90% of patients) 5) Intracranial tumors (astrocytomes > medullablastoma, epndymomas and harmatomas) also phraechromocytoma 6) Spinal/paraspinal tumors (arise from dorsal root extending into spinal cord = deficits) 7) Leukemias 8) Gastro-intestinal - Neurogenic tumors i.e Gastrointestinal Stromal Tumors (GIST)

What are the causes of ptosis? (incomplete)

1) Neurogenic (levator palpebral superioris vs superior tarsal muscle) - Third nerve palsy (intracranial +/- syndromes) - Third nerve aberrant regeneration - Horner's syndrome (1st vs 2nd vs 3rd order) 2) Myogenic - Myositis - Muscular dystrophy - Myasthenia Gravis/LEMs/Millerfischer - Chronic progressive external ophthalmoplegia - Oculopharyngeal muscular dystrophy 3) Aponeurotic - Senile ptosis - Post-operative ptosis - Post-traumatic psosis -> resulting in failure of levator aponeurosis 4) Mechanical - Compression e.g thyroid eye disease/ orbital compartment syndrome - External ophthalmic issues (lid edema/chalazion/symblepharon etc) /foreign body 5) Neurotoxic - Botulism 6) Congenital

What are the possible causes of a patient being unable to tip toe?

1) Neurological - Cerebral palsy - Spina bifida - Spinal cord injury 2) Muscular - Charcot-martie tooth - Muscular dystrophy - Disuse atrophy 3) Tendinous - Posterior tendon dysfunction

What are the possible manifestations of renal carcinoma?

1) Hematuria - Painless, gross - +/- clots and 2' ureteric colic/obstruction 2) Pain - Loin pain - esp when tumor breaks through false capsule 3) Paraneoplastic syndrome - Anemia - Hepatic dysfunction (Stauffer syndrome: dysfunction w/o hepatic mets due to cytokine release) - Fever - Hypercalcemia (via prostaglandins that stimulate boen resorption) - Polycythemia (from erythropoeisis) - 2' amyloidosis - Thrombocytosis - Polymyalgia rheumatica 4) Varicocele/LL swelling - Due to occlusion of Left renal and testicular veins via direct spread along renal vein - May obstruct IVC as well, causing LL swelling 5) Haemorrhage - Sudden severe if RCC ruptures 6) Mets - Back pain/fractures - LOW/LOA 7) Hypertension - rare but possible

What are the stages of wound healing?

1) Hemostasis 2) Defensive/inflammatory phase 3) Proliferative phase 4) Maturation phase

What are the breast screening algorithms for patients with increased risk and those on hormone replacement therapy?

1) High risk (fmhx etc) - Start screening 5 years before youngest age of CA ddx - Monthly breast self exam - 6 monthly clinical breast exam and u/s breast - Mammography yearly 2) Hormone replacement therapy: - 40 to 49: same as general population - 50 onwards: biannualy until 5 years after cessation of HRT

What kind of lower limb amputations are there?

1) Hip - Disarticulation 2) Thigh - Transfemoral (AKA AKA hahah) 3) Knee - Disarticulation 4) Tibial - Transtibial (BKA) 5) Foot (in ascending order, TLCS) - Transmetatarsal - Lisfrancs - Choparts - Symes (ankle disarticulation) Tender Loving CareS

How is bullous pemphigoid diagnosed?

1) Histological biopsy - Direct immunofluoresence (gold standard) - Skin is placed under microscopy w ab binding and fluoresence 2) Serum/blood test - Indirect immunofluoresence (serum ab applied to cadeveric skin to see if binding) - ELIZA test for presence of ab (more specific but limited to known, available markers)

What is the diagnostic criteria of ulcerative colitis?

1) History Diarrhoea > 4 weeks 2) Endoscopic findings - Loss of vascular markings (due to mucosal engorement) - Granular mucosa with erosions/shallow ulcerations - Exclusion of other causes 3) Biopsy findings - Crypt abscess/branching/atrophy - Lamina propriya cellularity, eosinophils, lyphoid aggregates

What is the triple assessment for breast disease?

1) History and clinical examination 2) Imaging 3) Pathology IF all 3 agree with each other i.e concordance = 95% certainty that diagnosis is correct If non-concordant, should reassess/repeat tests

How would you work up a suspected malignant soft tissue or bone tumor?

1) Hx and PE 2) Invx o Labs - FBC, RP (+Ca, P) - ESR, CRP - ALP, LDH (if bone tumor) - +/- myeloma panel o Imaging - XR +/- CT +/- - MRI o Staging - CTTAP - PET scan - Skeletal survey (if bone tumor)

How do patients with spondylodiscitis present?

1) Hx of UTI, pneumonia, skin infection, organ transplant etc 2) Fever 3) Pain - severe and insidious w onset - worse w activity and unrelenting - night pain 4) Neurological deficits (10-20%)

What are the radiological signs of subarachnoid haemorrhage?

1) Hyperdense basal cisterns 2) Star sign

What is Klumpke's palsy and its pathophysiology? What is an associated condition?

1) Hyperextension of all MCPJs and flexion of all IPJs (lumbricals all knocked out) 2) Wrist extension (wrist flexors out) C8-T1 lesions of brachial plexus, resulting in median and ulnar nerve palsy a/w Horner's syndrome

What are the dermatological changes in patients with Addison's disease ?

1) Hyperpigmentation - Diffuse but more intense in sunexposed areas - Worse at friction sites/flexures/ bony tuberositis - Muscosal pigmentation also visible 2) Loss of axillary and pubic hair

What are the clinical signs of thyroid storm?

1) Hyperthyroidism signs - Hyperthermia - Tachycardia - Clinical signs: (eye, peripheries, limbs, neck) - Weight loss - Hyper-reflexia - Diarrhoea etc 2) Multi-organ dysfunction - CNS (mandatory criteria): AMS, confusion, delirium, agitation, stupor and coma - CVS: arrythmia, haemodynamic instability, heart failure - GIT: abdominal pain, diarrhoea, vomiting, liver dysfunction with jaundice, renal failure - MSK**: Thyrotoxic periodic paralysis

What are the possible side effects of sulfonylurea?

1) Hypoglycemia - Avoid in elderly - Must ask about hypogly episodes if you see in list 2) Weight gain - Avoid in obese patients

What is the classical triad of fat embolism? What other signs would you look for?

1) Hypoxemia 2) Petechial rash 3) CNS abnormalities Major criteria of Gurd's criteria - ACHE 1) Hypoxemia (<60mmHg) 2) Axillary or subconjunctival petechiae 3) CNS depression disproportionate to hypoxemia 4) Edema (pulmonary)

How do you reverse warfarin?

1) IV Vit K to overcome warfarin effect (5-10mg) and add on the following if bleed is severe: 2) Prothrombin complex concentrate OR fresh frozen plasma (15ml/kg) if not available

What are the mimics of stroke?

1) Neurological - Epilepsy - Hemiplegia migraine - Infection (meningitis, encephalitis, abscess) - Intracranial mass lesions - Multiple sclerosis (rare) 2) Non-neurological - Hypogly, hypergly - Hyponatremia - Syncope (head, heart, vessels) - Sepsis - Drug overdose (alcohol overdose/withdrawal) - Functional disorder e.g conversion disorder

What is the treatment of Cryptococcal meningitis? What are the side effects?

1) IV amphotericin with 5FC (flucytosine) - 5FC increases effect of amphotericin 2) Therapeutic lumbar punctures - Relief of pressure since cryptoccocal meningitis has high ICP (thats how it kills) Side effects 1) Amphotericin - Risk of renal damage and electrolyte imbalances - Pre-hydration and pre-medication required 2) Flucytosine - bone marrow suppression

What are the international patient safety goals?

1) Identify 2) Effective Communication 3) High alert medications 4) Correct site, patients, procedure, surgery 5) Reduce risk of HAI 6) Reduce falls

What is the management of Colles fracture?

1) If stable - Manipulation and closed reduction 2) If unstable - Extraarticular or 2 part intraarticular fractures - Percutaneous pinning i.e K wire - External fixation - ORIF w plating + screws/intramedullary nail

What investigations would you order or perform to diagnose chondromalacia patellae?

1) Imaging - XR: may show subchondral slcerosis/degenerative changes on inferior aspect of patella - MRI: high signal abnormal cartilage suggestive of inflammation or complete loss of cartilage 2) Procedures - Arthroscopy: can be diagnostic and symptomatically therapeutic, to see damaged/ fibrillated cartilage.

What are the causes of a positive tredenlenberg test?

1) Neurological - spinal cord lesions, L5 radiculopathy - superior gluteal nerve injury (iatrogenic, peripheral neuropathy from infection, trauma etc) 2) Muscular - Myopathy (infection, injury) - Tendinous rupture (trauma) 3) Bony - Congenital (DDH, Perthes, SCFE) - Acquired (AVN, stress/fracture, dislocation)

What is the management of physeal distal radius fractures?

1) Immobilised fracture first in sling, then assess for N&V compromise 2) Salter I and II: M&R very gently (to reduce growth plate damage) - Traction and counter traction - Reduce the injury - Backslab, change to full cast in 1 week, healing in 3 weeks 3) Salter III: can attempt M&R, ORIF w pins/screws if not clear 4) Salter IV and V: ORIF w pins and/screws

What are the complications post-surgery in trauma?

1) Immobility related - Pressure sores - DVT/PE - Chest infection 2) Fracture related - Wound, pin, pin tract infection - Osteomyelitis - Swelling, compartment syndrome - Rhabdomyolysis - Delayed union - Non union - Malunion - Premature physeal closure - Fracture associated sarcoma

What are the possible skin lesions associated with Diabetes mellitus?

1) Immunocompromisation - Bacterial infections - Candidiasis 2) Vasculopathy - Peripheral vascular disease (ulcers, trophic changes, dystrophic nails) - DM dermopathy (red-brown shin papules -> circumscribed atrophic hyperpigmented areas +/- scaling) - DM rubeosis (flushing 2' decreased vascular tone) 3) Neuropathy - Dry skin (autonomic neuropathy) - Neuropathic ulcers 4) Metabolic changes - Necrobiosis lipoidica diabeticorum - Diabetic thick skin (scleredema) - Eruptive xanthomas (2' hypertriglyceridemia) 5) Insulin related - Allergic reactions (pruritis, urticaria, angioedema) - Lipohypertrophy

What factors does management of periprosthetic fractures depend on?

1) Implant stability 2) Osteoporosis

What are the possible neurovascular deficits in humeral shaft fractures?

1) Neurological: - Radial nerve (spiral groove, holstein lewis fracture) 18% of all cases, function should recover within 3-4 months, surgical exploration required if not) - Median, ulnar nerve 2) Vascular: - Brachial artery injury - Penetrating branches of brachial artery injury (bone ischemia) 3) Compartment syndrome

What are the findings of a normal cholangiogram?

1) No filling defects 2) Filling of intrahepatic ducts 3) Smooth tapering of common bile duct 4) Contrast flow into the duodenum 5) Normal diameter of common bile duct (4-8mm)

What are the risk factors for Diabetes Mellitus?

1) Non modifiable: - Diseases - Obesity - Hypertension - HLD - Coronary artery disease - Gestational DM - PCOS - PreDM diagnosis - Impaired fasting glucose - Impaired glucose tolerance

What are the risk factors for rheumatoid arthritis?

1) Non modifiable: - Gender: F:M = 3:1 - Age 40-50 - Genetic: HLA DR4 and DR1 - Functional: high impairment 2) No modifiable risk factors Treat early or it will progress, 20-30% become work disabled within 3 years

What is the management of patients with Sever's disease?

1) Non operative - Activity limitation, allow for recovery - NSAIDs for pain - Use of cushioning shoewear with heel pads or heel cups - Icing prior to and after sports acitivities - Short leg cast for immobilisation in cases of persistent severe pain 2) Operative - Not indicated, usually self resolves completely but prone to recurrence, especially in boys

How would you manage a patient with Posterior tibial tendon dysfunction?

1) Non operative - Ankle-foot orthosis (to support valgus collapse) - Immobilise in walking cast/boot for 3-4/12 2) Surgical o Flexor digitorum longus tendon transfer o Correction of deformities - Hind foot valgus: calcaneal osteotomy - Equinus contracture: tendon achillies lengthening or gastrocnemius recession - Forefoot ABDuction: corrective osteotomies o Additional stablisation - Triple arthrodesis (calcaneal-cuboid, talonavicular, subtalar joints with medial column stabilisation)

How would you manage equinovalgus?

1) Non operative - Bracing - PT - Botox injections 2) Operative - Calcaneal oestotomy + TAL and peroneus brevis lengthening - Indicated for rigid deformities.

What are the management options for pes planus?

1) Non operative - If assymptomatic, can be left alone (children will resolve w age) - Stretching of tendoachillies - Shoewear and orthotics 2) Operative - Achillies tendon and gastrocnemius lengthening - Indicated in flexible pes planus with tight heelcord and severe refractory pain on stretching.

How would you manage scheurmann's kyphosis?

1) Non operative - Indicated in patients w kyphosis < 60 degrees and assymptomatic or mild pain - Bracing with extention type orthoses 2) Operative - Indicated if kyphosis > 75deg, neurological deficit/ spinal cord compression, severe pain - Spinal decompression, deformity correction and stabilisation with instrumentation and fusion

What is the definitive treatment of knee dislocations?

1) Non operative - Most cases require surgical 2) Surgical o Indicated in: - vascular injury (assessed via doppler) - associated fractures (open) - irreducible dislocaiton - compartment syndrome - Obesity - Mulitrauma patient o Emergent surgical intervention - Repair of vessels - External fixation - Fasciotomies o Delayed surgery - Ligamentous injury for repair/reconstruction Above knee cast after swelling subsides

What is the treatment of osgood schlatter?

1) Non operative - NSAIDs - Activity limitation/rest - Strapping/ sleeves to reduce tension on the apophysitis - PT w quadriceps stretching - 90% have complete resolution 2) Operative - Ossicle excision - Indicated in refractory cases (10%) or in skeletally mature patients with persisteny symptoms

How is clostridium difficile severity determined? When is treatment indicated?

1) Non severe - WBC < 15000cells/ml and serum creatinine <1.5mg/dL 2) Severe - WBC > 15000 and or creatinine > 1.5mg/dL 3) Fulminant colitis - Hypotension, shock, ileus, megacolon Treat if acute diarrhoea >/= 3 loose stools in 24 hrs, signs of severe/fulminant colitis +/- positive diagnostic laboratory assay If not lab assay, can still treat, but if lab results positive w/o diarrhoea, NO TREATMENT INDICATED.

What are the possible presenting complaints of patients with gastric cancer?

1) Non specific symptoms - Abdo pain - N&V - Early satiety, LOA/LOW - Hemetemesis/ melena 2) Symptoms a/w complications o Bleeding - Anemia sx o Obstruction - I/O signs o Perforation o Malnutrition o Metastases - Bony pain - Dyspnoea - Abdo mass etc

What is the management of Clergyman's knee?

1) Non surgical - NSAIDs for pain relief - Cold compress - Activity limitation (stop kneeling) 2) Surgical (for chronic cases) - Aspiration - Excision

How would you manage pathological fractures?

1) Non-neoplastic - Osteoporosis - Metabolic bone disease etc 2) Neoplastic - Known hx of CA: re-stage, open biopsy and frozen section to confirm, KIV fixation - Known hx of bone mets: work up KIV fixation - Unknown hx of CA: treat as if bone tumor, local scans and staging invx, open biopsy w frozen section to confirm, KIV fixation

How would you treat ankylosing spondylitis?

1) Non-operative - Analgesia via NSAIDs, Cox 2 inhibitors - Physiotherapy to maintain flexibility - TNF alpha blocking agents e,g infliximab, etarnacept, adalimumab 2) Operative - Indicated for treatment of fractures, severe deformity or arthritis - Spinal fractures o Non operative: traction, orthosis or halo-immobilisation o Spinal decompression with instrumentation and fusion - Deformity o Corrective lumbar or cervical thoracic osteotomies - Large joint arthritis o THR or arthroplasty

What are the management options for lisfranc ligament injuries?

1) Non-operative - Cast immobilization for 8 weeks - Only if no displacement on weight bearing XRs and stress radiographs + no fractures - Or if patients are not fit for surgery 2) Operative - ORIF (if unstable > 2mm shift) - Arthrodesis (if chronic/delayed treatment and only ligamentous injuries.

How would you manage carpal tunnel syndrome?

1) Non-operative - Indicated as first line of treatment - NSAIDs - Night splints - Activity modification (avoid aggravating activity) - Adjunct: steroid injection 2) Operative - Indicated in failure of conservative treatment - Carpal tunnel release surgery (can be endoscopic or open, to cut the transverse carpal ligament aka flexor retinaculum)

What is the treatment for dequervains tenosynovitis?

1) Non-operative - Indicated as first line: NSAIDs, thumb spica, rest - 2nd line: Intracompartmental steroid injection 2) Operative - Indicated in refractory cases >6months, severe pain - Surgical release of extensor compartment

How do you manage prolapsed intervertebral discs?

1) Non-operative - Indicated for first line as 90% resolve without surgery - Rest - PT (extension exercises, traction, chiropractic manipulation) - NSAIDs - Selective nerve root corticosteroid injections (2nd line of therapy if medications fail) 2) Operative - Indicated in persistent disabiling pain > 6 weeks refractory to conservative tx, progressive and significant weakness or CES) - Laminotomy and discectomy

How would you manage acute calcific tendinitis?

1) Non-operative - Indicated for first line for all phases, often self resolving after resorptive phase - NSAIDs - PT, strengthening and stretching - Steroid injections 2) Operative - Indicated in progression of symptoms, refractory to conservative treatment, interference w ADLs - Surgical decompression of calcium deposit (basically they squeeze it out)

How would you manage spondylodicitis/epidural abcess?

1) Non-operative - Indicated for most cases - Bracing and long term (6-12 week abx) - Broad spectrum abx until susceptibility out (vancomycin + 3rd gen ceph) - Linezolid and daptomycin if resistant strains 2) Operative - Indicated in rafractory cases, neurological deficits, progressive deformity and spinal instability - Surgical decompression, debridement and stabilization w instrumentation

How would you manage a Tillaux fracture?

1) Non-operative - Indicated if displacement is < 2mm after reduction - Closed reduction with long leg cast for 4 weeks and short leg cast for 2-3 weeks - Reduction by internally rotating foot, hence casting must control rotation 2) Operative (usually done) - Indicated if displacement > 2mm after reduction - ORIF with percutaneous pinning/screws

What is the management of chance fractures?

1) Non-operative - Indicated if no neurological deficits, fracture stable (bony chance or intact posterior elements) - Immobilisation in cast or cervical/ thoracolumbosacral orthoses 2) Operative - Indicated if neurological deficits present or fracture is unstable (ligamentous chance w posterior element disruption) - Anterior or posterior decompression and stabilization with instrumentation +/- fusion

What is the management for acute mesenteric ischemia?

1) Non-operative - Indicated if patient haemodynamically stable with no signs of ischemic bowel - Anticoagulation - Longterm: CVS risk factors control w anticoagulation 2) Operative (depends on the cause) - Mesenteric embolism: open embolectomy or percutaneous clot aspiration and catheter directed thrombolytic therapy - Mesenteric artery thrombosis: bypass or SMA angioplasty + stenting - Surgical resection of ischemic, non viable bowel w primary anastomosis/hartmans - Relook laparoscopy in 24-48 hrs TRO bowel ischemia

How would you manage rupture of the achillies tendon? Who gets it?

1) Non-operative - Indicated in acute injuries, sedentary patients or medically frail patients - Functional bracing/casting in equinus 2) Operative - Indicated for acute ruptures < 6 weeks - Open/percutaneous end-end achillies tendon repair - Reconstruction - Flexor hallucis longus tendon transfer Outcomes ~ the same Tends to occur in 1) Weekend warriors (episodic athletes) 2) Steroid users 3) Fluoroquinolone Abx

How would you manage olecranon process fractures?

1) Non-operative - Indicated in undisplaced fractures, elderly patients, patients unfit for surgery - Long arm cast immobilisation in 45-90 degree flexion with early RoM and strengthening 2) Operative - Indicated in displaced fractures with no comminution - ORIF (plates and screws, tension band, intramedullary nail)

How would you manage Monteggia fractures?

1) Non-operative - Indicated more commonly in children due to higher success rates - Closed reduction with long arm cast in flexion (110deg and full supination) 2) Operative - Indicated in acute fractures that are open or unstable e.g long oblique fractures/comminuted fractures, also first line for adults - ORIF (plates + screws /IM nailing) of ulnar shaft +/- radial head reduction (usually reduces itself after pull by interosseus membrane)

How would you manage congenital muscular torticolis?

1) Non-operative - Passive stretching: if condition < 1y/o and limitation < 30deg 2) Operative - SCM relsease: indicated if failed 1 year of stretching or significant deformity

What is the treatment for developmental dysplasia of the hip?

1) Non-operative - Pavlik harness: indicated if < 6months old and hip reducible with norma muscle function (hence cannot in spina bifida and spasticity) 90% success after 3-4 weeks - Closed reduction and spica casting: indicated if fail pavliks or 6-18 months old, aim < 55deg ABDuction bec risk of osteonecrosis) 2) Operative - Open reduction and spica casting: indicated if fail closed reduction or > 18 months old - Open reduction w femoral osteotomy: indicated if > 2y/o - Open reduction w pelvic osteotomy: indicated if > 4y/o

What is the management of congenital talipies equinovarus?

1) Non-operative - Ponsetti method of serial casting o From within 1st week of life, weekly casting to correct deformities o +/- TAL o Followed by Foot ABduction orthoses 2) Surgical - Indicated if ponsetti casting fails - Soft tissue release and tendon lengthening.

How do you treat giant cell tumors?

1) Non-operative - RT alone (for surgically unfit patients, inoperable tumors) - Bisphosphonates/ denosumab 2) Operative - Extensive curretage and reconstruction w bone graft/ cement + adjuvant therapy (phenol, hydrogen peroxide, argon beam etc) - Amputation (hand lesions not amenable to resection)

How would you manage sternoclavicular joint dislocations?

1) Non-operative - Reassurance and local symptomatic treatment if atraumatic or > 3 weeks old - Sling for comfort - Return to unrestricted activity by 3 months OR 2) Operative - If acute dislocation: closed reduction under GA +/- thoracic surgery - If posterior dislocation w dysphagia, SOB, decreased peripheral pulses: open reduction, soft tissue reconstruction +/- thoracic surgery - Medial clavicle excision if persistent pain, recurrent/chronic dislocation

What are the managements of burst fractures?

1) Non-operative - indicated if no neurological deficit + mechanically stable (posterior elements intact, acceptable kyphosis/loss in vertebral body height) - Immobilisation with cervical/ thoracolumbosacral orthoses (may provide symptomatic relief, may not be needed) - Bony fragments are resorbed over time and do not cause neurological deterioration (ORTHOBULLETS) 2) Operative - Indicated if neurological deficit present or mechanically unstable (posterior elements not intact, unacceptable kyphosis/loss of vertebral body height, radiographic evidence of cord/nerve compression by fragment) - Surgical decompression and stabilization with instrumentation +/- fusion

How would you manage patellar fractures?

1) Non-operative - indicated in minimally displaced fractures with intact extensor mechanism - Knee immobilisation in extension with brace/ cylinder cast - Early ROM in hinged knee brace 2) Surgical - Indicated in extensor mechanism failure (cannot SLR) - open fractures - Displacement >3mm - ORIF with tension band construct - Partial/total patallectomy (for severe, comminuted fractures)

What are the symptoms of legionella pneumonia?

1) Non-productive cough 2) Fever w bradycardia (Faget's sign) 3) Myalgia

How should osteomyelitis be managed?

1) Non-surgical - IV or oral abx therapy for 4 to 6 weeks - +/- hyperbaric oxygen therapy (used as adjunct in refractory OM) 2) Surgical - Irrigation and debridement followed by organism specific antibiotics - + bone biopsy and culture of joint fluids/tissue - +/- internal fixation in bone fractures

How would you manage a Baker's cyst?

1) Non-surgical - Usually self-resolves in 10-20 months hence observation before surgery is indicated. - If symptomatic: needle aspiration with steroid injection for relief, plus NSAIDs/Icing to reduce inflammation, rest and elevation of the leg 2) Surgical - Indicated in persistently symptomatic or complicated cysts with failure of conservative treatment - Surgical excision

What are the 5 criteria of a normal cholangiogram?

1) Normal diameter 2) No filling defects 3) Intrahepatic ducts well visualized 4) Smooth tapering of the distal bile duct 5) Flow of contrast to the duodenum (prompt duodenogram)

What are the causes of Ricket's disease? What does it result in?

1) Nutritional - Most common cause: vitamin D deficiency - Dietary limit, insufficiency sun exposure, exclusive breastfeeding, coeliac's disease etc - Calcium and phosphate deficiencies too 2) Renal rickets Disease results in weak or soft bones in children

What are the risk factors of OA and RA?

1) OA - Family history - Trauma 2) RA - Family history - Smoking - Females

What are the joint deformities associated with psoriasis?

1) OA: degeneration of joints 2) RA 3) Anklosing spondylitis: psoriatic spinal arthropathy 4) Arthritis mutilans: severe form of chronic RA/PsA characterised by resorption of bones and consequent collapse of soft tissue, which manifests clinically as telescoping fingers or sausage fingers.

What are some important complication of spleenectomy?

1) OPSI - overwhelming post spleenectomy sepsid Usually caused by 3 H influenza type B Strept Pneumoniae N. meningitidis Provide pre-op vaccination, repeated 3-5 years If patient below 18 y/o, requires penincillin prophlyaxis until after 18y/o. 2) Thrombocytosis - need aspirin for thromboembolism prophylaxis

What are the complications of pancreatic cyst? How would you treat it?

1) Obstruction 2) Infection 3) Rupture 4) Bleeding - from pancreatic necrosis with digestion of surrounding arteries - Formation of pseudoaneurysm 1) Cystogastrostomy or cysto-something - Connection of pseudocyst to luminal structure to drain - Can be done endoscopically (puncture hole, place stent to connect - Can be done laparoscopically or open as well.

What are the possible mechanisms of raised lactate and their respective causes?

1) Pyruvate o Increased production - Sepsis - Glycogen storage disease - Beta blockers o Impaired utilization - Congenital (G6PD) - Reye's syndrome o Favoured conversion to lactate - Seizures - Diseases of increased metabolic exertion e.g asthma, severe exercise - Diseases of reduced oxygen delivery e.g shock, cardiac arrest/failure, hypoxemia - Diseases of reduced oxygen utilization e.g cyanide, drugs e.g zidovudine, stavudine, sepsis 2) Lactate o Decrease in lactate metabolism - Hypoperfusion - Alcholism - Liver disease 3) Unknown mechanism - Malignancy - DM, metformin - AIDs - Hypoglycemia - Idiopathic

How would you manage an elbow dislocation?

1) Posterior dislocation o Non-operative - If < 8y/o can reduce w anterior pressure on olecranon - If >8y/o need to flex elbow w distal traction (because reduction in hyperextended elbow a/w increased risk of median nerve entrapment and soft tissue injury) - Posterior splinting at 90 degrees with loose circumferential wraps OR if no assoc fractures: hinged elbow brace w stable arc of motion - Early gentle, active RoM 5-7 days after reduction - Full recovery after 3-6 months o Operative - Indicated if soft tissue/bone entrapment/ closed reduction impossible, unstable, loose fragments etc - ORIF w plates/screws 2) Anterior dislocation o Initial mx w M&R under anesthesia - Traction distally to flexed forearm w dorsally directed pressure on volar forearm and anteriorly directed pressure on humerus. - Assess triceps after because may have been stripped form olecranon. o ORIF - If olecranon also fractured.

What are the tests for posterior cruciate ligament tear?

1) Posterior sag 2) Posterior drawer test 3) Dial test (for posterolateral rotary instability of the knee from PLC injury, - Patient is prone w knee in 30deg flexion and ankles at max dorsiflexion to prevent rotation - Then cup hands at heels and maximally externally rotate the heels, measure the thigh foot angle. - Repeat at 90 deg knee flexion - > 10 deg difference is significant - isolated PLC injury if >10deg at 30 deg flexion, but not at 90deg flexion because restriction by PCL - Both can be injured.

What are the possible causes of urinary retention?

1) Obstruction Ureter -Intraluminal: stones, abscess -Mural: congenital PUJ narrowing, ureteric neoplasm, stenosis/scarring from instrumentation/injury/infection -Extramural: intra-abdominal masses, renal neoplasm, iliac artery aneurysm (common site), bladder CA (ureterovesical junction) Bladder -Intramural: clots, stones, abscess -Mural: bladder CA -Extramural: pelvic masses Urethra -Intramural: clots, stones, abscess, foreign body (ring pessary, toys) -Mural: stenosis, traumatic injury, urethritis, urethral neoplasm -Extramural: pelvic masses (pregnancy, constipation, tumors, fibroids), pelvic organ prolapse (cystocele, rectocele, procidentia) 2) Contraction Neurogenic (central, peripheral) -Central: stroke, parkinson's disease -Peripheral: diabetes mellitus Drugs -Anticholinergics (from PD: artane, benztropine but also: orphenadrine, atropine) -Antihistamines -Decongestants Decompensation -Chronic obstruction leading to decompensation Post-surgical -Pain -Post partum -Immobility

What pathological lesions can the CT IVP detect?

1) Obstruction along upper urinary tract and bladder (stones, strictures) 2) Kidney function (poor contrast excretion) 3) Malignancies (enhancing lesions) 4) Complications - Renal rupture: extravasation of contrast - Hydronephrosis: visible on plain+corticomedullary phase

What are the significant physical findings of congenital muscular torticolis?

1) Of course the torticolis: head tilt (towards affected side) and chin rotation (away from affected side) 2) Palpable SCM mass 3) NO PAIN (if painful suggests different diagnosis)

Why other areas should you examine after a spine examination? Why?

1) Offer to examine the hip: Hip pain may be due to sciatica (L4,5,S1) in lumbar radiculopathy, but important to rule out other hip pathologies. 2) Offer DRE - To assess for anal tone - Take hx on saddle anesthesia as well - TRO Cauda equina syndrome

What is the meds in triple therapy?

1) Omprazole 20mg BD 2) Clarithromycin 500mg BD 3) Amoxicillin 1000mg BD 7-10 days

What lumbar puncture findings are typical of TB and crytococcus?

1) Opening pressure >20 is high 2) Lymphocytosis

What are the possible manifestations of rheumatoid arthritis?

1) Orthopedic - Hands: lots + arthritis mutilans - Elbow: elbow RA - Shoulder: shoulder RA - Hip: protrusio acetabuli - Cspine: AA instability - Knee: knee RA - Foot and toes: claw toe, talonavicular arthritis 2) Non orthopedic - Rheumatoid vasculitis - Pericarditis - Interstitial lung disease - Still's disease (fever, rash, splenomegaly) - Felty's syndrome (splenomegaly and leukopenia) - Sjogren's syndrome (dry eyes, mouth) - Skin: Rheumatoid nodules, rash

What are the side effects of thiazolidinediones?

1) Osteoporosis 2) Weight gain 3) Fluid retention - Avoid in IHD/CCF 4) Hepatotoxicity

What are the differentials of blastic bone lesions?

1) Osteosarcoma (can be 2nd decade of life or 50y/o, look for sunburst reaction) 2) Paget's disease of bone (esp if elderly)

What are the scaphoid series radiographs?

1) PA (for inercarpal space and DRUJ) 2) Oblique 3) Lateral (TRO lunate d/l) 4) Angled PA (to show scaphoid facing forwards)

What are the signs of severe mitral stenosis?

1) PHTN 2) Long diastolic murmur 3) Shorter interval between S2 and opening snap (Early diastolic sound -> due to raised Left atrial pressures)

What are the investigations in the trauma series?

1) POCT: - ABG - ECG - CBG 2) Laboratory tests: - FBC - PT/PTT - GXM 3) Imaging studies: - FAST - CXR, PelvicXR

What are the causes of hypercalcemia?

1) PTH - 1' hyperparathyroidism - ESRF 2' hyperparathyroidism - 3' hyperparathyroidism 2) Familial hypocalciuric hypercalcemia 3) Drugs - Lithium associated hypercalcemia 4) Malignancy - Bone metastasis w osteolysis - Paraneoplastic syndrome o Parathyroid related peptide - Increased 1 alpha hydroxylase activity e.g in lymphoma, increases activation of vit D) 5) Antagonistic autoantibodies to the calcium sensing receptor

What are the signs of acute calcific tendinitis?

1) Pain - Over supraspinatus tendon, felt under acromion over humeral head 2) Supraspinatus tendinitis physical signs - See above

What are the signs of a Slipped Capitofemoral epiphysis?

1) Pain (for acute slips) 2) Limb length discrepancy, with external rotation and flexion 3) Coxa plana deformity: flattened femoral head due to necrosis with 2' bone healing 4) Limited abduction and internal rotation

What are the 3 or 4 anastomoses required for Whipples?

1) Pancreatio-enterostomy e.g jejunum or gastrostomy - No difference in outcome 2) Hepatico-jejunostomy - from common hepatic ducts to viscus 3) Jejuno-gastrostomy 4) +/- Jejuno-jejunostomy - To prevent biliary reflux e.g in Roux-en Y but not done because not as common in asian population

What are the histological findings of papillary thyroid carcinoma?

1) Papillary architecture 2) Orphan Annie nuclei - With pseudoinclusions 3) Psammoma bodies Possible tall cell variant where papillary architecture within a follicular lesion -> has worse prognosis.

What are the biopsy features of medullary thyroid cancer?

1) Parafollicular C cells 2) Amyloidosis due to calcitonin production

Post surgical dyspnoea: What are the possiblities?

1) Parenchymal - Atelectasis (poor inspiration-> poor alveoli inflation-> collapse w mucus and subsequent infection, can come with FEVER) - Barotrauma/pneumothorax (sharp pain on rupture but after its just breathlessness) - Lung contusion - Pneumonia 2) Thoracic - Fluid overload (cardiac failure or iatrogenic over infusion, make sure to listen clearly at bases, may have cardiac wheeze) - Effusion - Diaphragmatic splinting - Incisional pain 3) Vascular - Pulmonary embolism (pain if lung infarct) - AMI 4) Metabolic causes - Sepsis w lactic acidosis 2' to perforation

Causes of Parkinsonism? (6)

1) Parkinson's disease 2) Parkinson plus syndromes 3) Drugs (neuroleptics, antiemetics, MPTP: 1 methyl 4 phenyl 1,2,3,6 tetrahydropyridine - a prodrug of a neurotoxin that damages the substantia nigra) 4) Anoxic brain damage 5) Post encephalitis 6) Tumors e.g giant frontal meningiomas

What are the names of nail related infections and their respective locations?

1) Paronychia = eponychium infection 2) Felon = pulp abscess 3) Nail infection = onychomycosis

What are the treatment principles of psoriasis?

1) Patient education and counselling 2) Specific therapy for lesions 3) Screening and management of comobidities (baseline metabolic screening, referral to relevant subspecs)

What are the factors that affect bone healing?

1) Patient factors - Malnutrition - Steroid use - Old age - Immunocompromisation - Co-morbidities (e.g DM, osteoporosis) - Genetic disorders (e.g hemophillia etc) 2) Bone factors - Infection, debris, dead tissue in wound - Damage to nerves and blood vessels - Bone pathology (tumors) - Improper reduction/abnormal position

Name the parkinson plus syndromes and their respective unique characteristics when compared to classical parkinson's disease

"1) Shy-drager's syndrome (i.e multiple system atrophy): depends on subtype: MSA-C (cerebellar: a/w cerebellar signs of ataxia, falls, dysdiadokinesia), MSA-A (autonomic: a/w postural hypotension, erectile and urinary dysfunction), MSA-P (parkinsonism dominant: parkinson sx more obvious), All with corticospinal signs i.e hyperreflexia and upgoing plantars 2) Progressive supranuclear palsy (frontal lobe): downward gaze disturbance (first downgaze, then upward gaze then horizontal gaze palsy), blepharospasm, frontal lobe signs (palmomental reflex: stroking thenar eminence results in twitch in chin and grasp reflex: putting anything in their hand results in them grasping it = contralateral frontal lobe damage) 3) Lewy body dementia: associated with early falls, early dementia w/ hallucinations and postural hypotension 4) Corticobasal degeneration (frontoparietal lobe): limb apraxia or alien hand syndrome (patient has no control over limb) and acute dystonia (abnormal, sustained, often painful muscular spasms producing twisted postures) 5) Vs Parkinson's: NO early falls, early dementia or postural hypotension and symptoms tend to be assymetrical (where all parkinson plus are symmterical (CBDG may be assymetrical)

What is the diagnostic criteria of rheumatoid arthritis according to the 2010 American College of Rheumatology?

'2 Bloods and 2 Bones' - Diagnosis of RA requires both clinical and laboratory criteria. 1) Joints - 1 large joint = 0 pts - 2 to 10 large joints = 1 pt (large joints less likely) - 1 to 3 small joints = 2 pts - 4 to 10 small joints = 3 pts - >10 small joints = 5 pts 2) Duration of arthritis - At least 6 weeks = 1 pt 3) Serological (Rheumatoid factors and anti-citrullinated protein antibody) - RF(-) or APCA (-) = 0 pts - RF (low+) or APCA (low+) = 2 pts - RF (high+) or APCA (high+) = 3 pts 4) Acute phase reactants - Elevated CSP/ESR = 1pt RA is diagnosed if 6 or more points + synovitis not explained by other causes

What are the components of the sepsis bundle?

'6 things to do, investigation + the treatment of poor investigation result' 1) Urine output check + fluids 2) Lactate ABG + oxygen supplementation 3) Blood culture + IV antibiotics

Describe the lifecycle of a red blood cell after 120 days.

1) In phagocytes - Phagocytosis by macrophages that break RBC down to heme and globin component - Heme component broken down to iron and biliverdin - Globin component broken down into amino acids for recycling 2) In blood - Iron bonded to transferring and brought to liver - Biliverdin rapidly reduced to form unconjugated bilirubin by biliverdin reductase - Unconjugated bilirubin has to bind to albumin before subsequent transport to the liver 3) In liver - Iron is stored, subsequently bonded to transferrin again for transport to bone marrow for erythropoeisis - Unconjugated bilirubin is conjugated via glucuronidation by uridine disphosphonosyltransferase. 4) Gall bladder - Conjugated bilirubin transported via bile canaliculi to GB for storage until ingested fat triggers secretion of cholescystokinin to contract the gall bladder to secrete bile into the duodenum 5) In bowel - Conjugated bilirubin converted by gut bacteria to stercobilinogen and urobilinogen - Most are excreted via stool while some are reabsorbed via enterohepatic circulation and either recycled or renally excreted as urobilin in urine

What is the pathogenesis of acne vulgaris?

1) Increase in sebum secretion (seborrhea) 2) Pilosebaceous duct hypercornification/keratinisation that obstructs it (comedomes) 3) Increase in propionibacterium acnes in the duct 4) Inflammation (acne)

What is the double bubble sign?

1) Indicates dilated duodenum and stomach 2) Obstruction at level of duodenum - Luminal: duodenal atresia, duodenal webs, duodenal stenosis - Extraluminal compression: choledochal cyst, retroperitoneal tumor, superior mesenteric artery syndrome, annular pancreas, midgut volvulus 2' malrotation

What are the indications of good prognosis in patients with thyroid cancer?

1) Patient factors - Patient age < 55 y/o - Female 2) Tumor factors - Tumor size <4cm - No gross extra-thyroidal extension - Low grade tumor - No mets 3) Surgical factor - Resection complete e.g if attached to vessels, need to re-consent, do total if consent already obtained AGES/MACIS/AMES If prognosis is good, can consider hemithyroidectomy

What are the contraindications to joint aspiration?

1) Patient factors - ongoing systemic infection e.g bacteremia - patient immunocompromised - existing coagulopathies 2) Joint factors - prosthesis/implants in place - ongoing adjacent infection e.g OM/cellulitis

What are the indications and contraindications in use of rTPA in ischemic stroke?

1) Indications - Within 4.5 hours (of symptom onset, after which bleeding risk increases) - Ischemic stroke with neurological deficit - Patient > 18 y/o 2) Contraindications o PmHx - Head trauma/stroke < 3/12 - Arterial bleed @ non-compressible site <7/7 - Major surgery/serious trauma < 14/7) - Recent GIT/Urinary tract bleed <21/7) - Recent AMI (3/12) - Stroke and DM Hx - On oral anticoagulants o Investigations - HTN (SBP>185/DBP>110) - CBG <2.7mmol/L - Plt < 100,000 - INR > 1.7 or PT >15 - ASPECTS <7

What is the mechanism of injury for a lisfranc ligament tear?

1) Indirect rotational forces 2) Axial load through hyperplantarflexed forefoot i.e spinters, fall from height, RTAs

What are the causes of genu varum?

1) Infantile Blount's disease (idiopathic osteochondrosis of the medial physeal growth plate) 2) Rickets 3) Osteogenesis imperfecta 4) Proximal tibial physeal injury (trauma, infection, radiation)

What are the post-surgical complications of traumatic fractures and how might they be managed?

1) Infection - Cellulitis/osteomyelitis - Manage risk factors: o retained foreign body o limited soft tissue coverage o nutritional deficiency o multi-system injury 2) Compartment syndrome - High index of suspicion, serial N&V examinations, serial compartment pressure measurement (>30mmHg) - Surgical faciotomy during op

What are the red flags to be ruled out in a limping child?

1) Infection - Systemic signs: fever, dyspnoea, toxic appearance - Septic arthritis (medical emergency) 2) Trauma 3) Autoimmune - Stiffness - Family history of heritable arthropathies.

What are the complications of a ganglion cyst?

1) Infection 2) Chronic inflammation: distends tendon sheath above and below the flexor retinaculum 3) Compression e.g nerves 4) Pain

What are the possible causes of hematuria?

1) Infection 2) Obstruction 3) Menstrual 4) Malignancy 5) Congenital 6) Haematologic 7) Exercise induced 8) Glomerulonephritis

What are the causes of limb length discrepancy?

1) Infection (OM, septic arthritis, poliomyelitis) 2) Trauma (physeal injuries) 3) Neurological (cerebral palsy, muscle weakness) 4) Neoplasm (AVMalformations)

What are the complications of diverticulosis?

1) Infection/inflammation - Acute diverticulitis - Diverticular abscess - Chronic diverticulitis 2) Perforation 3) Obstruction - Small bowel (enteric adhesion to diverticulitis) - Large bowel (diverticular stricture formation) 4) Haemorrhage 5) Fistula formation - Vesicocolic - Colovaginal - Coloenteric (with enteritis and malnutrition)

What are the triggers of erythrodermic psoriasis?

1) Infections 2) Medications - Corticosteroids withdrawal (esp injected systemic steroids) - Strong coal tar preparations - Lithium - Antimalarials - Interleukin II 3) Metabolic conditions - hypocalcemia - excessive alcohol intake

What are the signs and symptoms of leptospirosis?

1) Infectious symptoms - Fever/chills/rigors - Myalgia - N&V 2) GIT - Diarrhoea - Hepatitis/liver failure (+ jaundice) - Renal failure (oliguria/anuria) 3) Skin - Rash 4) Eyes - Conjunctival redness 5) Brain - Meningitis (headache, neck stiffness, photophobia) - Delirium/psychosis/confusion 6) MSK - Calf muscle tenderness

What are the risk factors of hepatocellular carcinoma?

1) Infective: - Hep B and C - Risk factors of contracting blood bourne diseases (IVDA, Tattoos, Sex, FmHx w vertical transmission etc) 2) Irritative o Previous disease: - Alcoholic cirrhosis/steatohepatitis - NASH - Hemochromatosis - Acute intermittent porphyria - Cholecystectomy o Dietary - Toxins (alflatoxin, betel nut chewing) - Obesity o Congenital - Alfa 1 anti-trypsin deficiency

How would you apply a skeletal traction pin?

1) Patient is placed under GA or LA 2) Site is cleaned with iodine and alcohol 3) Pin/wire is mounted on hand drill 4) Hold limb at same degree of lateral rotation as normal limb and with ankle at right angles 5) For distal femur pin insertion: go medial to lateral (to avoid adductor hiatus where femoral artery and vein run) For proximal tibia pin insertion: go lateral to medial (to avoid peroneal nerve at head of fibula) 6) Insert pin at right angle to long axis of bone, be sure to avoid muscle and tendon 7) Place cotton buds soaked in alcohol around pin to seal wound.

What are the options of bowel/attachment after large bowel resection?

1) Patient stable - Primary anastomosis - Defunctioning stoma (distal in colon more physiologic and less risk of high output stoma but need MUST DO washout) - Double barrel if right sided lesion (ileocolic stoma) 2) Patient unstable - End colostomy +/- Reversal - Damage control surgery, close up and do later

Types and management of levadopa dyskinesia

1) Peak dose dyskinesia 2)

Types and management of L-dopa dyskinesia

1) Peak dose dyskinesia: reduce dose and frequency, add on COMT inhibitors (entacapone) that increase Ldopa half life, initiate therapy w dopamine agonist first before L dopa. 2) Wearing-off/off period dyskinesia: increase dose and frequency, adding on DA, MOAI, COMTI. 3) Diphasic dyskinesia

What are the stages of bone healing and how long does each take?

1) Inflammation (from time of fracture to ~ 1-7 days after) - Hematoma formation and release of inflamamtory cytokines to trigger cascade of inflammatory response - Eventually forms granulation tissue and deposition of cartilage and fibrous tisse 2) Soft callus formation (2-3 weeks post fracture) - With gradual deposition of cartilage, fibrous tissue and bone - Decrease in pain and swelling 3) Hard callus formation (3-4 months post fracture) - Bone formation moves from periphery to fracture site with combination of intramembranous and endochondral ossification (outside vs soft tissue inside) - Forms woven bone 4) Remodelling (months to years after) - replacement of woven bone with lamellar bone via osteoclastic and osteoblastic action.

What are the complications of a hiatal hernia?

1) Inflammatory - GERD - Gastric carcinoma - Barrett's esophagus - Bleeding 2) Mass effect - Respiratory compromise 3) Tension-related - Obstruction - Perforation - Strangulation

What vaccinations should patients with HIV be offered?

1) Influenza (seasonal) 2) DTaP (Diptheria, Tetanus, PErtussis) 3) Pneumococcal (both PCV13 and PPSV23) 4) Meningoccocal 5) HBV 6) HAV 7) HPV NOT Hemophillus influenzae because most evolve non-typable strains that vaccine does not protect against - Some indication if patient is asplenic

What are the complications of total parenteral nutrition?

1) Infusion/access related - Vascular injury/puncture - Lung injury - Air embolism, hematoma, thrombosis - Catheter related sepsis - Malposition. 2) Nutritional - Dehydration - Electrolyte imbalances - Hyper/hypoglycemia - Micronutrient deficiencies (vitamins and minerals) - Volume overload - Metabolic bone disease 3) Organ failure - Steatohepatitis - Liver failure - Cholelithiasis/sludge formation

What are the indications for external fixation?

1) Initial: - Open fractures with severe soft tissue injury - Closed fractures with severe soft tissue injury - Open book pelvic fractures - Rapid stabilization and reduction of long bone/periarticular fractures (damage control surgery) - Faster, can adjust outside 2) Definitive: - Some long bone/pelvis fractures

What are the indications for surgical intervention in shoulder dislocations and what might they entail?

1) Injury factors: - Soft tissue interposition: cannot reduce - Associated fractures: displaced greater tuberosity fracture, glenoid rim fracture 2) Patient factors: - Young active men with first time dislocation Involves arthroscopic reduction and repair of anterior/inferior labrum (Bankart lesion) +/- capsular shift, muscle/tendon transfers etc if recurrent case

What are the side effects of total parenteral nutrition?

1) Insertion complications (via CV line) - Haemothorax - Chylothorax - Pneumothorax - Air emboli - Nerve and arterial injury 2) Catheter-related - Thrombophlebitis - Embolism/ thombolism - Sepsis - Cath occlusion - Cath rupture 3) Metabolic o Early - Volume overload - Hyperglycemia - Electrolyte deficiencies (PO4, K, Mg, Cl) o Late - Metabolic bone disease - Hepatic steatosis - Hepatic cholestasis - Trace mineral deficiency - Vitamin deficiency

What diabetic drugs are at high risk of hypoglycemia?

1) Insulin 2) Sulfonylurea e.g glicazide, glimipride

What is the insulin carbohydrate ratio and the insulin sensitivity factor/correction factor?

1) Insulin carbohydrate ratio - How many grams of carbohydrates will one unit of insulin cover? - Must teach them to calculate or ask them if it has been calculated - Each patient has a different ICR 2) Insulin sensitivity ratio/correction factor - How many mmol/L one unit of insulin will lower the blood glucose by

What are the phases of TB treatment?

1) Intensive phase - 2 months of RHEZ 2) Continuation phase - 4 months of RH - Can extend if not cleared (sputum, XR) to 7 months (for bone and joint) or 10 months (for CNS) Can be daily dosing or 3X a week.

What are possible vessels for use in CABG?

1) Internal mammary artery 2) Radial artery 3) Long saphenous vein

What is 4, the flood and one more?

1) Intra-abdominal 2) Thoracic 3) Long bone 4) Pelvis Floor: Bleeding loss on the floor via massive bleeding One more: Retroperitoneal (requires CT/ laparotomy)

What are the vascular malformations associated with neurofibromatosis?

1) Intracranial - Vessel stenosis or occlusion - Vessel rupture due to berry aneurysm on b/g of hypertension 2) Gastrointestinal - Vascular compromise 3) Renal - Renal artery stenosis 4) Cardiac - Coarctation of aorta

What is the treatment of keloid?

1) Intralesional steroid injection 2) Surgery is not commonly offered due to risk of recurrance

How are thyroid storms investigated and managed?

1) Investigations - Assess degree of hyperthyroidism (TFT) - Assess for organ failure (RP, LFT,) - R/o Infection (UFEME/C/s, blood C/s, CXR) - R/o Complications (ECG, CXR) 2) Treat as per clinical diagnosis, lab investigations may take too long o Specific sequence (EPPIC) - Esmolol IV (ultra-short acting BBlocker for tachy and 2'AMI) - Propanolol - Propylthiouracil - Iodine (Lugol's iodine) - Corticosteroid (hydrocortisone, also blocks peripheral conversion of T4-T3)

How is bullous pemphigoid and pemphigus treated?

1) Pemphigus - Oral hygiene - Topical corticosteroids for mucosa - Systemic immunomodulaters e.g prednisolone, ciclo, aza, mycophen, mtx, cyclophosphamide, IVIG - Biologics e.g rituximab (anti CD20 ab) - Can use dapsone for foliaceous 2) Bullous pempigoid - Similar to pemphigus - But use POTENT topical steroids for skin

How is hypothyroidism investigated and managed?

1) Investigations - Determine severity of hypothyroidism (TFT) - Assess for organ dysfunction (RP, LFT, CXR, ECG, CBG, cortisol levels) - Assess for complications (ABG, creatinine kinase: rhabdomyolysis in hypothyroid myopathy) - TRO other causes (FBC) 2) Management o Supportive treatment + complications - ABCDs with heating blanket - CBG mx, electrolyte imbalances, pericardial effusion/heart failure) o Hypothyroidism - T3 (faster than T4 with greater biologic activity) o Adrenal insufficiency - Hydrocortison o Treat precipitating causes

What are the objectives in operative management of Traumatic fractures?

1) Irrigation and debridement 2) Foreign body removal 3) Fracture stabilization 4) Soft tissue coverage 5) Limb salvage 6) Prevention and management of complications

What are the indications for an intraoperative cholangiogram?

1) Jaundice related - Clinical scleral icterus, pruritis, yellow skin etc - Hyperbilirubinemia - Raised liver enzymes 2) Pancreatits related - Diagnosis of pancreatits - Raised amylase - Raised lipase 3) Imaging - Dilated common bile duct on imaging

What imaging modalities would you order to suspected osteochondroitis dissecans and what would you expect to see?

1) Knee XR, Tunnel view - Line of demarcation around the lesion - +/- loose body within joint space 2) MRI - Area of low signal intensity in T1 weighted images 3) Radionuclide scans - Increased activity around lesion

What is the use of statins in post AMI patients?

1) LDL lowering 2) Plaque stabilisation

How does acute diverticulitis present?

1) LLQ colicky pain - Relieved by defacation - a/w N&V, Constipation/diarrhoea - +/- urinary sx e.g urgency 2) Fever - Low grade 3) Physical signs - fever + tachycardia - abdo tenderness and palpable mass

What are the investigations for a lower BGIT?

1) Laboratory - FBC - RP - PT/PTT - GXM - ECG 2) Bedside +/- NGT and aspiration (TRO UBGIT) 3) Imaging - Colonoscopy (be sure to do PEG lavage) - Angiography 1) CT mesenteric angiogram (most valuable) - But cannot do embolization bec its from peripheral vein 2)Formal Mesenteric angiogram - only detects if bleeding 0.3 to 0.5ml/second bleeding - Can do selective embolisation of arterial bleeds via ileocolic vessels - If bleeding persistent, go for surgery

When is emergency cholecystectomy indicated?

1) Perforated gall bladder 2) Gangrenous (on radiology) 3) Diabetic patients (rapid decompensation in cholangitis)

What investigation would you conduct for ankylosing spondylitis?

1) Laboratory - Usually not significant - HLA B27 positive - RF negative (hence seroneg spondarthropathy) 2) Injection - SI joint injection with analgesia - Relief of joint pain is highly diagnostic

What are the investigation options for an upper GIT bleed?

1) Laboratory investigations: - FBC (anemia) - Renal panel (electrolyte imbalances) - PT/PTT - GXM - ECG (Exclude type II MI) 2) Imaging - Endoscopy

How is dermatomyositis investigated?

1) Laboratory tests - Serums creatinine kinase: elevated in dermatomyositis/drug induced myositis but normal in steroid induced proximal myopathy 2) EMG - Myopathic changes: spontaneous fibrillation, repetitive potentials, short intervals of low amplitude polyphasic potentials 3) Muscle biospy - Necrosis - Phagocytosis of muscle fibres - Interstitial and perivascular inflammatory infiltrates 4) Imaging: - MRI: Abnormal signal from affected striated muscle - MRI/CT: search for underlying neoplasm if patient is > 40y/o

What do you look out for in log roll? How many people do you need at least?

1) Lacerations 2) Step deformities/gaps 3) Hematomas 4) DRE for anal tone and prostate At least 4 people needed

How are anterior hip dislocations reduced?

1) Lateral traction to disimpact the femoral head from the pelvis/obturator 2) Axial traction to move femoral head into acetabulum 3) Internally rotate the femoral head to correct the external rotation

What are possible complications of fundoplication?

1) Perforation - Most feared, may lead to mediastinitis 2) Excessively tight or loose = dysphagia + gas bloat syndrome/failure of treatment 3) Slippage of stomach

What is the TMN staging of RCC?

1) Limited to kidney - T1 </= 7cm - T2 >7cm 2) Within gerota fascia - T3 - Extending into major veins/perinephric tissue 3) Beyond gerota fascia - T4 - Includes ipsilateral adrenal glands N0 = no LN N1 = LN involvement Stage 1 - T1 to T2N0 Stage 2 - T2N1 Stage 3 - T3 to T3N1 Stage 4 - T4 and above

What investigations should be done before starting anti TB treatment?

1) Liver function - RHZ all cause hepatic toxicitiy 2) Renal function - For drug dosing 3) HIV - TRO cause of TB 4) Color vision - E causes retrobulbar neuritis 5) HbA1c - TRO DM

What are some complications of HIV treatment?

1) Liver, renal toxicity 2) Metabolic disorders - HLD - DM - Osteoporosis - Lipodystrophy 3) Drug allergies/severe reactions

What are the possible presentations of lung cancer?

1) Local - Cough, dyspnea, chest pain, hemoptysis 2) Regional - Pleural/pericardial effusion, chest wall invasion, SVC syndrome, Horner's syndrome, hoarseness, phrenic nerve paralysis, fistula 3) Distant - Liver* - Adrenal, bone, brain 4) Paraneoplastic - Hypercalcemia - SIADH - Cushings - Eaton lambert

What are the presenting signs and symptoms of mycobacterium avium complex?

1) Localised disease - Lymphadenitis (fever, leukocytosis, focal inflammation in a LN) - IRIS in HIV patients 2) Disseminated - Non sepcific (fever, night sweats, abdominal pain, diarrhoea) - Lymphadenopathy - Anemia, elevated ALP and LDH

What must be looked for in staging of renal cell carcinoma?

1) Loco-regional - Renal vein invasion - Metastatic adenopathy - Perinephric invasion - Adjacent organ invasion 2) Distant mets - Lung - Bone

What are the lab tests to diagnose anti-phospholipid syndrome?

1) Lupus anticoagulant 2) Anti-cardiolipin 3) Beta2-glycoprotein

What are the skin manifestations of TB?

1) Lupus vulgaris (shit smeared on skin) - Around body - At BCG sites 2) Verrucosa cutis - Warty appearance w surrounding erythema 3) Miliary TB - Monomorphic, generalised, diffuse red spots 4) Erythema induratum - Nodular growth on LL (prediliction for posterior calf), can be ulcerated.

What are the MEN syndromes and their associated lesions?

1) MEN I (PPP) - Pancreatic tumors - Pituitary adenoma - Parathyroid hyperplasia 2) MEN IIa (PPM) - Parathyroid hyperplasia - Phaeochromocytoma - Medullary TC 3) MEN IIb (PMM) - Pheochromocytoma - Medullary TC - Marfanoid/mucosal neuroma

What are the risk factors for medullary thyroid cancer?

1) MEN2 syndrome MEN2a - PMP (parathyroid, pheo, MTC) MEN2B - MMP (pheo, MTC, Marfan) 2) Familial medullary carcinoma syndrome (occurs 20-30y/o vs 50 y/o in sporadic)

What are the possible descriptive terms of primary skin lesions and their respective size partners?

1) Macules/patches - Flat, erythema - <5mm/>5mm 2) Papule/nodule/tumor/plaques - raised, circumscribed - <5mm/>5mm/>1cm/>1-2cm coalescence of papules/nodules 3) Vesicles/bullae - localised fluid collections - <5mm/>5mm 3) Weals - transient <24hrs, will change around (if longer then need to suspect other causes) - Skin edema - White, compressible, surrounded by flare - AKA urticaria/hives - Can also be erythematous plaques

Please rank the most common causes of large bowel obstructions?

1) Malignancies/neoplasm (60%) 2) DIverticular disease (20%) 3) Colonic volvulus (5%)

How do you treat multiple myeloma?

1) Malignancy - Chemotherapy w bone marrow transplantation 2) 2' fractures - Internal fixation of pathological/impending fractures

What are the radiological features of Psoriatic arthritis?

1) Periostitis: 'fluffy' appearance of bone 2) Small joint destruction 3) Pencil in cup appearance: in arthritis mutilans where bone resorption results in deformity of joints 4) Non-marginal syndesmophytes: in ankylosing spondylotic type PsA

What are the causes of retention of urine?

1) Mechanical Luminal - Congenital valves - Foreign body - Tumor - Clots - Stones - Strictures Mural - Phimosis - Urethral rupture - Urethritis - Meatal ulcer - Tumor - BPH Extramural - Pregnancy (retrograde gravid uterus) - Fibroids - Ovarian cyst - Fecal impaction - Paraphimosis 2) Neurogenic - Post op retention - Spinal cord injury/disease e.g MS, tabes dorsalis - Hysteria - Drugs e.g anti-cholingergics, anti-histamines, smooth-muscle relaxants, tranquilizers

What are the peripheral and cardiac signs of mitral stenosis?

1) Peripheral - Malar flush (sign of PHTN) - Raised JVP (CCF) - Atrial fibrillation 2) Cardiac - Palpable P2 - Tapping apex beat - Opening snap (high pitched ED sound) - MDM murmur at apex - Loud S1 (forceful closure of Mitral valve)

What are the possible causes of acute urinary retention?

1) Mechanical o Extraluminal - Prostate (BPH*, CA, infection) - Pelvic masses (ovarian, fibroid, pregnancy etc) - Pelvic organ prolapse o Intramural - Bladder neck tumor (TCC) - Urethritis (UTI) - Stricture (STD, instrumentation) - Phimosis - Trauma/rupture o Intraluminal - Stones - Blood clot - Foreign body 2) Non-mechanical o Neurogenic - CES, Cord compression - Multiple sclerosis - Tabes dorsalis - DM neuropathy o Drugs - Anticholinergics * (cough medicine, overactive bladder tx) - Antihistamines - Antidepressants - Alcohol o Others - Prolonged immobility - Post-anesthesia - Pain

What are the triggering factors of psoriasis?

1) Mechanical trauma: Koebner's phenomenon 2) Infection: streptococcal pharyngitis (guttate psoriasis formed) 3) Drugs: beta blockers** (can de novo or retrigger), ACEI, ARB, lithium, withdrawal of systemic steroids 4) Endocrine: improvement during pregnancy and worsens after delivery 5) HIV infection

What vessels supply the femoral head?

1) Medial circumflex (majority) 2) Lateral circumflex 3) Inferior gluteal artery 4) Artery to ligamentum teres

What is the terrible triad of elbow dislocations?

1) Medial collateral ligament injury 2) Radial head fracture 3) Coranoid process fracture

What are the treatment modalities for achalasia?

1) Medical - CCB - nitrates 2) Endoscopic - Pneumatic dilation - Botulinum injection - POEM 3) Surgical - Heller's myotomy +/- fundoplication

What are the parts of the renal parenchyme and their associated pathologies in glomerulonephritis?

1) Mesangial cells - Mesangioproliferative GN 2) Endothelial cells - Endocapillary-proliferative GN 3) Basement membrane - Membrano-proliferative GN (immune complexes deposited inside GBM -> more inflammation) - Cresenteric GN (to Bowman's capsule)

What are possible differential diagnoses in appendicitis and what questions would you ask?

1) Mesenteric adenitis: self limiting inflammation of mesenteric LN a/w recent viral infections 2) Meckel's diverticulitis, seen in young 3) Terminal ileitis, commonly seen in Crohn's disease 4) Colitis 5) Psoas abscess/ intraabdominal abscess e.g PID 6) GE, UTI, URTI/LRTI 7) PREGNANCY, do bHCG 8) Appendiceal stump appendicitis: rare complication of appendectomy

What are the signs of mitral regurgitation?

1) Peripheral: - Jerky pulse (due to reduced systolic ejection time in regurgitation of blood to left atrium) - Hepatic pulsations (also seen in tricuspid regurgitation) 2) Central - Pansystolic murmur louder on expiration (VS tricuspid which is louder on inspiration) - Murmur radiates to axilla - Possible apex murmur with radiation to carotids due to regurgitant blood hitting walls of left atrium near aortic root - (Be sure TRO tricuspid regurgitation via accentuation maneuver: Carvallo's sign where inspiration increases the sound of the right-sided Tricuspid regurg)

What is the treatment of inflammatory bowel disease?

1) Pharmacological - 5 aminosalicylic acid: blocks the production of prostaglandins and leukotrienes, good for UC, maybe CD - Corticosteroids: offered in acute severe disease if 5 ASA not effective, can be orally, topically or parentally, mainly used for acute flares, not good for maintanance and comes with side effects - Immunosuppressants: azathioprine (UC and CD), methotrexate (CD), cyclosporin, infliximab (anti-TNAalpha therapy) 2) Surgery Crohns disease: - if perforated, massive haemorrhage, carcinoma, fulminant/unresponsive severe colitis - if obstructed, abscess/fistula, complicated disease - Limited resections/strictureplasty Ulcerative colitis - 20% will require surgery at some time, 30% of those with total colitis will require colectomy within 5 years - toxic megacolon, perforation, haemorrahge, severe non responsive fulminant colitis - chronic symptoms despite medical therapy, carcinoma - Emergency: total colectomy with ileostomy and mucus fistula - Elective: panproctocolectomy with ileostomy/ileal pouch anastomosis OR total colectomy and ileorectal anastomosis

What are the types of tibial bowing and their causes?

1) Posteromedial - Physiological - Treat limb-length discrepancy if >5cm 2) Anteromedial - Defomity, pathological - Knee: missing cruciates, Ankle: ankle valgus, Tarsal coalition, LL discrepancy DDx Congenital pseudoarthrosis of tibia - A/w NF type 1 - Crawford classification (4 grades) o Grade 1: medulla and cortex intact, cortical thinning o Grade 2: medullary canal thinned, cortical thickening o Grade 3: cystic lesion, unstable, high risk of fracture o Grade 4: non-union of tibia and fibula i.e pseudoarthrosis. - Treat with circumferential brace to protect from fracture, before surgical excision and reconstruction via fibular bone graft.

What are the physical findings of patients with scoliosis?

1) Postural - Listing - Limb length discrepancy - Gait abnormalities 2) Neurological deficits - LL - Abdominal reflexes - Sensation of back - CES 3) Special tests - Adams forward bend test (apex of kyphosis more visible laterally) - Scoliometer showing trunk assymetry - Plumbline with deviated spinal balance

What are the preferred pre-meal and post-meal blood sugar levels?

1) Pre-meal: around 6 2) Post-meal (2-3 hours after): around 8

What is would you do for the patient before surgery for colorectal CA?

1) Pre-op investigations - FBC, PT/PTT, GXM - ECG, CXR - As necessary for co-morbidities 2) Bowel prep 3 days before: low residue diet 1 day before: 4L PEG midnight to op: NBM 3) Prophylactic abx - cefuroxime + metronidazole - 30 minutes before induction

What are the phases of acute calcific tendinitis?

1) Precalcific - Fibrocartilaginous metaplasia of tendon - Clinically pain free 2) Calcific o Formative phase - Cell mediated calcific deposits +/- pain o Resting phase - period after formation but before reaction +/- pain o Resorptive phase - Phagocytotic resorption and vascular infiltration - Most painful part 3) Post calcific

What are the contraindications for radiotherapy?

1) Pregnancy 2) Inability to lie flat 3) Previous radiation in similar area 4) Connective tissue disease (RT may trigger onset of disease: self antigen expression, diminished T cell regulatory activity, activation of Toll-like receptors etc)

In patients with assymptomatic bacteruria, when is treatment indicated and why?

1) Pregnant women - Reduces risk of subsequent pyelonephritis - Reduces risk of low birth weight/PROM 2) Patients undergoing surgery 3) Immunocompromised patients

What are the causes of jaundice?

1) Prehepatic Unconjugated: - Haemoglobinopathy (sickle cell disease, thalassemia B and A) - Membrane instability (hereditary spherocytosis) - Enzyme defects (G6PD deficiency, pyruvate kinase deficiency) - Congenital hypoerbilirubinemia (Gilbert's syndrome - common, Criggler-najjar syndrome) Conjugated: - Congenital hyperbilirubinemia (Dubin johnson, Rotor syndrome) 2) Hepatic - Hepatitis (viral, bacterial, drug induced, NASH, alcoholic etc) - Malignancy (1' or 2') - Chronic liver disease - Fatty liver disease 3) Post-hepatic - Intraluminal - Mural - Extraluminal - Stones, malignancy, compression etc

What are the complications associated with use of Pavlik Harness?

1) Pressure sores, skin infections 2) AVN of femoral head 3) Femoral nerve palsy

What are the risk factors of Perthes disease?

1) Previous trauma/transient synovitis/infection 2) Family history 3) low birth weight

What are the most common pathogens for skin infection/pyoderma?

1) Primary - Coagulase +ve staph e.g S.aureaus - B-hemolytic strep e.g S.pyogenes - Also corynebacterium, mycobacterium 2) Secondary - Infection of previously broken/ diseased skin

What are the causes of coxa protrusio?

1) Primary - Familial - OA 2) Secondary (My PROTRUSIO) - Marfans - Psoriatic arthritis - RA - Osteogenesis imperfecta - Trauma - Rickets - Unknown (idiopathic) - Sickle cell anemia - Inflammatory arthritis (AS, infection) - Osteomalacia

What are the causes of hyperparathyroidism?

1) Primary - Parathyroid adenoma - Parathyroid hyperplasia - Parathyroid carcinoma 2) Secondary - Renal insufficiency 3 Tertiary - Chronic secondary hyperparathyroidism

What is the management of septic arthritis?

1) Pharmacological - IV abx (penicillin for strep, cloxacillin for steph, ceftriaxone for gonoccocal infection, use c/s to guide) - Oral only if adequate levels of blood abx, compliance good and no GI complications that impair absorption 2) Surgical - Mainly drainage (percutaneous or open) - For percutaneous, requires course to be < 6days, NO OM< easily accessible joint - Open drainage is indicated in: o Hip and shoulder joints o Peripheral joints not responsive to perc o Systemic illness COMBINE WITH IMMOBILISATION - traction and abduction for hip until infectionresolves - longer period required for unstable joint with spica casting Follow up clinical evaluation for long term complications - AVN, OA, joint instability, deformity 2' growth plate involvement.

What are the treatment options for achalasia?

1) Pharmacological CCB, Nitrates 2) Non surgical Endoscopic submucosal botox injection 3) Surgical Heller's myotomy (open) POEM (Peroral endoscopy myotomy)

What is the management of Grave's disease?

1) Pharmacological: Disease related: o Carbimazole - Dose depends on initial free T4 and size of goitre o NOT PTU because - Risk of liver injury and failure - More for pregnancy (esp 1st trimester) - Risk of thyroid storm (controversial) - More for patients unable to tolerate carbimazole (adverse drug interactions) Symptom related: o Betablockers e.g propanolol 10-40mg BD/TDS for tachycardia

What is the acute treatment of UBGIT?

1) Pharmacological: - IV somatostatin 250ug bolus w 250 ug/hr infusion + IV esmoprazole 80mg bolus + 8mg/hr infusion - IV Ros-flagy for SBP prophylaxis - Prokinetics: erythromycin/ metoclopramide 2) Non-invasive: - Sengstaken-blakemore tube if bleeding uncontrolled after IV somtatostatin - Endoscopy for ulcer hemostasis (clip/band ligation for varices, saline-adrenaline injection/histoacryl glue (glue for varices), heater probe/argon coagulator, hemostatic powder containing thrombin) 3) Surgical: - Angioembolization - Surgery for massive bleed - TIPS for variceal (Transjugular intrahepatic portosystemic shunt - to reduce portosystemic hypertension) - Liver transplant if all cannot (for variceal)

What are the muscles responsible for external rotation of the hip?

1) Piriformis 2) Internal and external obturators 3) Quadratus femoris 4) Superior gemellus

What are the nail changes of psoriasis?

1) Pitting (most common) 2) Onycholysis 3) Subungal hyperkeratosis 4) Discoloration (oil drops) 5) Dystrophy

What is the 2 bag method, why is it important and how much time does it buy?

1) Place amputated parts in plastic bag, tightly secured 2) Place bag into another bag containing ice cubes 3) Aim to keep COLD and DRY Cold ischemic time with muscle 12 hours, while warm is 6

What are the parts of the renal parenchyme and their associated pathologies in nephrotic syndrome?

1) Podocytes - Minimal change disease 2) Basement membrane - Membranous nephropathy (immune complexes deposited outside of GBM -> less inflammation) 3) Indiscriminate - Focal, segemental glomerulosclerosis - Amyloidosis (anyhow deposit)

What patients require antibiotics prophylaxis pre-operatively?

1) Post spleenectomy 2) Immunocompromised - Chemotherapy 3) Heart valve patients 4) Implants - Metallic implants - Mesh in hernia

What are the possible long term complications of hip dislocations?

1) Post traumatic arthritis 2) Femoral head osteonecrosis 3) Sciatic nerve injury 4) Recurrent dislocations 5) Myositis ossificans

What are the causes of polydipsia in patients?

1) Primary polydipsia - Primary increase in water intake - Middle aged women, psychiatric patients, hypothalamic lesions e.g in sarcoidosis - Sensation of dry mouth 2) Central diabetes insipidus - Deficient secretion of ADH production - Idiopathic or autoimmune injury, also trauma, surgery, hypoxia 3) Nephrogenic diabetes insipidus - Normal ADH but renal resistance - Childhood: X linked hereditary nephrogenic DI from mutation of ADH receptor OR AR and AD mutation in aquaporin - Adult: almost always chronic lithium use and hypercalcemia. 4) Osmotic diuresis in uncontrolled DM

What are the pharmacological options for treating osteoporosis?

1) Prophylactic - Calcium supplements - Vitamin D supplements 2) Therapeutic - Bone metabolism o Bisphosphonates (Alendronate, residronate) o Cyclical etidronate o Strontium ranelate - Hormonal o Hormone replacement therapy (only in cases of symptomatic menopause, to be taken with progestogen to reduce endometrial cancer risk) o Selective estrogen receptor modulators (SERM) o Synthetic steroids: Tibolone - Parathyroid o Intermittent parathyroid hormone o Calcitonin (via nasal spray)

How do you treat malaria then?! What prophylaxis?

1) Prophylaxis - Depends on region of travel, what the endemic strains are and their resistance profiles - Chemoprophylaxis + avoidance of mosquito (nets, insecticides, clothing etc) 2) Treatment of disease - Quinine combination therapy o Quinine + doxycycline/tetracycline/clindamycin for 7 days - Artemisinin based combination therapies (ACTs) (for quinine reseistant malaria) o Artemether-lumefantrine o Artesunate-amodiaquine or mefloquine etc (none have been proven superior to the others) - Atovaquone-proguanil 3) Prevention of transmission - Goal of destroying gametocytes - Artemether-lumefantrine OR primaquine (the latter has no effect on blood stage parasites)

What are the possible causes of thrombophillic state?

1) Protein C and S deficiency 2) Factor 5 leiden 3) Anti-phospholipid syndrome 4) Anti-thrombin 3 deficiency syndrome 5) Homocysteinemia

What are some causes of chondrocalcinosis?

1) Pseudogout 2) Gout 3) Hypercalcemia +/- 2' to hyperparathyroidism 4) Wilson's disease 5) Hemochromatosis

What are the causes of clubbing?

1) Pulmonary - lung abscess - bronchiectasis - pulmonary fibrosis - mitotic lesion/mesothelioma - cystic fibrosis 2) GIT - cirrhosis - IBD - coeliac disease - GI lymphoma 3) Cardiac - Cyanotic heart disease - Atrial myxoma - Endocarditis 4) Others/pseudo clubbing - Familial/idiopathic - Thyroid acropachy - Unilateral - Brachial AVM - Axilliary artery aneurysm

What are the stroke syndromes of small vessel (lacunar stroke)?

1) Pure motor 2) Pure sensory 3) Mixed 4) Clumsy hand dysarthria 5) Ataxic hemiparesis (ataxia out of proportion to weakness) 'Does not respect the homunculi'

What are the possible fracture patterns of metacarpals in the hand and how are they managed?

1) Metacarpal Head - Epiphyseal fractures, collateral ligament avulsion fractures, Oblique/horizontal/vertical head fractures, comminuted fractures, Boxer fractures with joint extension (of 5th MC), fractures a/w bone loss - Most require ANATOMICAL fixation with ORIF w K wires/headless compression screws - Fixation at 'protected position' of >70deg metacarpal phalageal flexion to minimize joint stiffness, EARLY RoM is essential 2) Metacarpal Neck - Occurs from direct trauma volar comminution and doral apex angulation - Most can do CLOSED reduction but hard to maintain reduction -> percutaneous pins - Acceptable degree of deformity varies depending on which metacarpal (10 deg for 2nd and 3rd, 30-40deg angulation for 4th and 5th) - if cannot reduce -> ORIF w plates + screws/ intramedullary nail 3) Metacarpal Shaft - Closed reduction if minimally displaced +/- percutaneous pinning - Acceptable deformity is 10deg for 2nd and 3rd metacarpal, 20 deg dorsal angulation for 4th and 5th - Malrotation is UNACCEPTALBLE (10deg is enough to cause overlap) - if closed reduction cannot -> ORIF w plates + screws or intramedullary nail 4) Metacarpal base - Risk of associated CMC fracture dislocations hence need to take AP, lateral and 30 deg pronated lateral view - Reverse Bennett fracture (of the 5th MC base/hamate) where fragment usually displaced proximally due to pull by extensor carpi ulnaris, requiring ORIF 5) Thumb o Extraarticular -> most can be closed reduction and casted o Intraarticular -> most will need CRIF or ORIF - Benentt's fracture (fracture line splits articular surface into 2, with different muscles pulling the fragments differently) - Rolando's fracture -) A comminuted Bennett's fracture at higher velocity - BOTH intraarticular will required CRIF or ORIF

What are the paraneoplastic syndromes associated with pancreatic cancer?

1) Migratory thrombophlebitis (suddenly here, suddenly there) 2) Dermatomyositis - Also common in other GIT manifestations 3) Lesser Trelat - Eruption of multiple seborrhoeic keratoses with inflamamtory bases 4) Hypercalcemia - Moans, bones, groans and stones 5) Carcinoid syndrome - Flushing, diarrhoea, vomiting - Heart failure, dyspnoea

What is the empirical broad spectrum antibiotic for surgical cases?

'I would like to give broad spectrum empirical antibiotics with coverage for gram negatives, main ly a 3rd generation cephalosporin.' I would give 2g of IV ceftriaxone stat after blood culture with 500mg of metronidazole.

What configuration should external fixation be applied in?

'Near-near, far-far rule' where the placement of pins span the fracture site and comprise of 2 pairs of pins, one of which are placed as close to the fracture site as possible without risking contamination and further comminution.

What is the management of acute limb ischemia?

'Super PAP Smears Punish Cervical Adenocarcinoma' 1) Staging - Determine limb viability via Rutherford scoring for acute limb ischemia - Determine level of obstruction via investigations 2) Pre-op investigations - FBC, RP - PT/Ptt, GXM - CXR, ECG 3) Anticoagulation - 5000 IU heparin bolus - 1000 IU/hr infusion 4) Perfusion measures - O2 supplementation - Haemodynamic support, BP maintainance 5) Surgical/definitive treatment 6) Post-op anticoagulation - Heparin - +/- vasodilators 7) Complications (watch for) - Compartment syndrome (+/- prophylactic fasciotomy in severe limb ischemia) - Reperfusion injusry (potassium/vasodilators -> stroke/AMI) - Rhabdomyolysis 8) Associated conditions - Treat underlying cause (AF etc) - Manage CVS risk factors

How do you calculate the adjusted calcium?

(40-albumin) x 0.02 + Total calcium In hypoalbuminemia, total calcium levels do not reflect the free (ionized) calcium.

What are the risks of adhesions?

(Others) Post op complications - Prolonged stay - Injury to bowel - Infection/sepsis

What are the risk factors of oesophageal cancer?

*PV syndrome - Plummer-vinson syndrome: genetic factors/nutritional deficiency resulting in dysphagia, angular stomatitis/glossitis, iron deficiency anemia and oesophageal webs.

What can you not eat during 24hr urine metanphrine collection?

-

What are some pre-malignant lesions that can progress to SCC?

- Actinic keratosis - Keratoacanthoma - Leukoplakia - Bowen's disease (SCC in situ)

What is the pathogenesis of melanoma

- Arises from melanocytes, via stepwise accumulation of mutations. - Radial growth phase and vertical growth phase

What is the pathogenesis of the trousseau's sign and chvosteck sign?

- Calcium is important in maintaining neuronal membrane stability ○ Hence in hypocalcemia, there is nerve hyperexcitability ○ Resulting in spasms on gentle palpation

What are the most common 1' bone tumors in patients over 30 years old?

- Chondrosarcoma - Soft tissue sarcomas

What are the risk factors for inter-trochanteric femoral fractures?

- Gender (M:F ~ 1: 2-8, ?post menopausal changes) - Advancing age - Multiple-comorbids - ADL dependance - Hx of fagility fractures

What are the most common blood borne pathogens in needle stick injuries and the timing for prophylaxis treatment?

- Hepatitis C - Hepatitis B (within 72 hrs) - HIV (within 24 hrs)

What are contraindications for venepuncture?

- Mastectomy side - Edematous areas - Fistula arm - Hematoma - Scarred areas - Burns - Tattoos - IV line side (switch off for 2 minutes at least before taking)

What are the risk factors for plantar faciitis?

- Men (30-60y/o) - High activity (jumpers, runners, ballet dancers, obese individuals) - Co-morbids (DM, enthesopathies like autoimmune arthritis)

What are the side effects of alpha-glucosidase inhibitors?

- Modest glucose lowering effect (0.3-0.5%) - Reduces post-prandial hyperglycemia by 50% - Avoid in renal impairment - Might reduce CV risk Bloating feeling

What is maturation speech delay?

- Most common developmental disorder, aka late-talkers/late-bloomers - Only expression of speech is affected i.e can understand people fine - More common in boys, (+) FmHx Good prognosis, should have normal speech by age of school entry

What are the most common 1' bone tumors in patients > 60 years old?

- Multiple myeloma - Chordoma (in spine) - Secondary ostegenic sarcoma But most common in this age group is METASTATIC Bone cancer

What are the complications of narcotic use post-op?

- N&V - Poor oral intake - Constipation/ileus - Lethargy - Sleepy/drowsy - Central respiratory depression - Potential dependancy

What are the most common 1' bone tumor in patients 5-20 years old?

- Osteogenic sarcoma (10-20 and 50 y/o) Ewing's sarcoma - Chondroblastoma

What is the treatment of variceal bleeding?

- PPI 80mg bolus, 8mg/hr infusion - Somatostatin 250mg bolus, 25mg/hr infusion - Antibiotics - Transfusion + platelet and clotting factor replacement - Go for early endoscopy for hemostasis, if not give sengstaken blakemore - Replace clotting factors etc - Repeat endoscopy again the next day (sengstaken blakemroe cannot > 24 hrs) - If still cannot, TIPPS (transjugular, intrahepatic portosystemic shunt) ○ Aim to and hemostase again - Treat underlying cause/liver transplant

What is the cause of pulp abscesses and how do you treat them?

- Penetrating injuries 1) Operative! - Irrigation and debridement distal to DIP joint crease (prevent contractures) - IV antibiotics (augmentin, Saureas most common)

What are the possible signs of Galeazzi fractures?

- Piano key sign - Tenderness over fracture site - Instability in pronation and supination - DRUJ stress causes wirst of midline forearm pain

What are the nail changes in Psoriasis?

- Pitting of nails (P for psoriasis) - Onycholysis - Subungal hyperkeratosis (keratin deposition under the nail) - Nail discoloration

What is the pathogenesis of BCC?

- Pluripotent epithelial cells in basal layer of epidermis and its appendages - Low metastaic potential (0.0029%), - Most common skin malignancy - 4-5x more common than SCC

Definition of Parkinson's disease

- Progressive neurodegenerative disorder - Associated with degeneration of dopaminergic nigrostriatal neurons It is clinically diagnosed with 2 out of 3 signs, comprising of 1) Resting tremour of 3-5 hertz 2) Bradykinesia 3) Rigidity with the fourth: 4) Postural instability, occuring later in the disease course

What is the terrible triad in elbow dislocation? How would you treat it?

- Radial head fracture (impt in valgus stabiliser, similar to medial ligament) - Coranoid process fracture (prevents posterior displacement) - Posterior dislocation Hence fracture will be unstable and unable to maintain reduction.

Regarding kidney stones, which are radiolucent and which are radioopaque?

- Radioopaque: Calcium phosphate and calcium oxalate - Relatively radiolucent: struvite and cysteine - Radiolucent: uric acid, indinavir

What is Richeter's hernia, sliding hernia, pantaloon hernia?

- Richter's hernia - protrusion of part of the circumference of the bowel wall (often in hernia w very small neck e.g femoral hernia, obturator hernia) - Sliding hernia - part of hernia wall or sac is formed by retroperitoneal structure due to large size e.g large hernia dragging organs in - Pantaloon hernia - concurrent ipsilateral indirect and direct hernias Reduction en masse - when you think youve reduced the hernia but it just moved the whole mass in and is still strangulated

What are the complications of epididymoorchitis?

- Scrotal abscess - Testicular infarction/atrophy - Chronic epididymoorchitis - Infertility

How do you use the incentive spirometry? When is it indicated?

- Sit at edge of bed - Breathe in deep and slow, holding for 5 seconds at end -> aiming for blue region - Do 10 times every hour when awake - When well enough to walk, cough, can stop unless otherwise indicated. Indicated in Types of surgery: 1) Upper abdominal surgery 2) Thoracic surgery Past medical hx 3) COPD hx 4) Atelectasis hx 5) Lung defects 6) Dysfunctional diaphragm

What is Bryant's traction?

- Skin traction used to immobile both lower extremities - Treatment of fracture femur or correction of congenital hip dislocation - Infants and children less than 20 kg

What are the principles of long term care in post-MI patients?

- Smoking cessation - Patient education and counselling - Vaccinations - Cardiac rehabilitation - Pharmacological methods - Surgery if needed

What are the U/S features suspicious of thyroid malignancy?

- Solid - Microcalcifications - Hypoechoic - Irregular margins - Internal vascularity - Absence of halo (normal hypoechoic rim that is absent in local invasion) - Taller than wide (similar to breast)

What thigh foot angle is considered diagnostic of tibial torsion?

-10 degrees or 10 degrees of internal rotation.

What is the temperature of liquid nitrogen

-196 degrees celsius

What are the various stages of diet available to patients?

-NBM -Moist lips -Sips of water -Clear feeds (water, TehO etc) -Full feeds (milk, milo etc) -Soft diet (porridge, blended cake) -Diet of choice

What are the stages of BIRADS in breast?

0 - Non diagnostic (usually due to dense breast tissue) 1 - Normal 2 - Benign 3 - Probably benign 4 - Suspicious 5 - Highly suspicious 6 - Known malignancy

What are the stages of breast cancer?

0 to 4 = 5 stages 0 - Ductal carcinoma insitu (given a stage on its own because mastectomy indicated for an almost 100% cure rate without need for regional/systemic tx), if not it recurs 50% of the time 1 and 2 - Early breast cancer: < 5cm AND No node involvement 3 - locally advanced breast cancer: > 5cm or skin/chest wall involvement AND 2-3 LN affected - Tx via local, regional and systemic 4 - advanced breast cancer: distant mets - Tx via some local/regional but mostly systemic + tx of mets - Bone: RT/fixation/decompression sx + hypocalcemia tx - Brain: RT - Pleural: drain effusion - Liver: liver failure and obstructive jaundice

What is the natural history of HIV without treatment?

0-10 weeks (immune depletion) - 'vague unease, flares of immune response' AKA viral syndrome - Fever - Myalgia - Arthralgia - Adenopathy - Malaise - Rash - Pharyngitis - Diarrhoea - Pancytopenia and LFT abnormalities - +/-Meningoencephalitis 10 weeks to 5 years (early, CD4 > 500) - 'immune system still strong but going haywire, injuring nerves, joints and blood cells' - Neurological: Gullian Barre, Chronic demyelinating neuropathy, Bell's palsy - Haematologic: idiopathic thrombocytopenia - Autoimmune: Reactive arthritis, Polymyositis, Sjogren's syndrome 5 years to 10 years (intermediate, CD4 500-200) - 'no more immune system, giving way to infections, cancer and ADI) - Fungal, viral infections - Kaposi's sarcoma - Malignancies: Cervical CA, lymphoma) - Characteristic infections (Cryptosporidiosis, PCP, toxo, cryptococcal, mycobacterium avium complex, CMV) >10 years (advanced, CD4< 200)

How many mls of insulin actrapid is given in hyperkalemia?

0.1ml of 100IU/ml insulin

How quickly are limbs lengthened in lengthening osteotomies?

0.25mm each time, 4 times a day. Totaling to 1 mm per day.

What is the daily requirement of potassium?

0.5-1mEq/kg

How is patient controlled analgesia set up?

0.5mg of morphine per dose (up to 2.5mg/dose) at a maximum of 8 doses in 1 hr (up to 15 doses, to max of 7.5mg/hr) Lock up set for 5-10 minutes

What is the dose of IV rTPA to be given?

0.9mg/kg over 60 minutes + Initial bolus of 10% total dose over 1 minute UPTO a maximum of 90mg

What is the risk of Recurrent laryngeal nerve injury in thyroidectomy?

1% per side

What are the features and typical history of keratoacanthoma?

1) 2) Grows quickly before regressing

What are the pathogens responsible for gastroenteritis and how many days after exposure do they present?

1) 1-2 days: o Bacterial - Salmonella o Viral - Rotavirus - Astrovirus 2) 1-3 days o Bacterial - Vibrio cholerae o Viral - Norovirus - Sapovirus 3) 2-3 days - Campylobacter 4) 2-4 days - Ecoli 5) 1-4 days - Shigella

How would you treat hypocalcemia?

1) 10mls of 10% calcium gluconate over 10 minutes 2) Oral calcium carbonate 500mg

How is needle thoracostomy done?

1) 16g or larger needle inserted with syringe filled with saline/water to observe for bubbles 2) Placed in the 2nd intercoastal space midclavicular line.

What is the typical follow up after an elective total knee replacement and the objectives for each visit?

1) 2 weeks - For soft tissue evaluation - Healing of scar, resolution of swelling - Functional assessment (should be able to ambulate +/- walking aid/wheelchair (depends on pre-op status as well) 2) 1 month - assessing for return of function, should be back to pre-morbid status 3) Beyond that o Assessment for long term complications - Implant failure - Peri-prosthetic fractures

What are the risk factors of SCC formation?

1) 4 Ss - Smoking - Spirits - Sharp teeth/chronic wounds - Spices 2) Immunocompromised 3) Sun damage 4) Actinic keratosis 5) radiation

What is the definition of scheuermann's kyphosis?

1) > 45deg kyphosis (normal is 20-45) 2) Anterior wedging of >5deg in 3 consecutive vertebrae 3) Not correctable with hyperextension i.e rigid

What are the indications for MRI evaluation of soft tissue tumors? What would you look out for?

1) > 5cm 2) Deep to subcutaneous structures 3) Neurovascular involvement Looking for calcification, depth of invasion

What are the principles of stroke management?

1) ABCDE stabilization 2) Rule out stroke mimics 3) Determine stroke type (hemorrhagic vs ischemic) 4) Evaluate for deficits (Small vessel vs hemispheric vs posterior circulation) 5) Treat as appropriate (DAVID) 6) Assess degree of disability (NIHSS) 7) Investigate for etiology, risk factors 8) Prevent secondary stroke (treat etiologies and manage risk factors) 9) Manage functional complications

What is the ED treatment for a supracondylar femoral fracture?

1) ABCs + rule out life threatening injuries 2) Rule out open fracture 3) Assess neurovascular status 4) Provide analgesia 5) Reduce the fracture and split using thomas splint until definitive treatment 6) Arrange orthopedic consultation

What are the possible investigations for peripheral vascular disease?

1) ABPI (ankle-brachial pressure index or toe-brachial index) 2) Handheld doppler - Triphasic waveforms: forward flow in systole + reverse flow in late systole/early diastole + forward flow in late diastole = NORMAL - Biphasic waveforms (can be normal/abnormal/mild-moderate stenosis) - Monophasic waveforms are abnormal 3) Arterial duplex (aka vascular ultrasound) - Combination of traditional ultrasound to assess artery lumen and doppler for flow 4) DIgital subtraction angiogram (Gold standard) - Invasive but clear - Requires high dose of potentially nephrotoxic contrast 5) Magnetic resonance angiography

What are the causes of angioedema without weals and what investigations/questions would you ask?

1) ACEI use 2) Hereditary angioedema (C1 inhibitor deficiency or C1 inhibitor dysfunction) 3) Acquired C1 inhibitor deficiency (B cell lymphoma or acquired autoimmune disease) Hence 1) Take hx on ACEI use 2) Serum tests for complement C4, C1 inhibitor levels and function C1 inhibitor deficiency -> excessive kallikrein, which in turn produces bradykinin, a potent vasodilato

What factors would prompt you to be suspicious of IRIS?

1) AIDs w low pre-treatment CD4 count (<100cells/uL) 2) Positive virologic and immunologic reaction to ART (increase in CD 4, fall in viral load) 3) Clinical manifestations of inflammatory condition 4) Temporal association of ART initiation and onset of illness 5) Exclusion of drug resistance infection, poor compliance, ineffective drug (drug drug interactions, malabsorption etc)

What radiological investigations would you order and how would you classify an IT fracture>

1) AP and lateral hip XR 2) MRI - imaging of choice to delineate non-displaced or occult fractures that are not apparent on plain radiographs - CT if contraindications to MRI exist IT fractures are classified according to Evan's classification: 5 types Type 1: Undisplaced, 2 part Type 2: Undisplaced, 3 part Type 3: Displaced 3 part, posteroLATERAL comminution Type 4: Displaced 3 part with posteroMEDIAL comminution Type 5: Displaced 4 part with comminution of both trochanters

What radiographs would you order for ankle pathologies and what are the normal values?

1) AP ankle - Distal tibiofibular overlap > 6mm - Equal horizontal and medial joint space ~ 3mm 2) Mortise view (AP w 10deg internal rotation of foot to better assess talocrural jt w/o overlap) - Distal tibiofibular overlap > 1mm - Lateral joint space <3-6mm - Fibular fossa clearly visible 3) Lateral ankle - Fibular superimposing distal tibial's posterior 1/3

What are the Xrays to order in hip pathologies?

1) AP pelvis 2) Lateral hip

What is the pathogenesis of colorectal cancer?

1) APC pathway (adenoma-carcinoma sequence) - 80% of sporadic colorectal carcinomas - Concept that cancers arise from pre-existing adenomas, which can be tubular, tubulovillous and villous (of which has the highest risk of cancer) in morphologically identifiable stages. - Due to chromosomal instability, where stepwise accumulation of mutation in oncogenes and tumour suppressor genes. - APC, DCC, KRAS, P53 where APC codes for protein for breakdown of B-cetenin that activates cell proliferation, KRAS mutation prevents apoptosis and p53 mutation = no DNA repair. - Localised epithelial proliferation to small adenoma to large dysplastic adenoma to carcinoma in-situ to invasive carcinoma. 2) Defects in DNA mismatch repair - 10-15% of sporadic cancers - Due to defects in specific DNA repair genes, cancer arises without morphologically identifiable stages. - MSH 2 and 6, MLH 1, PMS1 and 2. - Hence there is microsatelite (repeated segments of DNA bases) instability that affect coding/promotor regions in cell growth genes - Tumors that arise from this instability have a better prognosis than those of the APC sequence.

What are the possible complications of SCFE?

1) AVN 2) Coxa vara deformity 3) Degenerative changes i.e OA

What are the possible clinical findings of appendicitis?

1) Abdominal pain - in RIF @ Mcburney's point = 1/3 distance from ASIS to umbilicus +/- palpable mass LIF pain = situs inversus or long appendix 2) Peritonism - Localised: Cough sign w pain at RIF - Generalized: guarding, rebound tenderness 3) Classical signs: - Psoas sign (RIF pain on hip flexion) - Obturator sign (RIF pain on hip extension) - Rosving's sign (RIF pain on LIF/non RIF deep palpation) 4) Atypical: - Infants/children: inflamed hemiscrotum from pus migration via processus vaginalis - Right flank pain: retrocaecal appendix - RUQ pain: pregnancy/intraabdominal masses push appendix up - Suprapubic pain: pelvic appendix +/- dysuria and urge to defecate 'Do the back door, or be young and pregnant from the front'

What are the criteria for liver transplant in HCC?

1) Absence of extra hepatic disease/vascular invasion 2) No distant metastasis 3) University of California San Francisco criteria - Single tumor </= 6.5cm - </= 3 tumors of </= 4.5cm with total tumor size </= 8cm 653458

What are the indications of thyroidectomy?

1) Absolute - Suspicion of malignancy - Compressive sx - Prophylaxis for medullary thyroid cancer 2) Relative - Size - Cosmesis - Endocrine control e.g Grave's disease - Hyperfunctioning adenoma - Toxic MNG - Patient request

What are the signs of AC joint dislocation?

1) Ac joint tenderness 2) Ballotable AC joint 3) Step deformity 4) Scarf test positive

What are the components of Gerstmann syndrome?

1) Acalculia 2) Agraphia 3) Left-right disorientation 4) Finger agnosia (Cannot interpret sensations)

What is the treatment of herpes zoster?

1) Aciclovir - 800mg 5 times/day - for 1 week OR 2) Valaciclovir - 1g 3 times/day - for 1 week

What is the follicular occlusion tetrad?

1) Acne conglobata 2) Hiradenitis supprativa 3) Dissecting cellulitis of the scalp 4) Pilonoidal sinus

What are the complications of knee dislocations?

1) Acute - N&V compromise - Compartment syndrome - Associated injuries (bones, ligaments) - Skin necrosis (in dimple sign) 2) Chronic - post-traumatic degeneration - joint instability - recurrent dislocation - reperfusion injury - stiffness

What are the possible complications of IT fractures?

1) Acute - N&V compromise - Compartment syndrome - Hypovolemic shock - DVT, PE - Fat embolism - Osteonecrosis of femoral head 2) Chronic - Malunion - Non union - Loss of fixation (most commonly due to varus collapse of proximal fragment with lag screw cutout ~ 20% in unstable patterns ~ 3/12 after surgery) - Malrotation deformity (due to internal rotation of distal fragment during internal fixation)

What are the differentials for acute limb ischemia?

1) Acute DVT -> phlegmasia cerulean dolens 2) AAA/proximal artery w blue toe syndrome 3) Purple toe syndrome: warfarin therapy 4) Venous: - Insufficiency - Occulsion 5) Acrocyanosis

What are the possible causes of acute limb ischemia?

1) Acute embolism (60-80%) - b/g of AF, prostehtic heart valves 2) Acute thrombolism - b/g of vasculitis (arterial), hypercoagulable state (antiPLA, heparin-induced thrombocytopenia), atherosclerosis, HTN, HLD, DM - usually less severe because time for collaterals to develop in b/g of chronic ischemia 3) Arterial trauma - endovascular procedures - fractures/dislocation 4) Aortic aneurysm w dissection 5) Buerger's disease - distal vessel disease in young smokers - phlebitis w atherosclerotic risk factors

What are the management principles and options for rheumatoid arthritis?

1) Address symptoms - NSAIDs 2) Address disease pathology - Steroids (1st line) - S.DMARDS (methotrexate, hydroxychloroquine) - B.DMARDS (anti TNF, Anti IL1 or 6 etc) (DMARDS are 2nd line) - Involvement of rheumatologist 3) Treat/manage disease complications - Depending on joint involvement and nature of destruction: soft tissue release, tenotomy, decompression, fusion, etc

What are the possible histological types of esophageal cancer?

1) AdenoCA 2) SCC 3) GIST 4) Neuroendocrine tumor 5) MALT 6) Amelanocytic melanoma

What kind of polyps are there?

1) Adenomatous - Villous - Tubular - Tubulovillous 2) Hamartomatous - Peutz jaegher's syndrome - Disorganized, non-dysplastic, native tissue with excessive growth 3) Inflammatory - UC NOT Crohns - C.diff w pseudomembranous colitis 4) Hyperplastic

What are the goals of treatment of spinal metastasis?

1) Adequate pain control 2) Adequate functional independance

What is the anticoagulation bridging protocol?

1) Admit patient 2) Stop warfarin 3) Start heparin immediately - Aim therapeutic dose based on body weight - Aim 60-80s APTT - Q6-8hrly APTT check 4) Stop heparin pre-op - 6 hours prior to operation so that it will wear out by op. 5) Restart heparin post op.

What is Riggler's triad?

1) Aerobillia 2) Small bowel dilation 3) Ectopic gallstone (usually in right iliac fossa) Indication of gallstone ileus from the fistulated biliary tract, obstructing the small bowel.

When is surgery recommended for sigmoid volvulus?

1) After sigmoidoscopic decompression - Conversion of emergency surgery to semi-urgent operation - Done 24-72 hours later - Open instead of laparoscopic since sigmoid easily mobilised out of abdomen (no advantage for laparoscopy) - Primary colorectal anatomosis after or Hartmans 2) Contraindications to sigmoidoscopic decompression - Gangrenous bowel, risk of perforation

What are the risk factors for thyroid carcinoma?

1) Age: <15 or >45y/o 2) Male 3) Previous radiation exposure 4) FmHx - Thyroid cancer - FAP - MEN

What are the indications for bone biopsy?

1) Aggressive lesion 2) History of cancer 3) Unclear diagnosis in symptomatic patient

What are the various hospital transmission precautions and their requirements?

1) Airborne precaution - N95 - Glove and gown if doing invasive procedure 2) Droplet precaution - Surgical mask - Handwash 3) Contact precaution - Glove and gown - Handwash 4) Protective precaution (for immunocompromised patients) - Surgical mask - Glove and gown - Handwash

What are the causes of fatty liver disease? and its complications?

1) Alcoholic steatohepatitis 2) Non- alcoholic steatohepatitis - hyperlipidemia - obesity - insulin resistance and hyperglycemia Complications: - Cirrhosis - Hepatocellular CA

What is the difference between Allergic contact dermatitis and irritant contact dermatitis?

1) Allergic dermatitis = caused by antigenic agent that causes type IV cell mediated/delayed hypersensitivity reaction: hence may have systemic effect and affect surrounding skin (less clear margins) - Affects only sensitized people VS 2) Irritant contact dermatitis: - occurs after single exposure to toxic agent - dose dependant - affects most people but only confined to specific location and margination

What are the causes of circumscribed hair loss? And how do they differ?

1) Alopecia areata - Exclaimation mark hair, narrowing before the bulb 2) Fungal infection e.g tinea capitis? - Loss of proximal end, cut off 3) Trichotillomania - Pulling out of hair - Straight w small bulb at bottom

How would you treat pheochromocytoma?

1) Alpha blocker first - Phenoxybenzamine - Phentolamine - 3 to 4 weeks prior to operation to bring BP down - Patient will be intravascularly depleted, must tell the patient to hydrate and salt 2) Betablocker next - Propanolol - 1 week before surgery - For cardioprotection, as manipulation of pheochromocytoma will overstimulate the heart sympathetics, causing failure

What are the risk factors for aspiration pneumonia?

1) Altered consciousness - Alcoholism - Seizures - Head trauma - GA - Drug overdose 2) Dysphagia - Esophageal disorders (Stricture/neoplasm/diverticula) - Tracheoesophageal fistula - Incompetent cardiac spincter, achalasia - Gastric outlet obstruction 3) Neurological disorder - CVA, MS, PD, MG, Pseudobulbar palsy, ALS 4) Mechanical disruption - Endotracheal intubation, trachoetomsy, upper GIT endoscopy, brochoscopy 5) Others - Protracted vomitting, NGT feeding, pharygneal anesthesia, general debility, recumbent position, ascites, gastroparesis, ileus, glottic infussciency

How might bacteria gain resistance to drugs?

1) Altering antibiotic target protein binding site e.g penicillin binding protein in pneumococcus 2) Upregulation of abx deactivating enzymes e.g erythromycin ribosomal methylase in staphylococci 3) Down regulating or altering outer membrane protein channel used in abx entry e.g OmpF in Ecoli 4) Upregulating transmembrane pumps that expel abx from cell e.g fluoroquinolone efflux in S.aureus)

Why do females get ACL tears at a higher rate?

1) Anatomic - Hypermobility - Ligament laxity - Smaller, shorter ACL 2) Neuromuscular - Lower hamstring to quadriceps ratio - Lower hamstring recruitment 3) Hormonal - Pre-ovulatory phase of menses (hormones affect coordination, hence females on OCPs have less effect)

What are the principles of treament in intraarticular fractures?

1) Anatomical reduction - Reduces degenerative changes - Maintains function 2) Stable internal fixation - Allows early mobilisation - Prevents heterotrophic ossification

Clinical manifestations of Aortic stenosis and their respective prognoses.

1) Angina - 5 years 2) Dyspnoea - 3 years 3) Syncope - 2 years

What are the special tests of the foot and what are they looking out for?

1) Ankle instability - Anterior drawer test (foot in 10deg plantar flex, then pull forward, abnormal if >10mm/>5mm Vs normal side) - Valgus and varus stress tests - Talar tilt (inversion and eversion tests, compare w other foot) 2) Achillies tendon rupture - Simmond's test (patient prone on bed or w leg straight, squeezing the gastrocnemus to observe for plantar flexion -> if absent = rupture of achilies tendon) 3) Flexibility of an existing hind foot varus - Coleman block test: aims to eliminate 1st toe influence via insertion of a 2cm block under the lateral foot. - If varus is reversed -> varus is flexible and 1st metatarsal fixed flexion deformity is the issue - If not -> 1st metatarsal is not the issue ->varus is fixed and subtalar joint is the issue 4) Jacks test - if midfoot is in planus - check if flexible/dynamic or fixed

What are the techniques for closed reduction of shoulder dislocations?

1) Anterior dislocation - Traction-countertraction - Hippocratic - Stimson - Milch - Kocher 2) Posterior dislocation - Traction applied to aDDucted arm in line of deformity + gentle lifting of humerus into glenoid fossa 3) Inferior dislocation - Traction-counter traction

What are the possible causes of shoulder dislocations?

1) Anterior dislocation (96%) - Direct or indirect trauma: o Direct: Anterior-ly directed impact to posterior shoulder, esp w b/g of recurrent instability (minimal trauma can cause) o Indirect: Trauma to aBDuction, extended and externally rotated shoulder 2) Posterior dislocation - Posteriorly directed trauma with arm in ADDuction, flexion and internal rotation - Seizures and electrocution (can also cause anterior) 3) Inferior dislocation - HyperaBDuction force causing impingement of humeral neck on acromion which acts as a fulcrum to lever the humeral head out of G.fossa

What can the anterior tibial, posterior tibal and dorsalis pedis artery be assessed via doppler?

1) Anterior tibial - Anterolateral to tibialis anterior muscle bulk 2) Posterior tibial - Anteromedial to the tibial surface - behind medial malleolus along achilies tendon (1/3 of the line joining the medial malleolus to the heel) 3) Dorsalis pedis is felt between extensor digitorum and hallucis longus tendon (1/3 of the line perpedicular to a line bisecting the lateral and medial malloeli)

What drugs can cause post-op ARU?

1) Anti-cholinergics 2) Analgesics e.g opiods

What is the treatment of leprosy?

1) Antibiotics - 1st line: Dapsone + Rifampicin + Clofazimine 2nd line: Minocycline, olfloxacin, clarithromycin 2) Nerve protection - Thalidomides - Corticosteroids - Long courses needed to prevent neurological deficits 3) Deformities - Surgery for correciton

What drugs can cause photosensitivity?

1) Antibiotics - Tetracyclines - Fluoroquiniolones - Sulfonamides 2) NSAIDs 3) Diuretics - Furosemide - Hydrochlorothiazide 4) OHGAs - Sulfonylureas (glipizide, glyburide) 5) Cardiovascular - Amiodarone - Diltiazem 6) Quinines - Quinine - Quinidine - Hydroxylchloroquine ALOT MORE OMG.

What pharmacological therapy should IMMUNOCOMPROMISED patients with candidemia be managed? What are the possible complications and how should you assess for them?

1) Antifungals - IV anidulafungin or caspofungin given - NOT fluconazole because only C.albicans is S, C. krusei and Glabrata have variable resistance patterns 2) Assessment for complications - Fundoscopy for endophthalmitis (16%) - Echocardiogram for valvular issues

Differentials for Ejection systolic murmur

1) Aortic stenosis/sclerosis 2) Pulmonary stenosis 3) Hypertrophic Obstructive Cardiomyopathy (HOCM) 4) Mitral regurgitation/valve prolapse 5) Coarctation

What are the differentials for a mass in the right iliac fossa?

1) Appendix - Appendix mass - Appendiceal abscess 2) Colon - Carcinoma - Crohn's terminal ileitis - Spigellian hernia (aponeurotic layer between rectus and semilunar line) 3) Pelvic organs - Ovarian cyst/tumor - Fibroid - Tumor in undescended testes 4) Infection/inflammation - Iliac lymphadenopathy - Psoas abscess - Actinomycosis 5) Vascular - Iliac artery aneurysm - Ruptured epigastric artery 6) Bone - Iliac chondrosarcoma or osteosarcoma

What are the compartments of the arm, forearm, thigh and leg?

1) Arm (2): - Anterior - Posterior 2) Forearm (4): - Superficial and deep Volar - Dorsal compartment - Mobile wad 3) Thigh (3): - Anterior - Medial - Posterior 4) Leg (4): - Anterior - Lateral - Superficial and deep posterior (The proximal limbs have fewer compartments while the distal both have 4)

How do patients with proximal humeral fractures present?

1) Arm held closely to the chest by contralateral hand - Shoulder pain - Swelling - Limited RoM by pain - Variable crepitus Usually NO immediate flank ecchymosis, so need to check for thoracic injury if seen. Need TRO N&V - Axillary nerve: lateral forearm (lower), regimental badge (upper), deltoid atony (resulting in inferior translocation of distal humerus)

What kinds of foot ulcers may exist?

1) Arterial 2) Venous 3) Neuropathic 4) Traumatic 5) Infective 6) Malignant

What are the 3 phases of CT Urogram?

1) Arterial phase 2) Venous phase 3) Delayed phase (to look at ureters)

What are the differentials for pyoderma gangrenosum?

1) Arterial/venous ulcers 2) SCC 3) Drug eruptions

What are the surgical options for OA knees?

1) Arthroscopic debridement and washout 2) High tibial osteotomy to correct valgus (for young patients with unicompartmental OA) 3) Unicompartmental replacement 4) Total knee replacement

What are the possible sources of pyogenic hepatic abscess?

1) Ascending cholangitis (duct involvement with multiple microabscesses, 2) Portal vein/hepatic artery e.g IE (need to look for source), 3) Appendicitis, colitis, 4) Tumor (1-3% w abscess have colon CA, hence all need to go for completion colonoscopy), 5) Spread from local e.g cholecystitis, empyema, penetrating injury, fishbone perforation, post surgical etc)

After placement of a colonic stent, what would you do? What are the possible complications of stent placement?

1) Assess effectiveness - Abdominal exam - AXR for decompression 2) Check for immediate complications - CXR/AXR for perforation - I/O charting - Vitals monitoring Risks: 1) Operation related - GA risks - Bleeding, infection, perforation 2) Stent related - Stent failure (technical and clinical) - Infection - Migration 3) Tumor related - Seeding - Overgrowth

What is the initial workup and management for a post-op patient with haemorrhagic shock?

1) Assessment - Vitals - Clinical appearance - Operation site and drain - Limb neurovascular status 2) Initial management - Clamp drain and stop anticoagulants - Resuscitation: supplemental O2 and IV fluids - Investigations: FBC, U/E/Cr, ECG, CBG, PT/PTT, GXM, septic workup if the patient is febrie\ - Inform the doctor if necessary

What are the symptoms of chronic venous insufficiency?

1) Assymptomatic 2) Fluid accumulation - fullness/swelling in leg - heaviness - dull aching discomfort - bursting pain upon standing 3) Stasis - itching - nocturnal leg cramps

What are the possible presentations of peripheral artery disease?

1) Assymptomatic, incidental finding on artery ultrasound 2) Acute limb ischemia 3) Chronic - Claudication pain - Tissue loss (ulcers etc) - Rest pain (acute on chronic pain i.e critical ischemia)

What are the causes of abdominal aortic aneurysm?

1) Atherosclerosis 2) Cystic medial degeneration - Marfan's 3) Trauma/infection - Mycotic

What are the possible mechanism of injury for acromioclavicular joint dislocation?

1) Athletic injuries make majority - Due to direct trauma to the acromion +/- tear of the coracoclavicular ligaments (conoid and trapezoid) Results in longitudinal split of the periosteal sleeve: like a banana from it's peel

What are the types of non-union in bone and their causes?

1) Atrophic - Little to no osteogenic potential e.g in vascular compromise, bone disease) 2) Hypertrophic- Inadequate mechanical stability 3) Oligotrophic (only minimal callus formation seen) - Inadequate apposition of fracture fragments

What are the possible causes of PUO in adults and how would you investigate for each?

1) Autoimmune conditions (22%, MOST COMMON) - Vasculitis (Wegners, takayasu, cryoglobulinemia) - SLE - Polymyalgia rheumatica - Adult still's disease - Giant cell arteritis Investigations: ESR, RF, ANA 2) Infections (16%) - TB - Abscess - OM - Bacterial endocarditis - HIV - EBV Investigations: blood cultures x 3, heterophile antibody test, TST/IGRA, HIV ab-ag and viral load, creatine phosphokinase 3) Malignancies (7%) - Lymphoma (esp NHs) - Leukemia - RCC - HCC (1' and 2') - Multiple myeloma Investigations: CT TAP, serum electrophoresis, serum LDH, LN biopsy 4) Drugs - Drug reaction Investigate with drug charting Dont forget about the normal causes of fever!!

What are the complications of diabetes?

1) Autonomic dysfunction - Postural hypotension - Erectile dysfunction 2) Poor sugar control - Hypoglycemia - Hypoglycemia unawareness (look at book and ask if there are symptoms in episodes of hypoglycemia <4) - Hyperglycemia - DKA - HHS - Microvascular and macrovascular complications 3) Psychosocial - Depression - Burnout 4) Associated conditions - Metabolic syndrome - OSA - HTN - HLD - IHD

Why do patients with DM fall?

1) Autonomic neuropathy - Postural hypotension 2) Peripheral neuropathy - Loss of propioception - Loss of protective sensation 3) Ocular impairment - Cataracts - DM retinopathy 4) Syncope - 2' hypoglycemia

What is the age for Diabetes Mellitus screening to begin and what is the recommended frequency?

1) Average risk (no risk factors) - Above 45 years old - 3 yearly screening 2) High risk (with risk factors) - Above 30 years old - Yearly screening for those with Pre DM - 3 yearly screening for GDM patients

What is the treatment of rosacea?

1) Avoidance of aggravating factors - Cosmetics - Sunlight - Alcohol 2) Mild inflammatory: - Topical metronidazole - Topical azelaic acid - Topical brimonidine (vasoconstrictor that reduces redness) 3) Severe, papulopustular - Add on oral abx: doxycycline, minocycline - Switch to oral metronidazole for resistance cases - Add on oral isotretinoin - Laser treatment 4) Rhinophyma - Surgical reshaping

What is the difference between a Bankart lesion and SLAP lesion? How might they be treated?

1) Bankart lesion - Tear of anterior-inferior glenoid labrum -> repair w labrum repair +/- Bony bankart (glenoid rim also torn) -> Letarjet (moves tip of coracoid process down to inferior glenoid, additional stabilisation by biceps and coracobrachialis) 2) SLAP lesion - a superior tear extends from anterior to posterior - Treatment of SLAP repair - Biceps tenotomy and tenodesis (because biceps tendon attached to superior aspect, then tenodesis to humeral head)

What are some atypical fracture patterns of IT fractures?

1) Basicervical fractures - Anatomically neck fractures but are found outside of capsule hence treated as IT fractures - Higher risk of osteonecrosis than more distal IT fractures - Femoral head more likely to rotate during op (lacking in cancellous interdigitation) thus needs additional antirotation screw/pin during implant insertion 2) Reverse obliquity fractures - Fracture line extends from the medial cortex proximally and to the lateral cortex distally - More likely to have medial pull by aDDuctors causing medial displacement - Treated as sub-trochanteric fractures.

What is the diagnostic criteria of Ankylosing spondylitis?

1) Bilateral sacroilitis 2) HLA B27 positive 3) +/- Uveitis

What are the differences between each type of gastric reconstruction surgery and their limitations?

1) Billroth I - end to end gastroduodenostomy - difficult to mobilise duodenum to anastomose w remaining stomach 2) Billroth II - Mobilise the jejunum to anastomose w stomach - Risk of biliary reflux into stomach 3) Roux-en-Y - Mobilise jejunum to anastomose w stomach/oesophagus + reattach duodenum/jejunum distally along jejunum - 2 anastomoses w higher chance of leak

What malignancies are commonly associated with blastic lesions and which ones are also associated with lytic lesions?

1) Blastic lesions (paired organs) - Kidney - Lungs - Brain - Thyroid - Breast - Prostate 2) Both - Lungs - Prostate - Breasts

What are the possible complications of diverticular disease?

1) Bleeding 2) Infection (diverticulitis) 3) Perforation 4) Intestinal obstruction from diverticular stricture 5) Fistulation

What are the causes of microcytic, hypochromic anemia?

1) Bleeding 2) Thalassemia 3) Iron deficiency 4) Sideroblastic anemia 5) Lead poisoning

What are the posible manifestations of Meckle's diverticulumn?

1) Bleeding (ulceration of ectopic gastric tissue)

What are the requirements of safe bowel anastomosis?

1) Blood supply 2) Healthy tissue 3) Haemodynamic stability 4) Tension free

How do patients with inflammatory bowel disease present?

1) Bloody stools with mucus and pus - Bloating and distention - a/w pain, N&V, diarrhoea 2) Complications - Fistula (colovaginal, enterocutaneous) o Fecaluria/recurrent UTI, skin lesions, PID - Malignancy - PR bleeding

What are the angles of the calcaneum and their normal values?

1) Boehler's angle - Formed from tangential line drawn from anterior process to posterior facet and line drawn from posterior fact to tuberosity. - Normal value is 20-40 deg - Decrease represents the failure of weight bearing by posterior facet causing it to fall forward 2) Gissane's angle - Formed from line along posterior facet and line along anterior process - Normal is 110-135deg - Decrease also represents collapse of weight bearing by the posterior facet.

What are the important parameters in the Orthopedic trauma association classification of proximal and distal long bone fractures?

1) Bone: humerus, femur etc 2) Location: Proximal vs distal 3) Articular: intra-articular, extra-articular 4) Displacement: displaced vs non displaced

What are the 2 types of mallet finger?

1) Bony mallet - A/w avulsion fracture due to ligament - Need to operate (~ 1/3 of articular surface) 2) Non bony mallet - Only tendinous rupture - Finger splinting for 6 weeks and 6 weeks of night splinting - ~ 10 deg loss of extension (similar for surgery)

What types of braces are there for correction of scoliosis?

1) Boston brace - AKA Thoracolumbosacral orthoses 2) Charleston bending brace - AKA night time brace 3) Milwaukee brace - AKA Cervical TLSO

What are the neck stiffness physical signs in bacterial meningitis?

1) Brudzinski's - passive flexion of neck causes flexion of both legs and thighs - patient 'B'ring's legs up 2) Kernig's - with hip flexed to 90 deg, patient cannot fully extend knee - a 'K'nee problem

What are the serological markers of each autoimmune blistering disease?

1) Bullous pemphigoid - BP 180 autoantibody - BP 230 autoantibody -> triggers T cell mediated destruction and inflammation -> rarely affects mucosa/genitals 2) Pemphigus vulgaris - Desmoglein 1 autoantibody (skin blistering) - Desmogelin 3 autoantibody (mucosal blistering) 3) Pemphigus foliaceous - Desmoglein 1 autoantibody (hence no mucosal blistering 4) Paraneoplastic pemphigus - Variable, check for autoantibodies against other cell-cell adhesion proteins - Search for malignancy/lymphoproliferative disorder

What are the standard Xrays in PolyTrauma?

1) C-Spine (requires swimmers view to visualise c7 thats blocked by shoulder, nowadays replaced by CT Cspine) 2) Chest Xray 3) Pelvis

How is HIV disease burden assessed?

1) CD4 testing - <500cells/uL - ADIs occuring at < 200cells/uL 2) Quantitative viral load testing - Plasma conc of HIV RNA - No 'set point' unlike CD4 but useful in measuring treatment progress - Aim 50-200 RNA copies/ml (to keep replication low so resistance does not set in, also the lowest range machines can detect haha) - Resistance risk is high at 500-1000 RNA copies/ml

What are the causes of respiratory alkalosis?

1) CNS Stimulation - Pain - Anxiety - Psychosis - CVA - Infection (menigitis/encephalitis) - Neoplasm - Drugs (e.g progesterone) - Pregnancy 2) Hypoxemia or tissue hypoxia causing dyspnoea - High altitude - Pneumonia - Interstitial fibrosis - Pulmonary embolism - Pulmonary edema - Hypotension - Severe anemia - Aspiration 3) Chest receptors stimulation - Flail chest - Hemothorax

What is the investigation of choice for diagnosis of iatrogenic ureter injury?

1) CT Pyelogram 2) Intravenous urogram if CT not available. Suspected iatrogenic ureteric injury: An approach to diagnostic imaging J.H.BriggsL.WingA.C.MacdonaldC.R.Tapping

How does one diagnose a brain abscess?

1) CT brain - Fast, reasonable sensitivity - Looking for cerebritis (irregular area of low density) > ring enhancing lesion with contrast (abscess) > thin ring pre-contrast (fibrous encapsulation) OR MRI - If patient stable - Higher sensitivity, but difficult to get - Better visualisation of brains stem 2) LP - Contraindicated if patient has neurological deficits, requires CT TRO focal lesion - Send for culture (Gstain, bacterial, fungal, AFB), histology and serology (Toxo IgG and anti-cysterocical ab for tapeworm)

What investigations can be done to stage esophageal CA?

1) CTThorax 2) Transesophageal ultrasound 3) As needed: bronchoscopy, TTE+ECG, lung function test (for single lung ventilation)

How do you grade the severity of pneumonia?

1) CURB 65 score - Confusion - Uremia (>19mg/dL or >7mmol/L) - Respiratory rate >/= 30 - BP SBP<90 or DBP <60 Admit if 2 ICU if 3 or more 2) PSI (Pneumonia severity index)

What are the aspects of goal directed therapy in sepsis as well as their respective ranges?

1) CVP 8-12mmHg 2) MAP >/= 65mmHg 3) Urine output >/= 0.5ml/kg/hr 4) SCvO >/= 70%

What are the causes of aortic stenosis?

1) Calcific stenosis (most common, occuring in old age) 2) Congenital: Bicuspid aortic valve 3) Rheumatic heart disease

What are the dermatological changes in hyperparathyroidism?

1) Calciphylaxis - Obstruction of vessels by calcium - Painful, ischemic ulcers

What is the mechanism of action of carbimazole and propylthiouracil?

1) Carbimazole is a thionamide, Works by forming residues with iodine to prevent iodination of thyroglobulin in formation of thyroid hormone. 2) Propylthiouracil has 2 functions, First of which is similar to carbimazole, second is where it Works by blocking the peripheral conversion of T4 to the more active T3 in peripheral tissues e.g liver.

What are the non-muscular abnormalities associated with myotonic dystrophy?

1) Cardiac - Cardiomyopathy - Conduction abnormalities (heart blocks + severe Q wave abnomalities in precordial leads) 2) Immunological (decrease in serum IgG levels) - Respiratory infections 3) Neurological - Foot drop - Somnolence 4) Ocular - External ophthalmoplegia - Ptosis (bilateral) - Stellate/xmas tree cataracts

What are the contraindications for beta blockers?

1) Cardiovascular o Structural - Cardiomyopathy o Rhythmic - Bradycardia - Heart block o Vascular - Peripheral vascular disease - Raynaud's phenomenon 2) Respiratory - Asthma - COPD 3) Others - DM - Depression

What are the possible modes of action of antibiotics and which groups perform those actions respectively?

1) Cell wall synthesis interferance - Beta lactams - Glycopeptide agents - Interfere w enzymes required for synthesis of peptidoclycan cell wall - Vancomycin and teicoplanin prevent crosslinking by binding to the terminal D alanin residues 2) Protein synthesis disruption - Macrolides - Aminoglycosides - Tetracyclines - Bind to ribosomal subunit (30S) - Chloramphenicol (50S subunit) 3) Nucleic acid synthesis disruption - Fluoroquinolones - Rifampin - Causes lethal double strand DNA breaks) 4) Metabolic pathway disruption - Bactrim - Block folic acid synthesis that is needed for DNA synthesis 5) Bacterial membrane structure disruption - Polymyxin and daptomycin

What are the possible causes of ataxia?

1) Cerebellum - Hereditary: - AD: Spinocerebellar ataxia, episodic ataxias - AR: Frederich's ataxia, Wilson;s disease etc - Maternal inheritance: mitochondrial diseases - Acquired: - Nutritional deficiencies: e.g Vit E, alcoholism, drugs - Infections: Whipple's disease, Cruetz feld Jakob, HIV - Demyelinating disease: MS - Inflammatory: SLE, Sjogren's Behcet's, Sarcoidosis, Vasculitis - Paraneoplastic (a/w LEMS)/neoplastic - Vascular: infarction, haemorrhage 2) Sensory loss (propioception) - Peripheral neuropathy - Miller fischer - Sensory neuronopathy - Chronic immune sensory polyradiculopathy - Myelopathy - Degenerative - B12, copper - Posterior cord syndrome - Tabes dorsalis etc 3) Cerebral/brainstem (impaired postural reflexes) - Parkinsonism syndromes - Thalamic ataxia - Normal pressure hydrocephalus 4) Metabolic - Hypothyroidism: myxedema staggers - Alcohol: truncal ataxia, upper limb and nystagmus less common

What is the diagnostic criteria for HCC?

1) Characteristics CT triphasic findings 2) Risk factors for HCC 3) AFP >400 (to be tested if no risk factors) 4) Do second imaging e.g MRI, hepatic angiography +/- lipoidal/postlipoidal IF both imaging confirmatory, it is HCC until proven otherwise.

What are the treatment of acne scars?

1) Chemical peels - for superficial scars 2) Intralesional steroids - for keloids/hypertrophic scars - triamcinolone acetonide 5 or 10mg/ml 3) Surgery - scar excision/subcision - laser resurfacing w 10600nm

What are the causes of gallstone?

1) Cholesterol (80%) - High cholesterol - Obesity - Cystic fibrosis - Age 2) Pigment stones (20%) - Chronic hemolysis - Infection - Alcoholic cirrhosis - Age - Billiary tract infection - Sickle cell anemia 3) Mixed stones - Consisting of both

What are the predisposing factors for this sigmoid volvulus?

1) Chronic constipation 2) Sigmoid cancer (hence need to scope next day TRO cancer) 3) Congenital (long redundant sigmoid) 4) Adhesion band holding sigmoid up

What are the causes of palmar erythema?

1) Chronic liver disease 2) Hyperestrogenism 3) Hyperthyroidism 4) COPD 5) Rheumatoid arthritis 6) Kawasaki's disease 7) Polycythemia 8) Pregnancy 9) Pseudo: HFMD, ricketsial, rocky mountain spotted fever etc etc

What are the risk factors of trigger finger?

1) Chronic repetitive injury 2) Inflammatory joint disease e.g RA

What are the symptoms of Acute Appendicitis?

1) Classically: - Periumbilical pain radiating to RIF - a/w LOA - a/w N&V AFTER pain (if occuring before pain, exclude I/O) - +/- diarrhoea/ constipation from irritation 2) Possibly extensions of disease: - Bladder irritation -> frequency, urgency, nocturia (FUN: irritative symptoms) and haematuria - Fever: late presentation but common w rupture

What is the classification and management of inter-trochanteric fractures?

1) Classification: Evan's classification (5 grades) - 2 part fracture, undisplaced - 2 part fracture, minimally displaced - 3 part fractures with seperation of greater trochanter - 3 part fractures with seperation of lesser trochanter - 4 part fractures w severe comminution 2) Management - Non surgical: only in patients medically unfit or non-ambulatory patients w cognitive impairment and mild hip pain, splinting via thomas splint/straight leg traction. - Surgical: DHS/PFNA, replacement if cannot.

What are the clinical and histological features of a phyllodes tumor? How would you treat it?

1) Clinically: - Sudden aggressive growth from a b/g stable mass - Firm in consistency - rarely spread outside of breast - 40 to 50 y/o (vs 35 y/o in fibroadenoma) - Younger women tend to have benign 2) Histologically: - Fibroepithelial tumor: epithelial and stromal component - Can be benign (58%), borderline (12%) or malignant (30%) depending on histologic features of stromal cellularity, infiltration and mitotic activity 3) TREAT w WIDE LOCAL EXCISION, chemoRT not effective.

Why is the popliteal artery prone to injury in knee dislocations?

1) Close proximity to the knee joint - Situated deep in the posterior knee 2) Relatively immobile due to tethering - Proximal attachment: due to fibrous tunnel at adductor hiatus - Distal attachment: fibrous tunnel at soleus muscle (where it bifurcates into anterior and posterior tibial artery)

What is the possible blood loss in pelvic fractures?

1) Closed pelvic fracture: 4L 2) Open pelvic fracture: 6L

What are the possible complications of colloid infusion?

1) Coagulopathy - dilutional and impaired platelet aggregation 2) Anaphylaxis - itching, urticaria etc 3) Risk of cerebral edema - limited maximum amount

What are the significant radiological findings in scoliosis and how do they guide management?

1) Cobb angle - lines projected from upper and lower most vetebrae in the curve - > 10deg = scoliosis - > 20 then need to do shit 2) Risser's sign - Extent of ossification and fusion of iliac apophysis, suggests skeletal maturity - 4 grades - 1: 25% - 2: 50% - 3: 75% - 4: 100% - 5: complete fusion Management: 1) Non-operative - Observation: indicated if cobb angle < 20 or 20-40 in risser 3,4,5 (4 monthly f/u w full spine XR for progression) - Brace therapy (if 20-40 and risser 1,2) 2) Operative - Indicated if cobb > 40deg and risser 1,2 OR cobb angle > 50deg and risser 3,4,5 (because progression likely still present) - Aim for 50% correction via surgical reduction and fusion

What is are the differentials for a thyroid mass?

1) Cold Diffusely enlarged - Non toxic multinodular goitre - Endemic goitre 2) Hot Diffusely enlarged - Toxic multinodular goitre - Grave's - Hashiomoto's - Etc Thyroiditis 3) Cold solitary - Thyroid adenoma - Thyroid cyst - Thyroglossal cyst - Thyroid carcinoma - MNG w large nodule 4) Hot solitary nodule - Thyroid adenoma - Thyroid carcinoma - MNG w large nodule

Where are possible manifestations of FAP in the body?

1) Colon 2) GIT, non colon - Stomach - Duodenum 3) Extra-GIT - Skin: epidermoid cysts/lipoma - Bone: osteoma of skull/mandible, dental abnormalities - Eye: Congenital hypertrophy of retinal pigment epithelium - Others: thyroid CA (follicular/papillary) and periampullary CA

What are the differentials for a mass in the left iliac fossa?

1) Colon - Diverticulitis (The rest is in RIF, except appendix not counted hahah) 2) Colon - Carcinoma - Crohn's terminal ileitis - Spigellian hernia (aponeurotic layer between rectus and semilunar line) 3) Pelvic organs - Ovarian cyst/tumor - Fibroid - Tumor in undescended testes 4) Infection/inflammation - Iliac lymphadenopathy - Psoas abscess - Actinomycosis 5) Vascular - Iliac artery aneurysm - Ruptured epigastric artery 6) Bone - Iliac chondrosarcoma or osteosarcoma

What is the management for Chronic diverticulitis?

1) Colonoscopy TRO colon CA 2) Conservative management if uncomplicated: - Fibre, stool softeners 3) Surgery if recurrent/unresolving I/O or pain

What are the components of the screening program recommended for patients with Familial adenomatous polyposis?

1) Colorectal Cancer - Yearly colonoscopy starting at 10-12 years of age 2) Duodenal, periampullary and gastric cancers - 1 to 3 yealy OGD starting at 20-25 years of age. 3) Pancreatic cancer - Consider a yearly transabdominal ultrasound 4) Thyroid cancer - Yearly thyroid examination 5) CNS cancer - Yearly physical examination

What are the cancers HNPCC patients are at risk of and how would you screen for them?

1) Colorectal cancer - Yearly colonscopy starting at 20 years of age 2) Endometrial cancer - 1 to 2 yearly pelvic examination + endometrial sampling + transvaginal ultrasound starting at 20-35 years of age 3) Upper urinary tract cancer - 1 to 2 yearly U/S and urinanlysis starting at 20-35 years of age 4) Gallbladder + biliary system + small bowel + CNS cancer - No screening recommendations

What are the possible secondary lesions?

1) Colour - Hypo/hyper/depigmentation 2) Infective - Pustules - Crusting (dried pus/serum/epithelial debris) - Granuloma 3) Tissue loss - Scaling - Excoriation - Ulcers/erosions (epidermis + dermis, vs epidermis only) - Atrophic scars - Skin atrophy - Fissues 4) Tissue gain - Hypertrophic scars - Lichenification - Induration

By what factors might muscle viability be assessed?

1) Colour 2) Consistency 3) Contractility 4) Bleeding

What are some complications/presentations of hyperthyroidism?

1) Compressive sx 2) Eye manifestations 3) CVS, (AF, CCF, tachycardia, 2' stroke) 4) Thyroid storm 5) Thyrotoxic periodic paralysis (intermittent leg weakness) 6)Agranulocytosis

What is the role of antibiotics in pancreatitis?

1) Concomittant infections - Cholangitis 2) Infected pancreatic necrosis - Gas bubbles seen in pancreas - Endoscopic sampling confirmation of infected collections 3) Going to procedures - ERCP - lap etc

How do you manage urticarial vasculitis?

1) Confirm diagnosis - Skin biospy - Direct immunofluoresence (TRO a/w systemic lupus) - ANA (not ANCA because the vasculitides do not present w urticaria) - C3, C4 (for hypo/normocomplementemic status)

What are the causes of recurrent shoulder instability?

1) Congenital: Connective tissue disease (Marfan's, Ehler Danlos) 2) Acquired: Traumatic vs Atraumatic o Traumatic: TUBS (Traumatic, unilateral dislocations with a Bankart lesion, requiring Surgery) o Atraumatic: AMBRII (atraumatic, multidirectional, bilateral (frequently), treatable with Rehabilitation or Inferior capsular shift) = from microtrauma in overuse or ligamentous laxity

What stability do the coracoclavicular ligaments provide?

1) Conoid - Medially located 2) Trapzoid - Laterally located Both confer superior stability

How would you manage pilon fractures?

1) Conservative - Analgesia - Abx (in open fractures) - Elevate to reduce swelling - Splinting 2) Surgical - Must do because unstable - 1' stabilisation for 2-3 weeks via external fixation (acute setting, contaminated wound) +/- fibular fixation - Definitive surgery after soft tissues optimized: ORIF, exfix, mixed, percutaneous pinning etc 3) Post op - Elevation - PT - Partial weight bear after 6 weeks

How would you manage chondromalacia patellae?

1) Conservative - Analgesia (NSAIDs) - rest, activity modification - PT to strengthen quadriceps muscles 2) Surgical o Articular surface: - Arthroscopic debridement and washout - Articular resurfacing o Deformity correction - Lateral release/patellar realignment - Patellectomy

How do you treat hallux valgus?

1) Conservative - Change in footwear: open toed, wide shoes - For Hallux rigidus: Rocker sole (boat shaped shoe to relieve pain) - No role for PT or Splints 2) Pharmacological - Analgesia for pain - Treat complications e.g Hallux rigidus 3) Surgical - Soft tissue release/tightening and corrective osteotomy - Correction of associated over-riding 2nd toe - Hundreds of operations - For Hallus rigidus cannot osteotomise, need to arthrodesis

What are the management options for familial adenomatous polyposis?

1) Conservative - Close surveillance - Yearly colono from 12 onwards - 5 yearly OGD for periampullary CA - Genetic testing/ education 2) Surgical - Prophylactic proctocolectomy with ileal pouch anal anastomosis (IPAA) at around 20y/o - Subtotal colectomy is rectum relatively spared of polyps

How would you manage tibial shaft fractures?

1) Conservative - Closed reduction with cast immobilisation - Only if closed, low energy fractures with acceptable aligment - Place in long leg cast and convert to functional (patellar tendon bracing) in 4 weeks 2) Operative - External fixation - Internal fixation (IM nail, plates and screws) - Amputation if severe open fractures etc w nonviable limb.

What are the treatment options for limb length discrepancy?

1) Conservative - Elevated footwear (only for < 2.5cm) 2) Pharmagological - Magic beans to help them grow -JK, no pharmacological tx duh 2) Surgical (on the longer leg) - Physeodesis (obliteration of the growth plate) - Femoral shortening

How would you manage osteochondroitis dissecans of the knee?

1) Conservative - Lifestyle modification, activity modification/reduction for 6-12 months - Small lesions usually heal spontaneously - Only in early stage where cartilage intact and lesion is stable 2) Surgical - indicated in unstable fragments - Small, ill fitting fragement: Arthroscopic removal with drilling of defect base (eventually covered with fibrocartilage) - Large > 1cm fragement: insitu fixation with pins/cannulated screws - Also cartilage transplantation - Knee casting for 6 weeks, limited weight bearing until XR shows healing.

How can you manage a femoral shaft fracture?

1) Conservative - Long leg cast - Only for nondisplaced femoral shaft fracutres in patients with multiple medical co-morbids 2) Surgical - ORIF w plates and screws or intramedullary nail - Ex fix first (2-3 weeks) w subsequent conversion to intramedullary nail (for unstable poly trauma patients, vascular injury and severe open fractures)

What is the management of testicular torsion?

1) Conservative - Manual detorsion: done by twisting outward (like opening a book), 30-70% success rate, not usually done unless symptoms > 6hrs and surgery is delayed 2) Surgical - Exploration and orchiopexy

How is neurofibromatosis managed?

1) Conservative - Multi-disciplinary in nature - Genetic counselling (50% of child will have NF1) - Monitoring for complications - Early intervention of developmental delay - Pharmacological treatment of hypertension 2) Surgical - Removal of tumors causing neurological deficits - Orthopedic management of scoliosis and tibial bowing - Corrective/aesthetic surgery for disfiguring deformities - Cardiac surgery for coarctation of aorta - Correction for causes of secondary hypertension

How would you manage tibial torsion?

1) Conservative - Observation and parental education - Most cases resolve by 6 - Bracing, PT, orthotics DO NOT change natural history of condition 2) Surgery - Derotational supramalleolar tibial osteotomy vs proximal osteotomy - Only in children >6-8 y/o w functional problems and thigh foot angle >15degrees - Possible severe case: SERPENTINE foot - Opening wedge and closing wedge osteotomies

How would you manage femoral anteversion?

1) Conservative - Observation and parental ressurance - Most cases usually resolve by 10 - Bracing, inserts, PT, sitting restrictions DO NOT change natural history 2) Surgical - De-rotational femoral osteotomy - Indicated if <10deg external rotation in old child (>8-10y/o)

How would you manage IT fractures?

1) Conservative - Only for patients unfit for surgery e.g multiple co-morbids, unwell - Or patients with no benefit: demented, non-ambulatory patients with mild pain - Early bed to chair mobilisation to avoid cx of immobilisation e.g DVT, pressure sores, atrophy etc 2) Surgical ORIF: o Dynamic hip screw: - Must place within 1cm of subchondral bone to provide secure placement - Must be in the central position in femoral head (with tip-apex distance <25mm) o Cephalomedullary nail (PFNA?) - Can exten to subtrochanteric to treat reverse obliquity fractures as well. REPLACEMENT: - only if ORIF failure and not candidate for repeated internal fixation - Calcar replacement prosthesis EARLY PATIENT MOBILISATION with weight bearing as tolerated.

What are the management options of peptic ulcer disease?

1) Conservative - Patient education on causes, NSAID avoidance - Dietary changes: high fibre food, avoid coffee/caffeine, acidic fruit juices, alcohol, spicy foods - Antacids 2) Pharmacological - PPI (standard dose is 20mg OM) - H2 receptor antagonists e.g famotidine (40mg BD, only promotes ulcer healing if NSAIDS stopped vs PPI) - H.pylori eradication - Triple therapy for 2/52 - Quadruple therapy if not responsive 3) Endoscopic - Haemostatic procedures o Epinephrine injection therapy o Thermal coagulation o Haemostatic powder spray 3) Surgical (repair surgery vs definitive anti ulcer) o Repair surgery (parietal peritoeneum/ jejunum serosa/local flap from stomach/omentum patch repair) o Definitive antiulcer surgery - For Duoedenal ulcer = Duodenotomy + vagotomy w gastric drainage procedures (for elective setting in chronic duodenal ulcers - For Gastric ulcers = gastrectomy (partial if distal ulcer, total if proximal)

How would you manage transient synovitis?

1) Conservative - heat therapy, massage therapy - 7-10 days of bed rest in position of comfort 2) Pharmacological - Analgesia: NSAIDS (naprozen, ibuprofen, check for asthma!!)

How would you manage frozen shoulder?

1) Conservative o Main goals are to: - Provide analgesia - Prevent further stiffening while waiting for thawing phase - Promote early recovery of RoM and strength o Hence measure include - NSAIDs +/- PPI - PT w passive ABduction (towel pulley w/ other hand) or Assisted internal rotation (towel diagonally across back w normal hand pulling from above shoulder) - H&L injection (improve in 3-6 months) 2) Surgical - Only indicated for patients who fail conservative therapy (no progress in 12 months) - MUA (contraindicated in DM and osteoporotics) - Followed immediately by active exercises - Arthroscopic capsular release: division of interval between supraspinatus and infraspinatus - Followed immediately by active exercises - Saline injection for joint distension (distension arthrogram)

What is the management of boxer's fracture?

1) Conservative - Ulnar type splint/ posterior splint 2) Surgical fixation - if malrotation or > 40 degrees angulation

What is the treatment of bleeding diverticular disease?

1) Conservative (80% resolve spontaneously) - Supportive tx - Transfusion if necessary 2) CTMA +/- angioembolization 3) Resection of affected bowel/ - Blind total colectomy: if patient unstable

How would you manage torticollis?

1) Conservative (only if <1 y/o) - Physiotherapy (heat, massage, stretching) 2) Pharmacological - Only for drug induced torticolis - Treat with diphenhydramine, benztropine, BXD, anticholinergics 3) Surgical (if > 1y/o just do this straight) - Surgical SCM release (+/- reconstruction of the sternal head) + traction + custom made cervical collar - Aggressive physiotherapy after surgery

What management options exist to help improve function in spastic cerebral palsy?

1) Conservative management - Early intervention - Physiotherapy, occupational therapy: improve seating, trunk control, upper limb utilisation, walking ability, improve RoM and reduce contractures - Orthotics 2) Pharmacological - Botulinum A toxin for hypertonia (tone will recover in 60 days via sprouting of new nerve terminals) +/- EMG guidance 3) Surgical - Bone: Spine (scoliosis not treatable via bracing: remember critical cobb's angle, unstable/imbalanced and complicated hips, feet: to correct foot alignment) - Soft tissue: for when RoM limited by contractures (tendo-achillies lengthening, hamstring release if crouch gait, adductor release for scissor gait/adducted hips for hygiene)

What are the management options of humeral shaft fractures?

1) Conservative treatment (non-operative) - 90% will heal w/o surgery - Casting/splinting after M&R under anesthesia - Use of Hanging cast (weight of arm-forarm cast with hanging to provide traction for reduction), or Valpeau casting or U slab (like hanging cast but w better support also Functional bracing 2) Surgery - ORIF via anterolateral approach, anterior approach or posterior approach - Plates and screws or intramedullary nail OR - External fixation

How is pre-diabetes managed?

1) Conservative treatment: - Identify and treat cardiovascular risk factors - Diet and activity modifications (low sugar, high exercise) - Closer follow up (yearly FBG screening)

What is the management of diverticular obstruction?

1) Conservative treatment: - NBM - IV fluids - Prokinetics 2) Surgery: If unresolving - Resect affected bowel and 1' anastomosis

What is the management of epididymorrochitis?

1) Conservative/supportive treatment - Analgesia/anti-inflammatory - Rest + scrotal elevation - Cold compress - Management of sexual partners 2) Antibiotics For sexual heteros: - Ciproflox 500 mg bd + doxy 100 mg bd for 2/52 - Levo 500mg od for 10/7 - IM ceft 250mg x1 + doxy 100mg 10/7 - Macrolides For sexual homos/elderly: - Cipro 500mg bd x 2/52 - Zinnat 250mg bd x 2/52 For <17 y/o - Bactrim (co-trimoxazole) - Augmentin (co-amoxiclav) + Outpatient uro referal in 3-7 days Admission if: - Toxic - Complicated by abscess - Failure to respond to abx - Immunocompromised

What is the management of tarsal coalition?

1) Conservative: - Activity modification - Orthosis - Casting/splinting to reduce movement - Analgesia 2) Surgical: - Excision of coalition - +/- soft tissue interposition - if pain intolerable or late presentation: triple arthrodesis (subtalar, calcaneocuboid and talonavicularjoint)

What are the management options for renal trauma?

1) Conservative: - Admission w bedrest for 24 hrs/until haematuria clears (check via urine rack) - Close clinical observation: serial vital signs, FBC - Most blunt trauma and 50% of penetrating done this way 2) Surgical exploration if: - Patient unstable w renal bleeding - expanding/pulsatile renal hematoma - Grade 5 AAST/urine extravasation/penetrating injury/arterial injury

What is the management of facet joint dislocations?

1) Conservative: - Cervical orthosis or external immobilisation for 6-12 weeks - For facet fractures w/o significant subluxation/ dislocation/ kyphosis 2) Surgical - Immediate closed reduction w MRI followed by subsequent surgical stabilisation: o UNLESS MENTAL STATUS CHANGE - Immediated MRI then ORIF

What are the treatment options of renal stones?

1) Conservative: - Dietary advice: o adequate hydration (~ 2.5L/day) o Low animal protein and salt o Reduce oxalate intake NOT low calcium - Wait for stone to pass: o If stone does not pass within 4-6 weeks, likely not gonna pass 2) Pharmacological: - Medical expulsion therapy (for 1/12) - Dissolution therapy 3) Non-invasive - ESWL 4) Surgical - URS +/- LL +/- basket stone removal +/- double stent placement - Retrograde intrarenal surgery - PCNL (if >1.5-2cm) - Open surgery

What is the management of peripheral artery disease?

1) Conservative: - Lifestyle management: smoking cessation! - Exercise therapy of minimum 3x/week for 3/12 to increase claudiation distance 2) Pharmacological: - Aspirin/plavix for anti-coagulation - Statins for hyperlipidemia - Both tgt combined w risk factor modification help reduce need for amputation 3) Definitive - thrombosis: require by pass, if angioplasty fails (vein vs prosthetic) - Embolism: require embolectomy w fogarty catheter - Angioplasty: less invasive compared to by pass, easier to do

What is the treatment of CMC OA?

1) Conservative: - Splinting (thumb spica) - Analgesia (NSAIDS) - Steroid injections - Hyaluronic acid injections 2) Surgical - Corrective osteotomy - Trapizectomy +/- ligament reconstruction and tendon interposition)

Management options of Aortic stenosis and the criteria for surgical therapies.

1) Conservative: education 2) Medical: Antibiotic prophylaxis, Treatment of complications e.g AF and CCF, Statins possible role in reducing calcification 3) Surgical: indicated if symptomatic and severe OR assymptomatic but has area <0.6, LV dysfunction, hypotension on exercise, VT, LVH > 15mm or moderate AS but going in for other cardiac surgeries e.g CABG - Valve replacement - Valvuloplasty (for moribund/palliative patients unable to undergo surgery)

What is the imaging of choice in acute appendicitis + what to look for. What are some alternative imaging options?

1) Contrast CTAP - Greater sensitivity/accuracy and negative predictive value - Looking for: - Fat stranding - Dilated appendix/localised paralytic ileus - Fecolith/tumor/ LN - Appendiceal gas - Phlegmon 2) Abdominal U/S 3) Erect CXR - TRO perforation 4) AXR - fecolith, appendiceal gas, localised paralytic ileus, blurred right psoas, free air

What are the principles of damage control surgery?

1) Control haemorrhage in an unstable patient 2) Quick and short, as fast as possible 3) Laparotomy and packing 4) External fixation of fractures

What are ankle foot orthoses used for?

1) Correction of deformities (flexible ones) 2) Compensation of weakened limbs e.g foot drop 3) Stabilization of joints

What pertinent history questions would you ask about in glomerular disease?

1) Course 2) Cause - Chronic viral infections & high risk behaviour (Hep B/C) - Known hematological conditions (myeloma) - Recent severe infections (e.g pancreatitis, IE causing post-infectious GN) - Extra-renal autoimmune manifestations (rash, joints, eye) 3) Co morbidities - HTN (possible cause and complication) - Pregnancy (inhibits use of ACEI and other drugs) - Abortions (PLA) - GDM (DM) - Proteinuria (nephrotic/nephritc syndrome) - Metabolic syndrome (baseline before giving steroids) 4) Complications 5) Compliance - Previous treatments (may give clue to etiology) - Complications of treatment (cushings disease, neutropenic sepsis)

What conditions predispose a patient to recurrent anal fissue?

1) Crohns 2) TB 3) Anal cancer

How is Meloidosis diagnosed?

1) Culture - Ashdown's agar (contains gentamicin to allow selective growth) or Ashdown's liquid transport broth (contains colistin) - Gold standard - From sputum or swabs of throat, ulcers, skin lesions, rectum etc 2) Microscopy - Gram negative bacilli +/- safety pin appearance 3) Serology is NOT RELIABLE, - High false negative rates (in acute sepsis) and many healthy individuals in endemic areas have high ab levels.

What are the clinical manifestations of dermatomyositis?

1) Cutaneous signs: - Heliotrope rash around eyes (purplish blue) - Gottron's papules (erythema and papules of knuckles with phalangeal sparing vs SLE which is opposite) - Erythematous rash (neck, upper chest, shoulder i.e shawl sign, elbows, knees and malleoli) - Mechanic's hands (thickened/roughened/cracked cuticles and palms) - Raynaud's phenomenon - Photosensitivity 2) Muscle features: - Proximal weakness - Tenderness - Weakness of neck flexors (head drop) - Normal to absent reflexes - CN weakness (dysphagia, dysphonia) 3) Respiratory features: - Interstitial lung disease: fine, late/pan inspiratory crepitations

What are the cutaneous signs of Hep B?

1) Cutaneous vasculitis 2) Cryoglobinemic vasculitis

What are the possible causes of AAA?

1) Cystic medial necrosis - Familial - Marfan's/ehler danlos 2) Atherosclerosis 3) Infective -Syphillis - TB - Mycotic 4) Vasculitis - Takayasu - GCA

What are the symptoms of cystitis vs pyelonphritis?

1) Cystitis - Irritative and obstructive symptoms 2) Pyelonephritis - flank/costovertebral pain and tenderness - Fever and chillls

What are the possible complications of Baker's cysts?

1) DVT - Increased risk - Due to possible lateral deviation of popliteal vein -> compression and venous stasis 2) Arterial compression - Popliteal artery - Dorsalis pedis and tibialis posterior pulses weak 3) Nerve compression - Peroneal nerve - Foot drop

What reversal agents are available for NOACS?

1) Dabigatran - Idarucizumab 2) Rivaroxaban or apixaban - Andexxa

What is the mechanism of injury for a posterior cruciate ligament tear and how would you manage it?

1) Dashboard injuries 2) Non-contact hyperflexion with plantarflexed foot 3) Hyper-extension injury 1) Non-operative - Indicated in grade 1, 2 and 3 - 1 and 2: protected weight bearing and quadriceps rehab 2-4 weeks - 3: immobilisation in extension with quadriceps strengthening 2) Operative - Indicated in multi-ligamentous injuries (ACL, PCL injury), bony avulsion and functionally unstable knee - ORIF of avulsion fracture w reconstruction of ligaments (allograft achillies, hamstrings, anterior tibialis)

How might bacteria gain resistance to antibiotics?

1) De-novo acquisition - Selection pressure from antibiotics - AKA vertical evolution 2) Resistance gene acquisition - Conjugation (Bacterial sex) - Transformation (bacteria taking in new DNA) - Transduction with plasmids (Bacteria obtaining plasmid from phages) - AKA horizontal evolution

A patient presents with acute urinary retention. After inserting the urinary catheter: 1) There is some blood in the urine aspirated; what is your diagnosis and how would you manage? 2) Patient's urine output in the next 2 hours is 300ml/hr; what is your diagnosis and how would you manage?

1) Decompression haematuria: gross haematuria from release of pressure on the bladder wall. - Usually transient and mild - No management needed unless complicated 2) Post-obstructive diuresis - Diagnosed when urine output >200ml/hr for 2 consecutive hours - Replace with IV 0.45%N/S at HALF the volume lost, over next 1 hour

What are the complications of a limb-length discrepancy?

1) Deformity: - Scoliosis - Equinus contracture of ankle - Flexion contracture of knee 2) Pain and degeneration - Knee pain in atheletes - Lower back pain - Degenerative arthritis of hip of longer leg 3) Increased energy expenditure

What are the causes of spondylolisthesis? How would you grade it?

1) Degenerative (forward subluxation due to) - Facet joint degeneration - Facet joint sagittal orientation - Intervertebral disc degeneration - Ligamentous laxity 2) Isthmic/spondylolysis (pars interarticularis defect, which can be due to:) - Congenital dysplasia - Degeneration - Elongation due to multiple healed stress fractures - Acute fractures - Pathological fractures 3) Deformity - Scoliosis: assymetrical degeneration of facet joints and intervertebral discs. Myerding classification (5 grades) - Grade 1: <25% - Grade 2: 25-50% - Grade 3: 50-70% - Grade 4: 75-100% - Grade 5: >100%

How would you classify SJS/TEN?

1) Degree of involvement 10% - SJS 10-30%- SJS/TEN overlap >30%- TEN

What are the most common causes of hyponatremia in surgical patients?

1) Dehydration (Hypovolemic, UNa<20) 2) Diabetes insipidus (Hypovolemic, UNa>20) 3) SIADH (Euvolemic, UNa>20) 4) Drugs (diuretics in patients with CVS morbidities e.g thiazides -> Hypovolemic, UNa >20)

What are the WHO classification of dengue?

1) Dengue without warning signs - Diagnosis can be presumed if o Travel/stay in endemic area WITH fever IF 2 or more of following: o N&V o Rash o Headache, eye pain, myalgia, arthralgia o Leukopenia o Positive tourniquet test 2) Dengue with warning signs - Disease deemed as severe infection if above and any of the following o abdominal pain/tenderness o persistent vomiting o clinical fluid accumulation (ascites, pleural effusion) o mucosal bleeding o lethargy/restlessness o hepatomegaly o increase in hematocrit with concurrent fall in platelet count.

What is the difference between the incubation period of dengue, malaria and paratyphoid?

1) Dengue: 1-3 days 2) Paratyphoid: 1-10 days 3) Malaria: F.V.O Falciparum: 9-14 days Vivax: 12-18 days Ovale: 18-40 days Both vivax and ovale can vary due to latent hypnozoites in liver.

What are the possible causes of hyperpigmentation?

1) Derm conditions - Cafe au lait spots - Freckles (post infectious hyperpigmentation) - Melasma - Drugs (minocycline, amiodarone, tetracycline) 2) Endocrine disorders 3) Hemochromatosis 4) Lifestyle - Sunlight exposure

What is the definition of line-related sepsis

1) Development of blood stream infection 48 hrs of insertion 2) 3x CFU in line culture 3) 3 or more hour earlier growth in line culture than other peripheral culture.

What is the management algorithm of gastric cancer?

1) Diagnose - OGD visualisation and biopsy with histological confirmation 2) Staging - CT TAP for lung/liver mets, bone scan if bony pain reported - TMN staging via - CTTAP and/or - Endoscopic U/S and/or - Staging laparoscopy to look for peritoneal mets 3) Treatment - Can be split into curative or palliative CURATIVE: - Wide resection (at least 6cm margin) + en bloc (all together) removal of LN - Total Vs Partial gastrectomy + reconstruction PALLIATIVE: - Treat cx - Obstruction: endoscopic laser ablation/ total or subtotal gastrectomy/ gastrojejunostomy - Bleeding: Embolisation/ external beam radiotherapy for low level bleeding

What are the investigations to be done for myotonic dystrophy?

1) Diagnose via EMG - 'Dive bomber' effect where waxing and waning of potentials seen 2) ECG: - assess severity of conduction defects/CMP 3) CXR if patient dyspneic - TRO chest infections (most common cause of death) 4) Slit lamp: - TRO stellate/xmas tree cataracts

How would you manage a patient who might have GIST?

1) Diagnosis - EUS guided biopsy with C-kit immunohistochemistry 2) Staging - CTTAP w contrast - Bone scan if symptomatic - PET scan (GIST w high glucose metabolism) 3) Treatment - Laparoscopic resection if symptomatic or >/= 2cm and resectable - Imatinib first line (50% responsive) can be neoadjuvant/adjuvant for resectable and non-resectable tumors) - Sumatinib second line

What are the principles of initial pancreatitis management?

1) Diagnosis (lab, image, clinical hx) 2) Stratify - Clinically (signs of haemorrhagic pancreatitis, patient toxicity etc) - Lab (glasgow, CRP > 150IU/ml is immediately severe, can trend also for recovery)

What are the indications for radioactive iodine scan?

1) Diagnostic 2) Ablative: to ablate residual disease tissue 3) Systemic: to treat metastases

What is the post-scope follow up care of a patient with a previously bleeding ulcer?

1) Diet of choice after 1-2 days: longer if necessary 2) Medications: - Oralise PPI after 3/7 of infusion 3) Monitoring: - Monitor for rebleed via Hb - Monitor for H.pylori eradication via rebiopsy/urea breath test 4) Follow up: - TCU in clnic 3-4/52 - Re-scope 6-weekly - Look for ulcer status - Biopsy TRO CA, H.pylori

What are the modes of spread of gastric cancer?

1) Direct extension 2) Lymphatic spread - regional LN - Supraclavicular LN - Umbilical (sister mary joseph's node, indicative of peritoneal mets, via any of the listed modes) 3) Haematologic spread 4) Peritoneal seeding - to omentum/ parietal peritoneum/ ovaries (aka krukenberg tumor) or to cul de sac/ pouch fo douglas

What would you examine for in a patient with glomerular disease?

1) Disease - Edema, - Volume status - Blood pressure 2) Etiology - Hepatospleenomegaly (hepatitis, hematological conditions) - Vasculitic rash (autoimmune condition) - Acute murmurs (IE, w post infectious GN) 3) Complications - Chest for crepitations (fluid overload/pulmonary renal syndrome)

When is amputation is indicated for limb ischemia?

1) Disease factor - Limb unsalvageable (Rutherford 3, paralysis, complete sensory loss) - 3 Ds - Dying (gangrene) - Dangerous (infection) - Damn nuisance (severe uncontrolled rest pain) 2) Patient factor - Significant co-morbids affecting revascularisation op - High mortality risk from revascularisation

What findings should be seen on achalasia? How would you investigate further?

1) Distal narrowing 2) Proximal dilatation 3) Degree of stenosis 4) Pooling of contrast OGD TRO CA (aka pseudoachalasia) Manometry for 1) Resting tone 2) Failure of relaxation on peristaltic stimulation

What are the nerves that supple the skin of the foot and which areas are they innervating?

1) Dorsum of foot except for 1st web space - Superficial peroneal nerve (L4-S1) 2) 1st web space - Deep peroneal nerve (L4-5) 3) Medial border of foot - Saphenous nerve (L3-4) 4) Lateral border of nerve - Sural nerve (S1-2) 5) Sole - Tibial nerve (S1-2)

Where are 5 possible sites of ganglion cyst formation?

1) Dorsum of wrist 2) Volar wrist 3) DIPJ 4) PIPJ 5) A1 pulley

How does one administer vancomycin?

1) Dosage is initially calculation as 15mg/kg 6hrly 2) Serum vancomycin levels are measured after the 3rd or 4th dose to ensure the trough levels reach 15-20mg/L 3) Ensure regular renal panels TRO renal injury

How do you treat MAC?

1) Dual therapy - Azithromycin/clarithromycin + ethambutol 2) Ongoing ART 3) If failing ART/cannot tolerate ethambutol - Add rifabutin as 3rd agent or replace ethambutol with it 4) If cannot tolerate macrolide - Rifabutin + ethambutol + fluoroquinolone Guided by susceptibility testing

What are the differentials to peri-ampullary tumors?

1) Duodenal CA 2) Ampullar of vater adenoCA 3) Distal cholangiocarcinoma 4) Head of pancreas tumor

What are the full bloods available in the emergency department?

1) E blood (immediate) - for chinese/indian men: O+ bec of low O- incidence - for malay/european women of repoductive age group: O- 2) Rapid blood (5mins) - only matched for big groups e.g A+ etc 3) GXM bloods (15mins) - fully cross matched

What investigations should be ordered in mitral valve regugitation?

1) ECG: - P-mitrale: double humped and broad P waves - AF, LVH - Concomittant AMI from CAD 2) CXR - Pulmonary congestion - Cardiomegaly 3) TTE - Degree of regurgitation via color doppler - EF (low EF of RV is poor prognosis) - Anatomic cause of regurgitation (valvular Vs myocardial) 4) Cardiac catheterisation - TRO CAD

What are the clinical signs of aortic stenosis?

1) ESM heard loudest at right upper sternal edge (aortic region) 2) Pulse: pulsus parvus et tardus (slow rising and slow falling, low volume) 3) Soft second heart sound - poorly mobile and stenotic valve 4) Reversed splitting of S2 (late closure of aortic v relative to pulmonary v due to mechanical or electrical prolongation of systole) 5) Single second heart sound implies fibrosis and fusion of leaflets 5) Normal second heart sound implies mild stenosis

What are the possible complications of Pilon fractures?

1) Early - High number of associated injuries (ipsilateral and bilateral), vertebral compression, tibial plateau, acetabular, pelvic etc - Compartment syndrome - N&V injury 2) Late - 2' OA

What are the complications of fibular fractures?

1) Early - N&V - Skin compromise -> open fracture - 2' infection esp in DM 2) Late - Non union, malunion, delayed union - 2' degenerative changes - Stiffness - Complex regional pain syndrome

What are the complications of shoulder dislocations?

1) Early: - N&V compromise - Associated injuries o Bone: Hill-sachs, glenoid lip fracture, greater tuberosity fracture, acromion/coracoid fracture o Soft tissue: Rotator cuff tear, capsular tendon tears 2) Late: - Recurrent dislocations - Post-traumatic degenerative changes

What are the complications of a supracondylar fracture?

1) Early: - N&V compromise e.g median (most common, AIN), ulnar nerve injury, vascular injury (direct brachial artery) - Complications due to reduction: myositis ossificans (bone formation in muscle 2' traumatic injury), compartment syndrome 2) Late: - Malunion, non-union - Deformity (valgus and varus, varus more common = gunstock deformity, generally 10-20%)

How do you grade the severity of Aortic Stenosis?

1) Echocardiogram findings: - Aortic valve area - Transvulvular mean pressure gradient 2) Presence of LVH (solokov-lyon criteria: S wave depth in V1 + tallest R in V5-6 > 35mm, left strain: ST segment depression w T wave inversion in left sided leads) or 1st degree heart block or LBBB) 3) Presence of complications P.A.C.E (pulmonary hypertension, AF, CCF, endocarditis) 2 and 3 mean severe AS (see other card for features if severe AS)

What topical agents are available to treatment of psoriasis?

1) Emollients - Aqueous cream - White soft paraffin 2) Topical steroids - watch for skin atrophy, adrenal suppression - Use less potent types in flexural regions 3) Keratolytic agents - Salicylic acid 4) Tar - Inhibits mitosis - Might be staining and smelly 5) Calcipotriol - Vit D analogue that inhibits mitosis and induces cell differentiation - May be irritative 6) Anthralin - Inhibits mitosis - Can cause staining and irritation, limit to 30 minutes therapy

What are the aggravating factors of psoriasis?

1) Emotional stress 2) Alcohol 3) Drugs e.g BBlockers, ACEI, indomethacin, lithium and anti-malarials 4) Streptococcal infections (guttate psoriasis) 5) Skin injury (both mechanical and sunburn)

What are the principles of Colorectal surgery?

1) Enbloc resection of lesion 2) Achieving R0 surgery 3) With lymphovascular clearance 4) GI continuity and 5) Adequate nutrition

What is the most commoj primary bone tumor of the hand?

1) Enchondroma - Benign, arising from cartilage

What are the causes of secondary hypertension?

1) Endocrine - Conn's syndrome (1' hyperaldosteronism, most common, nothing to ask, maybe hx/sx of hypoK) - Apparent mineralocorticoid excess (genetic/acquired) - Pheochromocytoma - Cushings - Acromegaly - Hyper/hypothyroid - Hyperparathyroidism - CAH - Liddle syndrome - ?Vitamin D deficiency 2) Renal - Renal artery stenosis (nothing to ask) - Polycystic kidneys (FmHx since AD, of young onset renal failure) - Obstructive uropathy (change in urinary habits) - Glomerulonephritis 3) Vascular - Coarctation of aorta - Subclavian artery stenosis 4) Respiratory - OSA (STOPBANG) 5) Rheumatoid - SLE - Polyarteritis nodosa - Scleroderma 6) Drugs - OCP - NSAIDs - Sympathomimetics - Glucocorticoids - Cyclosporin, tacrolimus - Alcohol/cigarettes/illicit drugs

What are the possible causes of secondary osteoporosis?

1) Endocrine - DM (! and 2) - Estrogen deficiency - Hypogonadism - Hypercortisolism - Hyperthyroidism - Hyperparathyroidism (1' 2' and 3') - Vit D deficiency - Growth hormone deficiency 2) Hematologic - Multiple myeloma - Chronic hemolytic anemia - Systemic mastocytosis (accumulation of mast cells in tissue) 3) Connective tissue disease - Osteogenesis imperfecta - Ehler Danlos - Marfans - Homocysteineuria (high homocysteine levels interfere with collagen cross linking) 4) GIT - Malabsorption - Malnutrition (incl eating deficiencies) - IBD - Cirrhosis 5) Drugs - Alcohol - Glucocorticoids - Cyclosporin - Chemotherapy - HIV medications

What is the ED treatment of minimally displaced, simple fractures of the supracondylar femur?

1) Ensure ABCs, rule out life-threatening injuries 2) Rule out open fracture 3) Assess neurovascular status of patient 4) Provide analgesia 5) Splint and elevate the limb with thompson splint until definitive care 6) Orthopedic consult

What are the laboratory investigations for C.difficile?

1) Enzyme immunoassay for toxins in stool 2) PCR for toxin genes

What are the possible skin infections w relation to their depth?

1) Epidermis - Impetigo (honey-colored crust, non-tender because no nerve endings) 2) + Upper dermis + superficial lymphatics - Erysipelas (may have bullous component) 3) + Dermis - Cellulitis - Ecthyma (deep sores extending into upper dermis, trauma, poor hygiene, bug bites) 4) Hair follicle - Folliculitis (infection to hair follicle w pus at opening, erythema surrouding hair) - Furunculitis (deeper infection involving the pilosebaceous apparatus) - Carbuncle (coalescence of multiple furuncles) 5) Fascia - Faciitis (pictures to be added)

What are the complications of spondylodiscitis?

1) Epidural abscess 2) Neurologcal deficits (1-20%, 50% if 2' epidural abscess) 3) Vertebral collapse/osteomyelitis

Based on location along bone, what are the most common tumors?

1) Epiphysis - Chondroblastoma (because chondroblasts found here) - Giant cell tumor (in adults, e.g PVNS) 2) Metaphysis - Chondrosarcoma (near growth plate mah, plus bone parenchyme) - Osteogenic sarcoma (yeap, bone parehcyme) - Chondroma (along the periosteum) 3) Diaphysis - Ewing's sarcoma (small round blue cells, also arise in flat bones) - Adamantinoma (common in tibia)

What are the principles for investigations of a patient with glomerular disease?

1) Quantify the renal function - Renal panel (be wary as CrCL not accurate in AKI) 2) Quantify the severity of proteinuria - 24 hr urine protein - spot PCR (does not correlate well in acute disease) 3) General laboratory tests - FBC (cell line abnormalities, anemia/Hb before biopsy) - PT/PTT (coagulopathies in lupus, Anti PLA, before biopsy) - GXM (before biopsy haha) 4) Determine the etiology o Autoimmune (circulating) - ANCA - Complement levels - Lupus: anti DsDNA, complements, ANA - Anti PLA2 ab - Paraproteinemia: monoclonal gammopathy - IgA CANNOT be found o Virology - Anti HBS, hepBsAg, anti hep B Core +/- hep B viral load - Anti HCV +/- hep C viral load - Anti HIV o Metabolic - DM (FBG, HbA1c) - Lipids o Structural - U/S kidneys o Histopathological - Biopsy

What is Charcot's triad? What is Raynaud's pentad? What do they mean?

1) RHC pain 2) Jaundice 3) Fever Indicative of cholangitis 4) Hypotension 5) Altered mental status Indicative of hepatobiliary sepsis

What is the management for multinodular goitre?

1) Radioactive iodine 2) Surgery if - Large, cosmetic - Compressive - Refractory to medication - Total thyroidectomy

How would you investigate a patient suspected of spondylodiscitis?

1) Radiography - paraspinous soft tissue swelling (loss of psoas shadow) - disc space narrowing and destruction - end plate erosion 2) CT - can show extent of bone involvement and abscess - can guide biopsy if blood culture -ve 3) MRI - w gadolinium contrast (gold standard) - paraspinal and epidural inflammation - Disc and endplate enhancement w gadolinium - T2 weighted hyperintensity of disc and endplate (rim enhancing) 4) Open biopsy - indicated if tissue/organism diagnosis cannot be made w non-invasive techniques

What is the pathophysiology of the triphasic wave on arterial doppler?

1) Rapid, high amplitude antegrade flow on systole 2) Transient retrograde flow on early diastole 3) Slow antegrade flow on late diastole All 3 together give rise to the triphasic pattern. In distal/superficial circulation, wave may be biphasic i.e okay BUT monophasic is always bad

What can you do if lung cancer recurs?

1) Re-diagnose and re-stage 2) Systemic - Chemo +/- RT 3) Regional - RT/CT 4) Local - Re-resection or RT

What is the management of intracapsular femoral neck fractures?

1) Reduction and fixation - IF NON DISPLACED: Fixation in all age groups regardless of other conditions - IF DISPLACED BUT < 60Y/O W/O RISK FACTORS OF OSTEOPOROSIS - IF DISPLACED W SOME RISK FACTORS OF OSTEOPOROSIS BUT < 40 Y/O - Reduction via hip flexion + gentle traction + external rotation THEN slow internal rotation to achieve reduction - Internal fixation via multiple screw fixation/dynamic hip screw/patello femoral nail antirotation (PFNA) 2) Arthroplasty - IF DISPLACED AND > 60Y/O - Allows faster weight bearing, eliminates nonunion, osteonecrosis, failure of fixation (20-40% ORIF patients require 2' surgery) - But more expensive, extensive surgery w greater blood loss - Bipolar vs unipolar (no added benefit, over time bipolar may loss articulation to effectively become unipolar and unipolar is cheaper) - Cemented vs non-cemented (cement has lower incidence of intra-operative fracture and less thigh pain but may pressurize resulting in hypotension and death) - Total hip replacement (for independantly mobile, no cognitive impairment and no major PMHx)

What is the purpose of traction in orthopedics?

1) Regain normal length and alignment of involved bone 2) Reduction and immobilization of fractured bone 3) Relief or elimination of muscular spasms 4) Relief of pressure on neurovascular structures 5) Prevent or reduce skeletal deformities or muscle contractures Acute and chronic.

What are the objectives of management in rheumatoid arthritis?

1) Relief of acute synovitis - Analgesia - Anti inflammatory 2) Prevention of recurrent flares/progression - DMARDS - Patient education and counselling 3) Maintainance of function and prevention/management of disability - PT/OT - follow up for associated functional impairments 4) Management of associated co-morbidities - Treatment associated complications - Extra articular involvement - Prevention of subsequent disease

In cases of suspected nerve injury post op, what can be done immediately?

1) Remove all dressings 2) Positioning to relieve nerve tension e.g flexing knee in case of peroneal nerve palsy.

How would you instruct the patient in use of inhalers? What are the steps?

1) Remove cap and shake 2) Breathe out as hard as you can 3) Close lips tightly around the mouthpiece of inhaler 4) Breath in slowly and deeply while pressing the top of the canister once 5) Hold your breath for about 10 seconds then breath out slowly 6) If taking > 1 puff, WAIT 1 MINUTE before the 2nd puff and repeat the above steps 7) After finishing, wipe the mouthpiece and replace the cap.

What are the systemic diseases that cause pruritis?

1) Renal - Chronic renal failure 2) Endocrine - Hypo/hyperthyroidism - Hyperparathyroidism 3) Haematological - Polycythemia rubra vera - Iron deficiency - Iron excess - Other forms of anemia 4) Infective - HIV (infection itself + dermatological lesions) 5) Neurological - Brain (strokes, tumors, brain abscesses, multiple sclerosis -> localised/generalised pruritis) - Spinal cord (localised, dermatomal pruritis 2' to tumors, trauma, syringomyelia, spinal MS) - Nerve roots (radiculopathies) 6) Neoplasm (paraneoplastic) - Lymphomas - Lung, gastric, laryngeal CA 7) HEPATIC hahaha - 2' cholestasis - w/o obstruction also can

What are the causes of gross hematuria?

1) Renal - Congenital: polycystic kidney - Trauma: rupture, stone - Inflammation: TB - Neoplasm: Renal CA, renal pelvis CA - Coagulopathies - Vascular: right heart failure, renal artery stenosis, arterial emboli (AMI, IE) 2) Ureter - Stone - Neoplasm 3) Bladder - Stone - Neoplasm - Infection: cystitis, TB, schiostosomiasis 4) Prostate - BPH - CA 5) Urethral - Traumatic: stone, rupture - Inflammatory: urethritis - Neoplastic: TCC

What are the contraindications for potassium replacement?

1) Renal failure 2) Drugs - Potassium sparing diuretics - ACEI (RAAS impairment) 3) Metabolic acidosis - Should aim for alkalinising forms of K+ e.g potassium bicarbonate/acetate/citrate/gluconate

What are the contraindications of metformin?

1) Renal failure 30-50 eGFR = limit to 1g/day <30 = avoid 2) Contrasted scan 3) B12 deficiency

What to look out for on upper limbs in Parkinsonism and how they look like (6)

1) Resting tremors 2) Bradykinesia (slowness of movement, test via finger-to-thumb test, wrist rotation and twinkle stars) 3) Lead pipe rigidity and cogwheeling 4) Acute dystonia (abnormal twisting posture) and alien hand syndrome (inability to control movement of hand: ask patient to attempt to comb hair) 5) Pronator drift and cerebellar signs 6) Palmomental reflex (stroking of thenar eminence results in chin twitch, indicative of contralateral frontal lobe damage), grasp reflex

What are the components of damage control surgery in Trauma?

1) Resuscitation - Permission hypotension - Early transfusion 2) Trauma Laparotomy - Hemostasis - Decontamination - Temporary closure w ICU stabilisation 3) Definitive surgery

What are the options for treatment of chronic limb ischemia?

1) Revascularise first if critical - Investigate via angiography etc - Consider angioplasty/stent/bypass surgery - Remove gangrenous part before revascularisation 2) Conservative treatment - SMOKING CESSATION - Patient education on critical limb ischemia, lifestyle modification - Claudication diary - Referral to podiatrist - Manage CVS risk factors + refer cardiologist 3) Pharmacological treatment - Statins (check LFT) - Aspirin/clopidogrel

What are the causes of mitral stenosis?

1) Rheumatic heart disease 2) Congenital biscupid valve 3) Left atrial myxoma 4) Connective tissue disease

What are the possible risk factors and presenting signs of infectious endocarditis?

1) Risk factors - Old age > 60 - Male - IVDA - Poor dentition/dental infection - Co-morbid heart condition, heart device - Previous hx (IE, Hemodialysis, HIV) 2) Presenting complaints o Systemic: - FEVER (90%) - malaise, headache, arthralgias, night sweats, abdopain, etc) o Visceral - Cardiac: murmurs - GIT: splenomegaly - Peripheral: splinter haemorrhages, petechiae, janeway, oslers, roth's - Dental: tooth ache o Complications - Cardiac: valvular insufficiency, heart failure - Neuro: embolic stroke, intracerebral haemorrhage, brain abscess others - Septic emboli: kidney, spleen, other visceral infarction - Metastatic infection e.g vertebral osteomyelitis, septic arthritis, psoas abscess - SIRS

What vaccinations should individuals going to Africa (Ethiopia) obtain?

1) Routine Vaccines (Measles, DTaP, Influenza, MMR, Polio) 2) Highly Recommended: Hep A, typhoid 3) Recommended: Cholera, Hep B, Meningococcus, Rabies, YELLOW FEVER

What vaccinations should individuals going to Vietnam/south east Asia obtain?

1) Routine vaccines (MMR, DTaP, VZV, Polio, Influenza) 2) Highly recommended: Hep A, Typhoid 3) Recommended: Japanese encephalitis (if staying > 1/12), Hep B, Malaria, Rabies NOT yellow fever (no cases in vietnam and only required if coming from country with risk)

How would you work up a patient with fat embolism? How would you manage the patient?

1) Rule out differentials - Mainly ARDS, CXR will show nothing if fat embolism but severe diffuse edema in ARDS 2) Determine complications - mostly spread to end organs - ABG, CT brain 3) Supportive treatment - Transfer to ICU - High flow oxygen, intubation and mechanical ventilation if necessary - Aggressive fluid replacement - Splint/fix the fracture

What are the associated complications of Plantar faciitis?

1) Rupture (esp w hx of steroid injection) 2) Deformity: - Flattening on longitudinal arch - Heel spur formation (not cause of pain) 3) Loss of sensation - Heel hypoesthesia

What are the organisms responsible for spondylodicitis?

1) S.aureaus 2) TB 3) Salmonella 4) Gram negs 5) PAE

What are the possible pathogens responsible for infectious endocarditis?

1) S.aureaus (31%) 2) Strep viridans (17%) 3) Enterococci (11%) 4) CNS staphylococci (11%) 5) Streptococcous bovis (7%) 6) Other streptococci (5%) 7) HACEK (2%) - Haemophillus aphrophilus, Actinobacillus, Cardiobacterum hominis, Ekinella corrodens, Kingella Kingae) 8) Non HACEK gram negs (2%) 9) Fungi (2%) 'The Original Version Entered the CNS, only to see the Cow and it's Udders Laughing (HA) at the Fun guy'

What are the available colours of inhalers and what medication do they contain?

1) SABA inhalers - Salbutamol (ventolin) BLUE - Ipratropium bromide (atrovent) WHITE + GREEN cap - Fenoterol + Ipratropium bromide (berodual) WHITE 2) Corticosteroids - Fluticasone (flixotide) - Ciclesonide (alvesco) - Beclomethasone (becloasma) ALL ORANGE - Budesonide (pulmicort) WHITE + GREEN twisty 3) Corticosteroids + LABA - Fluticasone + salmeterol (seretide) PURPLE - Formoterol + budesonide (turbuhaler) WHITE + ORANGE twisty 4) LAMAs - Tiotropium (spiriva) - Glycopyrronium (seebri) BOTH WHITE twisties 5) LABAs omg who gives a shit why do we need to remember this - Salmeterol (serevent) GREEN + BLUE cap - Formorterol (oxis or foradil) WHITE + TURQUOISE twisty and BLUE

After 6 hours since symptom onset, what else can you do for stroke patients?

1) SAPT (i know the slides say DAPT but he changed on the spot) - Aspirin 300mg bolus, then 100mg OM FOR LIFE - Also given after treatment if they present <4.5 or 4.5-6hrs (NO aspirin for 24 hrs s/p rTPA) 2) Secondary stroke prevention (Depends on etiology) o Cardioembolic stroke - Warfarin - NOACs for non-valvular AF o Intracranial/lacunar - Long term SAPT o Extracranial (carotids) - Antiplatelets - Endartrectomy if stenosis > 70% - Carotid angioplasty/stenting if not suitable for endcartrectomy. 3) Risk factor control o CVS risk factors - Smoking cessation - Lifestyle modification - Stroke rehabilitation - Driving - Vocational/occupational rehab

What are the physical signs of sternoclavicular dislocation and what radiographic view would you order?

1) SC joint - Tenderness - Ballotability - Prominent on arm abduction and shoulder elevation - Improvement with head rotation to ipsilateral side 2) Serendipity view hahaha - beam at 40deg cephalic tilt

What is the diagnostic pathway of osteoporosis?

1) Screening and risk factors FOR PEOPLE WITH: - Hx of fragility/atypical fractures - Radiological evidence of osteopenia/ vertebral deformity - Peri/post menopausal women (according to OSTA) - Women with osteoporosis (yearly DEXA) - Women with osteopenia (2 yearly DEXA) 2) Investigations - For diagnosis (BMD test) o DEXA scan (T score < -2.5SD) o Xrays o Grade according to Singh's index - For secondary causes o Calcium, phosphate (normal in osteoporosis) o FBC, ESR, UECr, Vit D o Others if associated disease suspected (TFT, LFT, tumor markers/myeloma screen e.g bone marrow aspiration etc) - For complications i.e fractures o XR 3) Fracture risk assessment - FRAX score o 10 year probability of hip fracture/major osteoporotic fracture (spine, forearm, hip/shoulder) 4) Treatment - To start if T score </= -.25SD and secondary causes excluded - T score -1 to -2.5 + FRAX score 10yr hip >/=3% or major fracture >/= 20%

What are the tests for syphilis?

1) Screening: - RPR or VDRL - Can be false positive 2) Diagnostic - TPPA (treponema pallidum particle agglutination) - EIA if fail - Alternative tests: syphilis LIA IgM and IgG (line immunoassay)

What do the inguinal lymph nodes drain?

1) Scrotum/vagina 2) Lower limbs (nails, webspaces etc subungal melanoma) 3) Urethra 4) Perineum 5) Rectum/anal canal (can spread to inguinal LN) 6) Check contralateral side for lymphadenopathy 7) Check systemic (neck, axillary) 8) Check for hepato-spleenomegaly TRO lymphoproliferative disease Looking for ulcers

What are the complications of axillary clearance?

1) Sensory loss 31% 2) Lymphoedema 13% 3) Shoulder stiffness

What laboratory tests are useful in PCP?

1) Serum - LDH in HIV patients (not as useful in immunocompetent patients because can be due to malignancy) - Beta D Glycan (cell wall component of fungi, non specific but can lend weight) 2) Microbiology - Induced sputum - Brochoalveolar lavage - tinctorial staining, fluorescent ab staining or PCR

How does fournier's gangrene typically present? How do you treat it?

1) Severe pain STARTING at anterior abdominal wall -> Migrates to gluteal muscles, scrotum and penis 2) Tense endema with blisters/bullae, crepitus and subcutaneous gas - Systemic findings of fever, tachycardia, hypotension Treatment: 1) IV augmentin OR 2) IV cefazolin + PO metronidazole

What are the indications for surgical fixation of rib fractures?

1) Severe pain due to movement/non union - Causing respiratory failure 2) Significant chest wall deformity 3) Failure to wean from mechanical ventialtion - Non pulmonary contusion 4) Chest wall instability - Flail chest 5) Visceral damage - Pulmonary laceration/perforation

What are the lines of the hip in skeletally mature individuals?

1) Shenton's line 2) Inter teardrop line (horizontal line tangential to teardrop structure that is formed from ischium and superior pubic ramus) 3) Acetabular angle of sharp (line drawn from tip of tearbrop, to lateral edge of acetabulum, normal is 33-38) VS acetabular index, where > 30 is DDH)

How may patients present with in Frozen Shoulder?

1) Shoulder Sx - Gradual increase in pain, affecting posture and sleep - Subsides after several months, followed by increasing stiffness over 6-12/12 2) Sx of different stages o Stage 1: Freezing (Month 2 to 9) - Pain increasing with movement - Worse at night - Increasing stiffness - Surgical release contraindicated due to recurrence rate from ongoing and 2'inflammation o Stage 2: Frozen (Month 4 to 12) - Stiffness is maximal, esp in external rotation - Pain has decreased - Surgical release may be indicated in this period - MUA is contraindicated due to risk of labrum tears/fracutures/cuff tears o Stage 3: Thawing (Month 5 to 24) - Gradual return in range of motion - MUA and physio good for this time period.

What would a patient with ankylosing spondylitis present with in terms of signs?

1) Signs o Articular signs - Bilateral sacroilitis - Progressive spinal kyphosis - Cervical spine fractures - Large joint arthritis (hip and shoulder) o Extra articular signs - Red eye (acute anterior uveitis or iritis) - Heart rhythm abnormalities - Ascending aorta conditions (aortitis, stenosis, regurgitation) - Pulmonary fibrosis - Renal amyloidosis

How is an FOBT done?

1) Sit forward slightly on the toilet bowl facing the posterior wall such the faeces lands on the sloping part +/- toilet paper to prevent it from sloping 2) Using the dipstick, sample 6 different areas of the stool before re-inserting it into the tube. 3) Wash hands and set to Singapore cancer society 4) At least 2 a year, if 1 positive, need to come in for scope.

What are the dermatological changes in hyperthyroid patients?

1) Skin - Flushing (face, neck, palmar) - Diffuse hyperpigmentation - More on the eyelids (Jellinek's sign) - Pre-tibial myxedema (plaque, nodular, elephantisic) - Hyperhidrosis 2) Nails - Thin, brittle, w onycholysis starting in ring finger (Plummer's nails) - Thyroid acropachy 3) Hair - Dry, coarse - May have diffuse aloepecia - Loss of lateral third of eyebrows

What are the differentials of posterior knee joint swellings and the pertinent features to distinguish them?

1) Skin - Lipoma - Sebaceous cyst 2) Muscle - Rhabdomyosarcoma etc 3) Nerves - Neuroma - Neurofibroma - Neurilemmoma 4) Vessels o Popliteal artery aneurysm o Saphena varix of saphenopoliteal junction o DVT 5) Bursa o Semimembranosus bursa - enlargement of bursa between semimembranosus and medial head of gastronemius - found ABOVE joint line, medially. 6) Joint o Baker's cyst - from joint synovium, found BELOW joint line and deep to gastrocnemus - a/w degenerative knee joint

What investigations would you do for vasculitis and what features would make you suspect lupus?

1) Skin biopsy - To look as size of vessels involved, predominant cellular infiltrate - +/-Direct immunofluoresence 2) Laboratory tests - FBC (eosinophils) - Renal panel, urine analysis, creatinine - IgA (for HSP) - ANA, C3, C4 - ANCA (ddx between ANCA+ve and -ve) - CXR, ECG (for granulomatous polyangitis and esosinophillic granulomatous polyangitis) - Hep B, C, cryoglobulin (if suspecting cutaneous small vessel vasculitis) - Throat swab and anti-streptolysin O test (if suspecting HSP and has sore throat)

How would you treat rosacea?

1) Skin lesion - Topical: metronidazole cream +/- erythromycin abx - Systemic: oral abx (doxycycline 50-100mg BD or metronidazole: 500mg BD), oral BBlocker carvedilol may reduce erythema and telangiectasia - Oral isotretinoin (0.5mg/kg low dose) if refractory 2) Complications - Rhinopyma - Telangiectasia - Treated with surgery or laser

What are the differences between small and medium vessel vasculities?

1) Small - petichiae->palpable purpura->vesicobullous-> superficial erosion 2) Medium - plaques/tender nodules->deep ulcerations - dusky appearance w livedoracemosa - +/- digital gangrene

What type of lung cancer types are there?

1) Small cell 2) Non small cell - Epithelial cells - AdenoCA + adenoCA in situ - SCC - Large cell - Mixed - Minimally invasive carcinoma 3) Carcinoid - usually resected because not responsive to chemo

What are the associated features of congenital talipies equinovarus?

1) Small feet and calves 2) Shortened tibia 3) Medial and posterior foot creases.

What are the etiologies of lung cancer?

1) Smoking 2) Dietary 3) Oncogenes etc?

What is the STOPBANG questionnaire for OSA?

1) Snoring 2) Tiredness 3) Observed apnoea 4) Pressure (tx or diagnosis) 5) BMI >35 6) Age > 50 7) Neck circumference > 40 8) Gender if male If score > 3, refer for sleep study

What are the features of a breast cyst?

1) Solitary nodule w smooth regular border, firm and well circumscribed, mobile,TENDER, soft to firm 2) No change w menstruation 3) +/- Serous nipple discharge

What are the features of a breast fibroadenoma?

1) Solitary nodule w smooth regular border, firm and well circumscribed, very mobile, NON TENDER, rubbery 2) No change w menstruation

What are the types of cerebral palsy?

1) Spastic (the only one ortho can help in) - Hemiplegic (one side), diplegic (both legs), quadriplegic, total body, triplegic, monoplegic. 2) Ataxic 3) Athetoid (whole body w/ involuntary contraction of muscle groups) 4) Hypotonic

What are the signs and functional aims for spastic hemiplegia, diplegia and quadriplegia?

1) Spastic hemiplegia - Circumductive gait, early handedness (prefers functional side) - Efficient walking 2) Spastic diplegia - Scissor gait (legs cross due to adductor dominance when being carried), commando crawl (upper limb works but legs dragged behind) - if they CAN WALK: efficient walking (no crouch/jump gait, no tiptoeing, feet in neutral position w/o scissoring) - if they CANNOT WALK: treat as quadriplegic 3) Spastic quadriplegia: - All limbs affected, also with high seizures, cognitive impairment and swallowing impairment. - Aim for painless seating, with spine straight and stable, hip abduction for hygiene, adequate knee flexion.

In completion of Parkinsonism examination, one needs to request for: (5)

1) Speech assessment 2) Swallowing assessment 3) Handwriting assessment 4) Postural BP 5) Abbreviated mental test

How do you differentiate spinal shock from complete spinal cord injury?

1) Spinal shock - loss of sacral nerve function: triad of perianal sensation (S3,4,5), rectal tone (S2,3,4) and great toe flexion (FHL = S1,2) - loss of bulbocavernosus reflex - usually recovers in 1-2 days 2) Complete spinal cord injury - bulbocavernosus reflex may be present if lesion is above L1 - 30% of patients regain some function Hence in suspected spinal cord injury, assess bulbocavernosus injury, if present, suspect spinal shock, stabilize and wait for resolution. If recovery does not occur after 1-2 days = complete spinal injury, treat w stabilization and manage complications e.g ventilator

What are possible confounders in using HbA1c to diagnose DM?

1) Spleenectomy 2) Bleeding 3) ESRF 4) Hemoglobinopathies - Thalassemia - Sickle cell anemia

What are the causes of pneumothorax?

1) Spontaneous 1' Idiopathic Thin young men 2) Spontaneous 2' Infections Malignancy Chronic lung disease (COPD, Asthma, cystic fibrosis, emphysema) Traumatic Iatrogenic Connective tissue disease

How does bronchogenic carcinoma spread?

1) Spreads along alveolar wall from bronchus 2) 'Spread thru air space' STAS - poor prognostic factor Appears as fluffy lesions on lung CT

What are the clinical findings of shoulder dislocation?

1) Squaring of shoulder with palpable mass anteriorly 2) Regimental badge numbness 3) N&V +/- Sensation on anterolateral forearm

What are the possible radiological findings of ankylosing spondylitis?

1) Squaring of vertebrae 2) Vertical or marginal syndesmophytes 3) Joint space narrowing 4) Sclerosis around SI joint

What is the treatment of congenital diaphragmatic hernia?

1) Stabilisation of ABCs - Avoid mask ventilation (due to possibility of swallowing air) - Intubate immediately - Avoid high inspiratory pressures, watch for pnemothorax - Keep arterial blood pressure >/= 50mmHg 2) Insertion of vented orogastric tube on continuous suction (to prevent bowel distention and further lung compression) 3) Surfactant - Only if infant is pre-term (GA</= 34/52) and have findings of respiratory distress syndrome 4) Surgery - Primary repair of diaphragmatic defect after reduction of bowel into abdominal cavity - Patch repair of defect

What are the requirements of various precautions and some of their indications?

1) Standard precautions - Handrub, 5 moments of hygiene - Careful needle handling - Gloves and gown if handling patient/hazardous material The rest are IN ADDITION TO standard precautions: 2) Contact precaution - Mask +/- eye shield if risk of splash - Gown - Gloves e.g MRSA, C.diff (need soap) 3) Droplet precaution - Mask with eye shield e.g Meningococcal, influenza, mumps, rubella, pertussis 4) Airborne precaution - N95 mask e.g Disseminated zoster, chickenpox, measles, TB

What is the management of dermatomyositis?

1) Steroids: prednisolone 1st line 2) Methotrexate/azathioprine/IVIG if steroid resistant 3) Treat underlying cause: if due to tumor/malignancy, may remit with definitive treatment

How does HIV cause disease?

1) Stimulation of immune system (early) - Poly allergy - Immune mediated diseases e.g thrombocytopenia, acute/chronic inflammatory demyelinating polyneuropathy, GBS-like syndrome 2) Immunosuppression (advanced) - Opportunistic infections and malignancies - Reactivation of latent infections (TB, Herpes, HSV, VZV, CMV, Kaposisarcoma, toxoplasmosis) 3) HIV infection - CNS: meningitis, myelopathy, HIV enceophalopathy - PNS: peripheral neuropathy - Renal: nephrotic syndrome, FGS, ESRF - Heart: HIV cardiomyopathy - Wasting disease 4) Psychosocial - Stigma - Patient reaction to disease

What are the options for grafting a oesophagus?

1) Stomach 2) Colon 3) Jejunal

What are possible sites of peptic ulcer disease?

1) Stomach 2) Duodenum 3) Gastrojejunostomy site 4) Meckle's diverticulum 5) Esophagus

Principals of preserving amputated parts:

1) Stop bleeding with direct pressure 2) Bag it, then double bag it 3) Place double bag in ice slush = to reduce temperature to 0 deg, cold ischemia improves re-implantation success

What are the tests for nerve root tension and what do they indicate?

1) Straight leg raise (L5/S1 i.e L5): - Raise extended leg until patient says pain - Pain has to be of shooting nature to be +ve - Note angle of pain - Cross straight leg raise as well (where raising 'normal' leg results in contralateral radicular pain) ADDON 2) Lasegue's test - Lower SLR by 5 deg until no pain, then dorsiflex ankle to reproduce pain -> +ve Lasegue's test - SLR may be due to tight hamstrings (like mine hahaha) 3) Bow stringing test (L4, L5, S1) - Flex hip to 100 degrees and knee to 90 - Press medial to lateral biceps femoris tendon (where the peroneal nerve is) - If shooting pain radiating proximally or distally -> +ve straight leg test. 4) Femoral stretch test (L2, L3, L4) - Patient lies prone with hip extended and knee flexed to 90deg - gently further extend hip, if patient complains of anterior thigh shooting pain -> +ve

What are the causes of supraspinatus tendinitis?

1) Subacromial impingement 2) Overuse 3) Trauma

What are the pertinent components in summarizing a medical long case?

1) Summary of case and patient details 2) Issue list: - Medical issues (acute and chronic) - Functional issues - Psychosocial issues Must be tailored to patient Throw in health promotion/healthy lifestyle patients

What are the manifestations of cuff tear arthropathy?

1) Superior migration of the humeral head 2) Humeral head collapse or femoralisation +/- snow cap sign (from subchondral sclerosis) 3) Glenohumeral cartilage destruction 4) Subchondral osteoporosis 5) Acetabularisation of acromion

What is the management of diverticular bleeding?

1) Supportive: - ABCs esp if patient unstable 2) Conservative: - 80% self limiting, 40% will bleed again - Stop anticoagulation medications/correct coagulopathies - Iron tabs for bleeding 3) Non-invasive - Investigate as per LBGIT: colonoscopy - TRO UBGIT: OGD w biospy - ?small bowel bleed via capsule/enteroscopy - CTMA: diagnostic and potentially therapeutic (via angioembolisation) 4) Surgical - Exploratory laparotomy + enteroscopy via ICjx incision + colonoscopy large bowel with wash out - Bleeding segment resected

What are the dermatological changes in patients with Cushings disease?

1) Supraclavicular fat pads 2) Acne 3) Striae - Broad purplish striae (colour may fade but atrophy remains) 4) Infection - Pityriasis versicolor/dermatophytic infections

What are the special tests involved in shoulder examination and what do they test for?

1) Supraspinatus impingement - Neer's test (w elbow extended, flex the shoulder passively, watch for pain) - Hawkin's test (w elbow and shoulder flexed, passively internally rotate the shoulder, watch for pain) 2) Supraspinatus tendinitis - Jobe's test (beer can test, ask patient to flex shoulder against resistance) 3) Supraspinatus tear - Abduction test w drop off sign (when pt cannot flex shoulder, passively lift to 90 deg to show deltoid able to abduct) 1) Biceps tendinitis - Speed's test (bicep curls) - Yergason's test (supination test w/ elbow flexed) 2) Biceps rupture - Popeye's sign (rupture of long head, bulge drops down) 1) Stabililty - Anterior apprehension test (try to anterior dislocated shoulder, will be apprehensive) 2) Laxity - Sulcus sign (pull down to show sulcus) 3) Labrum tear - Obrien's (like jobes but w 10degree adduction, pain should diminish when done w arm in supination, otherwise specificity ~ 37% so not very useful)

What is the management of follicular and papillary thyroid carcinoma?

1) Surgical resection - Hemithyroidectomy if low risk - Total thyroidectomy for most - LN clearance (depending on involvement of LN) o Level 6 LN clearance o Neck dissection if more are involved 2) Adjuvant therapy - Radioactive iodine at ablative levels for remnant thyroid and any cancer tissue (for total) - External radiotherapy (good for locally advanced follicular CA) 3) TSH suppression - L thyroxine to suppress TSH levels to < 0.005U/L (so that any thyroglobulin produced is from CA cells) 4) Follow up - TSH levels - Thyroglobulin (for recurrance) - Radioactive iodine scan to detect recurrance, then f/u w radioactive ablation

What is the management of medullary thyroid carcinoma?

1) Surgical resection - Total thyroidectomy - Clearance of level 6 LN - Neck dissection if indicated 2) Follow up - Thyroxine replacement (for euthyroid state) - Serum calcitonin and CEA 6/12 after surgery (considered cured if normal, 5% 5 yr recurrance) 3) Recurrance - Only if calcitonin remains high, screen for metastatic disease, treat surgically or

When do you consider radiographic imaging in urinary tract infections?

1) Suspicion of structural abnormalities - REinfection with same species within short time intervals - Persistence infection despite appropriate therapy - Do U/S KUB 2) Complicated pyelonephritis - ill, complicated factors, due to underlying nephrolithiasis or reflux - CT looking for renal mass/abscess, calculi, hydronephrosis, BPH

How would a patient with carpal tunnel present in terms of symptoms and signs?

1) Symptoms - Shooting pain in lateral 3.5 fingers, worse at night from hand dangling off bed, can wake them - Clumsiness 2) Physical examination - Sensory deficit (2 pt discrmination) - Thenar wasting, weakness - carpal tunnel compression test (press w both thumbs over C.tunnel for 30 seonds, if pain/paresthesia = +) - Tinel's test + - Phalen's test +

How do you diagnose malaria?

1) Symptoms of malaria +/- 2) Laboratory confirmation of parasite infection - Microscopy (gold standard): giemsa stained blood smears, looking for intracellular parasite, parasite morphology and parasite density (which correlates with severity of disease) - Rapid diagnostic tests: Commonly looks for malaria antigens (HRP2 and pLDH), but antibody also exists - Serum aldolase: enzyme involved in malarial glycolysis pathway, correlates with parasite density and become undetectable with clearance of parasites.

How do patients with transient synovitis present?

1) Symptoms: - Unilateral hip/groin pain, inability to bear weight - Recent infection/illness/trauma (>50%) of cases) - If child very young, may just be crying at night 2) Signs: - **Log roll: showing involuntary guarding at one side - tenderness on palpation, pain on passive movement - limited RoM - Antalgic gait

What are the signs of breathlessness in children?

1) Tachypnoea 2) Increased effort in breathing: - Nasal flaring - Intercoastal/subcoastal retractions - Use of accessory muscles - Diaphragmatic breathing 3) Increase noise in breathing: grunting, stridor, wheezing 4) Reduction in activity level/speech to breath 5) Poor feeding 6) Appearing cyanosed/altered mental state 7) Apnoea

What are the managements of breast cancer?

1) Targeted endocrine therapy - ER - PR - HER2 (herceptin) 2) RT/CT Radiotherapy Contraindicated if: pregnancy, previous radiation in similar areas, connective tissue disease, patients who cannot lie flat/still e.g whole breast RT (esp post BCT), chest wall RT, axillary RT/ local mets/invasion Chemotherapy Adjuvant/neoadjuvant/control of metastatic disease e.g Taxanes 3) Surgery [Local therapy] - Breast conserving surgery (will require radiation after, or recurrence will be high, so patient have to go, cannot be contraindicated for + must consider possiblity of going for second surgery) - Oncoplastic breast surgery: doing perioperative plastic surgery to retain breast ideal shape. - Mastectomy (skin sparing/nipple sparing): indicated in multifocal, invasive, RT is contraindicated etc [Regional therapy] - Sentinel node biopsy, blue dye injected 5-10 minutes after GA and then massage in, to look for blue node after OR use radioactive isotope in radiology, then sent for intraoperative frozen section to determine need for axillary clearance - Axillary clearance: if frozen section reveals cancer, usually on levels I and II LNs [Breast reconstruction] - Implants/Autologous flaps etc

What is the definition of PUO?

1) Temp > 38.3 deg on several occasions 2) > 3weeks outside or > 1 week undiagnosed in hospital after investigations.

What are the indications for traction?

1) Temporary mx of neck of femur fractures 2) Femoral shaft fracture in children 3) Undisplaced fracture of acetabulum 4) Reduction of dislocated hip 5) To correct minor fixed flexion deformities of hip and knee

What are the side effects of isotretinoin?

1) Teratogenecity 2) Dryness (mucous membranes) 3) Hypertriglyceridemia (pancreatitis) 4) Transaminitis 5) Benign intracranial hypertension (when used w tetracycline) 6) Arthalgia, myalgia 7) Hair loss 8) Photosensitivity 9) Mood changes 10) Night blindness 11) Skin rash (almost like eczema)

What are the differentials for acute scrotal swelling and pain?

1) Testicular torsion 2) Epipididymoorchitis 3) Others: - Surgical - Torsion of testes appendix (i.e of hydatid of morgagni: remnant of mullein duct) - Inguinal hernia - Infection - Fournier's gangrene - Scrotal abscess - Mumps orchitis - Edema - Intrascrotal edema - Idiopathic scrotal edema - Autoimmune - Henoch scholein purpura - Neurological - Referred pain (lower ureter stone via L1 splanchnic, genito-femoral nerve compression, inguinal artery aneurysm, limbo-sacral spine pathology)

What are the indications for subacromial injection of lignocaine and triamcinolone?

1) Therapeutic - Subacromial OA - Rotator cuff tendinitis 2) Diagnostic - DDx of tendinitis vs complete rupture

What are the differentials for Osgood Schlatter?

1) Tibial tuberosity fracture 2) Jumper's knee (patellar tendinitis) 3) Osteochondroma of the proximal tibia 4) SLJ (sinding-larsen-johansson) syndrome: chronic apophysitis or minor avulsion injury of the interior patellar pole, occurs in 10-14 y/o children esp w cerebral palsy.

What are the possible sources of unilateral lower limb cellulitis?

1) Tinea pedis - Look between the web spaces 2) Chronic venous insufficiency - Look for varicosities, telangiectasias, edema, ulcers, venous eczema 3) Lymphedema - Look for swelling

What are the possible focal brain lesions in HIV and how do you differentiate them?

1) Toxoplasmosis - Multiple hyperdense lesion with ring enhancement and edema - Often at basal ganglia 2) CNS lymphoma - Usually single hyperdense lesion - Often at subependymal spread 3) Pyogenic brain abscess - Similar to toxoplasmosis 4) Progressive multifocal leukoencephalopathy - By JC virus - Patchy subcortical white matter lesions (hyperdense on T2 (watery) and hypodense on T1) - Usually no ring enhancement or edema 5) CMV encephalitis - Diffuse, non specific increased T2 flair in white matter +/- ventriculitis (ependymal enhancement and hydrocephalus) - No enhancement (w/o ventriculitis) and no mass effect/edema (often atrophied)

Where are the possible surgical entry points for thyroid surgery?

1) Traditional anterior approach 2) Axillary approach 3) Anterior chest 4) Breast approach 5) Combined breast and axillary approach

What are 5 steps of pain conduction?

1) Transduction (acute stimulation of nociceptors) 2) Conduction 3) Transmission (movement via axons from receptors to cerebral cortex) 4) Modulation (inhibition/facilitation of pain by mechanisms in ascending and descending pathway) 5) Perception (Cortical and limbic interpretation and awareness of pain)

What are the potential problems with massive blood transfusion?

1) Transfusion related o Immune acute - Immune mediated acute hemolytic reaction - Immune mediated non-hemolytic reaction - TRALI - Anaphylaxis o Non immune acute - Circulatory overload 2) Replacement related - Hypothermia - Electrolyte imbalances

What are the causes of pleural effusion.

1) Transudative - CCF - Cirrhosis - Nephrotic syndrome - Constrictive pericarditis - Peritoneal dialysis (due to hypervolemia) - Hypothyroidism - Atelectasis 2) Exudative - Parapneumonic effusion - Tuberculosis - Connective tissue disorders - Malignancy - Subphrenic abscess - Infections/ pancreatitis 3) Either - Pericarditis - Pulmonary embolism 4) Urothorax - Obstructive uropathy 5) Hemothorax - Trauma - Malignancy - Endometriosis (catamenial) 6) Chylothorax - Congenital chylothorax - Post-traumatic

What are the causes of cubitus varus deformity?

1) Trauma - Supracondylar fracture (unreduced medial column comminution or uncorrected rotation) 2) Congenital - Noonan syndrome - Turner's syndrome

What is the management of peptic ulcer disease?

1) Treatment of underlying cause - Biopsy for H.pylori, eradication therapy if have - Stop NSAIDs if possible 2) Anti-secretory therapy - Omeprazole 20mg OM for 8 weeks - Omeprazole 20mg BD if GERD. - +/- H2 receptor antagonists 3) Rescope in 6 weeks to document ulcer healing and re-biopsy for H.pylori

What are the causes of diabetes mellitus?

1) Type 1 2) Acquired - Type 2 - Insulinoma of pancreas

What are the 5 types of myocardial infarction and their causes?

1) Type I: spontaneous MI due to primary coronary artery disease 2) Type II: Due to imbalance in supply and demand of oxygen w pre-existing coronary artery disease -> ECG findings usually just T wave inversions w ST depressions 3) Type III: Sudden cardiac death, includes cardiac arrest, Verified coronary thombus on angiography/autopsy but death before biomarkers/blood tests can be done 4) Type IV: MI associated with verified stent thombosis via angiography or autopsy 5) Type V: MI associated with CABG

What are the phototherapy options available for psoriasis?

1) UVB 2) Photochemotherapy (psoralen + UVA) - Risk of pruritis, erythema, pigmentation - Photoaging, lentigenes, skin cancers (actinic keratoses, bowen's disease, BCC and SCC)

Functional assessments to be done in Parkinsonism examination. (3)

1) Unbutton shirt 2) Write something 3) Comb hair

How does varicella infection present as and what are the possible complications?

1) Uncomplicated varicella infection, 15 days aft exposure - Pharyngitis - Fever, malaise, LOA - Vesicular rash eruption (24 hrs after fever) 2) Complications - Skin and soft tissue: GAS infection (cellulitis, nec fash), myositis, toxic shock syndrome - Neurological: encephalitis (2 types: acute cerebellar ataxia: more in children and diffuse encephalitis: more in adults) and reye syndrome (esp w salicylate use, N&V, headache, delirium, combativeness, coma) - Respiratory: pneumonia - Hepatitis (esp in immunocompromised)

What are the causes of pes cavus? What is the use of the coleman block test?

1) Unilateral: - Previous trauma (talus fracture malunion) - Spinal cord tumor/impingement - Inflammtory joint disease 2) Bilateral - Charcot marie tooth - Muscle imbalances (strong tibialis posterior and peroneus longus w weak tibialis anterior and peroneus brevis) - Cerebral palsy - Fredreich's ataxia - Polio To determine if the cavus is hindfoot or forefoot and to determine flexibility of deformity

What are the investigations for testicular torsion?

1) Urine culture: - Usually normal with normal total whites (70% of time) 2) FBC: - Elevated whites in 60% of torsion (Both unreliable in differentiation from epididymoorchitis) 3) U/S scrotum - Useful to confirm absence of torsion assessing for blood flow - Should only be done in non-clear cases: 'A testicular torsion with classical signs should never see the inside of a radiology suite.'

What are the causative organisms of Necrotizing faciitis?

1) Usually polymicrobial (90% with clostridium, proteus, Ecoli, bacteroides and enterobacteria, hence the abx regimen) 2) Group A strep (10%, strep pygenes)

What advice should you give to asplenic patients?

1) Vaccinations - Seasonal influenza - Capsulated organisms (pneumococcal, meningococcal, H.influenzae) 2) Avoidance - Dog bites e.g capnocytophaga 3) Medications - Standby augmentin for febrile episodes that may potentially be meningococcemia

Complications of Prosthetic valve

1) Valve dehiscence/valve failure 2) Valve stenosis 3) Infection of valve 4) Hemolysis from shear forces - presenting as anemia and jaundice 5) Anticoagulation: overwarfarinisation or inadequate -> valve thrombosis

What are the surgical options for valve pathologies?

1) Valvuloplasty 2) Annuloplasty 3) Valve replacement

What are the indications for intense pulsed light therapy?

1) Vascular lesions including spider telangiectasis, port wine stains, broken facial veins, rosy cheeks, rosacea and red thread veins of the legs 2) Freckles and age marks 3) Facial lines and wrinkles 4) Removal of unwanted dark hair

What does one look out for in duplex venous ultrasound?

1) Venous dilation > 3mm 2) LSV/SSV incompetence 3) SFJ/SPJ incompetence 4) DVT

What are the different types of meniscal tears and how can they be managed?

1) Vertical 2) Bucket handle 3) Transverse 4) Parrot beak 5) Peripheral 6) Flap Management: 1) Non-operative - Indicated first line for degenerative tears - Rest - NSAIDS - Rehabilitation 2) Operative - Meniscal repair - Partial meniscectomy (if cannot repair: poor blood supply, severe damge) +/- Meniscal transplant

What discharge advice would you provide to patients after elective orthopedic operations?

1) Watch for complications - Bleeding, infection, DVT/PE and dislocation (e.g hip) - Watch for discharge, painful swelling of leg - Systemic symptoms of infection: fever, SOB 2) Activity limitation - Avoid extremes in range of movement i.e keep < 90 deg: no crossing legs, use shoe horn to wear shoes, grabbing tools/reachers, sleep w abduction pillow 3) Diet and exercise - diet of choice, keep nutrition high - weight bearing activities at least 4 weeks from op 4) Clinical follow up - TCU for STO and PT/OT

What are the important steps to ensure in set up of Russel skeletal traction with thomas splint and pearson knee attachment?

1) Weight not on floor 2) Pull direction aligns with longitudinal axis of limb 3) Patient in trendelenberg position 4) No interruption of line of pull 5) Adequate line of pull

What are the radiological signs of pulmonary embolism?

1) Westermark sign - sharp cut off of pulmonary vasculature 2) Hampton's hump - wedge shaped pulmonary infarction 3) Fleischner's sign - prominenct of central pulmonary artery 4) Fleischner's lines - focal atelectasis due to vascular occlusion leading to subsegmental brochial haemorrhagic exudate

What are the CXR findings of Aortic dissection? What other investigations would you order?

1) Widened medistinum 2) Paraspinous stripe 3) Obscured aortic knob 4) Calcium sign If stable: - CT aortogram: gold standard, contrast to be used. - Transthoracic echo If unstableish - Bedside transthoracic echo - CXR

What must you counsel the patient on before starting radioactive iodine treatment for Graves disease?

1) Will worsen ophthalmopathy 2) Cannot get pregnant while under treatment 3) Will likely be hypothyroid after ablation

What are the differentials for failed steroid therapy?

1) Wrong diagnosis - Extramammary paget's disease - Superficial fungal infection (tinea incognito) 2) Complications - Superimposed infection e.g S.aureaus, HSV 3) Allergy - Drug allery - Allergic contact dermatitis 4) Compliance - Poor patient compliance

What is the age distribution for Acute Appendicitis?

1) Younger patients (40-57%) 2) Older (>50 y/o) (55-70%) -> need to watch for misdiagnosis and have a high index of suspicion

What are the radiological signs of pneumoperitoneum?

1) free air under diaphragm 2) rigglers sign 3) football sign (small bowel in middle with air around) 4) falciform ligament (normally not visible)

What are the radiographic views for evaluation of pelvic fractures and what do they look out for?

1)AP pelvis - looking for assymetry, rotation, displacement of hemipelvis - Signs of pelvic ring fractures require additional imaging 2) Outlet view - ~45 degrees towards the head (cranial) - Good for sacral fractures and vertical translation of hemipelvis 3) Inlet view - ~45 degrees towards the tail hahah (caudal) - Good for anterior, posterior and rotational displacements of hemipelvis - Also good for sacral widening and impaction fractures of sacral ala

When is lymph node removal indicated in thyroid cancer?

1)Central lympnodes: pretracheal, paratracheal, precricoid, perithyroidal - Do if involved - Prophylactic removal is controversial 2) Lateral neck dissection: Iia III, IV, VB - Only in proven metastasis e.g FNAC, classic CT findings - So that spinal accessory nerve is spared

What are the trophic signs of chronic limb ischemia?

1)Loss of hair 2) Dry, shiny skin 3) Nail changes e.g ram's horn nail) 4) Wasting of muscles

How is S.Aureas treated and what if it doesn't freaking die?

1)Methicillin/oxacillin is typical treatment - But may have gene mutation coding for penicillin binding protein with less affinity for penicillins - Hence MRSA So use: 2) Vancomycin is typical treatment - But Vancomycin-intermediate/resistance S.aureus may be possibly due to thickening of cell wall that binds vancomycin or changes in cell growth metabolism OR vancomycin resistance E.faecalis conjugal gene transfer

What are the finger breadth measurements for annesthetic evaluation of airway?

1, 2, 3, 3, 4 rule hahaha 1 finger space behind ear when mouth open 2 finger space from thyroid to floor of mouth 3 finger spaces for hyoid-mental distance as well as interincisor distance 4 directions for range of neck movement.

How many toes are normally seen from the back?

1-1.5 toes, hence if > thatn that = too many toes sign.

What is the daily requirement of sodium?

1-2mEq/kg

What is the normal thickness of the gallbladder?

1-2mm, thickened is more than 3mm

What is the daily requirement of calcium?

1-3g/day

1) How much saliva is produced per day? 2) How much of gastric juice is produced? 3) How much of pancreatic juice is produced? 4) How much bile juice? 5) How much fluid will the small bowel produce?

1.5-2L 1.5-3L 500-800ml 500ml 4L

What is the formula of mean arterial pressure

1/3 DBP + (SBP - DBP)

What is the accompaniment of dermatomyositis and malignancies?

1/3 cancer before rash, 1/3 with rash, 1/3 after rash Hence need to follow up for 5 years

What is the normal 2 point discrimination length and what does it predict?

10 mm Predicts nerve injury

What is the rule of 10 in pheochromocytoma?

10% are malignant 10% are bilateral 10% are familial e.g MEN2b, von hippel lindau, NF-1, TS 10% are recurrent (search on MIBG scan, metaiodobenzylguanidine) 10% are found in children 10% are extra renal: bifurcation of aorta (aka organ zuckerkandl)

What is the definition of a urinary tract infection? What will make you suspect a sample is contaminated?

10^5 CFU/ml of a single pathogen Contamination should be suspected if 10^4 CFU/ml or 2 or more pathogens are cultured.

What is subtotal thyroidectomy?

10g of thyroid left over on either side.

How is calcium gluconate given in hyperkalemia?

10ml of 10% Calcium gluconate over 10 minutes -> 2.5 mls every 2.5 minutes for 4 times = slow bolus

What is the enzyme that breaks down cortisol, and to what end product?

11B hydroxysteroid dehydrogenase Breaks down cortisol to cortisone. Inhibited by liquorice, resulting in hypercortisolism!

What is the treatment of TB meningitis?

12 months of RHEZ therapy + adjunctive steroids 2/12 of Initial phase - RHEZ 8/12 of continuation phase - RHE

What is the normal femoral neck-shaft angle?

120-135 degrees

At what volume does the urge to urinate occur? How much volume can a bladder typically hold?

150-200ml at 300-400 ml is still tolerable

What urinary cath size would you put in and what if the patient has clots?

16Fr 20-24Fr if hematuria.

When can the hip screw in SCFE be removed?

18 years old - Age of fusion of the proximal femoral epiphysis

What is the Gartlands classification, how might it guide management?

1: Undisplaced - Treat with cast immobilisation for 3-4/52 and radiographs at 1 week 2: Displaced with intact posterior cortex - Treated with CRPP (closed reduction and perc pinning) 3: Displaced with no cortical contact - Treat w CRPP or ORIF if needed

What is the dosing of tranexamic acid in blunt trauma and how does it work + any contraindications??

1g over 10 minutes, then 1g in 8 hour infusion. It is an anti-fibrinolytic that reduces clot breakdown. Contraindicated in stroke/IHD.

What are the types of metallic valves?

1st gen: ball and cage But flow is not laminar, blood disperses around the ball -> turbulence results in coagulation Also cage is sutured onto the base -> after a while there is metal fatigue and the cage breaks off -> strut fracture 2nd gen: tilting disc Turbulence better but still present, with risk of strut fracture 3rd gen: bi leaflet

What are the drugs in each generation of cephalosporins?

1st generation - Cefazolin - cephelexin 2nd generation - Cefoxitin (only one that covers anerobes) - Cefotetan 3rd generation - Ceftriaxone (covers gram positives) - Ceftazidime (covers gram negatives, anerobes and PAE, but poor at gram positives) 4th generation - Cefepime (covers PAE and gram negatives) 5th generation (even covers MRSA) - Ceftaroline - Ceftolozane + tazobactam - Ceftazidime + avibactam 'Ft' found in 3rd gen 'F' found in 1st and 2nd gen (plus 4th gen)

What is the treatment for syphilis?

1st line - IM benzylpenicillin 2nd line (for allergic patients - PO doxycycline - PO tetracycline - PO erythromycin

How do you treat tinea capitis?

1st line - Oral antifungal: griseofulvin, terbinafine and itraconazole

How do you treat malaseezia folliculitis?

1st line - Oral fluconazole - Oral itraconazole 2nd line (if cannot tolerate oral or refractory) - Topical: selenium sulfide or econazole shampoo

How do you treat alopecia areata?

1st line: - Intralesional triamcinolone 5-10mg/ml 2nd line: - Topical diphenyl-cyclopropenone - Photochemotherapy

What is the treatment of gonorrhoea?

1st line: 1) Ceftriaxone 500mg IM x1 dose or 2) Cefixime 400mg PO x1 dose 2nd line: 1) Cefotxime or aztreonam 1g IM x1 injection WITH CHLAMYDIAL coverage - PO doxycycline 100mg BD or azithromycin 1g STAT Follow up test of cure cultures in 14 days.

How does one treat acute bacterial sinusitis? What antibiotics are contraindicated?

1st line: Augmentin 2nd line: respiratory fluoroquinolone (levofloxacin, ofloxacin) or doxycycline Bactrim and macrolides are NOT indicated due to increasing H.influenzae and strep pneumo resistance respectively

What is the diagnostic criteria of stress fractures and how would you manage?

1st line: XR of affected part - TRO non-stress fractures - Stress fractures only appear late (2-4 weeks after start of pain) - Subtle and weak radiolucency may be seen +/- delicate fracture line +/- callus formation in late stage - High false negative rate, hence follow up with: 2nd line: MRI - Most sensitive and specific imaging examination for diagnosis of stress fractures - CT is used if MRI contraindicated - Can identify 1-2 days after start of symptoms - Edema in bone tissue and adjacent areas - Able to differentiate cortical damage from medullary and periosteal. - +/- signs of bone remodelling (intramedullary endosteal edema) - Sensitivity is greater than or equal to scintigraphy and of higher specificity Radiology is important for grading, which will guide the required duration of rest to be prescribed. Treatment: 1) Conservative: - Activity modification: 3 to 16 weeks (across the 4 grades), change in training conditions - Footwear adjustments - Immobilisation generally not prescribed due to muscle atrophy risk - Low intensity pulsatile U/S 2) Pharmacological - Analgesics e.g NSAIDs - Recombinant parathyroid hormone 3) Surgery - Only in severe injuries complicated by malunion/nonunion e.g in lateral cortical bone of the femoral neck (torsion impairs healing) Patient can be allowed to return to sport after: - No pain in provocation tests at the injury site - Absence of abnormalities in imaging studies - No pain for 10-14 days

What are the causes of Horner's syndrome?

1st order neurone (Central): - Brainstem pathology (vascular/demyelination) - Spinal cord pathology (tumors/cord syndromes) 2nd order (preganglionic): - Intrathoracic lesions (pancoast, subclavian artery aneurysms) - Neck lesions (glands, trauma, lymph nodes) 3rd order (post-ganglionic) - Internal carotid artery disease - Cavernous sinus masses

What factors to be considered in tetanus prophylaxis?

2 Components: 1) Tetanus toxoid (antigenic for active immunity) 2) Tetanus immunoglobulin (confers passive immunity) Important factors 1) Completion of tetanus vaccination/duration since last dose 2) Incomplete (<3 doses) 3) Clean, minor wound vs all other wounds If you had full 3 doses within last 5 years, dont need toxoid. 10 years if the wound is clean. You only need TIg if the wound is dirty AND your vaccination is incomplete.

What is the rule of 2?

2 Inches long (5cm) 2 feet away from ielocacal valve (60cm) 2% of population 2 types of ectopic tissue Present before age of 2

How long does it take for a bedsore to develop? How would you prevent this?

2 hours ,hence need to turn or: Remove factors that inhibit mobility - Pain: adequate analgesia - Neurological deficit: rehab, physiotherapy

What is the cut off time for percutaneous cardiac interventions and what do you do while waiting fo transport to an equipped facility?

2 hours. If > 2hours, give thrombolytic therapy while waiting for PCI.

How many out of 7 criteria for NF1 is diagnostic?

2 or more

At what age is flat feet normal?

2 years old, 90% have flat feet.

What is a krukenberg tumor?

2' ovarian tumor spread from anywhere, typically the GIT.

What are the cutaneous signs of Hep C?

2) Lichen planus

What are the common causes of pain in the mid foot?

2) Midfoot pain - If children < 5y/o: crushing osteochondroitis of navicular (Kohler's disease), where XR shows flattened navicular with increased density (self resolving) - If adult w pes cavus: Overbone, can feel a ridge of bone over cuneiform/1st metatarsal + shoe pain due to rubbing. (Changes the shoes or remove the lump) - Infections, tumors, fractures/stress fractures over metatarsal base/tarsal bones

What are the Respiratory manifestations of rheumatoid arthritis?

2) Respiratory system: - Upper airway: cricoaythenoid joints - Pleura: pleurisy, effusions - Airway: BOOP (Brochiolitis obliterans organising pneumonia, suspect w culture negative pneumonia) - Parenchymal: pulmonary fibrosis, pneumonitis, pulmonary hypertension - Caplan's nodes (rheumatoid pneumoconiosis with homogenous and well-defined lesions on XR)

What is the incubation period of Entamoeba histolytica?

2-4 weeks

What is the incubation period of gonorrhoea?

2-5 days (up to 2 weeks)

How soon does DRESS occur after start of drug treatment?

2-8 weeks

What is the target for TSH suppression in follicular and papillary thyroid cancer??

20 to 0.1 mU/L

What is the frequency of concomittant lumbar spinal stenosis in cervical myelopathy?

20%

What is the proportion of patients with asymptomatic cholelithiasis will present with symptoms within the next 20 year?

20-30% (about 1/3)

How is pheochromocytoma diagnosed?

24 hr urinary metanphrine/catecholamines +/- CT imaging for adrenal masses and visible neurofibroma popping out.

What are the options for targeted/hormonal therapy of breast cancer?

3 categories: 1) Selective estrogen receptor modulators - Tamoxifen - CI: prev DVT/CVA, immobile patients - risk of menopausal sx, endometrial CA (0.1%/year) and DVT 2) Aromatase inhibitors - Lanastrazole/ letrozole - inhibit peripheral conversion of testosterone and androstenedione to estradiol - Can only use in post menopausal women bec will cause over activity of HPA axis in pre-menopausal women - SE: MSK pain, osteoporosis! 3) Trageted therapy: - Herceptin i.e Trastuzumab - SE: cardiomyopathy and CCF, pulmonary toxicity, infusion reactions, febrile neutropenia - Others: avastin (targets VEGF) and lapatinib (her 1 and her 2 targets)

How does one grade carpal tunnel syndrome and what are the factors that connote a poor prognosis?

3 grades Mild: sensory involvement only Moderate: sensory and mild motor involvement (minimal wasting) Severe: severe sensory and motor involvement (severe wasting) Poor prognostic factors: 1) Old age 2) Longer duration of disease (> 10 months) 3) Extensive paresthesia 4) Muscle wasthing 5) Loss of 2 point discrimination > 6mm 6) Failure to improve after steroid injection

What is the recommended follow up duration for diabetics?

3 monthly.

Until what age will Barlow's and Ortolani's be reliable tests?

3 months of age - Beyond that, contractures will form that limit the sensitivity of the tests.

When can patients with suspected TB be removed from isolation?

3 negative AFB smears and 1 negative PCR.

What are the extra-GIT manifestations of ulcerative colitis?

3 skin: - erythema multiforme - pyoderma gangrenosum - erythema nodosum 3 eye - Episcleritis - Uveitis - Conjunctivitis 3 joints - Seronegative spondarthropathies - Ankylosing spondylitis - Sacroilitis 3 HPB - Primary sclerosing cholangitis - Gallstones - Autoimmune hepatitis 3 Renal - Nephrolithiasis - Hydronephrosis - ?

How would you classify pemphigus vulgaris and what are their pathophysiologies?

3 types: Mucosal dominant or mucocutaneous type and paraneoplastic 1) Mucosal dominant - Involves mucous membranes - Due to dominance of desmoglein 3 binding antibodies - Mucosa have higher levels of Dg3 2) Mucocutaneous - Skin signs exist alongside mucosal signs - Both Dg1 and 3 implicated. 3) Paraneoplastic type - underlying malignancy, unknown cause - both benign and malignant tumors possible.

How long should post-reduction shoulder dislocations be immobilised?

3 weeks in patients < 30 y/o 2 weeks in patients 30-40y/o 1-2 weeks for patients > 50y/o Can remove 2-4 times per day for range of motion exercises and finger/wrist/hand exercises to improve active + passive ROM and strengthen upper limb

What is the time period before a finger skin flap can be split?

3 weeks.

What are the neurological manifestations of Rheumatoid arthritis?

3) Neurological: - Peripheral neuropathy - Mononeuritis multiplex (autoimmune destruction of axons) - Nerve entrapment - Cx of atlanto axial subluxation +/- myelopathy - Muscle atrophy, proximal myopathy due to steroids, 2' MG due to penincillamine use

How soon can patients with an elective TKR go home after the operation, assuming there are no complications?

3-4 days

When would you drain a liver abscess?

3-5cm. Prof Alfred Kow suggests 5 Also, - Refractory to medical treatment - If solid on scan, nothing to drain, need hepatectomy - If multi-loculated, need to drain multiple times Send off for histology/culture and susceptibilty testing

What is the recurrence rate of DVT?

30% recur within 3 years.

What is the degree of bone loss that must exist before osteomyelitis is seen on XR?

30-40%

How many patients with PUO do not have a diagnosis and what will happen to them?

30-50% will not have a definitive diagnosis But prognosis is good

What percentage of adverse events are preventable?

30-70%

What is the maximum dose of lignocaine?

3mg/kg Hence 1% lignocaine = 10mg/ml 100% lignocaine = 1g/ml Therefore for 1% lignocaine, for 70kg patient, max dose = 210mg = 21ml of 1% lignocaine.

What are the grades of surgical site contamination?

4 Grades that depend on entry into colonised viscus/lumen, potential sites of contamination to determine risk of surgical site infection. 1) Clean - No entry into colonised viscus/lumen within body - Infection originates from contaminants/Environment/skin colonists - 2% rate of surgical site infection - e.g Elective inguinal hernia 2) Clean-contaminated - Elective and controlled entry into colonized lumen/viscus - Infection often from endogenous sources e.g E.coli - 4 to 10% rate of surgical site infection - e.g elective intestinal resection/head and neck cancers involving oropharynx etc 3) Contaminated - Gross contamination at surgical site in absence of obvious infection - Infection from endogenous sources - SSI rate > 10% even w abx etc - e.g penetrating bowel injury/elective bowel op w spillage 4) Dirty - Active infection present - Pathogens may be of unexpected orgins - Risk depends on duration of op and patient co-morbids (ASA status) - e.g abdominal exploration in acute bacterial peritonitis/ intraabdominal abscess

What are the risk factors for oral squamous cell carcinoma?

4 Ss 1) Spices - Betel nut 2) Sharp teeth - poor dental hygiene, chronic mucosal injury/infection/ trauma 3) Smoking 4) Spirits - Alcohol

What is the diagnostic criteria of RA? What investigations would you order?

4 components 1) Number and type of joints involved - small joints stronger - more joints stronger 2) Rheumatoid serology - RF and ACPA 3) Acute phase reactants -CRP and ESR 4) Duration of illness >/= 6 weeks According to 2011 American Academy of Family physicians Hence for labs: Order all - RF, ACPA - ESR, CRP - Joint fluid testing

What are the differences between compression stockings and 4 layer bandaging? What is their mechanism of action?

4 layer bandaging comprises of 1) Padding bandage 2) Crepe bandaging 3) Compression bandaging 4) Cohesion bandage and is indicated in ulcerative venous disease. Compression stockings are also for venous disease but not for use in patients with ulcers due to lack of absorptive properties. Both are contraindicated in peripheral artery disease. Improve disease by 3 ways: 1) Reduce venous pooling 2) Improve venous return 3) Reduce venous hypertension

What are the ligaments that stabilise the hip joint? How do they ensure security?

4 ligamentous elements 1) Illiofemoral 2) Ischiofemoral 3) Pubofemoral Forming a spiral orientation that tightens on joint extension, increasing security and reducing energy expenditure on standing position. 4) Thickened joint capsule

How long should PPI treatment be extended in the case of complicated ulcers?

4 more weeks.

What are the mechanisms of injury for knee dislocations?

4 possible types (Kennedy classification) 1) Anterior (30-50%) - Most common - Due to hyperextension injury - a/w PCL tear - Arterial injury 2' traction (insidious onset, need to put on regular vascular charting KIV angiography with angioplasty) 2) Posterior (25%) - 2nd most common - due to axial load to flexed knee e.g dashboard injury - Highest rate of complete tear of popliteal artery 3) Lateral (13%) - Due to varus or valgus force - Usually with ACL + PCL tear - Highest rate of peroneal nerve injury 4) Rotatory (4%) - posterolateral is most common - Usually irreducible - A/w buttonholding of femoral condyle through capsule (look for Pucker's sign/dimple sign)

What are the radiological features of injury to the stabilising ligaments of the lisfranc joint?

4 signs ON AP view 1) Disruption of the line drawn from the medial base of the 2nd metatarsal to the medial side of the middle cuneiform 2) Widening of the interval between the first and second ray ON OBLIQUE view 3) Medial side of the base of the fourth metatarsal does not line up with the medial side of cuboid on oblique view ON LATERAL view 4) Metatarsal base dorsal subluxation

What are the transfusion components for a massive transfusion protocol?

4 units RBC 2 units FFP 1 unit platets Transexamic acid in trauma patients Cryoprecipitate if fibrinogen < 1g/L

At what age do the femoral ossification centres begin to form?

4-6 months of age Hence radiographs for diagnosis of DDH will only be useful then and onwards.

What is the normal diameter of the bile duct? What is the diameter of the ERCP scope?

4-8mm 12-15mm

How are patients follow up after treatment of HCC?

4/12ly CT/MRI for 2-3 years 6/12ly after that + AFP levels if initially raised

What is the normal 2 point discrimination in the fingers?

4mm

How many samples should one take in an intra-operative joint biopsy for septic periprosthetic arthritis?

5

What is the urine output (ml/hr) of a patient in class III haemorrhagic shock?

5-15 ml/hr

What is the rate of recurrence of renal stones in patients with previous renal stones?

50% in 10 years

What is the normal range of cerebral perfusion pressure and how is it calculated?

50-150mmHg If < 65mmHg = Cerebral hypoperfusion if < 50mmHg = Cerebral ischemia and necrosis CPP = MAP - ICP

What are the reversible causes of cardiac arrest?

5H5T Hypoxia Hypothermia Hypovolemia Hypokalemia/Hyperkalemia H - acidosis Thrombosis, pulmonary Thrombosis, coronary Tension pneumothorax Tamponade Toxins e.g TCA A cold frozen tampon about to be bleached, tension from menstruation.

What are the signs of thyroid eye disease and how would you grade it?

6 Grades: - No involvement - Only lid signs - Soft tissue - Proptosis - Exophthalmos - Cornea - Squished nerve

What is the duration of antibiotic treatment in septic arthritis?

6 weeks (3 weeks of IV via PICC and then 3 weeks of oral if able to tolerate + clinical improvement)

What are the possible extra-articular manifestations of ankylosing spondylitis?

6As 1) Apical fibrosis 2) Aortic regurgitation 3) Anterior uveitis 4) Anemia 5) Amyloidosis 6) Achilies tendinitis Also treatment complications - Lung fibrosis from MTX - Pneumonia from immunocompromisation

What are the radiologic features of a charcot joint?

6Ds of hypertrophic joint 1) Distended joint 2) Density increase 3) Debris in soft tissue 4) Dislocation 5) Disorganisation 6) Destruction With signs of bone resorption With clinically intact skin and loss of protective sensation.

What are the signs of limb ischemia and the characteristics of arterial ulcers? What are the risk factors for peripheral vascular disease?

6Ps - Pain - Parethesia - Paralysis - Pulselessness - Pallor - Perishingly cold Arterial ulcers - Lateral aspect of foot/distal toes/pressure points or bony prominences - Painful - Irregular margins - Slough and necrosis at base Risk factors - Existing disease DM CAD Stroke/TIA PAD elsewhere - Non modifiable Age Gender Ethinicity Family hx - Modifiable Smoking Obesity Hypertension Lipids Glycemic control

When does contrast nephropathy typically set in?

72 hours after infusion of contrast

What are the ages of resolution in femoral anterversion, tibial torsion and metatarsal adduction respectively?

8 years, 6 years, 4 years.

What age group is the Pavlik Harness useful in?

9 months and below

How is weight estimated for children?

< 12 months Weight = (age in months + 9)/2 1-5 years Weight = 2(Age in years + 5) 5-14 years Weight = 4 x age in years

What is the definition of acute abdominal pain in paediatrics and what are the broad causes?

< 2 months of pain 1) Surgical (PAST) - Peritonism (perforation or visceral pathology) - Abdominal masses (liver, spleen, intestinal, reproductive organs) - Status post Surgery - Traumatic (visceral laceration/perforation/inflammation) 2) Medical - Upper abdomen (Hepatic, Gastric, Splenic, Pancreatic, important others: respi and cardiac) - Periumbilical (GIT and extra-GIT) - Lower abdomen (RLQ, LLQ, Suprapubic, flanks) - Suprapubic

What is the acceptable range of ICP?

< 25mmHg

What constitutes a young stroke? What are the causes?

< 45y/o with no vascular risk factors like: 1) Autoimmune

What does neutropenia in febrile episodes indicate?

< 7 days - Likely bacteremia > 7 days - Likely fungal infection

What are the principles in surgical excision of SCC?

<2cm, take margins of 5mm >2cm, take margins of 1cm Consider Moh's micrographic surgery SLN biopsy may be done KIV RT

What are the causes of raised CEA and what is the normal value?

<5.0 normal (smokers have 5) Useful in assessing for recurrence and indicative of prognosis. 1) Cancers - Breast - Thyroid - Testicular - Cervical - Bladder 2) Inflamamtory/ infective - IBD - Pancreatitis - Hepatitis - Cirrhosis - Chronic lung disease

What is portal hypertension and the normal pressure range within the portal ciruclation?

> 12mmHg is hypertension Normal portal pressure is 5-8mmHg

What is the risk of limb loss in critical limb ischemia?

> 50% in 6/12

What is the definition of massive hemoptysis?

> 600ml/day 1.5 cups

What is considered heavy proteinuria?

>1g/m2/day (height based)

What is the duration of cough that makes one suspect TB infection?

>3 weeks

What is the difference between SJS and TEN?

>30% involvement

How would you manage metatarsus adductus?

?Bleck classification ahhaha 1) Conservative - 90% resolve by age 4 - IF flexible and can be actively corrected: NO treatment - IF flexible and can be passively corrected: serial stretching by parents at home - IF rigid deformity: serial casting with goal of obtaining straight lateral border of foot. 2) Surgical - Tarsometatarsal capsulotomies - Multiple metatarsal osteotomies - Only if age > 5 years, resistant cases that fail non-operative treatment, severe deformity impairing shoewearing and causing pain.

What is the definition and pathogenesis of a Chance fracture?

A Chance fracture occurs when there is a pure bony injury extending anterio-posteriorly, where the fracture line passes through the body, pedicles and spinous process. Most often due to a flexion-distraction injury of the spine. Be sure to check from a duodenal and pancreatic laceration.

What is the pathogenesis of complex regional pain syndrome?

A clinical course of variable duration and unknown cause characterized by pain, swelling and vasomotor dysfunction of an extremity, usually associated with trauma or operation.

What is the Z line in the esophagus?

A demarcation line of the squamocolumnar junction where squamous esophageal mucosa meets the columnar mucosa of the stomach. Normally about 38-40cm from incisors, if higher = reflux esophagitis, confirm adenomatous metaplasia to diagnose Barrett's oesophagus.

What does the mesorectal fascia look like on MRI rectum?

A fine line of low signal, below the seminal vesicles and prostate (base on lithothomy position)

What is the pathogenesis of an abdominal aortic aneurysm?

A localised, abnormal dilatation of the blood vessel wall. True aneurysm = all layers intact False/ pseudoaneurysm = breach in vessel wall that allows communication between vascular lumen and extravascular space. AAA most commonly caused by athersclerosis where: 1) Plaque formation = destruction of tunica media and internal elastin fibres 2) Arterial wall thins and loses elastic recoil = dilation Diagnostic where aortia 3-15cm (normal aorta is 2 cm wide)

What is the pathogenesis of a wrist ganglion?

A mucinous cyst of unknown pathogenesis: myxomatous cystic degeneration, formation of one way valve between synovial body (tendon sheat or synovial joint).

What is a Glomus tumor?

A rare benign neoplasm arising from arteriovenous anastomoses in skin, previously for temperature regulation. Occurs commonly at nail beds, forming a small, tender, vascular swelling. Treat w surgery or radiotherapy

What is Muir-Torre syndrome?

A rare hereditary, autosomal dominant cancer syndrome that is thought to be a subtype of HNPCC. Individuals are prone to develop cancers of the colon, breast, and genitourinary tract, and skin lesions, such as keratoacanthomas and sebaceous tumors Pts have sebaceous gland adenomas, adenomatoid sebaceous hyperplasia and a visceral malignancy

What is the definition of an outbreak?

A single case of a communicable disease long absent from a population, or caused by an agent (e.g. bacterium or virus) not previously recognized in that community or area, or the emergence of a previously unknown disease, may also constitute an outbreak and should be reported and investigated.

What is a lung diffusion study and what are the implications of it's results?

A test gas is allowed to be inhaled by the patient, with the exhalation concentration measured to assess for the degree of diffusion. DLCO = diffusion capacity of the lungs for carbon monoxide. 1) Reduced DLCO (Airway, parenchymal, vascular, hematologic) - Obstructive lung disease - Parenchymal disease - Pulmonary vascular disease - Anemia 2) Increased DLCO - Asthma/normal - Pulmonary haemorrhage - Polycythemia - Left to right shunt

What subtypes of chlamydia are responsible for which lesions?

A-C - Trachoma (ocular infection) D-K - Chlamydia urethritis L1-3 - Lymphogranuloma venerum: genital ulcers and lymphadenopathy. Others - C.pneumoniae (atypical pneumonia) - C.psittaci

Which pulley do you cut in trigger finger?

A1 pulley A2 and A4 causes bowstringing A3 and 5 lie across joints and volar plate

Where are the pulleys of the finger flexors and which are important in preventing bow stringing?

A2, 4 and palmar aponeurosis are important to prevent bowstringing. A1, 3 and 5 lie across the joints while 2 and 4 lie across the phalages.

What is the use of ABPI in diagnosis of limb ischemia, the relevant values and the limitation?

ABPI - Ankle Brachial Pressure Index aka ArmSBP/AnkleSBP 0.9 or less is diagnostic of peripheral vascular disease 0.5 or less is diagnostic of critical ischemia However, in diabetic peripheral vascular disease, calcification of vessels may cause overestimation of ABPI (> 1.4). Hence TBI (toe-brachial index) should be used as small vessels are relatively spared and easier to occlude. For TBI: </= 0.7 is diagnostic of PAD

What are the features of inflammatory osteoarthritis and how would you manage it?

AKA Erosive arthritis 1) Epidemiology - Middle aged women (10:1 F:M) 2) Joints - typically fingers - Acute pain, swelling, erythema and warmth - XR: Gullwing and saw tooth appearance Management: 1) Non-operative - First line treatment for mild symptoms - NSAIDS - Observation 2) Operative - Indicated in debilitating pain and deformity: fusion - Indicated with mucocyst formation: excision + osteophyte resection

What is Faget's sign and what are it's causes?

AKA Sphygmothermic dissociation Where general rule of 1 deg rise in temperature equating to 10 bpm rise in heart rate no longer holds true. Causes: 1) Infection - Insect borne (ticks, mosquitoes) - animal borne (farms, rabbits, birds, rats) - airborne (aircon, air) - sexually transmitted (chlamydia) 2) Chronic fever

What are the signs of clubbed feet?

AKA congenital talipies equinovarus CAVE - Midfoot in Cavus - Forefoot Adduction and supination - Hind foot in Varus - Heel in Equinus

How would you manage a subtrochanteric fracture? What are the complications of management?

ALL SHOULD BE OPERATED Surgical: o ORIF with implants - Cephalomedullary nail - 95deg fixed angle device +/- bone grafting Cx: 1) Loss of fixation (Screw cutout, failure to lock etc) 2) Malunion 3) Non union

What are the differences in treatment of HSV, VZV and varicella?

ALL can use oral aciclovir 1) HSV - Topical aciclovir cream - Oral aciclovir within 72 hours - ABx for 2' bact infection 2) VZV - 800mg aciclovir 5x/day for 1 week 3) Varicella - 800mg aciclovir 4x/day for 1 week

What are the features that should be commented on in bone tumors?

ALPRO 1) Age - skeletally mature vs immature 2) Location - which bone - Diaphysis, metaphysis, epiphysis 3) Tumor to bone - Lysis vs sclerosis vs both - Expansility - Zone of transition, degree of demarcation 4) Periosteal reactions - Onion skin - Codman's triangle 5) Others: - Sunburst reaction

What is the use of AP XR, CT and MRI in evaluation of chance fractures?

AP view: - Can see empty vertebral body sign from displacement of posterior spinous process - Can see widening of pedicles which suggests concurrent burst fracture CT: - Can see retropulsion of vertebral body fragment into spinal canal MRI: - Can see degree of involvement in posterior ligamentous elements.

What is Gaucher's disease?

AR genetic defect in glucocerebrosidase. Leading to buildup of glucocerebroside, a lipid component of cell membranes, thus accumulation within cells in bone marrow.

What are the features of tuberous sclerosis?

ASHLEAF Ash leaf macules Shagreen's patches Heart rhabdomyosarcoma Lung hamartomas Epilepsy from cortical tubers Angiomyolipoma on kidney Facial angiofibroma

What is the most common presentation of chlamydia?

ASSYMPTOMATIC Mostly upper reproductive tract signs e.g infertility in women o

What are the margins of an abdominal Xray Vs a KUB xray?

Abdominal Xray: Diaphragm to iliac crests KUB: Diaphragm to pubic symphysis (can also be used to include the rectum)

Who are more likely to get psoriasis?

About 1-2% of the population M=F Bimodal: 20-30y/o and 50-60y/o 16% chance if 1 parent 50% chance if both parents

What are the criteria for initiating insulin therapy in diabetics?

Absolute: 1) All type 1 diabetics 2) Ketoacidosis/severe hyperglycemia (>27.7mmol/L) 3) Co-current serious medical condition e.g AMI/pneumonia 4) During/after surgery 5) During pregnancy 6) Failure to achieve ideal glycemic control with 2 or 3 oral agents - HbA1c > 10% - HbA1c > 7.5% plus FBG > 14 mmol/L

What does 'Best supportive care' entail?

According to American Cancer Society: 1) Symptom relief - Nausea - Pain - Dyspnoea 2) Psychosocial support - Depression

1) Educate patient on disease and recurrence, with diet and lifestyle modifications 2) Assess TRO secondary causes of hyperuricemia 3) Remove non-essential medications predisposing to hyperuricemia 4) Assess disease burden in terms of frequency and severity of signs and symptoms, as well as presence of tophi.

According to American College of Rheumatology 2009 guidelines: What are the baseline recommendations for management of all patients with gout.

1) Tophaceous gout (clinical or imaging confirmed) 2) Frequent flares (>2/year) 3) CKD stage 2 or worse 4) Past urolithiasis

According to American College of Rheumatology 2009 guidelines: What are the criteria for initiating pharmacological management of hyperuricemia in gout?

1) Xanthine oxidase inhibitors: Allopurinol or febuxostate If 1) is contraindicated 2) Probenecid

According to American College of Rheumatology 2009 guidelines: What are the lines of pharmacological management of hyperuricemia in gout?

How would you classify AVN? How does this guide management?

According to Ficat Arlet Staging - Grade 0 to 4 (total 5) Grade 0: pre-clinical features, but hip at risk. No pain - For risk factor management Grade 1: pre-radiographic phase, no structural change noted but patient symptomatic with hip/groin pain Grade 2: Pre-collapse phase, visible early structural change with repair, possible pain and stiffness For Grade 1 and 2 can be managed conservatively or surgically 1) Conservative - For early stage disease or medically unfit - Bed rest, weight relief (via crutches), splinting - NSAIDs/other analgesics - Bisphosphonates (slows resorption of necrotic bone) - PT - Risk factor control 2) Surgical - Core compression +/- bone grafting Grade 3: - Major structural failure, possible crescent sign (subchondral lucency from collapse) and step off sign (depressions in subchondral bone) - Bone collapse has occured - Treatment: o Realignment osteotomy o Sugioka transtrochanteric rotational osteotomy o Femoral head resurfacing o Hemiarthroplasty Grade 4: - Articular destruction, secondary OA changes, very painful - Only treatable w surgery: o Articular resurfacing o Total hip replacement

How can SCFE be classified and how does this guide management?

According to it's morphological appearance: degree of slip III grades Grade 1: <1/3 epiphyseal width - Treatment with ONE GOOD SCREW i.e percutaneous pinning - Position is acceptable\ - Removal at 18y/o (fusion of proximal femoral epiphysis) Grade 2: 1/3-1/2 epiphyseal width - Percutaenous pinning +/- compensatory osteotomy if residual deformity after remodelling Grade 3: >1/2 epiphyseal width - Percutaneous pinning WITH compensatory osteotomy OR - Epiphyseal replacement with piece of femoral neck + fixation with pins

What are the recommended cholesterol levels?

According to the CPG guidelines: Very high risk: 2.1 mmol/L High risk: 2.6 mmol/L Intermediate risk: 3.4 mmol/L Low risk: 4.1 mmol/L

How may humeral shaft fractures be classified?

According to the Orthopedic trauma association classification of fractures. 1) Bone: Humerus 2) Location: diaphyseal (vs proximal vs distal) 3) Type: simple vs wedge vs complex 4) Nature of injury: torsional, bending, direct trauma

Why is transaminitis present in obstructive jaundice?

Accumulation of toxic bile causes death of hepatocytes.

What is the difference between acidic and alkalotic skin necrosis?

Acid - coagulative necrosis, from which a protective layer forms Alkaline - liquefactive necrosis, which keeps burning down

What are the differences in pathogenesis and treatment of acute and chronic osteomyelitis?

Acute - Rapid onset, via hematogenous spread from other sources - Medical treatment - 6 or more weeks of appropriate antibiotics Chronic - Due to repeated, persistent exposure and unresolved infection, possible baseline immunocompromisation - Surgical treatment - Repeated debridement and long term antimicrobial suppression (months) - ABx to continue until ESR returns to baseline - Also to control underlying co-morbidities e.g DM

What is the pathogenesis of posterior tibial dysfunction?

Acute injury or long standing tendon degeneration results in loss of medial longitudinal arch

What is the difference between acute and chronic urticaria?

Acute/chronic urticaria - Duration attacks lasting <6 weeks/>6 weeks

What are the antibiotic regimens for treating C.difficile?

Acute: o Mild-moderate - PO Vancomycin o Severe - PO Vancomycin + IV metronidazole

What is the treatment of anaplastic thyroid caricnoma?

Acute: Tracheostomy Localised: RT Metastatic: Surgical debulking

What is the Knutsson sign?

Acuumulation of gas in intervetebral disc due to degeneration, principally nitrogen.

What is the difference between an Adult and Pediatric Colle's fracture?

Adults tend to have transverse fractures, paediatric bones can fracture in multiple ways, necessitating varying management. 1) Buckle fracture: very stable, cast for 2 weeks + 2 weeks of activity limitation 2) Physeal fracture (i.e Growth plate fractures): M&R w backslab (wrist slight flexion, ulnar deviation + elbow at 90deg) 3) Green stick fracture: M&R if deformity unacceptable (30deg in < 10y/o, 15 deg in > 10y/o), full cast from elbow to wrist, XR in 10 days, remove in 6 weeks) 4) Complete transverse fracture: same as adult

What regions are at risk of Yellow Fever?

Africa South and Central America (non USA)

What are the imaging options for small bowel bleed?

After excluding a LBGIT and UBGIT, a small bowel bleed is rare and classified as an obscure BGIT. Imaging options include: 1) Video capsule endoscopy 2) Deep enteroscopy 3) Radiologic imaging e.g angiography

Signs of Levadopa induced dyskinesia (3)

After long term use of levadopa, patients may exhibit dyskinesia in the form of 1) Dystonia (gradual tight painful contraction of muscles) 2) Chorea (sudden jerking of hands, neck etc) 3) Athetosis (writing snake like movements)

What is sezary syndrome?

Aggressive form of Cutaneous T cell lymphoma, where cancerous T cells (sezary cells) move from blood into skin and presents as erythroderma.

What is the treatment for brain abscesses?

Ahem. 1) Successful treatment of a brain abscess requires a high index of suspicion for the infection, which can have subtle presentations 2) Requires a combination of antimicrobial therapy and surgical draining for both therapeutic and diagnostic purposes - Antibiotics: o Empirical - Vancomycin (15-20mg/kg per dose IV every 8-10 hours PLUS Ceftriaxone (2g IV every 12 hours PLUS metronidazole (7.5mg/kg IV every 6-8 hours) (so like 8, 10, 12 hrly) - Surgery o Aspiration is first line o Excision indicated if Traumatic (remove the foreign material and chips) Encapsulated fungal brain abscess Multi-loculated abscess No improvement after aspiration/increased ICP/increasing size

What is the goal of TSH suppression in treatment of papillary and follicular thyroid carcinoma?

Aim TSH levels <0.005U/L

What is the ASPECTS CT trial and how does it guide management? (If got time, please draw the brain and label the regions).

Alberta Stroke Program Early CT Score (ASPECTS) Provides a score based on brain regions of the MCAaffected in ischemic stroke, to guide treatment with rTPA. Deduct points from total of 10pts for each region involved = early ischemic change (edema, loss of grey white differentiation) If score < 7 -> cannot rTPA as thrombolysis is a/w increased bleeding risk and reduced functional outcome at 3 months

What does a cross straight leg raise imply?

Always raise the normal leg first in SLR If raising normal leg ->the other side hurts ->cross SLR positive ->implies large central disc prolapse, large enough to compress nerves on both sides Patient is at risk of cauda equina syndrome

What is the AAST grading for renal injury?

American Association for the Surgery of Trauma grading system for severity of renal traumatic injury, 5 grades. 1 - Subcapsular hematoma 2 - Cortical laceration < 1cm 3 - Cortical laceration > 1cm 4A - Cortical laceration into collecting system 4B - Vascular injury with contained hematoma 5A - Shattered kidney 5B- Hilar injury/avulsion Also important to note: - Patient haemodynamic stabiltiy - Number of functional kidneys If unstable/grade 5 and above - exploration and nephrectomy If stable - Conservative mx w CT f/u because kidney healing good

What is amiodarone facies?

Amiodarone use manifesting as a deep blue discoloration around the malar region.

What types of hepatic abscesses are there and how do you tell them apart on CT?

Amoebic Vs Pyogenic Amoebic abscess - Usually no rim enhancement with peripheral edema - Usually from endemic region e.g 3rd world countries/middle east - Usually no jaundice/raised bilirubin Pyogenic abscess - E.coli, Kliebsiella, PAE, Schiostosomiasis - May come with jaundice - Treat with Ros-flagy

What is the antibiotic treatment for listeria monocytogenes infection?

Ampicilin or peniciliin G is the drug of choice But pen/amp have slower rate of onset because they have poor penetration into BBB and other less vascular regions. Hence combination therapy with Gentamicin If allergic to penicilins: - Desensitize OR - Treat with bactrim.

What is the mechanism of action of lignocaine? In what kind of wounds is it ineffective in and why?

An amide anesthesia, prevents opening of sodium gated channels in nerves, inhibiting pain potential conduction. It is a weak base, hence in acidotic regions e.g in infected wounds, it is INEFFECTIVE.

What is the NS1 antigen? What is it's use?

An antigen released by cells infected by dengue. Useful in detected acute dengue infections as it's level rises within the 1st days of infection, even before IgM or IgG rises.

What is a Tillaux fracture? What mechanism of injury causes it?

An avulsion fracture of the anteriolateral distal tibial epiphysis due to pull by the anterior inferior tibiofibular ligament, characteristically a salter-harris 3 injury. Due to external rotation injuries in skeletally immature individuals as the anteriolateral tibial epiphysis is the last to ossify.

What is the difference between the surgical and anatomical humeral neck?

Anatomical neck/surgical neck - higher risk of AVN/lower risk - ORIF/conservative tx - less common/85% of #

What is Baumann's angle and why does this damn thing matter??

Angle between line parralel to longitudinal axis of the humeral shaft and a line drawn along the lateral epicondyle. Important in supracondylar fractures BECAUSE (wait till you hear this shit): every 5 degrees change from it's normal of (70-75deg or < 5deg difference from normal side) can potentially lead to a 2 degree change in carrying angle. AND if your carrying angle is shit e.g varus deformity, you will freaking HIT YOUR HIPS while walking! WTF? Also internal rotation will be inhibited luh.

What does angular stomatitis and smooth tongue w loss of papillae imply?

Angular stomatitis - Riboflavin(B2) and niacin (B3) deficiency Smooth tongue - Riboflavin(B2), niacin(B3), iron, B12 and folate (B9) deficiency. Riboflavin is needed for cellular respiration as a co-enzyme Niacin (aka nicotinic acid) is also needed for cellular respiration as a precursor to NAD Folate is needed for DNA replication e.g in RBC (Megaloblastic anemia)

How does leptospirosis present and how is it diagnosed?

Anicteric vs icteric 1) Anicteric: - Abrupt onset of fever, chills - Myalgia, headache, abdominal pain - CONJUNCTIVAL SUFFUSION - transient skin rash - Asceptic meningoencephalitis 2) Icteric - Jaundice - Proteinuria, hematuria, oliguria/anuria - Pulmonary haemorrhages/ARDS - Myocarditis - Asceptic meningoencephalitis Diagnosed with 1) Clinical history (exposure to rats or contaminated water) 2) Leptospirosis serology (positive after 5- 10 days) 3) Microscopic agglutination test (gold standard, positive after 10-12 days) NOT cultures because positive only in 3weeks to 3 months, not useful except for research.

What are the boundaries of the anterior and posterior triangle of the neck?

Anterior triangle: - Posterior belly of digastric (superior) - SCM (lateral) - Superior belly of the omohyoid (inferior) Posterior triangle: - SCM (anteriorly) - Trapezius (posteriorly) - Omohyoid muscle inferior belly(inferiorly)

What is the surface marking of the diaphgram?

Anteriorly = 6 Laterally = 8 Posteriorly = 10 Hence triangle of safety is above rib 5 @ 4th intercoastal space, cutting parallel to upper border of 5th rib. Get patient to sit 45, or liver may rise

What can cause DRESS and how do you treat it?

Anticonvulsants, allopurinol, sulfonamides, other abx, NSAIDs 1) Remove drug 2) Check for systemic organ involvement 3) Systemic steroids 4) Ciclosporin in severe prolonged cases

How do you manage malassezia folliculitis?

Antifungals 1) Topical: - Ketoconazole shampoo 2) Oral: - Itraconazole tablets Isotretinoin 1) Oral isotretinoin

How would you treat urticaria?

Antihistamines - Non sedating 2nd generation H1 antihistamin e.g Loratidine/cetirizine 10 OD - Can be titrated up to 4x stated dose. - +/- sedating antihistamine at night e.g hydroxyzine 25mg - +/- H2 antihistamine may boost gen 1 H1 affect. Others - Indicated in patients refractory to antihistamine tx - Mast cell stabilisers e.g montelukast - Immunosuppressants e.g prednisolone, cyclosporin

What is amyand's hernia?

Appendix pulled to form part of hernia sac.

What are the differences in bases and locations of arterial/neuropathic/venous ulcers?

Arterial/ Neuropathic/ Venous Necrotic base/ sloughy base with granulation tissue/ sloughy base Pretibial, lateral malleoli, toes and heel/ pressure areas/ gaiter region e.g medial and can be circumferential.

What are the principles of an AvF formation?

Arterilisation of native vein by connection to an adjacent artery to create a high flow, low pressure system.

What is the spectrum of urinary tract infections and how do you treat them?

Ascending severity: 1) Asymptomatic bacteuria - Only treat if pregnant, under going op, immunocompromised 2) Uncomplicated cystitis - PO bactrim or - PO augmentin or - PO nitrofurantoin/fosfomycin/pivmecillinam (cannot for pyelonephritis) - If all cannot, fluoroquinolones (ciproflox) 3) Uncomplicated pyelonephritis - IV cefazolin + IV gentamicin or - IV augmentin + IV gentamicin' - If no response in 72 hrs, suspect perinephric abscess (need to image, consult uro and drain) 4) Complicated pyelonephritis - Emphysematous pyelo (drainage required along w abx and support, KIV nephrectomy) - Xanthogranulomatous pyelo (looks like RCC, patient unwell, surgical excision required) Same-same, but different 5) Recurrent cystitis (2x/6months or 3x/1 year) - Determine cause (Stones vs reflux) -Abx and lifestyle change if non structural - Surgery if structural (refer uro) 6) Prostatitis - Fluoroquinolones, bec can attain high levels in urine, despite resistance - 3rd gen cephalosporins - Carbapanems - If non responsive to abx or chronic (>3 weeks), image and consult uro 7) Catheter (Cath removed<48hrs ago) - stop/limit/switch cath methods - IV cefepime + IV amikacin - IV meropenem (if severe illness) 8) Candiduria - Fluconazole if symptomatic - +/- Surgical removal of fungal balls

What are the doses of aspirin and clopidogrel in DAPT?

Aspirin: 300mg bolus followed by 100mg OM maintenance Clopidogrel: 600mg bolus then 75mg OM maintenance

What are the causes of equinovalgus? What deformities are associated with it?

Associated with cerebral palsy (spastic or quadriplegic) OR fibular hemmelia (congenital lack of fibular, complete or partial) Possible deformities include: - Forefoot ABDuction - Heel valgus - Equinus contracture

What are the muscles displacing fragments in a Bennett's/Rolando's fracture?

At metacarpal head: APB FPB (Adductor Policis Brevis and Flexor Poliics Brevis) At base: APL (ABductor pollics Longus

When does treatment with ABduction splint for congenital tailipies equinovarus start and how long is it indicated for? Is there another name for the shoes?

At the 4th to 8th month of age After serial ponsetti toe to groin casting to reverse CAVE +/- tendon achillies lengthening if equinus To be used 23 hours a day for 3 months after correction After 3 months, do nighttime/nap timem use until 4 years of age Dennis Browne boots (FAO: Foot abduction orthoses)

Where is the pelvic binder place in a patient?

At the level of the greater trochanter.

What is the pathogenesis of alopecia areata and what are it's features.

Autoimmune destruction of hair follicles Non-scarring alopecia On pulling hair: exclamation mark

What is the pathogenesis of familial adenomatous polyposis?

Autosomal dominant inheritance of germline mutation of APC gene, results in > 100 adenomatous polyps all over colon. Polyps take 5-6 years to turn malignant and 90% will have colorectal CA by 45 years. Diagnosed by Genetic testing/colonoscope showing > 100 polyps

What is the pathophysiology of osteochondroitis dissecans?

Avascular necrosis of a fragment of subchondral bone predisposes overlying cartilage and subchondral bone itself to fragment. Subsequent osteochondral fragments (loose bodies) within joint space causes further pain and damage.

What is a de-gloving injury?

Avulsion injury resulting in removal of skin from it's underlying tissue, subsequently resulting in loss of blood supply.

If you could order 1 Xray on suspected shoulder dislocation, what would it be?

Axillary view Y scapula is shitty, cannot see.

What are the malignant features of breast masses in ultrasound?

B.I.T.C.H Borders ill defined Internal calcifications Taller than wide Central vascularity Hypoechoic

What are the components of the Child-pugh score and the scores that are relevant?

BATMAN saving a child with liver disease - Bilirubin - Albumin - proThrombin - Mental status - Ascities - Nutrition (replaced by pro-thrombin time)

Which anti hypertensive is contraindicated in phaeochromocytoma?

BBlockers. Need to give Alpha blocker first because if Beta receptors are blocked, all of them will move to alpha receptors causing significant vasoconstriction (alpha receptor activation with beta receptor blockade that is supposed to vasodilate).

What is the treatment of SCC, BCC and melanoma?

BCC: excision, RT SCC/bowen's

When is biopsy recommended in breast lumps?

BIRADS score of 3 or above RE: Grade 3: some centres offer short term interval f/u but some also offer histological biopsy

What is the definition of resistant hypertension?

BP > 140/90 despite 3 antihypertensives, 1 of which is a diuretic

What is the definition of multi-drug resistant bacteria?

Bacteria resistant to 3 or more antibiotic classes.

What are the pathogens that exist within normal skin flora?

Bacterial 1) Staphyloccocal - Epidermidis - Aureas 2) Streptococcus - Alpha-hemolytic (pneumoniae, viridans) and non-hemolytic (entercoccus) 3) Micrococcous 4) Peptostreptococcus 5) Neisseriae (non pathogenic) - i.e not meningitidis or gonorrhoea - e.g baciliformis, elongata etc 6) Propionibacterium 7) Diptheroids 8) Acinetobacter Fungal 1) Candidal

What is the BCLC criteria for HCC?

Barcelona Clinic Liver Cancer Staging. PS = performance status Associated diseases = ? Portal hypertension = hepatic venous pressure gradient >/= 10mmHg (difference between portal vein and IVC pressures) Increased bilirubin = >3mg/dL Childs Pugh A = 100-80% 2 yr B = 80-50% 2 yr C = <50% 2 yr survival

What are the important lines and angles of the hip joint on radiology and how are they useful in diagnosis of developmental dysplasia of the hip?

Based on AP Hip XR 1) Lines: - Shenton's line: smooth curved line from the medial border of the femoral metaphysis to the superior border of the obturator foramen - Hilgenreiner's line: horzontal line passing through the inferior aspect of the tri-radiate cartilage of the acetabulum - Perkin's line: line vertical to hilgenreiner's line that passes through both the lateral acetabular roof and the lateral border of the ossification centre of the femoral head. - Acetabular line: line drawn from the inferior aspect of the tri-radiate cartilages 2) Angles: - Acetabular angle: angle between the Hilgenreiner's line and the acetabular line - Lateral centre-edge angle: angle between vertical line from centre of femoral head to the outer edge of the acetabular roof Hence DDH, which presents as a supero-lateral migration of the proximal femur, will have - Disrupted Shenton's line - Perkin's line not moving lateral to ossification centre - Acetabular angle > 30deg

When is metabolic surgery indicated?

Based on BMI >/= 40: recommended regardless of glycemic control 35-39.9: Recommended in suboptimal glycemic control despites meds and lifestyle changes 30-34.9: Consider if glycemia control poor despite meds and lifestyle changes

What are the categories in the Bethesda criteria for Thyroid and their respective risks of cancer?

Based on FNAC findings, there are 6 grades: 1) Non-diagnostic or unsatisfactory (5-10, 10%) 2) Benign (0-3%) 3) Atypia of undetermined significance (10-30, 30%) 4) Follicular neoplasm or suspicious of follicular neoplasm (20-40, 40%) 5) Suspicious for malignancy (50-75, 50%) 6) Malignancy (97-99%)

What are the differentials for lumps in hand?

Based on anatomical distribution: 1) Skin: lipoma, sebaceous cyst, implantation dermoid, pyogenic granuloma, glomus tumor, SCC, melanoma 2) Muscle: hematoma, rhabdomyoma, myosarcoma 3) Tendon sheath/joint synovium: ganglion cyst, PVNS (diffuse and nodular tenosynovitis) 4) Tendon: trigger finger (sclerosing tenosynovitis) 5) Nerves: neuroma, neurofibroma, neurilemomma 6) Vessels: AV malfomation, hemangioma 7) Bone: enchondroma, osteochondroma, callus ALSO OA, RA, Gout and tophi

Depends on symptoms, tophi and achievement of target serum urate levels. 1) At least 6 months OR 2) Asymptomatic WITH tophi: 6 months after achieving target urate levels 3) Asymptomatic WITHOUT Tophi: 3 months after achieving target urate levels 4) No resolution of symptoms - continue indefinitely

Based on possible outcomes, what is the management pathway of prophylaxis of gout arthropathy?

How would you manage a Galeazzi fracture

Basically same as Monteggia 1) Non-operative - Closed reduction with casting in supination - Indicated more often in children (much higher rates of success) 2) Operative - Indicated for all cases in adults - Early ORIF of RADIUS with stabilisation of DRUJ - Must achieve anatomic reduction of DRUJ

What are the risk factors of osteochondroitis dissecans?

Being a traumatised male. (Story of my life) 1) Male (15-20y/o) 2) Trauma - single impact with edge of patellar - repeated contact with adjacent tibia ridge.

What are the 1' cancers of the heart?

Benign: 1) Myoxma (most common) 2) Lipomas 3) Fibromas Malignant: 1) Rhabdomyocarsoma - more in infants, fatally outgrows heart

Which is more potent: Betamethasone valerate 0.1% or betamethasone dipropionate 0.05%

Betamethasone dipropionate 0.05% is STRONGER despite having an apparently lower concentration.

When is Frozen section of the thyroid indicated? On the bestheda scoring?

Bethesda I, III (AUS) and V II - benign, very low risk hence no need III - FLUS no need because follicular cannot tell conclusively IV - follicular lesion hence frozen section is insufficient to rule out cancer VI - 99% chance of cancer, higher accuracy of FNAC than frozen section, thus no point to do.

What analgesia would you give for a 12 year old child in manipulation and reduction?

Bier's block: 1) Insert IV catheter to affected limb 2) Insert IV catheter to opposite arm for emergency use with fluids running 3) Place a working tourniquet to affected limb after checking that it works, inflate to 100mmHg above systolic BP. 4) Confirm absence of peripheral pulse (radial or dorsalis pedis) 5) Inject up to 21ml of 1% lignocaine (3mg/kg) for 70kg male 6) Wait ~ 5 minutes for onset of anesthesia 7) Observe patient for signs of LAST: perioral numbness, slurred speech, altered mental state, seizures. 8) When procedure is complete, leave tourniquet on for 20-25 minutes before deflating it.

What method is used for reduction of posterior hip dislocations and how is it done?

Bigelow's maneuver 1) Patient lies supine 2) Physician wraps arm under flexed knee to apply axial traction 3) Assistant applies downward pressure at hip for counter traction 4) Physician externally rotates the hip and extends it (since deformity of injury is internal rotation and flexion) Allis maneuvre 1) Patient lies supine 2) With assistant applying downward stabilising pressure on the pelvis 3) Physician flexes hip and knee to 90degrees and pulls upwards Both are done under GA in OT.

How would you differentiate a bipartite patella from a patella fracture?

Bipartite/Patella fracture - bilateral in 50%/ usually unilateral - rounded edges/ straight edges - usually in superolateral region/ varies depending on mechanism of injury

How would you counsel a patient taking bisphosphonates?

Bisphosphonates are associated with increased risk of reflux and esophagitis. Thus ensure they: - Take on an empty stomach, in the morning - Sit upright - Drink about 250ml (one glass) of water after taking the meds - Remain upright after consumption

What is the resolution criteria of DKA?

Blood sugar levels less that 11.1mmol/L AND 2 or more of the following 1) pH > 7.3 2) Bicarb > 18mmol/L (N 22-28) 3) Anion gap < 10

What organisms cause blood diarrhoea and which ones do not? What is their pathogenesis?

Bloody - Campylobacter (endotoxins and extracellular toxins) - Salmonella (direct invasion?) - Shigella spp (direct invasion and shiga toxin) - EHEC (shiga toxin production) - EIEC (invasion) - EPEC (invasion) Non Bloody - Salmonella (multiply and direct damage) - ETEC (shiga-like toxin) - Vibrio (enterotoxin that stimulates adenyl cyclase -> ater and sodium excretion)

What is inflammatory diarrhoea and what are the causes?

Bloody diarrhoea with white blood cells 1) Infective - Campylobacter - Shigella spp - Salmonella spp - ETEC - Clostridum difficile 2) Inflammatory bowel disease

Why is H pylori found in the antrum and not the fundus/body?

Body has parietal cells that secrete acid and intrinsic factor, the former which damages H pylori. Thus it hides in the antrum pits below an alkaline mucus layer of it's own making.

What are some locations to obtain bone for grafting?

Bone grafts can be autologous or cadaveric Autologus: - Illiac crest (often more painful than the grafting itself) - Femur, tibia (take from medullary and add BMP)

What are the causes of rigid pes planus?

Bone: tarsal coalition Neuromuscular: cerebral palsy Ligament: connective tissue disease (ehler danlos, marfans) Joints: inflammatory joint disease

What are the clinical signs of hypercalcemia?

Bones, Stones, tummy Groans and stupid Moans. Bones - painful bone condition e.g osteitis fibrosa cystica Stones - renal stones +/- 2' renal failure Groans - abdominal symptoms: N&V, Constipation and indigestion Moans - psychiatric effects: lethargy, fatigue, memory loss, psychosis and depression

What antibiotics should one give if the resistance of a staphyloccocus aureus are unknown?

Both vancomycin and cloxacillin Vancomycin for MRSA coverage Cloxacillin for MSSA since vancomycin coverage for MSSA limited.

Which gender is more likely to have transient synovitis? At what age?

Boys!! (2:1) Especially those of the ages of 3-10 years old.

What is BIRADS and it's implication?

Breast Imaging Reporting and Data System for both U/S and Mammogram 0: Need more information 1: Normal 2: Benign 3: Probably benign 4: Highly suspicious 5: Malignant 6: Known cancer - 0: futher imaging studies - 1 and 2: routine screening mammography - 3 and above need to do something e.g short term follow up or biopsy - 4 and above definitely biopsy (also applies for anything palpable)

What are the bullae flaccid or tense in bullous pemphigoid and pemphigus vulgaris? How come pemphigus fiolaceous does not have bullae?

Bullous pemphigoid - Due to subepidermal seperation of skin layers - Autoimmune destruction of dermo-epidermal junctional adhesion proteins Pemphigus vulgaris - Due to intraepidermal seperation Pemphigus fiolaceous - Seperation at subcorneal layer, not thick enough to retain fluids.

What is the difference between a burst fracture and chance fracture? What other vertebral fractures are possible?

Burst fracture - axial compression fracture with retropulsion of posteriorsuperior fragment Chance fracture - flexion distraction fracture with a transverse fracture line through the vertebral body or disc and ligaments. (stable if bony, unstable if ligamentous) Both have disruption of the posterior body but one is compression, one is distraction. Others e.g - Compression fracture (flattening but +/- retropulsion) - Flexion distaction (may be bony or ligamentous) - Translation (ouch)

What is the CEAP classification of chronic venous insufficiency?

C for clinical class, depending on characteristics: 0 = no findings 1 = telangiectasia/ reticular veins 2 = varicosities 3 = edema (from venous etiology) 4a = pigmentation/ eczema 4b = lipodermatosclerosis 5 = healed ulceration 6 = active ulcer E for etiology: C = congenital S = secondary P = primary A for anatomy: S = superficial P = perforator D = deep P for pathophysiology: R = reflux O = obstruction RO = both N = no evident disease

What is C-Kit and what does it imply in histological studies?

C-kit/CD117 is a proto-oncogene: a tyrosine kinase receptor responsible for cellular signal transduction for purposes of cell proliferation, survival and differentiation. It's over-expression is associated with GIST development. Non C-kit GISTs (small subset) are due to PDGFRA mutations (C kit negative, leiomyosarcomas etc are also C-kit negative)

What are the nerve roots involved in Erb's palsy?

C5-6

For myotomal testing in the ASIA score, what are the muscles from C5 to T1?

C5: Elbow flexors C6: Wrist extensors C7: Elbow extensors C8: Finger flexions T1: Finger ABDuctors

Which nerve roots are affected in Horner's syndrome?

C8 and T1

What are the pathogens responsible for community acquired vs hospital acquired pneumonia vs ventilator associated pneumonia and how does it guide empirical antibiotic therapy?

CAP 1) Strep pneumoniae 2) H.influenzae 3) Staphylococcus aureas 4) Influenza 5) PAE Treat with IV Augmentin 1.2 8hrly (IV Ceftriaxone 2g 24hrly if allergic) with PO clarithromycin 500mg 12 hrly for 7 days (14 days if PAE, A.baumannii or S.aureaus) HAP (48hr or more after admission, not incubating on admission), VAP (48-72hr after intubation) - MRSA - PAE - ESBL enterbacteriacea - Actinetobacter spp - Stenotrophomonas maltophilia Treat w IV Pip-tazo 4.5g 8hrly (IV ceftazidime 2g 8hrly WITH IV amikacin 15mg/kg STAT if allergic) with IV vancomycin 15mg/kg 12 hrly 7 days (14 days if PAE, A.baumannii or S.aureaus)

What is the treatment of osteogenesis imperfecta?

CARE IS MULTIDISCPLINARY - OT - Speech Therapist - PT - Nutritionist - Audiologist - Orthopedic - NES - Respi - Renal - Genetic counselling 1) Conservative: - Walking aids - Orthotic supports (for stability and prevention of deformity progression) - Physiotherapy - Diet: increase calcium, phosphorus and Vit D 2) Pharmacological: - IV Pamidronate (Bisphosphonate) 3) Surgery: - Bone: Intramedullary rod placement/pins/Kwires, corrective osteotomies, scoliosis surgery for painful/complicated deformities to stabilise - Soft tissue: LL contractures e.g achillies tendon

Ischemi

CArdioembolic vs large artery

What are the tumor markers to test for cholangiocarcinoma?

CEA and CA 19-9

What are the causes of pes planus?

CHILDREN 1) Flexible/dynamic - Generalized ligamentous laxity (all children have till about 7y/o) - Tight tendoachillies (bone growth exceeds muscle lengthening, tendoachillies causing eversion and elimination of arch) - Connective tissue disorder e.g Marfans - Differentiate with Jack's test (great toe flexion) and tip toe test to observe for arch 2) Fixed - Tarsal coalition +/- peroneal spasm - Inflammatory joint condition - Neuromuscular condition e.g cerebral palsy - Idiopathic ADULT - Inflammatory arthritis (RA) - Muscular: Generalised muscular weakness - Posterior tendon dysnfunction: synovitis or rupture (2' trauma/repetitive impact in sports)

What is the pathogenesis of Charcot Marie Tooth?

CMT AKA peroneal muscular atrophy is: - Hereditary motor and sensory neuropathy - Can be AR, AD or X-linked

What are the differences in presenting complaints of acute vs latent CMV infections?

CMV Infection Vs CMV disease 1) CMV Infection: - Isolation of virus or detection of viral proteins or nucleic acids 2) CMV Disease: - Evidence of CMV infection with symptoms and signs - Can manifest as : o Viral syndrome (fever, malaise, leukopenia, neutropenia, atypical lymphocytosis) o Tissue invasive disease

What are the differences in presenting complaints of acute vs latent CMV infections?

CMV Infection Vs CMV disease 1) CMV Infection: - Isolation of virus or detection of viral proteins or nucleic acids 2) CMV Disease: - Evidence of CMV infection with symptoms and signs - Can manifest as : o Viral syndrome (fever, malaise, leukopenia, neutropenia, atypical lymphocytosis) o Tissue invasive disease - Pneumonitis - GIT - Hepatitis - CNS - Nephritis - Myocarditis - Pancreatitis etc

What investigations should you order in suspected multiple myeloma?

CRAB 1) Calcium - Ca levels 2) Renal failure - Urine protein electrophoresis (bence jones protein) 3) Anemia - FBC (microcytic hypochromic) - Serum electrophoresis for monoclonal band - ESR - Peripheral blood smear (for rouleux stacking of RBC) - Serum B2 microglobulin 4) Bone lesions - Skeletal survey - Bone marrow aspirate

How does one use CRITOE to judge a patient's age?

CRITOE guess the APPEARANCE of ossification centres of each site-> once ossification seen, age reached.

What is the sequence of elbow ossification?

CRITOL Capitulum Radial head Internal epicondyle Trochlear Olecranon Lateral epicondyle

What is the Bosniak classification and how might it guide management?

CT classification of renal cysts that estimate the percentage of malignant cells. 1: Simple cyst (imperceptible wall, rounded), 0% malignancy 2: Minimally complex cyst (<1mm septa, calcification, minimal thickening of wall. 3: Measurable enhancement (cystic mass with thickened irregular 3 and 4 will offer partial nephrectomy.

What diagnostic tests can you do for lung cancer?

CT guided needle biopsy Histological: o Architecture o Receptor status

What is the utility of CT in suspected stroke patients?

CT helps in answering 3 questions: 1) Stroke or not - Vascular vs mass lesions/abscess 2) Ischemic or haemorrhagic - 50% of ischemic strokes have no evidence in 24 hours-1 week (may have hyperdense clots visible) 3) Severity/Complications - Midline shift/raised ICP - Regions involved (ASPECTS)

When is chemotherapy indicated in breast CA?

CT may be indicated as neoadjuvant therapy, adjuvant therapy and palliative therapy. 1) Neoadjuvant therapy - Reduce tumor size before resection to: - Ensure adequate skin closure (locally advanced tumor i.e stage 3) - Improve margins in breast conserving surgery 2) Adjuvant therapy - To reduce micrometasteses and recurrence rate after breast conserving surgery 3) Palliative - Reduce disease load to allieviate symptoms and increase survival

What is the management of diverticular abscess?

CT/US guided per-cutaneous drainage

What would you look for on CT for acute diverticulitis?

CTAP with contrast - Additional diverticulosis - Signs of colitis: o mesenteric fat infiltration, concentric bowel thickening - Phlegmon/abscess - Free gas - Segmental bowel thickening - Spill of bowel feces into abdominal cavity

What is the pathophysiology of chronic venous insufficiency?

CVI results from ambulant venous hypertension that stems from a combination of 3 factors: 1) Obstruction - e.g intra-abdominal/ pelvic mass, DVT 2) Valvular dysfunction - e.g varicose veins 3) Venous pump failure - muscle + valve failure - e.g stroke/muscular weakness

What are the causes of torticollis?

Can be Orthopedic (congenital or acquired), and Non orthopedic ORTHOPEDIC 1) Congenital - Congenital muscular torticollis (Most common cause) - Klippel Feil syndrome (short neck, low hairline, restricted neck motion 2' fusion of 2 or more C spine vertebrae) 2) Acquired - Atlantoaxial rotatory subluxation (post traumatic, post infectious i.e Grisel's disease, painful!!) - Epidural hematoma - Myositis NON ORTHOPEDIC 1) Eye (ocular torticolis) - Occulumotor nerve palsies - Strabismus 2) CNS/PNS - Syringomyelia - Dystonic syndromes 3) Infections - Lymphadenitis - Peritonsillar, retropharyngeal abscess - Epidural abscess 4) Drugs - Neuroleptic drugs e.g antipsychotics, antiemetics etc

How would you classify disc prolapse?

Can be classified by location and anatomy Location classification - Central - Posterolateral (most common, where posterior longitudinal ligament is weakest, affects the traversing/descending/lower nerve root e.g L4/5 affects L5) - Foraminal Anatomic: - Protrusion (eccentric bulging with intact annulus) - Extrusion (disc materal herniates through annulus but remains continguous with disc space) - Sequestrated fragment (disc material herniates through annulus and is no longer continuous with disc space)

What are the possible causes of glomerular disease?

Can be classified into pathophysiology. 1) Circulating IC deposition - Lupus - IgA - Hepatitis viral related - Post-infectious 2) Pauci immune - ANCA related 3) De-novo antibody binding - Anti PLA2 - Anti GBM 4) Single Ab deposition - Monoclonal Ig deposition - C3 glomerulopathy

What are the causes of pyloric stenosis?

Can be congenital or acquired Congenital: 1) Infantile hypertrophic pyloric stenosis: - M:F 4-6:1 Acquired: 1) Ulcerative - PUD involving pylorus - Chronic gastritis 2) Neoplastic - Carcinoma arising at pylorus (intestinal adenoCA, signet ring cell CA, GIST) - 2' obstruction: - HOP CA - Gastric lymphoma 3) Autoimmune: - Autoimmune gastritis

How are subarachnoid haemorrhages managed?

Can be conservative or surgical 1) Conservative (to reduce ICP) - Sitting up 30 degrees - Infusion of IV mannitol @ 0.25g/kg - Intubation w hyperventilation with pCO2 30-40mmHg 2) Surgery for AVM/aneurysm - Endovascular coiling - Clipping

What types of traction are there?

Can be divided according to methods and mechanisms METHODS: 1) Manual - Me pulling 2) Skin - Traction force applied over large area of skin - Adhesive vs Non-adhesive 3)Skeletal traction - Applied to bone directly via pin/wire - Steinmann pin, denham pin, kirschner wire (Kwire) MECHANISMS 1) Fixed - force is applied to fixed point of body - i.e something is placed against the body to provide counter traction - e.g Thomas splint 2) Sliding traction - Body is used as weight - Hamilton Russel traction or Perkins traction

What are the options for definitive treatment of acute limb ischemia?

Can be divided into endovascular or open. 1) Endovascular - Mechanical - Thrombectomy - Embolectomy (under LA via fogarty catheter) - Angioplasty +/- stenting - Chemical - Thrombolysis (via urokinase/streptokinase/rTPA) 2) Open - Endartrectomy - Bypass 3) Hybrid - Mix of multiple methods

What are the various types of breast cancer?

Can be divided into epithelial Vs non-epithelial tumors (WHO classification) 1) Non-epithelial: arising from supporting stroma (very uncommon) - Angiosarcoma - Malignant phyllodes tumor - Primary sarcomas 2) Epithelial tumors - Ductal: DCIS and invasive (IDC) - Lobular: LCIS and ILC - Others: - Specialised: medulalry/ colloid/ tubular/ papillary - Inflammatory carcinoma

What are the classifications of implant infection? What are the common presentations?

Can be divided temporally 1) Early <3 months - Due to Surgical innoculation - e.g S.Aureus 2) Delayed 3-24 months - More indolent pathogens - CNS staph, propionibacterium (esp if above shoulders) 3) Late >24 months - Hematogenous spread from extra-articular spread.

What are the causes of clubbed feet and how would you treat them?

Can be flexible or rigid 1) Flexible - Often due to intrauterine positioning - Can be treated with physiotherapy and stretching 2) Rigid - Mostly idiopathic - Can be neuromuscular/paralytic (CP, spina bifida w S1 palsy), syndromic (distal type arthrogryposis) - Treatment: o Conservative (only if within 1-2 days of birth), via manipulation and serial ponsetti (ankle to groin) casting followed with aBDuction orthoses, manipulation and reduction in order of CAVE, then ABDuction orthoses (Dennis Browne boots etc) o Surgical (only for resistance cases) - Release of capsular/ligamentous contractures - Tendon lengthening - Reduction and percutaneous pinning with casting - OR corrective osteotomies and fusions

What are the indications and complications of catheter-based angiography?

Can be for diagnostic and therapeutic purposes. 1) Anesthesia related - LA used peripherally - LAST 2) Contrast related - Allergy - Nephropathy (72 hrs later) - Asthma induction - Metformin (metabolic acidosis) - Cutaneous reaction 3) Procedure related o On insertion - Bleeding - Injury to surrounding structures e.g nerves o While in vessel - Catheter tip clot -> need to operate - Vessel rupture -> internal bleeding - Endothelial dissection o On removal - Pseudoaneurysm (from poor compression) -> hypotension, pain

What pathogens and conditions can cause brain abscesses?

Can be hematogenous spread or direct 1) Hematogenous - Lungs (lung infections) - GIT (intra-abdominal sepsis, pelvic abscess, esophageal variceal dilation) - Cardiac (IE, right to left shunt) - Skin infections 2) Direct - Ethmoidal sinus infection - Dental infections - Mastoiditis/otitis media Pathogens: - Strep (*S.viridans) - Staph (*S.aureaus) - Ears: PAE, bacteroides - Lungs: Fusobacterium, actinomyces, - Penetrating head trauma: clostridium, enterobacter - Immunocompromised: toxoplasma, listeria, fungal infections - Immigrants: parasites

What are the contributing factors to acne?

Can be intrinsic or extrinsic 1) Intrinsic o Genetic o Hormonal: - Androgens increase, estrogen decrease acne. - Levels of sex hormone binding globulin (SHBG) affect circulating testosterone - Sensitivity of pilosebaceous glands also differ - Idiopathic/disorders e.g Cushings, CAHyperplasia, PCOS o Fluid retension - Increased hydration and swelling of duct o Diet - Controversial - Mainly high glycemic index and dairy 2) Extrinsic - Oils and tars (oil foliculitis, common in thighs and lower arms) - Chloracne (due to halogenated hydrocardons -> multiple comedomes +/- inflamamtory lesions behind ears, axilla and groin) - Acne cosmetica - Drugs (corticosteroids: perioral dermatitis, antiepileptics, anti TB isoniazid and lithium)

What are the causes of scoliosis?

Can be postural or structural 1) Postural - Due to secondary/compensatory tilt to a condition outside of the spine - e.g short leg, pelvic tilt in hip contracture, muscle spasm in PID etc - Curve disappears when patient sits down 2) Structural scoliosis (I MOANS) - Fixed deformity that does no disapear with sitting down o Infantile - < 4y/o onset - may resolve o Myopathic - muscular dystrophy, SMA o Osteopathic - congenital vertebral anomalies e.g hemivertebrae, wedge vertebrae, block vertebrae o Adolescent** (Most common) - >/= 10y/o onset idiopathic o Neuropathic - due to assymetrical muscle weakness e.g CP, polio, myelomeningocele o Syndromic - Due to ehler danlos, marfans, NF, VECTERL

What are the causes of varicose veins?

Can be primary or secondary 1) Primary - unknown, genetic (20% risk inf gen pop, 50% if 1 parent, 80% if both parents) 2) Secondary - Post DVT valve thrombosis - proximal venous obstruction - AVF (increases flow pressure) - Pelvic masses

What are the causes of frozen shoulder?

Can be primary or secondary 1) Primary - Idiopathic - a/w DM, duputryen's contracture, hyperlipidemia, hyperthryoidism, cardiac disease 2 Secondary (intrinsic/extrinsic) o Intrinsic - Previous trauma, rotator cuff tear, fractures etc - Previous infection e.g septic arthritis o Extrinsic - Prolonged immobilization - Cervical radiculopathy - Post AMI - Post hand/elbow surgery

What are the causes of frozen shoulder?

Can be primary or secondary 1) Primary - Idiopathic in cause - A/w Females in fifties, DM (type 1), duputryen's contracture, HLD, hyperthyroidism, cardiac disease - Results in global contracture of shoulder joint 2) Secondary (Intrinsic vs extrinsic) o Intrinsic - Post traumatic scarring after injury (rotator cuff, proximal humeral fractures) o Extrinsic - Painful disorders limiting RoM (referred pain for Cspine radiculopathy, post hand/wrist/elbow surgery, post MI.

What are the causes of torticollis?

Can be primary/congenital or secondary/acquired. 1) Primary/congenital: - Complicated breech presentation - Skeletal deformities, atlantoaxial instability (e.g in downs) - Klippel feil syndrome (bony fusion of 2 or more C-spine) 2) Seconary/acquired: - Inflammatory muscle damage e.g myositis, lumphadenitis, TB, other infections) - Traumatic injuries causing contracture via scar tissue - CNS condition causing spasms e.g nerve root irritation via arthritis, OM, PID - Benign paroxysmal torticolis - Ocular torticolis (compensation due to superior oblique or trochlear nerve palsy) - Drug induced (part of dystonia) - Others: hysteria, hearing loss, flat head (plaigocephaly)

What are the possible types of pancreatic cancer?

Can be split by location and cell type 1) Location - Uncinate process i.e head of pancreas - Tail and body of pancreas 2) Cell type - Adenocarcinoma - Neuroendocrine tumors: - Insulinoma - Zollinger ellison (Gastrinoma)

What are the causes of hypothyroidism?

Can be split into acute precipitation or long standing conditions 1) Acute precipitation - Infection - Trauma - AMI - Cold exposure - Sedative drugs i.e opiates, BZDs - Stroke 2) Long standing - Autoimmune thyroiditis - Previous thyroid surgery/radi0-iodine tx - Medications (amiodarone: high iodine levels and lithium)

What are the causes of genu valgum?

Can be split into bilateral or unilateral 1) Bilateral o Physiological o Renal osteodystrophy (renal rickets) o Rickett's disease o Skeletal dysplasia - Morquio syndrome - Spondyloepiphyseal dysplasia - Chondroctodermal dysplasia 2) Unilateral o Physeal injury from trauma/infection or vascular insult o Proximal metaphyseal tibia fracture o Benign tumors: - Fibrous dysplasia - Osteochondromas - Ollier's disease (rare nonhereditary sporadic disorder where multiple enchondromas form near growth plates)

What are the possible complications of gastrectomy?

Can be split into early and late 1) Early - Bleeding - Infection - Anastomotic leak - Injury to surrounding structures e.g pancreas 2) Late o Filling - Early satiety - BIliary/intestinal reflux -> dyspepsia o Reservoir - Intestinal hurry -> indigestion o Emptying - Early dumping syndrome (flushed down by intestinal secretions) - Late dumping syndrome (reactive hyperinsulinemia) o Nutritional - Nutritional deficiecny (iron, B12 etc) - Weight loss

What are the indications for a chest tube?

Can be split into elective or emergency Main goal is to prevent fluid/air accumulation within a closed body cavity. E.g Hemothorax from trauma Vs E.g post esophagectomy to watch for leaks

What are the complications of AAA surgery?

Can be split into intraoperative or post-operative 1) Intra-operatively o Anesthesia related - AMI - Stroke o Procedure related - Bleeding - Renal insufficiency - Infection - Damage to surrounding structures - Spinal cord ischemia -Thrombotic - Trash foot 2) Post-operatively o Vessel - Aortoenteric fistula o Graft/stent - Graft infection - Graft dehisence/leak i,e endoleak - Stent migration o Lifestyle - Sexual dysfunction

What are the causes of compartment syndrome?

Can be split into intrinsic and extrinsic 1) Intrinsic - Fractures - Contusions - Surgery - Vascular trauma - Bleeding (coagulopathy) - Reperfusion after ischemia - Burns 2) Extrinsic - Tight dressing - Lying on the limb

How would you treat glomerular disease?

Can be split into non-immunosuppressive and immunosuppressive 1) Non-immunosuppresive o Optimize BP control via RAAS (ACEI or ARB) - Contraindicated if hyperkalemic, poor renal function, pregnancy (teratogenic) o Optimise BMI, metabolic targets o Smoking cessation o Lifestyle changes etc 2) Immunosuppresive o General pattern is induction, maintainence and decision on when to stop. o Offered if heavy proteinuria, neprhotic/nephritic syndrome, AKI, CKD, NOT IN ESRF o Prednisone if: - Bone healthy - PUD low risk + preventive strategies in place - Metabolic issues/hyperglycemia - Infections

What are the management options for venous disease?

Can be split into open and endovascular. 1) Endovascular o Thermal ablation - Radiofrequency - Laser o Non-thermal - MOCA (mechanico-chemical ablation) - Venoseal - Injection sclerotherapy 2) Open (for LSV vs SSV) - SFJ ligation (i.e high tie) + LSV stripping and Multi-step avulsion - SPJ ligation (no stripping because of risk of damage to sural nerve)

What are the possible associated conditions with schuermann's kyphosis?

Can be split into orthopedic or non-orthopedic 1) Orthopedic - Scoliosis (33%) - lumber hyperlordosis - spondylolysis (30-50%) 2) Non-orthopedic - pulmonary issues in curves exceeding 100 degrees

What are the management options for breast cancer?

Can be split into palliative or curative: depending on stage of cancer. RE: Curative 1) Hormonal therapy 2) Chemotherapy 3) Radiotherapy 4) Surgery - Local - Breast conserving - Mastectomy (skin/nipple sparing, radical mastectomy) - Regional - Sentinel LN biopsy - Axillary LN clearance - Systemic - Tumor board - Looks at patient and tumor factors - Sx for cx of mets - Oncotype testing for additional chemotherapy RE: Palliative: 1) RT in brain mets or bone mets to painful areas/impending fracture 2) Chemotherapy for symptom improvement and survivability 3) Surgery: palliative/toilet mastectomy with LN clearance

What are the red flags in orthopedic histories?

Can be split into symptoms and etiologies 1) Etiologies - Recent trauma - Fever/night sweats/chills&rigor - LOW/LOA - Morning stiffness 2) Symptoms - Saddles anesthesia/bowel or bladder incontinence - Rest pain/night pain

What are the causes of interstitial lung disease?

Can be split into upper, lower lobes and both. 1) Upper (CHARTS) - Coal worker pneumoconiosis - Histiocytosis - Ankylosing spondylitis, allergic bronchopulmonary aspergillosis - Radiation - TB - Silicosis, sarcoidosis 2) Lower (I SOAR) - Idiopathic pulmonary fibrosis - Scleroderma - Others I.e drugs - Cytotoxics: - Cytotoxics - MTX, Aza, bleomycin, bulsulphan, cyclo, chlorambucil - CNS - Amitryptyline, phenytoin and carbamazepine - CVS - Amiodarone, hydralazine, procainamide - Antibiotics - Nitrofurantoin, isoniazid - Antirheumatics - Gold, sulphasalazine - Asbestosis - Rheumatoid arthritis 3) Both (NEPAL) - Neurofibromatosis/tuberous sclerosix - Extrinsic allergic alveolitis (within 6 hrs of exposure to inhaled pathogen e.g farmers lungs) - Pulmonary haemorrhage syndromes e.g pulmonary renal syndrome in Goodpastures - Alvoelar proteinosis - Primary - Secondary (malignancy, inhaled organic dusts e.g silica, chronic infection) - Lymphangiomyomatosis (genetic in tuberous sclerosis and sporadic forms, proliferation of lymphoid tissue a

How would you differentiate candidiasis from inverse psoriasis?

Candidiasis comes with satelite lesions and systemic fever.

What are the signs of CMCJ OA?

Carpometacarpal joint OA 1) Squaring of thumb 2) Painful limitation in active RoM 3) Crepitations on movement 4) Grind test +

How would you classify the severity of Perthe's disease and how does it affect prognosis?

Catterall grading: according to degree of femoral head movement 4 grades 1: Involving only anterior portion of epiphyses, with NO collapse or sequestrum (REVASCULARISATION my be complete w/o bone collapse) 2: </= 50% epiphyseal involvement with a sequestrum 3: ~ 75% epiphyseal involvement with collapse and sequestrum 4: ~ Whole epiphyseal involvement Grades 2-4 means bony collapse is inevitable, but 3-4 comes with poor prognosis.

What is the difference between cauda equina and conus medullaris?

Cauda equina - compression of nerve roots beyond the spinal cord (below L1) i.e the horse tail - Presents like a LMN lesion - Areflexia, flaccid paralysis - Asymmetrical - Saddle anesthesia is unilateral and asymmetrical - Presents late and with urinary incontinence Conus medullaris - compression of the tip of the spinal cord (at L1) - Presents like an UMN lesion - Dimished reflexes at level, brisk reflexes below - More pain - Impotence is common - Saddle anesthesia is symmetrical and bilateral. - Presents early with BOTH urinary and fecal incontinence

What are the possible causes and associations of atlantoaxial instability?

Causes 1) Traumatic (flexion distraction forces with rotational component) Associated conditions: 1) Rheumatoid arthritis (40%) 2) Downs syndrome 3) Osteogenesis imperfecta 4) Neurofibromatosis 5) Ligamentous degeneration 6) Metabolic bone disease.

How do steroids cause AVN?

Causes 2 things 1) Increase in circulating lipids - leading to microemboli 2) Increased intravascular coagulation - ischemia

1) Primary: - Idiopathic 2) Secondary: - Prolonged hyperuricemia - Acquired myeloproliferative disease - Renal failure - Diuretics

Causes of Gout

USED CRAP Ureteroenterostomies - Cl in urine passed to colon, exchanged for HCO3 which is lost in stool. Steroids Extra Chloride (iatrogenic, into GIT, exchanged for bicarb) Diarrhoea (dehydration > hypoperfusion w lactic acidosis +/- contraction alkalosis) Carbonic anhydrase inhibitors (inhibit bicarb reabsorption) Renal tubular acidosis Addison's disease (RAAS dysfunction) Pancreatic fistulas (bicarb rich fluid secreted into intestines and lost)

Causes of NAGMA

1) Parkinson's disease 2) Parkinson plus syndromes 3) Drugs (neuroleptics, antiemetics, MPTP: 1 methyl 4 phenyl 1,2,3,6 tetrahydropyridine - a prodrug of a neurotoxin that damages the substantia nigra) 4) Anoxic brain damage 5) Post encephalitis 6) Tumors e.g giant frontal meningiomas

Causes of Parkinsonism? (6)

Causes -Infections Viral Bacterial Fungal Parasitic -Neoplastic Solid tumors Leukemia Lymphoma -Inflammatory/autoimmune SLE RA Polyarteritis nodosa GCA Polymyalgia rheumatica -Endocrine Hyperthyroidism Thyroiditis Adrenocortical insufficiency (1o: Addison's disease, 2o: head injury, Sheehan's) -Vascular Lesions Stroke! Intracranial bleeding SAH Pulmonary embolism -Drugs Abx (penicillins, sulfonamides) Anti-convulsants (carbamazepine, phenytoin) NSAIDS (salicylates, ibuprofen) Anticancer (bleomycin) Others (barbiturates, cimetidine)

Causes of fever

Spurious: - Drip arm - venous glucose (uncommon, usually CBG) - Wet fingers/alcohol rub - Gross anasarca (interstitial fluid squeezed out LOL) - Glucometer/strips issue (expired/spoil) Drugs: - Overdose OHGA/lipodystrophy w erratic insulin absorption/ miscalculation - Alcohol - Power one walnut Nutritional: - Poor intake Organ failure: - Sepsis - Acute Liver failure - Worsening renal failure: poor insulin clearance - Hypocorticolism/adrenal insufficiency - Hypopituitarism - Pancreatitis Neoplasm: - Insulinoma

Causes of hypoglycemia?

What are the causes of Fournier's gangrene and how is it treated?

Causes: - Trauma - Immunocompromisation - Scrotal abscess Treat with - Broad spectrum IV abx - Surgical wide debreidement - Treat underlying cause

What are the antibiotics that cause clostridium difficile? And what are the antibiotics that can treat a C.Diff infection?

Causes: fluroquinolones > cephalosporins > penicillins > clindamycin Treatment: Oral vancomycin > oral metronidazole

What are the caveats to courvoisier's law?

Caveats to courvisiers law 1) Between episodes of colic 2) Chronic cholecystitis due to stone but 2nd mucocele formation with one of the stones falling out and obstructing the CBD (as cholecystitis the gall bladder is usually constricted) 3) Recurrent pyogenic cholangitis 4) Mirrizi's syndrome

What is the antibiotic treatment of meloidosis?

Ceftazidime OR Carbapanems (meropenem and imipenem)

What is the treatment of chacroid ulcers?

Ceftriaxone 250mg IM x1 dose or Azithromycin 1g PO x 1 dose

What factors affect the radius of cerebral blood vessels

Cerebral metabolism CO2 and O2 Autoregulation

What are the differences between the cervical vertebrae, thoracic and lumbar vertebrae?

Cervical vs Thoracic Vs Lumbar 1) Body - Cervical has smallest body - Lumbar has largest body 2) Articulating joints - Cervical has uncovertebral joints, superior ones directed posterior superiorly/ inferior ones directed anterioinferiorly - Thoracic intervertebral joints: superior ones directed posteriolaterally and inferior ones are directed anteromedially, ALSO has rib articulating surfaces - Lumbar intervertebral joints: superior ones are directed medially and inferior ones are directed laterally 3) Processes - Cervical has small bifid spinous processes - Thoracic has larger, longer and inferiorly projecting spinous processes - Lumbar has shorted and blunter but posteriorly projecting spinous processes, also has transverse processes.

What is the difference between cervarix and gardasil?

Cevarix: bivalent with 16 and 18 Gardasil: quadravalent with 6, 11, 16 and 18 Both are recombinant vaccines

What is Foucher's sign? What is the underlying mechanism?

Change in pressure of Baker's cyst with extension and flexion of the knee, also visible on echography, arthography and computed tomography. In extension, gastrocnemius and semimembranosus oppose each other and the joint space, tending the out-pouching cyst.

What are the physical findings in a neonate with congenital diaphragmatic hernia?

Chest: - Barrel shaped chest - Assymetrical chest rise - Loss of breath sounds on ipsilateral side - Displaced apex beat Abdomen: -Scaphoid-appearing abdomen

What is the most common cause of PUO in children?

Children: self-limiting viral illnesses

What is frozen shoulder and what is it's pathophysiology?

Chronic inflammation involving entire cuff and joint capsule, characterized by progressive pain and stiffness which resolves spontaneously after 18 months. AKA Adhesive capsulitis

What are the clinical signs of hypocalcemia?

Chvostek sign - perioral spasms when percussed Trousseau's sign - carpopedal spasm when blood pressure cuff inflated 20mmHg above SBP

What are the features of drug urticaria? What causes it? How would you treat it?

Circumscribed, erythematous wheals, rapid onset after drug administration (type 1 hypersensitivity reaction: mast cell degranulation via IgE or non-immunological pathways) Resolves within 24 hours Penicillin (IgE) NSAIDs (prostaglandin, leukotriene pathway) Mx: 1) Assess haemodynamic status, angioedema (TRO anaphylaxis) 2) Withdraw suspected drug 3) Antihistamines (clear in a few days)

What are the stages for loss of function in orthopedics?

Class 1: normal function Class 2: Normal function with discomfort Class 3: Limited function, only a few tasks of work/self care Class 4: Almost/complete incapacitation

What are the clinical signs and complications of penile fracture?

Classically an 'eggplant deformity', occuring after a 'crack' in trauma/sexual misadventure Complications: - Urethral injury - Erectile dysfunction - Penile deformity

What are the clinical features of rheumatoid arthritis?

Classically, it is a bilateral, symmetrical and deforming polyarthropathy. Often affecting the: 1) Small joints of the hands o Fingers - Z thumb - Swan neck deformitiy - Boutonniere's deformity - Bouchards nodes - Ulnar drift o MCPJ - Volar subluxation w prominent metacarpal heads o Wrist - Radial tilt - Volar subluxation - Pain key sign in prominent ulnar head - Tenosynovitis: carpal tunnel, trigger finger, tendon rupture 2) Elbows - Limited extension 3) Shoulders - Limited abduction 4) Lower limbs o Knees - Fixed flexion deformity - Valgus deformity o Toes - Clawed toes Subcutaneous rheumatoid nodules over joints - firm to touch like an unripe fruit.

How do you classify PCl tears?

Classification based on posterior subluxation of tibia relative to femoral condyles (with knee in 90deg flexion) Grade 1: - 1 to 5mm posterior tibial translation - tibia still anterior to femoral condyles Grade 2: - 6 to 10mm posterior tibial translation - complete injury in which the anterior tibia is flush with the femoral condyles Grade 3: - > 10mm posterior tibial translation - tibia is posterior to the femoral condyles and often a/w ACL tear and or PLC injuries

What is the Salter Harris classification?

Classification for growth plate fractures I: fracture involving growth plate II: fracture involving growth plate and metaphysis (most common) III: fracture involving growth plate and epiphysis IV: fracture involving growth plate, metaphysis and epiphysis V: crush injury of growth plate

What is the Borrmann's classification?

Classification of Gastric cancer morphology, split into 4 grades. I - non ulcerating, polypoid and intraluminal growth II - ulcerated, circumscribed with sharp margins III - uclerated with blur margins IV - diffuse, infiltrating +/- linitis plastica +/- ulceration

What is the difference between Clergyman's knee and Housemaid's knee?

Clergyman's knee/Housemaid's knee - Infrapatellar bursitis/ prepatellar bursitis - Due to kneeling for long duration/ due to repeated kneeing and standing: friction between skin and patellar - Found below the patellar/ found superficial to the patellar.

What is the most common antibiotic cause of C.diff?

Clindamycin

What are the clinical and radiographic features of osteomyelitis?

Clinical features (signs and symptoms) 1) Symptoms: - Joint pain - Fever (more common in acute OM) 2) Signs - Erythema, effusion, tenderness - limping, antalgic gait - joint tenderness Radiographic features: 1) Lytic lesions surrounded by area of sclerosis - may mimic a neoplastic process 2) In chronic OM - +/- sequestrum: small piece of dead bone surrounded by healthy bone - +/- involucrum: thick sheath of periosteal new bone surrounding a sequestrum - +/- cloaca: single or multiple openings in involucrum due to rupture from pus under tension.

1) Angina - 5 years 2) Syncope - 3 years 3) Dyspnoea - 2 years

Clinical manifestations of Aortic stenosis and their respective prognoses.

Pronator drift is slow pronation of the forearm 1) Contralateral pyrimidial tract (corticospinal/corticobulbar i.e voluntary muscle movement) lesion, where the pronator muscles are stronger than the supinators->pronation +/- downward drift of arm 2) Ipsilateral dorsal column or cerebellar lesion where propioception is affected which presents with an upward drift of the arm as well.

Clinical presentation of pronator drift and it's pathological implication.

Whipple's Triad: 1) Typical symptoms (sympathetic drive, neurohypoglycopenia (dont forget seizures/coma)) 2) Proven low BSL (<2.2-2.6 mmol/L in non diabetics, <4.0 mmol/L if diabetic), generally investigate if <3 mmol/L 3) Both improves with ingestion of glucose

Clinical signs of hypoglycemia

What are the signs of severity in mitral regurgitation?

Clinical: 1) Significant apex beat deviation 2) 3rd heart sound (due to rapid ventricular filling instead of the usual LV dysfunction or increased filling pressure; due to high volume retained in left atrium from regurgitation) Investigations: 1) TTE w color doppler to quantify degree of regurgitation 2) Reduced right ventricular ejection fraction

What are the signs of tension pneumothorax?

Clinically - Dyspnoea - Hypotension - Tachycardic - Distended jugular veins (from RVF) - Unequal chest expansion and absent breath sounds.

How would you follow up for patients after colorectal CA surgery?

Clinically: - 3/12 for first 2 years - 6/12 for next 3 years - 12/12 after - CEA at each visit if CEA raised previously Colonoscopy: - Yearly Others: CXR and U/S TRO mets

Please rank common steroids used according to their potency.

Clobetasol Betametasone Mometasone Triamcinolone Fluocinolone Desonide Hydrocortisone Ointment > cream > lotion 'CB Man Terrorizes Foreign Domestic Helper'

What is a Colles Fracture and how does one reduce it?

Colles fracture is a distal radius fracture (within 2.5cm of wrist) with dorsal angulation, dorsal displacement, radial shift and radial deviation, resulting in a 'Dinner fork appearance'. 90% of distal radius fractures, due to a fall onto a hyper-extended, radially deviated wrist with the forearm in pronation. Hence reduction involves: 1) Exaggeration of deformity via hyperextension of distal radius 2) Traction 3) Volar tilt (i.e palmar) 4) Ulnar deviation 5) Application of backslab from below elbow to metacarpal necks 6) Check Xray after for reduction + assess N&V Otherwise, elevate arm for next 1-2 days, start shoulder, elbow and finger exercises immediately Recheck XR in 10 days Remove backslab in 5 weeks

What is the treatment of acne with regard to the types of lesions present? (

Comedones: - Topical retinoids Inflammatory lesions - Benzoyl peroxide (2.5, 5, 10) - Topical abx - Oral Abx Nodulocystic acne - Oral Abx - Oral isotretinoin

How is carbimazole and propylthiouracil dosed?

Comes in 3 quantities: 30 mg OM, 20 mg OM and 5-10 mg OM. Severe hyperthyroidism i.e young, fT4 > 60, large goitre = 30 mg OM Moderate i.e middle aged, fT4 30-60, = 20 mg OM Mild i.e fT4 < 30, small goitre, very old = 5-10mg OM Propylthiouracil is about 30x the dose of carbimazole i.e 30 mg OM = PTU 100mg TDS

What is fascioscapulohumeral dystrophy?

Common AD muscular dystrophy that results in degeneration of muscles in upper body.

What is the pathogenesis of seborrhoeic keratosis?

Commonest benign growth in elderly due to proliferation of epidermal cells. No identifiable cause found.

What radiographs are recommended in evaluation of proximal humeral fractures?

Complete trauma series 1) True AP 2) Y scapular 3) Axillary

HHHIII (from nephrotic syndrome) Cardiovascular events Infections Malignancies Anemia Mineral bone disease

Complications of Chronic Kidney Disease.

1) Valve dehiscence/valve failure 2) Valve stenosis 3) Infection of valve 4) Hemolysis from shear forces - presenting as anemia and jaundice 5) Anticoagulation: overwarfarinisation or inadequate -> valve thrombosis

Complications of Prosthetic valve

What are the complications of HSP? How would you treat it?

Complications: Long term f/u w rheumatologist, systemic treatment required.

What are the causes of radiculopathy?

Compression of nerve root due to: 1) Spinal - Lateral canal (ligamentum flavum, PID, facet joint hypertrophy, facet joint degeration) - Foramen (OA, degeneration) 2) Extraspinal compressive radiculopathies - Neoplasms - Hematoma - Vascular aneurysms - Neurilemmoma

What are the signs of posterior circulation stroke? What is the pathogenesis?

Comprises of brain stem/cereballar dysfunction or an isolated occipital lobe deficit. 1) Brain stem - ipsilateral CN III to XII palsy with contralateral motor and/or sensory deficit - Bilateral motor and or brainstem deficit - Disorder of conjugate eye movement 2) Cerebellum - Nystagmus, explosive speech, dizziness (can be seen in anterior circulation too), dysdiadochokinesia, pendulum knee reflex etc 3) Occipital lobe - Isolated homonymous hemianopia/quadranopia (can be seen in anterior circulation too) - Cortical blindness Vertebral artery that forms basilar artery before splitting PCA x2 that supplies the brain stem, cerebellum and occipital lobe.

WTF is a bipartite patella? How common is it and amongst who?

Congenital condition where patellar has 2 ossification centres instead of 1 and the bones fail to undergo fusion after adolescence, only being connected by a synchondrosis. The fragment most commonly occurs in the superolateral corner but can be at the bottom or the sides. Males: Female 9:1

What is the clinical presentation of myopathic lesions and their causes?

Congenital: myotinic dystrophy, inherited muscular dystrophies (duchennes, beckers), central core disease Acquired muscular dystrophies: oculopharyngeal dystrophy Metabolic: hypothyroidism, thyrotoxic periodic paralysis, cushings disease Autoimmune: polymyositis, dermatomyositis, myasthenia gravis Drugs: drug induced myositis (statins, colchicine etc), cushings syndrome, drug related myopathy (zidovudine, ethanol) Infectious: trichinosis

What is the treatment for acute pancreatitis?

Conservative: - ABCs - NBM +/- NGT (prevent pancreatic stimulation) - Aggressive fluid resuscitation + electrolyte replacement Pharmacological: - Anti-emetics - Gastroprotection: PPI (prevent stress ulcer formation) - Antibiotics (only if cholangitic/infected pancreatic necrosis) Support for organ failure: - ARDS (PEEP) - ARF (dialysis and CVP monitoring) - Hypotension (fluid resuscitation and inotropes) Monitoring for complications: - Pseudocyst formation - Acute fluid collections - Pancreatic necrosis (sterile/infected) - Organ failure - Hypo/hyperglycemia Treat underlying cause: - Avoid triggering factors - Gallstone: ERCP, cholecystectomy

How is myotonic dystrophy managed?

Conservative: - Genetic counselling +/- prenatal diagnosis if family plans Pharmacological: - For myotonia: - Phenytoin/ acetazolamide/ mexilitene (non selective voltage gated sodium channel blocker) - For weakness: - No treatment available For complications: - Foot drop: orthoses - Heart blocks: AICD should be considered - Cataract: cataract surgery - Infections: close follow up + treat w appropriate agents

What are the management options of psoriasis?

Conservative: 1) Education + avoidance of aggravating factors 2) Pharmacological therapy: o Topical symptomatic: - Parrafin/ salicylic/ aqeous creams o Topical treatment: - Steroids, coal tar, retinols, dithranol and calcipotriol + UVB therapy o Systemics: Methotrexate and cyclosporin, retinoids, systemic steroids, tacrolimus and mycophenolate mofetil o 2nd line: Biologics - Immunomodulators e.g infliximab and etarnacept

What are the management options Atrial Fibrillation?

Conservative: 1) Reduce stroke risk: - Lifestyle modifications: smoking, diet, alcohol etc - Anticoagulation as guided by CHA2DS2VAsc and HASBLED scoring - Aspirin/ warfarin/ NOAC 2) Rate control - AV blockers: - BBlocker (good for CCF, avoid in COPD/asthma) - Non-dihydropyridine CCB e.g verapamil and diltiazem - Digoxin - Amiodarone (last resort, broad range of action + risk of QTc prolongation) - Surgical: - AVN ablation + pacemaker 3) Rhythm control - More agressive, better for young diseases after failure of rate control and paroxysmal/persistent classes - Cardioversion - Chemical i.e amiodarone - Electrical (if < 48hrs give heparin, if > 48hrs TTE TRO thrombus or 3 weeks pre + 4 weeks post cardiovert anticoagulation)

What is the initial treatment of knee dislocations?

Considered an orthopedic emergency, requiring immediate closed reduction. 1) Ensure ABCs 2) TRO associated open fractures 3) Assess N&V status 4) Attempt closed reduction 5) Re-examine N&V w doppler and PE 6) Splint in 20-30deg flexion 7) Pre and post reduction radiographs

What is post TACE syndrome?

Constellation of symptoms including transaminitis, fever and abdominal pain due to tumor lysis, hypoxia and possibly rupture that causes localised inflammation manifesting systemically.

What is Kasabach-merritt syndrome?

Consuptive coagulopathy and thrombocytopenia in large liver hemangiomas. Due to the rapidly enlarging tumor. Aka hemagioma thrombocytopenia syndrome.

What are the features of pyogenic granuloma?

Contact bleeding, eyrthematous (more)

What are the contraindications for stenting of an obstructed bowel tumor?

Contraindications for stent - Local: rectal tumor, sigmoid/flexural, right sided, extensive/long strictures - Bowel factors: ischemic, perforated bowel - Patient factors: cannot lie flat(e.g pleural effusion, aspiration pneumonia from vomitting, diaphragmatic splinting), haemodynamically unstable

What is the difference between cord compression and myelopathy?

Cord compression = radiological diagnosis Myelopathy = clinical diagnosis comprising of classical symptoms arising from spinal cord compression (hand clumsiness, unstead gait).

What are the 7Cs to ask about in long case?

Course of disease Cause of disease Compliance Control Complications Co-morbidities Cognizance

What are the views of breast mamography and the findings suspicious for malignancy?

Cranio-caudal and mediolateral-oblique. (+ Left and Right breast) 'Go inside to out' - Mass seen w spiculations/architectural distortions - Microcalcifications - Nipple retraction - Skin thickening - Axillary lymphadenopathy - Mutifocal/ multicentric/ contralateral tumors

What is the difference between craniotomy and craniectomy?

Craniotomy - burrhole drainage Craniectomy - removal of part of skull

What are the types of respiratory crepitations and rubs and their causes?

Crepitations may be due to: 1) Re-inflation of collapsed airways on inspiration, (from proximal to distal) o Fine, early inspiratory - small airways disease e.g COPD o Fine, late/pan- inspiratory crackles - disease confined to alveoli e.g interstitial lung disease w pulmonary fibrosis 2) Movement of air through airways retaining secretions, affected by coughing. o Medium pitched - Left ventricular failure/pulmonary congestion o Coarse crepitations (w gurgling quality) - Bronchiectasis - Pneumonia 3) Rubbing together of thickened/roughened pleural surfaces; continuous, grating sound o Pleurisy - 2' to pulmonary infarction/pneumonia - Pleura malignancy - Pneumothorax - Pleurodynia (aka devil's grip from cocksackie B causing severe msk chest pain with fever/myalgia/malaise)

What muscle abducts the vocal cords and which nerve innervates it?

Cricothyroid muscles abduct the vocal cords. Innervated by the recurrent laryngeal nerve.

What are the pathological differences between Crohn's and Ulcerative Colitis?

Crohns Vs UC 1) Pathological features: - Extent: skip leisions + full thickness + entire GIT Vs continuous + superficial + ascends from rectum - Rectum? 50% Vs Always - Terminal ileum? 30% Vs 10% - Strictures? Common vs Rare - Fistula? Enterocutaneous or intestinal Vs Rare - Malignant change? Possible Vs Well known - Morphology: thick walled bowel with nodular cobble stone appearance and deep, linear ulcers VS granular appearance of mucosa with loss of vascular markings and possible pseudopolyps interspersed with areas of shallow ulceration 2) Microscopically - Transmural involvement with non-caseating granulomas Vs mucosal and submucosal involvement with crypt abscess.

What is the pathogenesis of crushing osteochondroitis of the navicular and what radiological findings would you expect?

Crushing osteochondroitis of the navicular AKA Kohler's disease Occurs due to avascular necrosis of the navicular given it's blood supply: supply to the central one third is a watershed zone that is susceptible to avascular necrosis and stress fractures. It is also the last bone to ossify, which increases it's vulnerability to mechanical compression and injury. Radiologically, one would see sclerosis, flattening and fragmentation of the tarsal navicular.

How do Supracondylar fractures result in BOTH cubitus valgus AND varus deformities and then tardy ulnar nerve palsy?

Cubitus valgus: due to non-union or growth plate arrest - Causes TUS due to stretching of nerve Cubitus varus: due to malunion from failure to reduce a collapsed medial column or rotation at fracture site. - Causes TUS due to compression by medial triceps head

What are the causes of a shift in the oxygen dissociation cuve?

Curve shifts rightward with: 1) Temperature increase 2) Increase in PCO2 3) Increase in H+ aka decrease in pH 4) Increase in 2,3 dibisphosphoglycerate (deceases affinity of deoxygenated Hb to O2, allowing easier diffusion of O2 to poorly oxygenated tissues) Meaning that oxygen affinity of Hb is decreased

After treatment of stroke with the achievement of blood flow, what is the management plan for stay in the acute stroke unit?

DAVID 1) Diet - NBM - Speech therapist to review 2) Ambulatory status - PT/OT - SOOB TDS - DVT prophylaxis (mechanical/pharmacological) - Rehab referral to assess rehab potential 3) Vitals monitoring - Q1hrly + SpO2 > 94% - Permissive hypertension (<185/110 for rTPA, 220/120 for non rTPA for perfusion of ischemic penumbra) Permissive hypotension - Q1hrly CLC (inform if GCS drop >/=2) - Q1hrly fits charting - Q4hrly hypocount monitoring (aim CBG 4-10mmol/L) 4) Investigations o For confirmation of ddx - CT brain TRO bleed/mass lesions - MRI brain (higher sensitivity, can ddx new stroke from past via diffusion weighed imaging) o Invx for etiology o Invx for risk factors 5) Drugs - Aspirin for antithrombotic effect (UNLESS s/p rTPA, need wait 24 hours) - SC unfractionated heparin (5000U TDS) for DVT prophylaxis - as appropriate for PmHx

What are the manifestations of chronic venous insufficiency?

DESU 1) Venous dilatation - Telengiectasia - Reticular veins - Varicositis - Corona phlebectactica (dilated vessels under medial/ lateral malleoli in severe hypertension) 2) Edema - hallmark change - Better w elevation vice versa 3) Skin changes - Hyperpigmentation: due to repeated hemosiderin deposits - Atrophie blanche: due to scarring of healed venous ulcers - Eczema: due to stasis - Cellulitis - Lipodermatosclerosis: fibrosing panniculitis of subcutaneous tissue = champagne bottle appearance 4) Ulceration - shallow flat ulcer w sloping edges - moist looking w sloughy/granulating base - 2' SCC malignancy i.e Marjolin's ulcer (esp if enlarging, tender, malodorous, thickened/raised edges w inguinal LN)

What are the possible differentials in bamboo spine? How do you differentiate them?

DISH: diffuse idiopathic skeletal hyperosteosis - Can lead to ankylosing hyperosteosis DISH/AS 1) Syndesmophytes - Nonmarginal/marginal 2) Disc space - Preserved/ossified 3) Osteopenia - Not present/present 4) HLA - B8/B27 5) Age group - Older/younger 6) SI joint - Not involved/bilaterally involved 7) Radiography - Flowing candle wax/bamboo spine 8) DM association - Yes/No

What is the diagnostic criteria for Diabetic Ketoacidosis and Hyperosmolar hyperglycemic syndrome?

DKA: 1) CBG >/= 14 2) pH <7.3 3) Elevated serum/urine ketones (Betahydroxybutyrate more sensitive, urine test only acetoacetate/ acetone) HHS: 1) CBG >33 2) pH >/= 7.3 3) No elevated serum/urine ketones 4) Effective serum osmolality (2Na + glucose) > 320 mOsm/L

What are the blood pressure targets in a diabetic patient?

DM + HTN = <140/80 DM + nephropathy = < 130/80

What are the lipid targets in a diabetic patient?

DM = LDL < 2.6mmol/L DM + IHD/CKD w/o dialysis = <2.1mmol/L

What are the side effects of incretin based therapy?

DPP-4 - Nasopharyngitis - Diarrhoea, constipation, nausea - Arthralgia - Pancreatitis (rare but important) GLP-1 - Nausea, vomiting, constipation, diarrhoea - Headache - Weight loss

What are the differentials for assymetrical calf swelling/suspected DVT?

DVT Cellulitis Necrotizing faciitis Compartment syndrome Lymphangitis

What is considered unstable according to the Denis three column system?

Damage to 2 contiguous columns (radiopedia) Or 1) Damage to the middle column - Evidenced by widened pedicles or decreased vertebral body height 2) Disruption of the posterior ligament complex along with anterior AND middle column involvement (Orthobullets) Just for revision, the posterior ligament complex comprises of 4 ligaments - Ligamentum flavum - Supraspinous ligament - Interspinous ligament - Facet joint capsule

What is the underlying pathophysiology and causes of achalasia?

Damage/failure to the auerbach myenteric plexus resulting in loss of neuromuscular coordination in the oesophagus. I.e hirschsprung disease in esophagus Can be 1' or 2' 1' - Congenital 2' - Chagas disease - parasitic infection by trypanosoma cruzi carried by the kissing bug (can affect sigmoid colon causing volvulus)

How would you classify fibular fractures and how does this guide management?

Danis-Weber classification A - Infrasyndesmotic o Usually stable o Conservative treatment: short leg walking cast/boot B - transsyndesmotic o Tibiofibular syndesmosis intact or only partially torn o Variable stability o Talar shift/displacement > 3mm = unstable o If ORIF, need syndesmosis fixation via syndesmosis screw C - suprasyndesmotic o Unstable, requires ORIF The more proximal the fibular injury, the more severe

What are the post op complications according to each day?

Day 1-2 - Basal atelectasis - Transfusion/medication related complications - Post-op fever (due to tissue damage and necrosis at op site) Day 3 onwards - Acute kidney injury - Chest infection - Pulmonary embolism - Wound infection Day 5 onwards - Anastomotic leak - Deep infections Anytime: - Haemorrhage, hypovolemia, electroly disturbance - Atrial fibrillation, myocardial infarction - Urinary retention - Nausea && vomiting, ileus, abdominal compartment syndrome - Wound pain

How long does it take for neuropraxia to recover and how would you investigate lack of progress?

Days to 3 months Investigated with 1) NCS (nerve conduction studies) 2) EMG (electromyography)

What is diverication of recti and how is it treated?

Defect in midline of rectus along linea alba due to weakness that may arise from patients with raised intraabodminal pressure or multiparous women. No treatment needed unless cosmetically bothering patient.

- Progressive neurodegenerative disorder - Associated with degeneration of dopaminergic nigrostriatal neurons It is clinically diagnosed with 2 out of 3 signs, comprising of 1) Resting tremour of 3-5 hertz 2) Bradykinesia 3) Rigidity with the fourth: 4) Postural instability, occuring later in the disease course

Definition of Parkinson's disease

Abnormally high levels of protein in urine, measured by protein/creatinine ratio > 15 mg/mmol Normal to mild = <15 mg/mmol Moderate = 15-49 mg/mmol Severe = >50 mg/mmol Nephrotic range proteinuria defn: 1) p/c ratio > 20 mg/mmol OR 2) 24hr Urine protein > 3.5g

Definition of proteinuria and nephrotic range proteinuria.

What is the definition and pathogenesis of high output stoma syndrome?

Definition: > 1L/day, esp if patient dehydrated/electrolyte imbalance Pathophysiology: 1) Infective/ inflammatory - Intraabdominal sepsis - Enteric infection e.g C.difficile - Radiation enteritis 2) Non-infective/ non-Inflammatory - Short gut (< 2m of small bowel) - I/O (invx w CT) - Medication (prokinectics, laxatives, sudden steroid withdrawal)

How is Wolff-parkinson-white diagnosed and treated? (incomplete)

Delta waves are seen Tx: 1) Stabilise patient 2) Give some drugs

What are the components of the deltoid ligament in the ankle and what other ligaments are involved in it's stability? What kind of stability to they provide?

Deltoid ligament (on the medial aspect) Superficial aspect (adds little to ankle stability): 1) Naviculotibial (prevents anterior displacement) 2) Talotibial 3) Tibiocalcaneal (prevents valgus displacement) Deep aspect: 1) Deep tibiotalar (main stabilizer against lateral displacement of talus) Fibular collateral ligament (on lateral aspect) 1) ATFL - anterior talofibular ligament (prevents anterior displacement) 2) PTFL - posterior (prevents posterior displacement and rotatory subluxation of talus) 3) Calcaneofibular ligament (prevents excessive inversion, it ruptured = postive talar tilt)

What is morgallen's syndrome?

Delusion of parasitosis

Male (90%), over their 30s Females (rare), only after menopause Risk factors: - Diet: high purine diet (sardines, red meat, soy/bean products, alcohol) - Obesity - Hypertension - Smoking - Medications: diuretics, aspirin - Lead poisoning - Family history

Demographic group and risk factors of Gout

Women over their 60s

Demographic group of Pseudo-gout

What are the risk factors of thyroid malignancy?

Demographic: - Gender (female higher risk) - Age (>45) - Family hx - Genetics (FAP, Gardner's, MENII) Previous conditions: - Radiation exposure - Thyroid follicular adenoma - Long standing goitre - Previous thyroid CA - Previous lymphoma - Previous thyroiditis

What is the Denis three column system and what does each column comprise of?

Dennis spinal columns classification 1) Anterior - Anterior longitudinal ligament - Anterior annulus - Anterior 2/3 of vertebral body 2) Middle - Posterior longitudinal ligament - Posterior 1/3 of vertebral body - Posterior annulus - Injury is evidenced by: widening of interpedicular distance on AP radiograph OR loss of height of posterior cortex of vertebral body 3) Posterior - Posterior elements o Pedicles o Facets o Lamina o Spinous process - Posterior ligaments o Ligamentum flavum o Supraspinous ligament o Interspinous ligament o Facet capsule

How can you classify tibial shaft fractures?

Depending on open or closed Open: Gustillo-anderson classification Closed: Tsherne's classification (4 grades) Grade 0: Indirect forces w negligible soft tissue damage Grade 1: Superficial contusion/abrasion, simple fractures Grade 2: Deep abrasions, muscle/skin contusion, direct trauma, impending compartment syndrome Grade 3: Excessive skin contusion, crushed skin or destruction of muscle, subcutaenous degloving, acute compartment syndrome and rupture of major blood vessel or nerve. (basically, if complicated = 3)

What are the options for fasciotomy in compartment syndrome?

Depending on the limb involved: 1) Arm: Volar henry approach + longitudinal incisions over 2nd and 4th metacarpals 2) Leg: dual medial lateral incision 3) Foot: dual dorsal incision

What is the pathogenesis of osteogenesis imperfecta?

Depending on the type, maybe due to collagen deficiencies (COL1A1 and 2 gene mutations) or non-collagen protein deficiencies (transmembrane proteins, scaffolding proteins etc) *Collagen type 1 alpha 1 and 2

What is the treatment of vitiligo?

Depends on BSA% involvement <10% - Topical steroids - Tacrolimus 10-40% - Phototherapy NBUVB

What is the critical Beighton score to be hypermobile? What are it's components?

Depends on age and pubertal status, children 7 or more, adults 4 or more.

What are the possible mycobacterial skin infections?

Depends on causative organism 1) Mycobacterium tuberculosis - Lupus vulgaris - Tuberculosis verrucosa cutis - Scrofuloderma - Orificial TB - Erythema induratum - Erythema nodosum 2) Mycobacterium leprae - Leprosy - Erythema nodosum leprosum 3) Atypical mycobacterium: (M. marinum/avium-intracellulare) - Fishtank granuloma (looks juicy, infiltrated plaque) (includ

What are the possible imaging investigations for suspected renal trauma?

Depends on haemodynamic stability. If patient is stable: Contrast CT is choice - More sensitive + sepcific for staging via AAST - Can detect other abdominal injuries - Contrast may be nephrotoxic. If patient is UNSTABLE - Intravenous urethrogram/pyelogram (IVU/IVP) - Single XR shot after IV contrast - Consider retrograde pyelogram if obstructed/deformed kidney/chronic pyelonephritis etc

What are the management options for abdominal aortic aneurysm?

Depends on if ruptured or non-ruptured. RUPTURED: 1) Stabilise ABCs - avoid intubation because neuromuscular blockers will reduce tamponade effect and worsen hemorrhage 2) Call for vascular surgeon 3) Pre-op meds: - To reduce post-op renal insufficiency - Furosemide or mannitol infusion 4) Emergency Surgical repair - Laparotomy and clamping of aorta (~30 minutes without visceral ischemia) - AAA incised, surrounding hematoma and mural thrombus cleared - Synthetic graft placed - Mortality rate ~ 50% NON-RUPTURED (depends on size of aneurysm: - Conservative if < 5.5 cm o Optimise CVS risk factors (DM, HLD, HTN) o Smoking cessation o Regular follow up w U/S 6/12ly or more if low risk - Surgery if: o > 5.5 cm o increase in diamter > 1 cm/ year o symptomatic/ complicated aneurysm o Either open OR endovascular grafting - Endovascular is less invasive, also under GA, just need to deploy stent with good neck and landing site - Need to do angiogram 6/12 to check position of stent (possible migration)

What is the management for urethral injury?

Depends on if the injury is a partial or complete tear. 1) Partial tear: urologist can attempt gentle catheterisation and guide catheter in 2) Complete tear or failure of urethral cath in partial tear: Suprapubic catheterisation (only done when bladder full from patient urine) 3) Delayed reconstruction: only when patient is stable and recovered from other injuries

At which levels do nerve roots exit with reference to their vertebral bodies?

Depends on level Above T1 (C1 to C7), nerve exits ABOVE it's vertebrae - C7 exits at C6/7 - C8 exits at C7/T1 Below T1, (T1 to S5), nerve exits BELOW it's vertebrae - T1 exits at T1/T2

How is transaminitis for patients on TB drugs managed?

Depends on levels and symptoms 1) Symptomatic (abdo pain, jaundice) + 3x upper limit - Stop drug 2) Asymptomatic + 5x upper limit - Stop drug After liver enzymes normalise, re-introduce drugs one at a time ending w pyrazinamide while observing liver enzymes.

What is the treatment for Psoriasis skin lesions?

Depends on manifestations 1) Stable plaque - Topical steroids, coal tar - Calcipotriol - If extensive, UVB or methotrexate, acitretin, ciclosporin, biologics or photochemotherapy 2) Facial - Mild to moderately potent steroids 3) Palms/soles - Moderate to potent steroids - Soak photochemotherapy, acitretin, methotrexate or ciclosporin A 4) Guttate psoriasis - Topical steroids, coal tar - UVB 5) Erythroderma or generalised pustular - Methotrexate - Acetretin, ciclosporin A, biologics

How long is antimicrobial therapy indicated in meningitis?

Depends on organism 1) 7 days - N.meningitidis and H.influenzae 2) 10-14 - Strep pneumoniae 3) 21 days - Gram negatives 4) >21 days - Listeria

What are the most common causes of small bowel obstruction?

Depends on previous surgery 1) Post-surgery - Adhesions 2) Virgin abdomen - Hernia

What are the treatment options for acne?

Depends on severity Mild acne: - Topical Abx (clindamycin and erythromycin) - Topical benzyl peroxide (2.5%) - Topical retinoids (for night use, retinoic acid, adapalene or tazarotene 0.01-0.05%) - +/- surgical excision of comedomes Moderate acne: - + oral abx to regimen of mild acne (doxycycline 50-100mg BD) - OR oral isotretinoin (do not use tgt, can prevent scarring) Severe acne: - Topical treatment + systemic tx of: - Isotretinoin (0.5-1mg/kg taken w food) - Oral steroids (for acne conglobata, acne fulminans) - Intralesional triamcinolone for isolated inflammatory cysts/nodules Additional therapy - Hormonal: Estrogens/anti-androgens e.g Diane 35, indicated in failure of abx therapy or if PCOS/contraception and menstural regulation required

What is the treatment of PCP?

Depends on severity of disease and HIV status of patient. HIV positive - Mild disease: Atovaquone (anti-malarial, doesnt cause BM suppression like bactrim) - Mild to moderate disease: PO bactrim or PO dapsone with trimethoprime or PO clindamycine with primarquine - Severe disease: IV bactrim or IV clindamycine + primaquine or IV pentamidine - GIVE ART ALONGSIDE!! - Steroids in moderate-severe PCP Non HIV patient - IV bactrim - Adjunctive steroids - PCP prophylaxis with bactrim

What is the treatment of syphilis?

Depends on the level. 1) Primary - single dose 2.4 MU IM B.penincillin 2) Late latent, indeterminate, CVS or late benign syphilis - 2.4MU IM B.P weekly x3 (instead of single dose) 3) Neurosyphilis - Aqueous procrain penicillin 2.4 MU IM daily for 10 days AND probenecid 500 mg oral qds - FOLLOWED by: 2.4MU IM B.P weekly x 3 4) Assymptomatic - 50,000U/kg (3.5 MU in 70kg) IM single dose OVERALL - Throw in corticosteroid cover to minimize effects of harish-herxheimer reaction that may occur 4-12 hours after the first dose of abx - Indicated in CVS, neurosyphilis and laryngeal gumma

When are traumatic renal injuries indication for imaging?

Depends on the nature of injury: penetrating vs blunt trauma Penetrating: - Image ALL (9% will have no hameturia) Blunt: - Gross hematuria OR - Microscopic hematuria (>5RBC/hpf) with shock (SBP<90) - Any child with > 50 RBC/hpf (shock is late presentation in children)

What is the management of spinal metastasis?

Depends on the prognosis 1) If < 6/12 - Palliative care 2) If >/= 6/12 - Surgical decompression with stabilisation - + Post op radiotherapy

What are the treatments for anal fistula

Depends on type of fistula, but generally involves 1) Fistulotomy - Curettaging base of fistula 2) Fistulectomy - Removing the fistula along with the base 3) Seton - For complex, long, high tracts

How do you treat acanthosis nigricans?

Depends on underlying pathology, which can be due to obesity, diabetes mellitus (and other endocrine disorders with impaired glucose metabolism), malignancies e.g stomach CA, drugs. - Lose weight - Treat underlying disease - Stop causative drugs Cosmetic treatment - Topical retinoids - Dermabrasion - Laser therapy

What is the treatment of periprosthetic fractures?

Depends on whether patient is osteoporotic and if implant is loose/unstable 1) If Yes - treat the fracture and revise the implant 2) If no - treat the fracture according to general principles of management.

What is the pathophysiology of acute calcific tendinitis? How do patients present?

Deposition of calcium hydroxyapatite crystals in supraspinatus tendon (often in critical zone: medial to insertion) with pain 2' to vascular and inflammatory reaction. a/w subacromial impingment, DM and hypothryoidism (endocrine disorders) Patients complain of rapidly progressing severe pain after unaccustomed activity, with painful loss of RoM of shoulder

It uses the time that a person takes to rise from a chair, walk three meters, turn around, walk back to the chair, and sit down. During the test, the person is expected to wear their regular footwear and use any mobility aids that they would normally require. If total time exceeds 10s, its considered abnormal and of functional impairment.

Describe the Get up and Go test.

Secondary hyperparathyroidism Mx: 1) Control of serum phosphate, calcium and parathyroid hormones - Diet control (phosphate restriction) - KIV Phosphate binders 2) 3-12 monthly monitoring of levels 3) Treatment of patients not meeting targets - CKD + elevated PTH = treat with active vitamin D sterols

Describe the management of mineral bone disease in Chronic kidney disease.

Why is Logol's iodine used in thyroid storms and Grave's disease?

Despite the counter intuitive of supplying iodine to a system that requires it as a precursor to synthesize thyroid hormones, Acute increase in iodine levels actually inhibit secretion of thyroid hormones via a mechanism yet unknown. Additionally, iodine also inhibits thyroid iodination (i.e organification) via the Wolff-Chaikoff effect, an autoregulatory phenomenon that prevents hyperthyroidism in excess iodine intake. While in individuals with normal thyroid, the gland may escape from this inhibitory effect (around 10 days), in patients with underlying autoimmune disease, the suppressive action of high iodide may persist. The Escape phenomenon occurs when the intracellular iodide levels fall below the threshold due to downregulation of the sodium iodine symporter (NIS) on the basolateral membrane of the thyroid follicular cell.

What happens if a lisfranc injury is left untreated?

Destabilization of the midfoot architecture with - Progressive arch collapse and - forefoot ABDuction with subsequent - Degenerative OA (need to treat with arthrodesis)

What are the developmental and growth complications of neurofibromatosis?

Developmental: - Leaning impairment - Developmental delay - Poor motor coordination - Hearing impairment (more common in type 2) Growth: - Premature/delayed puberty

What is the Amsterdam criteria?

Diagnostic criteria for HNPCC syndrome [3, 2, 2, 1] > 3 family members from same side w HNPCC related CA (2 or more are 1st degree) 2 successive generations >/= 1 of the HNPCC cancers must occur before 50 y/o FAP must be excluded

What are possible lifestyle habits for GERD?

Diet: - Avoid coffee, chocolate, fatty foods, anything that worsen's symptoms - Small meals Lifestyle: - Avoid eating 2 hours before sleeping - Walk after eating - Smoking and alcohol cessation - Elevation of bed - Weight reduction if obese

1) Aortic stenosis/sclerosis 2) Pulmonary stenosis 3) Hypertrophic Obstructive Cardiomyopathy (HOCM) 4) Mitral regurgitation/valve prolapse 5) Coarctation

Differentials for Ejection systolic murmur

G.H.O.S.T -Gout -Haemarthrosis -Osteoarthritis -Septic arthritis -Traumatic injury

Differentials for Monoarticular arthritis

1) Acute gout 2) Traumatic haemarthrosis 3) Septic arthritis 4) Reiter's disease (a/w conjunctivitis, urethritis and colitis)

Differentials for acute joint pain

1) Chronic gout 2) Chronic pseudo-gout 3) Inflammatory arthropathies (RA, SLE, Seronegative etc) 4) Metabolic disorders: - Haemachromatosis (a/w liver cirrhosis, diabetes, bronze pigmentations of skin, joints include hands, fingers and back, XR shows chondrocalcinosis and destructive arthropathey typically in MCPJ, with raised plasma iron and iron binding capacity. - Alkaptouria (congenital absence of homogentistic acid oxidase in liver and kidney, causing its accumulation that results in calcifications of hyaline and fibrocartilage. XR shows calcification of intervertebral discs, spinal osteoporosis, chondorcalcinosis and OA of peripheral joints) - Hyperparathyroidism: a/w hypercalcemia

Differentials for chronic joint pain

How does one diagnose tuberculosis?

Differing investigations for each stage on the spectrum of TB. 1) Latent - Tuberculin skin test (induration > 10mm in normal individuals, > 5mm in HIV, recent contact, +ve CXR findings, organ transplant.) - Interferon gamma release assay (quantiferon: IFN-g concentration vs Tspot/elispot: number of IFN-g producing WBCs) 2) Active - AFB Smear (non-specific mycobacterial detection) - TB PCR (limited to sputum, low sensitivity if smear neg) - TB Culture in LJ medium (6-8 weeks) - TB Mycobacteria growth indicator tube (MGIT) (6 weeks, gold standard)

What are the levels determining hypoglycemia?

Differs between diabetics and non-diabetics. Diabetics: <2.8 Non diabetics: <4mmol/L

What are the associated injuries with hip dislocations?

Differs with posterior and anterior dislocations 1) Posterior dislocations - Posterior acetabular wall # - Anterior femoral head # - Sciatic nerve injury (check L4,5,S1,2,3) - Ipsilateral knee injury - Osteonecrosis 2) Anterior dislocation - femoral head impaction/chondral injury

What are the mechanism of injury for a humeral shaft fracture?

Direct vs Indirect 1) Direct: - Most common - Blow or RTA to arm resulting in fracture 2) Indirect: - Fall on outstretched hand esp in elderly - Throwing injuries/extreme arm wrestling

In description of dislocations, is direction described in reference to the proximal or distal limb?

Direction is in reference to proximal limb, hence: Anterior shoulder dislocation = humeral head in front of shoulder Anterior knee dislocation = tibia is anterior to femur.

What is the difference between medial and lateral winging of scapula?

Direction refers to part of scapula that wings. Hence medial winging: - Due to serratus anterior palsy - From long thoracic nerve injury - Can be brachial plexus, nerve root, fascioscapulohumeral dystrophy syndrome - Emphasised when patient asked to push against wall Lateral winging: - Due to trapezius palsy - From spinal accessory nerve palsy - Can be intracranial, extracranial - Emphasised when patient asked to externally rotate against resistance

What is the difference between discitis and spondylodiscitis?

Discitis affects only disc spaces while spondylodiscitis affects the vertebral bodies as well. Discitis - More common in children because blood vessels extend from vertebral end plate into nucleus pulposes: allowing direct innoculation - Also because adults blood vessels extend only to annulus fibrosis thus isolated disc infection less common

What is the cut off sign in AXR?

Discontinuity in intraluminal bowel gas on AXR, indicating mechanical obstruction. Exception in pancreatitis w sentinel loop sign, (non mechanical but localised obstruction)

What are the factors of consideration and the difference in indications in deciding between warfarin and NOAC?

Disease factor - Prosthetic valve, valvular AF, LV clot etc Patient factor - Comorbidities (currently on many drugs) - Psychosocial (e.g ability to come for regular follow ups) - AF ○ Cannot use NOAC in valvular AF or mechanical heart valves or LV clots ○ Can be used in NON-valvular AF e.g hyperthyroidism - Mechanical heart vavles ○ NOACS CANNOT USE (RE-ALLIGN trial) only WARFARIN can be used ○ Increased haemorrhagic risk - DVT ○ NOACS cannot use in malignancy, only clexane (CLOT, LITE) that shows reduced risk of recurrance ○ Otherwise, NOACs can be used as well (EINSTEIN DVT) - Renal impairment ○ NOACs cannot use - On many drugs ○ Warfarin has high drug drug interactions. ○ E.g rifampicin EVERYTHING interacts w warfarin - Psychosocial ○ Warfarin need multiple clinic visits for INR ○ NOAC expensive ○ Tolerance of multiple venepunctures

What is the pathogenesis of rheumatoid arthritis?

Disease of cell mediated autoimmune inflammation and destruction of soft tissues, cartilage and subsequently bone and tendon, occurring within genetically susceptible individuals (HLA DR4 and DW4).

What is the difference between dislocation and subluxation?

Dislocation - full seperation of all articulating surfaces Direction depends on distal fraction

Where is the capitellum and trochlear in the elbow?

Distal humerus Capitellum is lateral, articulates with radius Trochear is medial. articulates with ulnar.

What is a Smith's fracture? How would you manage it?

Distal radius fracture with volar angulation, volar displacement of hand and radius, ulnar deviation, giving rise to a 'Garden spade deformity' Due to fall on a flexed wrist with forearm in supination i.e opposite to Colles Management: 1) If stable: - M&R w backslab, XR to confirm, examine TRO N&V compromise - Elevate for 1-2 days, exercise immediately, XR in 10 days to check again, remove backslab in 5 weeks, exercise again 2) If unstable: - K wire - External fixation - ORIF w plates & screws or intramedullar nail

What is considered a major tissue loss vs a minor tissue loss in Rutherford classification of chronic limb ischemia?

Distal to meta-tarsal phalangeal joints = minor, proximal = major tissue loss.

How do you measure the size of thomas splint?

Distance of adduction tendon at groin to heel + 25 cm.

What are the types of gastric cancer?

Divided into histological types: 1) Adenocarcinoma (90-95%) Lauren classification: o Intestinal type (most common) - high risk population - distal third of stomach - a/w erbB2 and B3 receptor stimulation o Diffuse type - low risk population - Proximal third and cardioesophageal junction - more aggressive, present later, worse prognosis - a/w K-sam oncogene - Can form linitis plastica (N&V before anemia) 2) Non-adenocarcinoma (<10%) - SCC - Neuroendocrine - Leiomyocarcoma - GIST - MALT

Theres no question, please remember what the light bulb sign looks like hahaha.

Does this fit what you remember?

Why are valgus and varus stress tests in physical examination done in both full extension and 30 degrees?

Done in 30 degrees to eliminate use of cruciate ligaments for stability, which are the secondary stabilisers of lateral and medial movement.

How is a discography done and what are the indications?

Done via injection of sterile liquid for radio-graphic assessment via CT (fluoroscopic discography) and/or to see if symptoms worsen (provocative discography). Useful in determining if cause of back pain is due to discs

What is the gold standard to check for drug allergy?

Drug challenge, unless anaphylactic reaction?

What are the possible causes of pseudohematuria?

Drugs - Rifampin - Phenothiazines - Phenytoin - Phenolpthalein laxatives Foods - Beets, blackberries, rhubarb Metabolic - Myoglobinuria - Hemaglobinuria - Porphyria Infections - Red diaper syndrome (serratia marcescens infection)

What are the causes of a continuously bubbling chest tube?

Due to air leak 1) Chest - Bronchopulmonary fistula - Penetrating lung injury - Ruptured bullae 2) Tube and drain - Puncture/defect in drain or tubing

Why are there hypopigmentation spots in pityriasis versicolor infections?

Due to azelaic acid production by the fungus that disrupts melanocyte function.

What is the pathogenesis of pretibial myxedema in thyroid disease?

Due to deposition of hyaluronic acid within the dermis and subcutis, unknown cause. Possible theory is stimulation of fibroblasts to produce glycosaminoglycan due to high levels of circulating thyroid receptor antibody and antigen-specific T cells.

What is the pathogenesis of the fat pad sign and what does it indicate?

Due to displacement of fat pad around joint. 1) Posterior fat pad sign: In adults - . radial head fracture In children - condylar fracture of humerus Any joint inflamamtion resulting in effusion can lead to abrnomal fat pad sign: - Toxic synovitis, septic srthritis, JRA, OM etc) 2) Anterior fat pad - Can be normal

Why is the PVNS tumor yellow?

Due to hemosiderin and lipid deposits.

What is the pathogenesis and implications of acanthosis nigricans?

Due to insulin binding insulin-like growth factor receptors on keratinocytes and dermal fibroblasts, stimulating growth Seen in association with diseases/increased insulin resistance (DEN): 1) Drugs - Nicotinic acid - Corticosteroids - Estrogen 2) Endocrine syndromes: - Diabetes! - Cushings - Acromegaly - PCOS - Hypothyroidism - Obesity 3) Neoplasia - Stomach - Pineal tumors

Why is the pulse jerky in mitral regurgitation?

Due to lack of sustained forward volume ejection from the regugitation, resulting in a normal amplitude pulse with shortened ejection time and sudden fall.

What is the pathophysiology and causes of ingrown toenails? How would you treat it?

Due to nail invasion into surrouding tose tissue, causing epidermal breakage and subsequent inflamamtory response due to foreign body entry. Possible causes include: - Hereditary - Trauma - Infection - Tight foot wear Management 1) Conservative - Packing - Taping - Guttering - Nail braces 2) Surgical - Excision and phenolisation of nail bednail (chemical matricectomy)

What is the pathophysiology of hypochloremic, hypokalemic metabolic alkalosis?

Due to vomiting and loss of water = hypovolemia Hypovolemia results in decrease renal arteriole perfusion, reduced efferent arteriole flow that stimulates the juxtaglomerular apparatus to trigger the RAAS. Aldosterone upregulates transporters e.g K H exchanger. With each proton lost, the tubules form one HCO3 that leads to metabolic alkalosis.

Which antibiotics are useful for E.faecalis and E. faecium?

E. faecalis - Imipenem (not meropenem or erta) - Any ampicillin - Vancomycin - Piptazo E.faecium - Only vancomycin

What is the cause of bloody diarrhoea without white blood cells?

EHEC. Only Shiga toxin - which impairs absorption. STrain does not invade/directly damage mucosa, thus no white blood cells

What is the difference between ESBL and AmpC mutation?

ESBL confers resistance to - Penicillins - Cephalosporins (up to 3rd and 4th gen and cephamycins) - Monobactams AmpC confers resistance to - Penicillins - Cephalosporins (up to 3rd gen) + 2nd gen and cephamycins - NOT 4th gen

What are the complications of ESWL?

ESWL complications - Failure of treatment - Bleeding (common) possible peri-renal hematoma - Infection

What is the pathogen responsible for traveler's diarrhoea?

ETEC: Enterotoxic E.Coli

What are the physical findings of testicular torsion?

Early: - Testes elevated, tender, with abnormal lie (usually transverse) - Spermatic cord shortened, swollen - Loss of cremasteric reflex - Prehn sign negative Late: - Reactive hydrocele - Scrotal wall edema/erythema

What are the early and late complications of stoma?

Early: 1) Bleeding/ischemia 2) Infection 3) Mucocutaneous dehiscence and retraction 4) Skin excoriation 5) High output stoma syndrome Late: 1) Stenosis 2) Fecal impaction/obstruction 3) Peristomal hernias 4) Prolapse Others for Ureteroileal conduits 1) Pylonephritis 2) Renal calculi

What are the complications of Colles fracture?

Early: 1) N&V compromise - Carpal tunnel syndrome from swelling 2) Associated injuries: - Scaphoid fracture - Radial fragment comminution - Triangular fibrocartilage complex injury from RUJ subluxation Late: 1) Neurological - Median nerve dysfunction (surgical exploration if not improved after reduction +/- carpal tunnel release) 2) Muscular - Extensor pollicis longus tendon (may be frayed, rupturing weeks after, do tendon transfer with index finger extensor tendon) 3) Skeletal - Malunion - Non union - Post-traumatic OA - Stiffness

How would you grade Charcot's foot? How would it guide management?

Eichenholz Classification (3 stages) Stage 1: Fragmentation/dissolution - Swelling, warmth, fractures/dislocations and osteopenia - Off loading with clinical follow up Stage 2: Coalescence - Reduced warmth and swelling, resorption of bone, early fusion and sclerosis - Off loading with walking air Stage 3: Reconstruction - Consolidation of deformity with joint arthritis and subchondral sclerosis - Treatment depends on whether foot is plantigrade - Plantigrade foot: custom shoe inlays - Non-plantigrade foot: debridement, Exostectomy, correction or fusion with internal fixation.

How do you treat syringomas?

Electrocautery or laser.

What are the causes of posterior shoulder dislocation?

Electrocution/seizures

What is Buerger's test?

Eliciting pallor and guttering on lifting of patient's legs and subsequent reactive hyperemia on sitting up with legs down to assess for severity of limb ischemia. Buerger's angle is 20deg, below which implies critical limb ischemia.

What is the difference between embolus and thombus?

Embolus - clot that travels until reaching terminal/narrow blood vessels, may have broken off larger clot Thrombus - clot that forms within the blood vessel that it obstructs, i.e not traveling T doesnt Travel.

When is surgery indicated in UC?

Emergency - Despite medical tx developes complicatiosn e.g toxic megacolon, bleeding, CA w obstruction Elective - Failed to respond to medical therapy - CA picked up on screening

When is surgery indicated for adhesion I/O?

Emergency Vs Elective 1) Emergency - Perforation - Bowel ischemia - Unresolving w conservative tx (fail gastrograffin study/obstruction in large bowel) 2) Elective - Recurrent - Done w adhesiolysis +/- anti - adhesion placement

What is the diagnostic criteria of Crohn's disease?

Endoscopy with intubation of terminal ileum for ileocolic CD. Endoscopy: - Focal ulcerations adjacent to areas ofnnormal appearing mucosa - Along with polypoid mucosal changes giving a cobblestone appearance. +/-Biopsy: - Focal ulcerations, full thickness involvement

What are the causes of erythema nodosum?

Erythema nodosum - erythematous nodules found on bilateral LL 1) Infections - Strep pharyngitis, mycoplasma, chalmydia, histoplasmosis, coccdiodomycosis, mycobacteria - HSV, EBV, Hep B/C, HIV 2) Drugs - Abx: Sulfonamide, amoxicillin - OCP 3) Autoimmune/inflammatory diseases - IBD - Sarcoidosis 4) Metabolic conditions - Pregnancy 5) Malignancies - Lymphoma 6) IDIOPATHIC

What is the presentation of Paget's disease of the breast?

Erythematous, scaly and weepy eczema involving the nipple (often misdiagnosed by GP and presenting late after an ineffective course of steroids) +/- discoloration/depigmentation

When is nutrition support indicated for a patient with eventual need for surgery?

Espen 2006 guidelines General indication is the prevention and treatment of catabolism and undernutrition. Nutrition support should be given to malnourished patient 10-14 days prior to surgery if: (WAB) - Weight loss 10-15% in last 6 months - Albumin < 30g/L (w/o hepatic or liver impairment) - BMI < 18.5 or SGA grade C -

What are the possible causes and risk factors of stroke? What investigations would you order for each one?

Etiology: Intracranial, extracranial, cardioembolic 1) Intracranial - MRI/MRA* - CT/CTA - Catheter angiogram 2) Extracranial - Carotid U/S* - MRA - CTA - Catheter angiogram 3) Cardioembolic - TTE/TEE - 24 hr Holter monitoring Risk factors: 1) DM - HbA1c, fasting blood glucose 2) Hypertension - BP measurement 3) Hyperlipidemia - Fasting lipids 4) Coagulopathies - Start with PT/PTT and INR - Further investigate if thrombophillic

What is the difference between eumelanin and pheomelanin?

Eumelanin: Brown to black Pheomelanin: red to blue hue.

What is the classifications of inter-trochanteric fractures?

Evan's classification 2, 2, 3, 3, 4

What type of tumor is associated with onion skinning of the cortex?

Ewing's sarcoma

What is the pathogenesis of scheuerman's kyphosis?

Exact cause unknown, current theory suggests kyphosis and vertebral wedging are due to developmental error in collagen aggregation that results in abnormal endplate.

What is Hutchinson's sign in melanoma?

Extension of the subungal hyperpigmentation to the paronychium and proximally.

What do you need to check before starting hydroxychloroquine in a patient?

Eye check G6PD screening

Name a condition where multiple sebaceous cysts can manifest?

FAP, Gardner's syndrome.

What blood tubes are used for FBC, renal panel, liver panel, PT/PTT, GXM, glucose, ESR , cardiac enzymes and amylase?

FBC - Purple (Contains EDTA) Renal panel - Red/yellow tube (clot activator)/green Liver panel - Red/yellow/green tube PT/pTT - Blue tube (with citrate, shake 3-4 times because may activate platelets) GXM - pink tube (with EDTA but larger volume) Glucose - grey tube (with fluoride that inhibits cellular glycolysis) ESR - Purple (w EDTA Cardiac enzymes - Green (has heparin) Amylase - yellow/red/green depending on validation of lab Red/Yellow/Green are interchangable, just that green has heparin and is accepted only in labs w validation for serum, where they dont have to wait for the sample to clot.

What is a complication of long term torticolis?

Facial hemiatrophy, smaller eye Reduced stimulus results in decreased growth.

What are the neurovascular structures in close relation to the parotid gland?

Facial nerve and its branches: The Zebra Bit My Cow External carotid artery and its branches: Maxillary, posterior auricular, superficial temporal. Retro-mandibular vein

What are the factors in the intrinsic pathway?

Factor 12 Factor 11 Factor 9 Factor 10

What are the clotting factors affected in obstructive jaundice?

Factor 2, 7, 9, 10 due to lack of fat soluble vitamin K due to poor fat absorption. (half life of 7 is the shortest -> belongs to the extrinsic pathway)

What is the Bell Clapper deformity?

Failure of anchoring of the gubernaculumn, epididymis and testis, leaving the testes free to swing and rotate within the tunica vaginalis.

What are the risk factors of OA?

Family history Trauma

How would you manage Necrotizing faciitis?

Fasciectomy

Other than allopurinol and probenecid, what else can you give?

Febuxostat Indicted when allopurinol and probenecid are contraindicated: 1) Adverse drug effects (SJS, TEN, DRESS) 2) Urolithiasis (increased uric acide predisposes stone formation)

What kind of patients tend to get posterior tibial dysfunction and how do they present?

Females > males Patients in their 50s Risk factors: - Obesity - HTN - DM - Steroids - Inflammatory joint disease Symptoms: - Medial ankle/foot pain and swelling - Lateral ankle pain due to subfibular impingement Signs: - Pes planus - Hindfoot valgus - Forefoot ABduction - Tenderness posterior to medial malleolus (where the tendon runs) - Unable to tip toe

How would you differentiate a femoral hernia from an inguinal hernia?

Femoral hernia - lateral and inferior to the pubic tubercle Inguinal canal - superior and medial to pubic tubercle.

What are the symptoms of acute HIV seroconversion and what are the possible differentials?

Fever Lymphadenopathy Sore throat Rash Myalgia/arthralgia Diarrhoea Headache And full body picture involving head, neck, limbs, GIT and skin. Differentials: - Infectious mononucleosis (EBV, CMV, with fever, LN, pahryngitis, rash) - Influenza - Rhinoviral infection - Viral hepatitis - Secondary syphillis (fever, malaise, pharyngitis, lymphadenopathy, rash) - Dengue

How would you diagnose Necrotising faciitis in the emergency department?

Finger test 1) Local anesthesia injected 2) Small incision made 3) Finger inserted, assessing for: - Dishwater like fluid - Easy finger dissection of necrotis tissue - Lack of /minimal bleeding

What is the Gleeson scoring?

First number is the predominant cell type. 3 + 3 = 6 is the lowest grading for prostate cancer 8 to 10 may be considered poorly differentiated or high grade cancer. Total score prognosticates the risk of recurrence after resection as well as mean survival without locoregional therapy.

What is the cause of aerobillia? How do you treat it?

Fistulation of the biliary tract to the GIT. Treat with decompression, the fistula is left alone usually.

What is the mechanism of action for facet joint dislocation and how would you classify it?

Flexion and distraction forces +/- element of rotation. Can be divided according to Allen and Ferguson Classification (4 stages): Stage 1: Facet subluxation Stage 2: Unilateral - Frequently missed on plain XR - seen as 25% subluxation on XR - a/w monoradiculopathy that improves on traction Stage 3: Bilateral - leads to 50% subluxation on XR - Ofter a/w spinal cord injury Stage 4: Fracture dislocation - 100% displacement - More frequently of the superior facet - Maybe unilateral or bilateral

What finger deformity is associated with a flexor tendon rupture at DIPj? What about extensor?

Flexor: rugger jersey finger Extensor: mallet finger

What is the holiday segar method for fluids?

Fluid maintanance formula for those above 1 year old. 3-10kg = 100ml per kg/day 11-20kg = 1L + 50ml per kg/day > 20kg = 1.5L + 20ml per kg/day For hourly maintanance: 3-10kg = 4 ml/kg/hr 11-20kg = 40ml + 2ml/kg/hr >20kg = 60ml + 1 ml/kg/hr

What is Todd's paralysis and when does it occur?

Focal weakness in part or all of the body after a seizure, typically affecting appendages and is localized to either left/right of the body. Typically subsides completely within 48 hours. Focal seizures only.

What are the causes of urticaria?

Food Medications Bug bites (incomplete)

What is the relation between food allergy and atopic dermatitis?

Food allergies are rare even if they appear to exacerbate AD. - AD can induce food sensitivity due to poor skin barrier BUT - Worsening of AD w some foods may be due to stress from new foods (will resolve over time) - RAST test (for IgE levels) high false positive bec AD patients have high circulating IgE - Skin prick test high false positive bec AD patients have baseline sensitive skin To check for TRUE food allergy causing AD - Take comprehensive clinical hx - Do RAST and Skin prick - Then do Food elimination test.

pH = increases by 0.08 HCO3 = decreases by 2

For 10mmHg decrease in PaCO2, what is the expected change in pH and HCO3 in acute respiratory alkalosis?

pH = increases by 0.03 HCO3 = decreases by 5

For 10mmHg decrease in PaCO2, what is the expected change in pH and HCO3 in chronic respiratory alkalosis?

pH = decreases by 0.07 HCO3 = increases by 1

For 10mmHg increase in PaCO2, what is the expected change in pH and HCO3 in acute respiratory acidosis?

pH = decreases by 0.03 HCO3 = increases by 4

For 10mmHg increase in PaCO2, what is the expected change in pH and HCO3 in chronic respiratory acidosis?

What is the treatment of anal SCC?

For SCC of the anal canal RT is the first treatment: 5FU Surgery after (for salvage)

What are the principles of long bone fixation?

For a long bone to be considered properly fixed, it should demonstrate: 1) Acceptable length and alignment (some fractures require anatomical reduction e.g forearm fractures to maintain function) 2) No mal-rotation 3) Early exercises to maintain range-of-motion in patients by Dr Murphy in NUH Trauma Ortho

What is the management of Perthe's disease?

For all: skin traction on affected leg until pain subsides (~ 3 weeks) Guided by Prognosis (caterall grading?): 1) Good prognosis: - No intervention required - Activity modification to maintain/improve RoM 2) Poor prognosis: - Containment of femoral head required (i.e need to better cover it) o ABDuction splint (for 1 year, unpopular hahah) to allow bone changes to run their course o Varus osteotomy of femur o Innominate osteotomy of pelvis

What are the principles of articular fracture fixation?

For articular fractures, they require: 1) Anatomical reduction 2) Mobilisation/RoM (some movement must be permitted) 3) Fixation should be stable.

What is the Burch Wartoffsky score and what is it use for?

For assessment of risk of thyroid storm. > 45 = highly likely thyroid storm 22-24 = possible, consider ICU stay < 22 = unlikely a thyroid storm It's Precipitating over my Heart in another Timezone (GMT) o Precipitating factors o Cardiac - Heart rate - AF - CCF o Timezone - G: GIT/hepatic effects - M: mental state changes - T: Temperature

What is the Gustillo Anderson classification for and what does it comprise?

For classification of open fractures I, II, III I - <1cm, mild soft tissue injury 2 - >1cm, moderate soft tissue injury, mild crush injury 3 - Severe soft tissue injury, severe crush injury 3A - Periosteum intact 3B - Periosteum stripped 3C - Vascular injury requiring repair

What is the use of a skyline view XR?

For clear visualisation of the patellofemoral joint space.

What is the amsel criteria and what is it for?

For diagnosis of bacterial vaginosis, requiring: 3 out of 4 of the following: 1) Thin homogenous vaginal discharge that coats the vaginal wall and vestibule 2) Vaginal fluid pH of > 4.5 3) Positive amine (fish like) odour test before or after addition of 10% KOH 4) Presence of clue cells on microscopy of vaginal discharge (squamous cells surrounded by gardnerally vaginalis and other bacteria.

When are steroids indicated in meningitis and how are they dosed?

For empiral/severe or known N.meningitidis and H.influenzae meningitis (outcome improvement only seen in the latters) 0.15mg/kg 6hrly 15 minutes before or together with antibiotics.

What are the possible causes of pain in the forefoot?

Forefoot pain (generalised vs localised) o Generalised - Metatarsalgia** (most common cause), due to foot strain via deformities/muscular weakness/prolonged use etc that causes faulty weight distribution. - RA - Infection, fractures/stress fracture, tumor o Localised 1) Under first metatarsal head - Sesamoiditis due to trauma/unaccustomed stress/sesamoid displacement/local infection/AVN etc. Walking/dorsiflexion aggravates it - Treat conservatively/infection/operative removal 2) Enlarged second metatarsal head - Friedberg's disease (crushing osteochondroitis of 2nd metatarsal head) - Young adults esp women - XR shows wide and flat 2nd MT w increased density. - Mx: modify footwear w moulded insoles OR surgical synovetomy w debridement and trimming of metatarsal hear) 3) Pain in toes, radiating from forefoot - Morton's metatarsalgia - Due to compression of interdigital nerves (that vary the pain distribution), resulting in 2' thickening of nerves - Signs: tenderness over intermetatarsal space, pain and distal tingling when pressing proximally, clicking when MT heads squeezed together (Mulder's click) - Pain improves if LA injected under transverse metatarsal ligament - Mx w shoe change (padding and widening) OR surgery to reduce compression or excise the 2' neuroma 4) Tumors, infections, fractures/stress fractures

What is sezary syndrome?

Form of cutaneous T cell lymphoma that manifests as erythroderma.

What is a Barton's fracture? How would you manage it?

Fracture-dislocation/subluxation of the wrist due to dislocation of dorsal/volar (volar more common) rim of radius along with hand and carpus. Due to fall on flexed wrist with forearm fixed in pronation Management: - Fracture is by nature unstable - ORIF w buttress plate + screws

What is a Holstein Lewis fracture?

Fractures of the distal third of the radius that impinge on the radial nerve.

What is the classical appearance of a patient with femoral hernia?

Frail, thin, female patients. Due to low fat in the femoral canal, little support.

Where is aeromonas hydrophillia found?

Fresh/brackish water, unlike vibrio vulnificans which is found in sea water, common amongst the damn fishermen.

What are the pediatric symptoms of abdominal pain?

Fretfulness, crying and/or curling up of legs into abdomen

Where does the ligament of Treitz attach?

From the right crus of diaphragm to the duodenojejunal junction.

What is the difference between a full thickness and partial thickness skin graft?

Full thickness includes the dermis Split thickness - less than the entire dermis involved The thicker the skin - More ideal conditions required for survival, higher metabolic demand - Less contraction in healing - Better tolerance to radiation therapy - Less intensive wound care regimen - Cosmesis - Better color match

1) Unbutton shirt 2) Write something 3) Comb hair

Functional assessments to be done in Parkinsonism examination. (3)

What are the sypmtoms of hiatal hernias?

GERD (heart brun, regurgitation, dysphagia) Intermittent epigastric/substernal pain Early satiety, bloating Nausea and vomitting

What are the clinical signs and symptoms of DDH?

Gait - Waddling gait - Limping gait - Delayed walking - Unstable gait Clinical - Limb length discrepancy (femur shortening) - Abnormal flexion creases - Limited ABDUction Special tests - Ortolani - Barlow's - Tredelenberg test

What are the signs of Developmental dysplasia of the Hip?

Galeazzi's sign: unequal knee height when patient is supine, with knees flexed and ankles touching buttocks Inability to abduct equally Barlow's (the bastard): aDDuction with posterior force to promote dislocation. Ortolani's: aBDuction with anterior force to reduce hip joint

How do you spell GAH LEE AH ZI?

Galeazzi's test, fracture etc.

What is the difference between Ganciclovir and aciclovir?

Ganciclovir Vs Aciclovir 1) MOA G: phosphorylated into substrate that competitively inhibits deoxyguanosine triophosphate to DNA polymerase resulting in inhibition of viral DNA synthesis. A: Converted to aciclovir monophosphate by virus specific thymidine kinase and then further converted to aciclovir trisphosphate by other cellular enzymes -> inhibits DNA synthesis and viral replications by competing with deoxyguanosine triphosphate for viral DNA polymerase and being integrated into viral DNA. 2) Indications G: CMV retinitis/pneumonitis/colitis, CMV prophylaxis A: HSV (genital, encephalitis), VZV, Zoster 3) Side effects G: BM suppression, CNS effects (headache, behavioral change, psychosis, coma, convulsion) A: Nephrotoxicity, CNS effects (tremor, myoclonus, confusion, agitation, lethargy, hallucination, impaired conciousness)

Why is ganciclovir better than aciclovir? How

Ganciclovir differs from aciclovir by a single carboxyl chain, a difference that confers about 50 times greater activity against CMV than aciclovir. Ganciclovir is able to reach 10 times the intracellular concentration in CMV cells than aciclovir because it counts on multiple phosphorylation methods for convertion to ganciclovir triphosphate (compared to aciclovir which can only use viral thymidine kinase).

In shortened/impacted neck of femur fractures, what Garden's grade is it?

Garden's 4. Garden's is evaluated on AP view, which doesnt show posterior/anterior displacement, shortened NoF means there is complete fracture with significant displacement.

What are the 2 types of tissues found in meckel's diverticuli?

Gastric and pancreatic

What is Bouveret's syndrome?

Gastric outlet obstruction secondary to gallstone obstruction of the duodenum, passed via a biliary-duodenal fistula.

Mask-like facies, monotonous speech and dyskinesia

General Inspection findings of Parkinsonism (3)

What are the colonoscopy guidelines for general population VS FAP VS HNPCC?

General:

What are the features of Acute Generalised Exanthematous Pustulosis?

Generalized exanthem and pustules. From 48 hours of drug adminstration DDx pustular psoriasis (more annular?) Terbinafine, diltiazem, penicillin etc

What is the pathogenesis and complication of xeroderma pigmentosum?

Genetic disorder that impairs DNA repair after damage from UV radiation. Results in increased risk of sun-related malignancies e.g BCC.

What are the differentials for a child with delay in the speech/language domain?

Genetic/congenital: - Cerebral palsy - Mental retardation - Autism Biological: - Hearing loss/impairment - Receptive aphasia - Expressive language disorder Environmental: - Psychosocial deprivation - Elective mutism - BIlingualism

What is a complication of stopping HAART?

Genital herpes

What are the most common 1' bone tumors in patients 20-30 years old?

Giant cell tumors

In a patient with IHD (CABG, PCI) and ESRF, old age, multiple co-morbids, what is the HbA1c target?

Given the patient status, risk of hypoglycemia and lack of compensatory reserves are more problematic than long term organ damage from poor control. Hence HbA1c targets may be relatively relaxed to about 7-7.9%. FOR ALL HbA1c targets: always qualify by saying 'provided there are no episodes of hypoglycemia'

What shapes of an AAA are there?

Globous Pear shaped Beaked Bilobar Multilobar Fusiform

How would you treat atopic dermatitis?

Go by principles 1) Moisturization - 5-10% urea cream - Aqueous cream - White soft' paraffin - Emollients - Wet wraps 2) Immuodulation - Topical low potency steroids (1% hydroxyzine) for all areas - Moderate potency for adults 3) Trigger removal 4) Manage complications - Bacterial infection - Eczema herpeticum

What are the mamographic findings in breast carcinoma?

Go from inside out: 1) Mass (may be spiculated, with architectural distortion) 2) Microcalcifications 3) Skin thickening 4) Nipple retraction 5) Axillary lymphadenopathy 6) Look at other side for contralateral/multifocal/multicentric masses

What are the causative organisms of septic arthritis?

Gonococcal and non-gonococcal, suppurative and non-suppurative SUPPURATIVE: 1) Gonococcal - Neisseria gonorrhoeae (most common cause in sexually active individuals) 2) Non-gonococcal - Staphylococcus aureaus (most common in adults and children older than 2 years, esp if b/g of RA) - Streptococcal species (S. viridans, S. pneumoniae, GBS in 20%) NON SUPPURATIVE 1) Bacterial - Borrelia burgdorferi (lyme disease) - Mycobacteria 2) Viral - HIV, HBV, rubella 3) Fungal - Histoplasma - Sporothrix schenckii (rose gardener's disease) - Cocciodiodes immitis - Blastomycosis

What are the requirements for a successful Ray amputation?

Good blood supply via the dorsalis pedis and the posterior tibial artery must be patent.

What are the features of good and poor prognosis in Perthe's disease?

Good vs Poor prognosis

Composition: -Gout: Monosodium Urate -Pseudogout: Calcium pyrophosphate Appearance on polarised light microscopy: -Gout: strongly negatively birefringent needle-shaped crystals, yellow when parallel to compensator -Pseudogout: weakly positively birefringent rhomboid-shaped crystals, blue when parallel to compensator

Gout crystals Vs Pseudo gout crystals (3)

What is the implication of the Gower's sign and how is it performed?

Gower's sign implies proximal muscle weakness 1) Patient instructed to lie flat and prone on ground 2) Patient is asked to get into standing position Observe for: 1) Use of upper limbs to 'climb' along floor -> knees -> thighs to stand up 2) Patient mantains a lordotic posture on standing.

What are the risk factors for AVN with regard to the Garden's classification?

Grade 1-2: 10-20% Grade 3-4: 50-80%

What antibiotics would you give depending on the Gustillo-anderson classification of open fractures?

Grade 1: Cefazolin Grade 2: Cefazolin + gentamicin Grade 3: Cefazolin + gentamicin + penincillin

What is the recommended follow-up action for a grade 3 on the Bethesda criteria for thyroid?

Grade 3 - atypia of unknown significance 1) Repeat FNAC, molecular testing or hemithyroidectomy

What is the recommended follow up action for a grade 4 on the Bethesda criteria for thyroid?

Grade 4 - follicular neoplasm or suspicious for follicular neoplasm 1) Molecular testing or hemithyroidectomy

What is the Rutherford classification for chronic limb ischemia?

Grades 0 to 6 0 - Assymptomatic 1 - Mild claudication 2 - Moderate claudication 3 - Severe claudication 4 - Ischemic rest pain 5 - Minor tissue loss 6 - Gangrene/ulceration

What is the Fontaine classifications?

Grades 1 to 4 1 - Assymptomatic 2a - mild claudication 2b - moderate to severe claudication 3 - ischemic rest pain 4 - gangrene/ulceration

What is the classification for trigger finger?

Green Classification 4 grades Grade 1: Palm pain and tenderness at A1 pulley Grade 2: Catching of digit Grade 3: Locking of digit, passively correctable Grade 4: Fixed, locked digit *Btw, A1 is located at metatarsal-phalangeal joints

What is the role of the growth plate, metaphysis and epiphysis?

Growth plate: zone of proliferating, hypertrophying, calcifying and ossifying chondrocytes, site of endochondral ossification. Metaphysis: functions to transfer load from weight bearing joints to diaphysis. Epiphysis: round articulating end of long bone, for pressure bearing and traction via attachment of ligaments/tendons.

When is amputation indicated in Traumatic fractures?

Gustillo-anderson III and above (controversial) Additional indications: - Non viable limb - Severe injury with post op function < prosthesis - Patient unstable, multiple-comorbidities - Personal, social and economic limitations to multiple, prolonged surgery - Injury severity score > 20, high risk of multi-organ/pulmonary failure from high metabolic demand of limb salvage therapy (controversial, scoring systems predict limb salvage poorly)

Is there a papillary thyroid adenoma?

HAHAHA no. 100% malignant

What is the HASBLED scoring and how is it applied?

HASBLED - Hypertension (1) - Abnormal renal function/liver function (1) - Stroke(1) - Bleeding (1) - Labile INR (1) - Elderly > 64 (1) - Drugs e.g NSAIDs, Aspirin (1) - alcohol > 8 drinks/week (1) If score >/= 3, The European Cardiac Society recommends avoiding warfarin as the patient is at high risk of bleeding.

What antipsychotics can be used in hyperactive delirium?

HROQ Haloperidol 0.5mg (IM) Risperidone 0.25-0.5mg Olanzepine 2.5-5mg Quetiapine 25mg (note the multiplication by 10) Start from Halo and work your way up Check the QTC (haloperidol and quetiapine)

What does a high signal zone on MRI indicate?

HSV Found on T2 weighted MRI sequences (water) that is indicative of rupture of disc annulus.

What does hair pulled from tinea capitis look like?

Hair shaft filled w hyphae and spores

What CTTAP findings in neutropenic patients are pathognomonic for aspergillosis? How do you treat it?

Halo sign - Focal hyperdense circular lesions with a surrounding ground glass opacity IV voriconazole

What are the complications related to Peutz Jegher syndrome and it's hereditary pattern?

Hamartomatous polyps Can be lead point of intussuception Can bleed, obstruct, perforate. Increased risk of colorectal cancer. It is inherited via AD (all phenotypic conditions are AD)

Why is abdominal pain important in peutz jegher's syndrome?

Hamartomatous polyps - Lead points for intussuception GIT - Increased risk of colorectal cancer - Increased risk of HBS cancer

What are the options for imaging in Hand Xrays?

Hand: PA, Lateral and Oblique Finger: PA, Lateral Wrist: PA, Lateral

What are the causes of syncope?

Head, Heart, Vessels 1) Head - Hypoxia - Epilepsy - Anxiety - Dysfunctional brainstem 2) Heart - Heart attack (AMI) - Embolism - Aortic outlet obstruction - Rhythm abnormalities - Tachycardia 3) Vessels - Vasovagal - Exsanguination - Subclavian steal - Situational syncope - ENT (glossopharyngeal neuralgia) - Low systemic resistance (DM, autonomic dysfunction) - Sensitive carotid sinus

What are the characteristics of cervicogenic headache?

Headache due to compression of C2 dermatome. Throbbing in nature with - Pain on one side of head/face - Stiff neck - Pain around eyes - Pain while coughing/sneezing - Worse with neck movement/some positions

How do you clinically differentiate NF 1 from 2?

Hearing loss

What are the differences CT findings of hemangioma and hepatocellular carcinoma?

Hemangioma vs HCC CT triphasic Arterial phase - slow peripheral enhancement or rim VS rapid enhancement (HCC well vascularised) Portal venous phase - slow progressive centripetal enhancement VS washing out to look dark against liver Delayed phase - continued filling in, potentially iso/hyper attenuated to liver VS Other features - Bright dot sign that remains hyperattenuating in arterial and PV phases for hemangioma

What are the features of a ganglion cyst and where might it be found?

Hemispherical/multilocular, smooth, well defined. Firm (ocassionally compressible), non tender (usually, unless flexor sheath ganglion) Fluctuant Transilluminable Filled w fluid like uncooked egg Attached to and arising from underlying structures but not to skin Can be found in 1) Wrist: flexor tendon sheath or carpal joints (volar), scapholunate junction (dorsal) 2) Hand: flexor tendon sheath, MCPJ (volar) 3) Digits: flexor tendon sheath (volar), PIPJ (dorsal)

What is the TNM staging of exocrine pancreatic cancers and what management do they entail?

Hence, Stage 1 = resectable, stage 2 to 3 = borderline resectable, stage 4 = not resectable

What are the risk factors of SCC esophageal cancer?

Hereditary hyperkeratosis (aka thilosis) Plumer vinson? esohageal webs, gutzi ring?

What is a hiatal hernia and how would you classify it?

Herniation of elements of the abdominal cavity through the esophageal hiatus of the diaphragm. Can be sliding (type I) or paraesophageal (type II, III, IV) Type I (95%): - Displacement of GE junction above diaphragm - Stomach remains in normal alightment and fundus remains below GE junction - Due to progressive disruption of the GE junction that allows gastric cardia to herniate upwards Type II, III, IV (5%): - Upward dislocation of gastric fundus - Due to abnormally lax gastrosplenic or gastrocolic ligaments that allow greater curvature of the stomach to roll up.

What are the high, intermediate and low risk ultrasound features of thyroid carcinoma?

High (FNA if >/= 1cm) - Irregular margins - Highly vascular - Microcalcifications - Invasion into surrounding structures Intermediate (FNA if >/= 1cm) - Solid - Hypoechoic - > 1cm Low (FNA if >/= 1.5cm) - Hyperechoic - Partially cystic Very low (no need biopsy) - Cyst

What kind of fever does abscess/skin infection present with?

High grade fever.

What is the difference between hip dysplasia and DDH? How would you treat it?

Hip dysplasia occurs in adults or adolescents via: 1) Previously treated DDH (resistant) 2) DDH that was not treated 3) Adolescent onset hip dysplasia Management: 1) Non-operative - Supportive measures as first line 2) Operative - Pelvic +/- femoral osteotomy - hip resurfacing - THR

What is the pathogenesis of Perthes disease in children?

Hip joint effusion secondary to trauma/infection/ non-specific synovitis results in increased pressure on the lateral epiphyseal vessels in retinacula. The disturbance in blood supply causes avascular necrosis of the femoral head, resulting in bone death and collapse + fragmentation. Subsequent healing and remodelling results in progressive deformity and early degenerative disease. Effusion -> necrosis -> death -> remodelling -> deformity

Eye signs - Ptosis - Diplopia Speech signs - Dysarthria - Dysphagia (isolated bulbar palsy in 20%) Respiratory signs - Respiratory failure (1%) Systemic signs - Generalised weakness - Reduced exercise tolerance Generally, trend is extraocular muscles > facial and bulbar muscles > limbs and truncal

How can Myasthenia Gravis present?

1) Inability to bury sclera on abduction (scleral show) 2) Esotropia on corneal reflection test (Hirschberg test)

How can a 6th nerve palsy be clinically detected?

Gout pain manifests maximally within a day while septic arthritis takes more time to build up. There is complete resolution between gouty attacks befoer subsequent flares/precipitants e.g drugs/diet change Vs septic arthritis which is unlikely resolves spontaneously. They can co-exist! Septic arthritis risk is increased in degenerated joints.

How can gout be differentiated clinically from septic arthritis?

1) Echocardiogram findings: - Aortic valve area - Transvulvular mean pressure gradient 2) Presence of LVH (solokov-lyon criteria: S wave depth in V1 + tallest R in V5-6 > 35mm, left strain: ST segment depression w T wave inversion in left sided leads) or 1st degree heart block or LBBB) 3) Presence of complications P.A.C.E (pulmonary hypertension, AF, CCF, endocarditis) 2 and 3 mean severe AS (see other card for features if severe AS)

How do you grade the severity of Aortic Stenosis?

According to GFR 1 = Kidney damage with GFR >/= 90 ml/min/1.73m^2 2 = Kidney damage with GFR 60-90 ml/min/1.72m^2 3 = Kidney damage with GFR 30-59 ml/min/1.73m^2 4 = Kidney damage with GFR 15-29 ml/min/1.73m^2 5 = Kidney damage with GFR <15 ml/min/1.73m^2 or kidney failure treated with dialysis or transplantation

How is Chronic Kidney Disease Graded?

Delta ratio = change in anion gap/change in HCO3 = (12 - anion gap)/(24 - HCO3) <0.4 = Pure NAGMA 0.4 - 0.8 = Mixed NAGMA and HAGMA 0.8 - 2 = Pure HAGMA >2 = HAGMA + concurrent metabolic alkalosis OR chronic compensated respiratory acidosis *Remember to calculate for albumin, whose normal value is taken as 40g/L **Standard value of HCO3 is taken as 24mmol/L

How is Delta ratio calculated and what are the important values?

Urine Output 1) Anuric - patient becomes acidotic very quickly due to inability to excrete acid. 2) Oligouric 3) Normal (0.5-1ml/kg/hr)

How is acute renal injury stratified?

For every 10g of albumin decrease, one needs to increase anion gap by 2.5. Normal albumin is 40g/L

How is hypoalbuminemia adjusted for in anion gap? What is the normal value of albumin?

1) Multidisciplinary Approach (PT/OT, Speech, Swallowing, Neuro etc) 2) Pharmacological therapy - Dopamine angonist - Ldopa therapy - Catechol-O-Methyl transferase (COMT) inhibitors that reduce breakdown of neurotransmitters - Anticholinergics (artane/benzatropine) for tremors - MAOB Inhibitors for rigidity (dopa-agnonist and Ldopa helps) 3) Surgical - Lesion specific: thalamotomy (to reduce tremors), pallidotomy (probe inserted and heated up in globus pallidus I.e basal nuclei to destroy small area of cells) - Deep brain stimulation (for all features esp tremors via electrical neurostimulation at areas depending on symptoms that failed to resolve via medical therapy)

How should Parkinson's disease be managed?

What are the risk factors of contracting S.typhi/paratyphi?

Humans are the only reservoir Spreads through feco-oral route Hence travel to areas of poor sanitation via - Food (Paratyphi) - Water (typhi)

What are the causes of calcified aortic valves in a young patient?

Hypercalcemia - Hyperparathyroidism (1' or 2') - CKD

What is the definition of lung hyperinflation and what are the normal margins of the lung?

Hyperinflation is when lungs exceed 8 ribs anteriorly. Normally lung fields do not exceed 6 ribs anteriorly.

What is the difference between hypertrophic and keloidal scars?

Hypertrophic scar tissue remains within scar borders.

What are the 5 year survival rates for each stage of lung cancer?

I - 80 to 100% II - 60 to 80% IIIa - 40 to 50% IIIb - 30 to 40% IV - 20%

In the classes of haemorrhagic shock, what are the volumes of blood loss of each grade?

I - <750ml II - 750-1500ml III - 1500-2000ml IV - >2000ml

What are the grades of the ASA physical status classification?

I - normal, assymptomatic II - mild disease III - severe disease w functional decline IV - severe disease w high risk of death V - moribund patient that is not expected to survive 24 hrs w/o operation VI - Brain dead patient

What are the causes of acute pancreatitis?

I GET SMASHED by CANCER Idiopathic Gallstone Ethanol Trauma Steroids use Mumps and other infections e.g VZV, HIV, CMV Autoimmune (SLE, Sjogrens) Scorpion toxin and others (e.g organophosphates) Hypercalcemia/hyperlipidemia ERCP Drugs (Sulfamethoxazole, Azathioprine, NSAIDs, Diuretics) Neoplasm

What is the QSofa criteria for sepsis?

I left my HAT on the SOFA 1) Tachypnoea (>22 bpm) 2) Hypotension (</=100 SBP) 3) Altered mental state (GCS <15)

What are the differentials for bloody and mucoid stools and how would you exclude them?

IBD Gastroenteritis - Bacterial (e coli, campylobacter, salmonella, shigella) - Parastitic (entamoeba histolytica) - Viral (rotavirus, norovirus) Other infections: diverticulitis, TB colon Ischemic colitis Bleeding diverticulosis Colon CA

What is the Rutherford grade for an ischemic limb that has lost distal sensation with preserved motor function, as well as inaudible arterial but audible venous waveforms on doppler? And what is the limb risk status and recommended treatment?

IIa. The limb is marginally threatened and will benefit from prompt treatment.

What are the dermatological manifestations of crohn's disease?

INCOMPLETE 1) ORal granulomatosis

What is IRIS and what diseases are associated with it?

IRIS = Immune reconstitution inflammatory syndrome - Collection of inflammatory disorders associated with paradoxical worsening of pre-existing infectious processes following the initiation of ART in HIV individuals. - Paradoxical IRIS: recognized pre-existing infection - Unmasking IRIS: unrecognized pre-existing infection Possible infections: 1) MAC, TB 2) Crytococcal 3) CMV 4) JC virus 5) PJP 6) HSV 7) Hep B/C 8) Kaposi's sarcoma 9) Leishmaniasis

How is each rotator cuff muscle tested in a shoulder exam?

ITS SASI (like calling a puppy dog...with a broken shoulder) 1) Supraspinatus - Involved in ABduction - Get patient to hold inverted beer can and resist downward push 2) SubscapularIs - I for internal rotation - Gerber lift off test: patient puts hand behind back and physician presses the patient's hand onto patient's back, patient attempts to push out - Belly press test (for use if the patient RoM limited and cannot bring hands behind back): patient presses against his/her own abdomen while preventing forward movement of elbows (if elbows move forward -> weakness of subscapularis 3) Infraspinatus and teres minor - Involved in external rotation - Arms tucked into side, resist internal rotation.

What is the treatment of neurosyphillis?

IV Benzylpenicillin for 14 days NOT benzathine penicillin (used for other forms of syphillis)

What is the treatment of acute hyponatremia?

IV Hypertonic saline (3%) 100-150ml bolus over 10-20 minutes for 2-3x Check clinically for GCS improvement or do renal panel if possible

What can you do to ensure circulation in ABC?

IV fluids via 2 large bore IV cannula +/- urinary catheter for urine charting

What is IGH white spots in eldery?

Idiopathic guttate hypomelanosis Biopsy shows lack of pigment cells/production of pigment Classically < 1cm

What is the treatment of developemental dysplasia of the hip?

If < 9 months: Pavlik's harness/abduction harness for 6/52ly over 3-4 months in (100deg flexion and 50 deg abduction) OR can observe for resolution. - Repeat U/S 1/52 in harness to ensure reduction - Repeat U/S in 6/52 out of harness for stability If > 9 months: ADDuctor release (tenotomy) and closed reduction with hip Spica cast

What is Courvoiser's law?

If patient has 1) Jaundice 2) Gall bladder palpable and non tender The jaundice is likely not due to gallstones. (But gallstones can cause intermittent pain, absent at time of examination)

What antibiotics is E.coli resistant to and why? What should you use then?

If they have: 1) Plasmid encoded B-lactamases - Resistance to Penicillins and narrow spectrum cephalosporins 2) ESBLs (extended spectrum beta lactamases) - Third generation cephalosporins and monobactams (e.g aztreonam) - USE cephamycins e.g cefoxitin and cefotetan 3) Cephamycin resistance - Due to outer membrane porin change that decrease flow of B-lactams across membrane - Often comes in ESBL - USE carbapanems!! e.g ertapenem, meropenem

What are the sick day instructions you would give a patient with adrenal insufficiency taking steroids?

If unable to keep tablets down - Call GP - Take 100mg of IM hydrocortisone as soon as possible - If unable, take double dose of steroid medication and seek medical advice In varying degrees of illness 1) Mild to moderate illness - Vomitting, diarrhoea - bad cold, high fever, broken bones - Double dose of steroid for 2 days and then go back to usual dose - E.g on 10-5-5 dose -> 20-10-10 dose for 2 days 2) Severe illness - illness requiring antibiotics or hospital admission - Double dose for 2 days, continue until feeling better or go see doctor Going for procedures - For all procedures: ensure doctor is aware you are on steroid - If going under LA: take double dose steroids for that day only - If going under GA: will require IV hydrocortisone at start of surgery and 6 hourly afterward until able to tolerate orally: then stay on double dose for 2 to 10 days until full recovery before going back to usual dose.

What injury in anterior spinal surgery will result in a hernia?

Illioinguinal nerve injury, leading to weakness in abdominal wall as it innervates the internal oblique and transversus abdominis.

How should erythroderma be managed?

Immediate 1) Supportive treatment 2) Remove/treat underlying cause: drugs, psoriasis, sezary etc Long term 1) Patient education 2) Drug avoidance w drug bracelet

Treatment of severe hypertriglyceridemia (55.7mmol/L)?

Immediate admission for IV heparin and insulin (required if levels >10mmol/L). Start fenofibrates, fish oil, niacin. Watch for acute pancreatitis (increased risk at TG > 4.5mmol/L) Insulin lowers TG by enhancing lipoprotein lipase activity to accelerate chylomicron and VLDL metabolism to free fatty acids, also inhibits lipase in adipocytes to reduce lipolysis.

What is the mechanism of hypercoagulability in heparin induced thrombocytopenia?

Immune-mediated adverse drug reaction that forms antibody complexes with heparin to activate platelets and cause release of prothrombotic platelet derived microparticles and platelet consumption, leading to a thrombocytopenia and paradoxically thrombotic state.

What is the treatment for cryptosporidiosis?

Immunocompetent hosts: - Self limited diarrhoeal disease (no symtoms in days and parasites cleared in months) , can treat conservatively with hydration and proper nutrition - If symptoms are protracted and severe, can consider o Antidiarrhoeal drugs: Loperamide, tincture of opium o Parenteral replacement of fluids and electrolytes o Antimicrobials: paromomycin or nitazoxanide. Immunocompromised hosts: - Disease is severe, debilitating diarrhoea with malabsorption and weight loss. - Aim is immediate start of ART to bring immune response back - +/- antimicrobial therapy of paromomycin or nitazoxanide.

What are the possible pathogens that cause chronic prostatitis?How can you treat this?

In HIV patients: Cryptococcus or dimorphic fungi In DM patients: melioidosis abscess Prostatodynia: prostate pain w/o systemic symptoms and negative cultures or elevated cell count or no improvement after course of abx - Treat with NSAIDs and alphablockers

What is the risk of cancer in MNG vs a single thyroid nodule?

In MNG the OVERALl risk of cancer is about the same as a single thyroid nodule, but cancer usually arises from a non-dominant nodule. Hence, determine which nodule to FNAC via U/S.

1) Mobility - Community/home; ambulant/ambulant with aid/non-ambulant; walking air and duration of need 2) ADLs - Dressing, Eating, Ambulating, Transferring, Hygiene 3) Feeding - Diet type, mode of adminstration (PO, NGT, PGT and IV) 4) Waste excretion - Bowel and bladder incontinence, use of diapers/IDC

In baseline functional assessment of patients, what components are there?

Why and how does it matter if Adults or Children get Monteggia fractures?

In children, the fractures are more likely to be incomplete hence more stable. Thus adults require ORIF sooner than children.

1) Speech assessment 2) Swallowing assessment 3) Handwriting assessment 4) Postural BP 5) Abbreviated mental test

In completion of Parkinsonism examination, one needs to request for: (5)

What are the common mechanisms of injury for a subtrochanteric fracture?

In elderly: common site of pathological fractures, bone here is largely cortical with low vasculature compared to femoral neck and IT (high vasculature) (17-35% due to pathological fracture) In young: high velocity RTA, fall from height or gunshot wounds.

When is post-surgical radioactive iodine indicated in thyroid carcinoma?

In high risk patients: - Metastasis - Lymph node involvement - Extrathyroidal extension - > 4cm - Follicular/hurtle cell

PaCO2 = increases by 0.75 mmHg

In metabolic alkalosis, what is the expected PaCO2 change per 1 mmol/L of HCO3 increase?

In what order does the distal tibial epiphysis ossify?

In order: 1) Central portion 2) Posterior portion 3) Medial portion 4) Anterolateral portion

What is the indication of the IVC filter?

In patients with distal DVTs below the IVC and in patients unfit for anticoagulation

When is Tamiflu indicated?

In patients with influenza community acquired pnuemonia (can detect via rapid antigen tests) where: - Symptoms have persisted > 2 days (immunocompetent individuals can fight off after 2 days) - or severe symptoms warranting hospitalisation - immunocompromised patients

When should bladder rupture be suspected and what investigations should be ordered?

In trauma cases with gross hematuria and pelvic fracture. - Cystogram: fill with at least 300ml of dilute contrast via urethral catheter, do pelvic XR or - CT with delayed phase

At what age does incidence of gastric cancer increase?

Incidence increases steeply after age 50.

Why has patient safety not improved?

Income, Inertia and Invisibility

What is Left Shift of White blood cells?

Increase in number of immature leucocytes in the peripheral blood, particularly neutrophil band cells.

What are the radiological features of Necrotising faciitis? What is the gold standard investigation for it's diagnosis?

Increased soft tissue thickness and opacity, usually normal radiographic studies until the infection and necrosis are advanced. +/- Gas in soft tissue CT - liquefied, necrosis tissue and inflammation resulting from bacterial exotoxins released in the fasical layers. +/- gas Gold standard is intraoperative frozen section.

Causes of increased and decreased vocal resonance?

Increased when there is greater transmission of voices and breath sounds through the lung. Increased in: - Consolidation Decreased in: - Pleural effusion - Pulmonary edema - Atelectasis - Pneumothorax - Pleural thickening

When is surgery indicated for oesophagitis and what does it consist of?

Indicated in o Failure of medical therapy/incomplete resolution of symptoms o Complicated oesophagitis - ulceration, structure, stenosis, Barret's - Respiratory complications o Poor compliance or patient wishes Surgery: - Aim to increase pressure at LES without causing dysphagia. - Fundoplication - Nissen - Partial fundoplication (in foreshortened esophagus bec higher risk of stomach intussuception through plication)

What are the indications and contraindications of halo orthoses?

Indicated in: - Upper Cervical spine (C1-C2) injuries, provides most rigid form of cervical external immobilization (90-95%), provides distraction force to reduce load on C-spine. Contraindicated in: - Lower cervical spine injuries, as it still allows for intercalated paradoxical motion in the subaxial cervical spine (C3-C7) - Bascially, lower Cspine allows for most lateral flexion, which is poorly stabilized by the orthoses, allowing for a 'snaking phenomenon'

What are the indications and contraindications for endo-vascular clot retrieval treatment in ischemic stroke?

Indications: - 4.5-6 hours from stroke onset - NIHSS >/= 6 at baseline - ASPECTs critieria fits - Large vessel obstruction on ICA/MCA - 18-80 y/o Contraindications: - >8 hours - ICH on imaging

What are the indications and contraindications of non-invasive ventilation in COPD?

Indications: 1) Severe dyspnoea with signs of respiratory muscle fatigue or increased work of breathing 2) pH <7.35, PaCO2 > 45mmHg 3) Persistent hypoxemia despite supplemental O2 Contraindications: 1) Facial trauma/fractures 2) Drowsiness 3) Haemodynamic instability

How do you differentiate a vertebral body infection/OM from a metastatic invasion?

Infection always involves the disc while tumors start from the bone.

What is the pathogenesis and presentation of community acquired MRSA? What pathognomonic sign does it exhibit on culture?

Infection by a specific strain of MRSA with the PVL gene (panton valentine leukocidin) coding to an exotoxin. Patient presents (often from America) with recurrent skin abscesses, non-improving on antibiotics. It exhibits the 'D' sign on culture, where upon exposure to erythromycin, surrounding colonies gain resistance to clindamycin, allowing them to encroach on the 'ring' of clinamycin slightly, forming a straight line that looks similar to a 'D' from what would otherwise be an 'O'. Similarly, patients with this strain would be resistant to clindamycin if they are exposed to erythromycin, this ability conferred by the bacteria's erm gene.

What are the risk factors for HCC?

Infective - Hep B and C - Risk of infection: o Sex, IVDA, Tattoos, FmHx (vertical transmission) Irritative - NASH - Alcohol, smoking - Previous liver disease: o Cirrhosis o Autoimmune hepatitis o Recurrent pyogenic cholangitis - Dietary o Aflatoxins o Alcohol o Betel nut

What are the common causes of pain in the Heel?

Inferior vs posterior heel pain 1) Inferior heel pain - Plantar faciitis** (most common) - Infections, tumors, paget's disease, fractures/stress fractures over calcaneum\ 2) Posterior heel pain - Traction apophysitis (pulling osteocondroitis, due to traction injury via achillies tendon, may see increased density or irregularity of apophysis, treat w activity limitation for a few weeks and raising heel of shoe) - Retrocalcaneal bursitis (shoe friction causing swelling and prominence in posteriolateral calcaneum, HAGLUND's deformity seen on XR, change footwear to open back shoes/heel cup in heels to raise bump above back of shoe OR OPERATIVE removal of deformity if conservative tx fails - Others: achilies tendon synovitis, gouty tophi, exostoses

What are some causes of erythema nodosum?

Inflamamtory - IBD - Lymphoma - Behcet's disease - Sarcoidosis Infective - TB - Streptococcal Drugs - Oral contraceptives - NSAIDs - Sulfur antibiotics Pregnancy

What is the pathogenesis of transient synovitis? How frequently is it seen and how is it diagnosed?

Inflammation of the synovium of the hip joint, possible due to previous infections or trauma. E.g URTI, pharyngitis, bronchitis, otitis media, trauma Most common cause of hip pain in 3-10 y/o but is a diagnosis of exclusion.

What is kyphoplasty?

Injection of radiolucent bone cement to prevent further collapse in cases of spinal osteoporosis.

What is the landmark for a parapatella joint aspiration?

Insertion of needle 3-4mm under the lateral/medial midpoint of the patella. Direct the needle toward the femoral intercondylar notch.

What are the features of PVNS?

Insidious onset Multilobulated, well circumscribed, tan coloured joint swelling W dull aching pain Most commonly at volar DIPJ of index and middle finger Firm to soft rubbery Attached to tendon sheath or joint NO TRANSILLUMINATION Non tender

What are the presenting complains ot Pneumocystis jirovecci pneumonia?

Insidious onset with progressive dyspnoea but FEW pulmonary physical signs Cough (non productive) Fever LOW Oral thrush CXR may be normal or with mild infiltrates in early disease

What is the pathogenesis of Slipped Capitofemoral epiphysis? What would patients present with?

Insufficiency/stress fracture through the hyperthrophic zone of cartilaginous growth plate. Results in an upward gliding of femoral neck while head stays in place. Hence patients may present with: 1) Pain (in cases of acute/severe slippage) 2) Limb shortening w external rotation and adducted (Coxa vara deformity) 3) Limited RoM (especially in internal rotation and aBDuction) **Most common cause of hip pain in 11-15y/o

What is meant by Sick Day Advice for hypocortisolism?

Intercurrent illness: >37.5deg for infection/sepsis requiring abx, or in stressful situations: - Double the dose Nausea: - take 20mg hydrocortisone and sip rehydration Vomiting: - Use emergency injection immediately and call a doctor, saying there is an Addison's emergency Injury: - Take 20 mg hydrocortisone to prevent shock Subsequent care: - Ensure doctor, dentist, endoscopist, surgeon knows and provides adequate cover - IV hydrocortisone 100mg pre-op

What is Thomas's sign?

Intermittent bleeding from peri-ampullary tumor manifesting as silvery stool when jaundice wanes. Due to a mix of steatorrhoea and melena.

What cells does GIST arise from and symptoms do patients present with?

Interstitial cells of Cajal i.e gastric pacemaker cells that form the interface between autonomic innervation of bowel wall and smooth muscle. Possible presentation - Bleeding - Incidental - Abdominal pain/discomfort - Early satiety/bloating - Acute abdomen - Abdominal mass - Sx of Mets (liver, peritoneum, lungs rare) - Paraneoplastic syndromes (rare) o consumptive hypothyroidism o non-islet cell hypoglycemia

What is the pathogenesis of Paget's disease of the nipple?

Intraductal carcinoma/invasive carcinoma just beneath the nipple where malignant cells invade across the epithelial-epidermal junction and enters epidermis of nipple -> allowing fluid to be extruded into nipple. Treat according to the underlying cancer or do a wide excision if no cancer found (via PE or mammography).

What is the criteria for disseminated zoster?

Involving more than 3 dermatomal regions.

What are the risk factors for follicular thyroid carcinoma?

Iodine deficiency MNG

On a CT brain, what are the differences between ischemia and haemorrhagic stroke?

Ischemia - +/- hyperdense MCA/vessels (seen in hyperacute setting, suggests embolus) - Loss of grey-white differentiation - Cortical hypodensity (2' to edema) Haemorrhagic stroke - White bleeding, with black peripheral edema - Loss of grey-white differentiation - Mass effect: Ventricular compression, midline shift

What is a Nightstick fracture? How would you manage it?

Isolated ulnar fracture with minimal displacement Due to holding arm up to protect from blow to face. Manage: - Below elbow cast for 6 weeks - ORIF if angulation severe

In a young man with severe sudden onset septicemia (due to meningococcemia), where would you place him and what antibiotics would you start him on?

Isolation with droplet precaution for at least 24 hours after appropriate abx given. Give 2g ceftriaxone 8hrly

How many ossifications centres are there in the tibia and what is the ossification sequence of the tibial tuberosity?

It has 3, which ossify via endochondral ossification 1) Diaphyseal (primary ossifcation centre) 2) Upper epiphysis (includes the tibial tuberosity) 3) Lower epiphysis RE: tibial tuberosity i.e upper ossification centre <11y/o tubercle is cartiliginous 11-14y/o apophysis forms 14-18y/o apophysis fuses with tibial epiphysis >18y/o epiphysis (and apophysis) is fused with rest of tibia.

What is the Golden S sign and what does it imply?

It implies a obstructive cause of upper lobe collapse. It is formed by a collapsed upper lobe moving medially and upwards with a centrally arising mass e.g bronchogenic carcinoma or other mediastinal/central masses on CXR.

Why do they throw in probenecid in treatment of neurosyphyllis?

It inhibits tubular excretion of penicillin, increasing it's serum concentrations up to 2 to 4 times.

What is the significance of S.aureus found in a urine culture?

It is NEVER a contamination, definitely pathological and source must be identified. To check - Line - Catheter - Prostate etc - TRO bacteremia

What is the pathogenesis of myotonic dystrophy?

It is a autosomal dominant disease with excessive CTG replications in the MDPK gene (myotonic dystrophy protein kinase), manifesting with myotonic and progressive muscle weakness (along with other manifestations) with anticipation across future generations. It usually manifests in the 3rd to 4th decade.

What is the pathogenesis of pigmented villonodular synovitis and what are the types?

It is a benign proliferation of the synovial lining in joints/bursa/tendon sheaths, resulting in localised inflammation/thickening of synovial membrane. Can be diffuse or localised 1) Diffuse: affects entire synovial membrane thus whole joint e.g knee, hip, ankle, shoulder 2) Localised: affecting tendon sheaths of small joints hence nodular tenosynovitis

What is the definition of cervical myelopathy and what are the causes?

It is a clinical syndrome caused by compression of the spinal cord that is characterised by clumsiness in the hands and gait imbalance. 1) Degenerative - spondylosis - OPLL 2) Deformity - congenital stenosis - cervical kyphosis 3) Tumors 4) Infection - Epidural abscess 5) Trauma Neurological findings can be due to direct compression or compression of the anterior spinal artery.

What is a pilon fracture and what is the mechanism of injury? What types are there?

It is a comminuted fracture of the tibial plafond, involving the talus and distal tibial metaphyses with articular extension +/- distal shaft involvement MOI: severe axial compression of the ankle joint, driving the talus into tibial plafond. Rudi-Allgower classification, 3 types: Type 1: undisplaced Type 2: minimally displaced Type 3: markedly displaced

What is the pathogenesis of tarsal coalition and what types are there (how may each type present? Any radiological studies?)?

It is a congenital connection between 2 or more tarsal bones. AD in dominance and bilateral in 50% of cases Can present as 1) Talocalcaneal (CT scan to see) - Rarer - Presents as constant activity-related pain, significant reduction in subtalar motion and tender medial subtalar joint 2) Calcaneonavicular (Oblique foot XR) - Most common - Causes intermittent symptoms - Little reduction in subtalar motion - +/- Talar beak

What is arthrogryposis?

It is a congenital contracture of 2 or more areas of the body

What is the pathogenesis of a liver hemangioma?

It is a congenital vascular malformation that enlarges to due ectasia.

What is a butterfly fragment of the humeral shaft fracture? What mechanism of injury causes it?

It is a large, triangular fragment found in com-minuted fractures. Due to torsional and bending forces on the humeral shaft.

What is Balthazaar's score?

It is a scoring system for radiological stratification of the severity of pancreatitis. Looking at the presence of necrosis and other complications?

What is colophony and why do we give a shit?

It is also known as rosin, derived from pine sap. Adhesive and chemical properties means it finds it's way into may cosmetic, medicinal etc products that we use. e.g salicylic plasters, opsite It is also within the top 10 causes of allergic contact dermatitis.

What is the pathogenesis of dermatomyositis?

It is an idiopathic inflammatory myopathy with characteristic cutaneous signs.

What is the inheritance pattern of neurofibromatosis?

It is autosomal dominant, with close to 100% penetrance.

What is the natural history of PVNS? What age group does it typically affect?

It is benign but locally invasive. Gradual growth with increasing involvement of joint causing progressive hand dysfunction 20-50 year olds

What is the definition of critical limb ischemia?

It is defined as rest pain > 2 weeks OR ulceration/gangrene WITH ankle pressure < 50mmHg or toe pressure < 30mmHg or ABPI < 0.5

What is the significance of HbcAg? How is it different from HbeAG?

It is the Hepatitis B viral core protein Usually only found in hepatocyte biopsies because it is not secreted into the patient's blood. While HbeAg is secreted into the blood and may have effect in inhibiting immune response to viral replication. Both are markers of infectivity and active viral replication, just found in difference places. Presence of HbCAg can be inferred from presence of anti-HbC antibodies in blood.

What is the terry thomas sign, what causes it and how can it be managed?

It is the widening of the scapholunate distance, Indicative of scapholunate dissociation from ligamentous injury 2' to fall on outstretched hand or degenerative changes Non-operative - Indicated for acute, undisplaced or incomplete tears - NSAIDS - Rest - +/- imobilisation Operative - Ligament repair (if reducible) - ORIF (if not reducible and a/w scaphoid #)

Another reminder question, what does myositis ossificans look like?

It looks like a sheet of calcification in muscles and soft tissue, resulting from direct or intramuscular trauma. Commonly occurs in elbow and hip from traumatic joint dislocation reductions.

What does a positive straight leg raise mean? What are your differentials?

It means nerve root tension. Most common cause if prolapsed intervertebral disc, but other possibilities like schwanoma, mass lesions with compression.

How does William's syndrome cause aortic stenosis?

It results in a congenital narrowing of large blood vessels, manifesting as supravalvular aortic stenosis.

What is the pressure to be applied in an underwater seal for a chest tube?

It should be equivalent to negative intrathoracic pressure at -20 cmH2O

What is the Monroe-Kellie Doctrine?

It states that the sum of volumes of brain, CSF, intracranial blood is constant and incompressible. Thus increase in volumes of either blood or CSF against poorly compliant brain tissue will result in raised ICP. Likewise if brain tissue expands ICP will increase.

What are the features and pathogenesis of thyrotoxic periodic paralysis?

It usually manifests in the late evening or early morning as a proximal, bilateral flaccid weakness/pain/cramping/stiffness progressing from the legs to the arms, resolving completely between episodes lasting hours to days and triggered by physical exertion/alcohol/ carbohydrate and salt-rich foods. It is associated with hyporeflexia and sensory function is maintained. Pathogenesis is due to underlying defects in several ion transporter/exchanges that result in increased Na/K ATPase activation in high thyroid hormone circulation, resulting in hypokalemia that manifests as MSK and CVS signs.

What is the difference between pseudojones and jones fracture? Is there another possible distal 5th metatarsal fracture?

Jones - Zone 2 fracture - Metaphyseal-diaphyseal junction - Involves 4th and 5th metatarsal articulation - Is at a vascular watershed area (hence increased risk of non-union) Pseudojones - Zone 1 - At the proximal tubercle (rarely enters the 5th tarsometatarsal joint) - Non union is UNCOMMON Zone 3 - Proximal diaphyseal fracture - Stress fracture site in athletes - A/w cavovarus foot deformities of sensory neuropathies - Increased risk of non union.

What is the diagnostic criteria of rheumatic fever?

Jones Criteria: 2 Major criteria OR 1 Major criteria + 2 minor + PmHx streptococcal infection Cardiac.MESS.FRAP 1) Major - Carditis (peri, endo, myocarditis) - Migratory polyarthritis (rapid onset of swelling in 1-2 joints with resolution over next few days) - Erythema marginatum - Sydenham Chorea (jerky movements) - Subcutaenous nodules 2) Minor - Fever - Arthralgia - Raised acute phase reactants - Prolonged PR interval

What is Lemierre's syndrome? What pathogen causes it and what are the initial presentations and complications?

Jugular vein thrombophlebitis of Fusobacterium necrophorum leading to sepsis & septic pulmonary emboli Presents with initial fever, sore throat and malaise lasting for days before progression to pulmonary signs (dyspnoea, pleuritic chest pain) Due to septic embolus moving down to Right heart and to lungs.

For dermatomal testing in the ASIA score, what are the landmarks for testing L1 to S1?

L1 : medial half of proximal thigh L2: Mid thigh L3: medial knee L4: medial malleolus L5: dorsum of foot/1st webspace S1: heel of foot

For myotomal testing in the ASIA score, what are the muscles for testing L2 to S1?

L2: hip flexion L3: knee flexors L4: ankle dorsiflexors L5: long toe extensors (EHL) S1: Ankle plantar flexors

What kind of gait do patients with L4 and L5 deficits present with respectively?

L4 - foot drop = high steppage gaing L5 - tredenlenberg gait = loss of gluteus medius via superior gluteal nerve

What is the level of the bifurcation of the aorta?

L4-5

When is low anterior resection, ultra-low anterior resection and abdominoperineal resection indicated in rectal cancer?

LAR - >7cm from anal verge (2cm from dentate line, which is 5 cm from anal verge) ULRA - 6 cm from anal verge APR - < 6cm from anal verge

What are the complications of lignocaine overdose and its management?

LAST (Local anesthesia systemic toxicity) CNS - Dysarthria - Perioral numbness - Confusion/altered mental status - Seizures CVS - Arrythmia - Tachycardia - Hypotension Treatment is ABC SLE - Ensure ABCs - Seizure suppression (via BZD) - Lipid emulsion therapy - ECMO (Extracorporeal membrane oxygenation)

What are the components of the LIRINEC Score and what does it prognosticate?

LIRINEC prognosticates the likelihood that a patient had necrotizing faciitis. Parameters included are: 1) CRP 2) WBC 3) Hb 4) Sodium 5) Creatinine 6) Glucose if < 6 considered low but present risk

What is a collar stud abscess?

LN abscess 2' TB

What is the difference between a laminectomy and laminoplasty?

Laminectomy - removal of the lamina, hence requires fusion and instrumentation to compensate for loss of stability to prevent progressive kyphosis. Laminoplasty - opening of the lamina while preserving bony structure, requires re-attachment of muscles, more stability but less degree of decompression.

How do you spell LASAG's test?

Lasegue test

What is the course of the small saphenous vein?

Lateral end of dorsal venous arch of foot Passes posterior to lateral malleolus Runs up Midline of calf Pierces deep fascia of popilteus to drain into popliteal vein

What are the possible ankle fractures and which are usually stable/unstable?

Lateral malleoli (only fibula) - common - Can usually be treated w/o surgery Medial malleolus (tibia only) - usually treated with surgery, esp if displaced Bimalloelar ankle fracture - Usually unstable - Surgery required Bimalleolar equivalent fracture - Fibular fracture + ligaments torn - Also unstable - Also need surgery Trimalleolar fracture - Posterior malleolus also involved - Need surgery Maisonneuve fracture - Medial malleolus fracture/deep deltoid ligament tear) with proximal fibular fracture and torn syndesmosis - Unstable, requires surgery

What are the lifespans of latex and silicone urinary catheters?

Latex (brown) - 2 weeks max Silicon (grey/transparent) - 3 months Remove as early as possible and swap to silicone if discharging/tell the nursing home

What is Goodsall's Law? Are there any exceptions?

Law for relating the external opening of an anal fistula to it's internal opening. If perianal skin opening is posterior to the transverse anal line (line drawn through ischial spines), the fistulous tract will open into the anal canal in the midline posteriorly, sometimes taking a curvilinear course. Also, an external opening anterior to the transverse anal line is usually associated with a radial fistulous tract. Exceptions exist 1) if the fistula is more than 2.5-3cm from the anal verge, where the fistula is almost always indirect 2) If there are multiple anal fistulas, their courses are more likely curvilinear due to the branching and communication of the openings.

How do you treat scrotal cysts?

Leave them alone

Which side is diverticular disease more common?

Left more than right but Asians trend to have it in the right. Right sided has better prognosis

What are the levels of axillary clearance and how do you landmark them?

Level 1 is below the lower edge of pectoralis minor Level 2 is underneath the pectoralis minor Level 3 (most superior) is above the pectoralis minor Traditional LN clearance removes level 1 and 2.

What lymph nodes are likely affected in thyroid carcinoma and when are they removed?

Level 5 (lateral neck) - Therapeutically removed (if clinical/radiological evidence of spread) - NOT prophylactically removed. Level 1-6 (central) - Therapeutically removed - Prophylatic removal is controversial

What are the causes of flexible pes planus?

Ligament: generalised ligament laxity, posterior tendon dysfunction Muscular: tight tendoachillies/gastrocnemius, posterior tendon achillies

How is radiolabeled lipoidal computed tomography useful in assessment of hepatocellular carcinoma?

Lipoidal is an oily compound. In normal liver, Kupffer cells (macrophages) line the sinusoids and are involved in lipid metabolism. Due to their absence in hepatocellular carcinoma, lipodals remain present longer, allowing an enhancement on imaging specialised to detect them. Hence it plays a role in detection/diagnosis of HCC as well as evaluation of post-TACE tumor presence.

What are the pathogens that cause recurrent pyogenic cholangitis?

Liver flukes: clonorchis sinensis, opisthorchis viverrini Intestinal round worms: Ascaris lumbricoides

What are the causes of pnuematosis intestinalis?

Local causes - Bowel ischemia - infection - iatrogenic (endoscopy, colonoscopy, operative, enteric tube placement, CT colonography) Systemic causes - Autoimmune disease and immunosuppression - connective tissue disease - pulmonary disease (COPD, asthma, cystic fibrosis) - Medication induced (steroids, chemotherapy)

How do thyroidectomies result in tracheomalacia?

Long standing goitres, when removed, result in sudden loss of structural support that predisposes the trachea to collpase.

What structures are at risk of injury during axillary clearance?

Long thoracic nerve, innervates the serratus anterior Thoracodorsal nerve, artery and vein that supply the lattisimus dorsi. Median pectoral nerve that supplies the lateral portion of pectoralis major. Axillary vein and artery and brachial plexus are also at risk but not normally exposed (means you cut too high).

What is the are the differences between a burst and compression fracture?

Look at anterior vertebral line - If AP diameter (view on lateral) is same or increased -> burst fracture (due to anterior displacement of vertebral body) Look at vertebral body - Increased in pedicular distance -> burst fracture (due to flattening and widening of vertebral body) Anatomical: - Compression fracture: involves anterior and middle portion of vertebral body - Burst fracture: involves the posterior portion as well Mechanism: - Compression fracture due to degenerative changes - Burst fracture due to traumatic axial forces

What is the are the differences between a burst and compression fracture?

Look at anterior vertebral line - If AP diameter (view on lateral) is same or increased -> burst fracture (due to anterior displacement of vertebral body) Look at vertebral body - Increased in pedicular distance -> burst fracture (due to flattening and widening of vertebral body) Anatomical: - Compression fracture: involves anterior and middle portion of vertebral body, less severe than burst: which affects the entire vertebrae - Burst fracture: involves the posterior portion as well, with RETROPULSION fragment

What are the radiological and management differences between an extraperitoneal and intraperitoneal bladder perforation?

Looking at the contrast flow in a Cystogram: 1) Extraperitoneal if contrast confined below level of acetabular roof - Conservative treatment w cath drainage for 10-14 days - Cystogram after to confirm healing - Surgical repair only if: - patient already in OT for other reason - associated rectal perforation/open pelvic fracture - Bone fragments projecting into bladder - Multiple/large ruptures 2) Intraperitoneal if contrast flows beyond level of acetabular roof - Surgical repair indicated - Open cystostomy - Multi-layer closure of bladder injury - Bladder drainage via IDC +/- suprapubic catheter - Perivesical drain

Which antihypertensive is preferred in patients with gout?

Losartan The only ARB that is associated with reduced risk of gout.

What is the Crowded Carpus sign?

Loss of intercarpal spaces, with multiple superimposition of carpal bones, due to 1) Perilunate fracture dislocation/lunate dislocation 2) With scaphoid fracture

What are the complications of erythrodermic psoriasis?

Loss of skin cover - Dehydration - Infection - Hypothermia Organ failure - Heart failure (high output) - Odema - Protein loss and malnutrition - Death

What causes erythema migrans?

Lyme disease

What are some differentials for a swelling at the angle of the jaw?

Lymph node Swelling of the inferior pole of the parotid

What mode of spread does papillary thyroid cancer spread by?

Lymphatics Hence need to assess for lymphatic spread via clinical exam and radiological imaging. Hence need to follow up after resection, check for lymphadenopathy.

How can vasculitides be classified histologically?

Lymphocytic Vs Lymphocytoclastic Vs Granulomatous vasculitis 1) Lymphocytic 2) Lymphocytoclastic 3) Granulomatous

What is the Alvarado score and what numbers are significant?

MANTRELS - Migrating of pain to RIF (1 pt) - Anorexia (1 pt) - N&V (1 pt) - Tenderness in RIF (2 pt) - Rebound pain (1 pt) - Elevated temperature (1 pt) - Leucocytosis (2 pt) - Shift of WBCs to left (1 pt) 0-3 points can d/c w/o imaging 4-6 points should do imaging study > 7 points should arrange GS consult

What must you counsel the patient on before starting steroids? (MAP IQ format)

MAP IQ (format for all meds counselling) 1) Mechanism - Hormones, reduces inflammation - Different from anabolic steroids - Useful because of disease etiology/pathophysiology 2) Adverse effects + follow ups - 2' infection from immunosuppression (wear mask, avoid sick people, hand washing) - HTN (must come back to BP monitoring or self monitor) - DM (look for 3 Ps of DM and go for check up if seen) - Osteoporosis (calcium and Vit D suppplements provided) - Appetite increase - Skin thinning (bruising) - Acne - Mood change - Dyspepsia (PPI, antacids provided/can buy) - Must be careful not to stop taking (body will produce less, if lacking - confusion, abdominal pain, need go hopsital) PmHx - Osteoporosis - Epilepsy - Excessive bleeding - Stomach ulcers - DM - Be sure to tell your doctors you are taking Interactions/allergy - Avoid NSAIDs - Any concurrent medications - Drug allergy - Have you taken before Questions/brochure - Any questions - Nurse outside will give you a brochure

What is the course of the great saphenous vein?

MAPAC Medial end of dorsal venous arch of foot Passes Anterior to medial malleolus Runs up Posterior leg to pass behind medial surface of knee Then runs Anterolaterally up thigh Pierces Cribriform fascia at saphenofemoral junction to drain into femoral vein

What are the features of drug exanthem, what drugs can cause it and how soon does it start? How do you treat it?

MAculopapular rash cephalocaudal or cetripedal spread ABX AntiTB Anticonvulsants NSAIDS etc Days to 2 weeks after drug administration Mx - Rule out organ dysfunction (DRESS) - Stop drug - Topical steroids and anithistamins

What are the possible combinations of HIV treatments for different patients?

Main combination: 2 NRTIs + 1 other drug 1) Most popular NRTI combo - Tenofovir + emtricitabine OR - Abacavir + lamivudine (aba needs hypersensitivity and CVS screening) 2) Other drug can be - NNRTI (efavirens most popz, but teratogenic)) - Ritonavir (P450cyp inhibitor to boost conc) + Protease inhibitor (lopinavir, darunavir, atazanavir etc) - Integrase inhibitors (raltegravir but needs BD dosing)

What are the causes of appendicitis?

Main pathogenesis is obstruction: 1) Mechanical obstruction: - Fecoliths - Foreign body - Parasites 2) Inflammatory obstruction: - Inflammatory bowel disease - Systemic infections (GE, URTI, IMS, measles, TB, parasites) All which may lead to lymphoid hyperplasia 3) Malignancy obstruction - GIT tumor

What is the 5 year survival rate after a major amputation?

Major amputation = BKA or AKA < 50% 5 year survival after amputation, where mortality stems from mostly stroke and AMI.

What is the pathogenesis of chondromalacia patellae?

Malcongruence of patellofemoral surfaces or malalignment of the extensor mechanism (large Q angle), leading to softening and degeration of articular hyaline cartilage of the patella.

Who gets Osgood schlatter disease?

Male > female Male: 12-15y/o Female: 8-12 y/o (pubertal age?) Jumpers (basketballers, volleyballers) or sprinters.

What is the gender distribution of thyroid carcinoma?

Male with thyroid nodule = higher risk of cancer Females overall higher risk of cancer because more of them have nodules.

What are the causes of lymphocytic rich pleural effusion?

Malignancy and tuberculosis most common. - Infective (Parapneumonic effusion, TB, abdominal organ infection e.g hepatitis, pancreatitis) - Autoimmune (sarcoidosis, rheumatic pleuritis) - Malignant (lung CA, mesothelioma) - Traumatic (chylothorax cisterna chyli rupture)

What is the management for Hasiomoto's thyroiditis?

Management consists of treating the acute hyperthyroidism as well as the subsequent hypothyroidism 1) Acute hyperthyroidism o Symptomatic management - Propanolol (BBlockers) 2) Subsequent hypothyroidism - Replacement of L thyroxine or T3 - Aim TSH below 3mIU/L

Acute immune-mediated reactions: 1) Stop transfusion and check blood match 2) Treat 2' hypotension/breathlessness/anaphylaxis supportively 3) KIV antipyretics + (for TRALI: ventilator support and leukoreduced products) Acute non-immune mediated reactions: 1) Stop transfusion and check blood match 2) Treat 2' hypotension, dyspnoea or fever supportively 3) Investigate for sepsis (w C/S) and DIVC (w FBC, PT/PTT, clinically) 4) If fluid overloaded: provide diuretics, slow transfusion rate and KIV ventilator support Chronic immune-mediated reactions: 1) Either self resolving (delayed transfusion reaction) or high mortality (GvHD) 2) Prophylactic investigations + leukoreduction AND irradiation Chronic non-immune mediated reaction: 1) Iron chelation 2) Pre-tranfusion screening or treat pathogen as appropriate.

Management of transfusion reactions

1) Conservative: education 2) Medical: Antibiotic prophylaxis, Treatment of complications e.g AF and CCF, Statins possible role in reducing calcification 3) Surgical: indicated if symptomatic and severe OR assymptomatic but has area <0.6, LV dysfunction, hypotension on exercise, VT, LVH > 15mm or moderate AS but going in for other cardiac surgeries e.g CABG - Valve replacement - Valvuloplasty (for moribund/palliative patients unable to undergo surgery)

Management options of Aortic stenosis and the criteria for surgical therapies.

Why is a large bowel loop colostomy higher risk than an ileostomy?

Marginal artery is lone supply between the ends, damaging it will cause necrosis of distal bowel.

What factors determine recurrance of BCC?

Margins, Size, histology, location

General Inspection findings of Parkinsonism (3)

Mask-like facies, monotonous speech and dyskinesia

What is radical mastectomy and when is it indicated?

Mastectomy that involves removal of the chest wall muscles namely the pectoralis major, along with axillary lymph node clearance. It is indicated when there is advanced local spread into the muscle as surgical margins require one layer above and below the mass to be resected.

What are the maximum doses of lignocaine +/- adrenaline and bupivucaine +/- adrenaline in a 70kg patient?

Maximum doses of Local Anesthesia: 1) Lignocaine (3mg/kg body weight) a. 1% (i.e 10mg/ml) = 21ml in a 70kg person 2) Lignocaine w adrenaline (7mg/kg body weight) a. 1% (10mg/ml) = 49ml in a 70kg person b. Contraindicated in terminal capillaries due to risk of ischemia 2' to vasoconstriction 3) Bupivucaine (2mg/kg BW) Marcaine = bupivucaine + adrenaline (4mg/kg)

What is the treatment of bacterial vaginosis?

May no require treatment: only if complicated by infection 1) Antibiotics - Intravaginal creams: metronidazole/clindaymycin - Oral metronidazole 2) Avoid douching 3) pH rebalancing - Boric acid pessaries - Lactigel/actigel 4) Vaginal antiseptic - Dequalinium

What are the complications

Mechanical GIT -

How does the intra-aortic balloon pump work?

Mechanical device with polyethylene balloon that deflates in systole to increase forward flow and inflates in diastole to increase retrograde aortic flow to the coronary arteries. Helium used: smaller and denser molecues that can flow faster and is less compressible. Also non-combustible vs oxygen. Indicated in refractory ventricular failure, cardiogenic shock, unstable refractory angina etc

What is the mechanism of ACL tears? Hence what is the ACL responsible for stability in?

Mechanism is usually a non-contact pivoting injury. ACL works to prevent anterior translation of tibia relative to the femur and actas as a secondary restraint to tibial rotation and varus/valgus rotation.

What are the borders of the Hasselbach's triangle?

Medial border: lateral border of rectus abdominis Lateral border: Inferior Epigastric artery and vein Inferior border: Inguinal ligament

What are the definition and differentials for widened mediastinum?

Mediatinum > 6cm (on erect PA) or 8cm (on supine AP) measured at above the cardiac borders. 1) Mediastinal Mass - Thymoma - Tuberculosis (Lymphnode) - Teratoma - Terrible lymphoma 2) Vascular - Aortic dissection (watch the aortic notch closely)/traumatic aortic injury - Aortic aneurysm - Anomalies (unfolded aorta, double SVC, aberrant subclavian artery) - Enlarged pulmonary arteries 3) Pneumomediastinum 4) Lung - atelectasis - Pulmonary masses extending into mediastinum 5) Technical factors: - Rotation - Poor inspiration - Supine position - Lordotic position

When do you start antiplatelet therapy in diabetic patients?

Men >/= 50 Women >/= 60 w risk factor: - smoking - HTN - LDL > 2.6 mmol/L - FmHx - Albuminuria Can start clopidogrel

What are the causes of Chronic Paediatric Abdominal pain?

Menstural causes: - Physiological (1st 2 days of cycle) - Mittleschmerz (mid cycle pain) - Endometriosis (severe progressive dysmenorrhoea > 2 days) - Pregnancy (amenorrhoea) Non-menstrual causes: 1) Complicated/prolonged acute causes of abdominal pain 2) GIT - Inflammatory bowel disease - Irritable bowel syndrome - Celiac disease - Tumors/malignancy 3) Metabolic - Porphyria - Sickle cell crisis - Lead poisoning

How is a femoral hernia surgically managed?

Mesh plugging, secured with stitching to 3 sites (not laterally as femoral vein is there)

1) Patellar tap (60cc) Milk suprapatellar pouch -> gently press on patella to assess for tap on femur 2) Cross-fluctuance (30-50cc) Milk suprapatellar pouch -> w hand placed on lateral, apply pressure on medial side to feel for flow 3) Fluid shift test (10-20cc) Milk suprapatellar pouch -> stroke medial to empty, then stroke lateral to watch for refill of medial.

Methods for evaluating joint swelling

What causes miliaria rubra and cholinergic urticaria.

Milaria rubra - obstruction of sweat glands in the deeper epidermis, manifesting as red, itchy, follicular rash when body temp rises. Cholinergic urticaria - small 1-4mm weals appearing when sweating/body heat rises, itchy with surrounding red flares.

What is the classification of lateral condylar fractures?

Milch classification Type I: Capitellum affected, Salterharris IV, fracture is stable. Type II: Capitellum is not affected, fracture line passes to capitotrochlear groove, Salterharris II but unstable.

How do you classify acne vulgaris?

Mild - Comedome - Few/superficial inflammatory lesions Moderate - Inflammatory lesions present Severe - Nodular

How do you grade the severity of a mitral regurgitation?

Mild - no pulmonary hypertension Moderate - pulmonary hypertension Severe - LVH, S3

What is Miller Fischer syndrome and it's presenting symptoms?

Miller fischer is a variant of guillian barre (ascending paralysis) that manifests with a triad of classical symptoms. 1) Opthalmoplegia/ diplopia 2) Ataxia, waddling duck-like gait 3) Reduced deep tendon reflexes +/- dysarthria

What are the fasting guidelines in preparation for operations?

Minimum 6 hours for milk and food. Minimum 2 hours for clear fluids

What is the pathogenesis of inverted supinator jerk?

Misnomer, is actually the inverted brachioradialis jerk. Due to relative impairment of C5-6 reflex (due to cervical myelopathy) with relative loss of inhibition of C7-8 reflex. Hence normal brachioradialis reflex is reduced and fingers flex in response to C8 reflex.

What is the characteristic murmur of mitral stenosis and what are the possible differentials?

Mitral stenosis: - Mid diastolic murmur loudest in the Apex +/- opening snap - Accentuated by placing the patient in the left lateral position and in held expiration Differentials: - Tricuspid stenosis (Rarer and louder in inspiration) - Atrial myxoma (rare, with fever and constitutional symptoms) - Austin flint murmur (AR murmur radiating back)

What are the risk factors for SCFE?

Modifiable - Obesity Non-modifiable - Family history - Gender(M:F = 2:1 just like transient synovitis) - Previous trauma - Precocious puberty - Ricket's - Down's syndrome if BILATERAL SCFE (1/3 of cases), think Endocrinopathies: - Hypogonadism, hypothyroidism, hypopituitarism, growth hormone deficiencies

What are the risk factors for RCC?

Modifiable - Smoking - Obesity - Occupational exposure to toxic compounds e.g cadmium, abestos - Chronic heavy use of NSAIDs Non-modifiable - HTN - Polycystic kidney disease - FmHx - Inherited kidney cancer syndromes: o Tuberous sclerosis o Von hippel lindau o Hereditary papillary renal carcinoma

What is the diagnostic criteria of Marfan's syndrome?

Modified Ghent criteria places emphasis on aortic root dilation and displaced lenses(ectopia lentis). Where if: 1) Both are present = diagnosis 2) 1 with 7 or more skeletal signs

How do you spell MON TEE GIUH?

Monteggia's fracture

What are the differences between Moore's and Thompson's hemiarthroplasty?

Moore's (i.e medullary prosthesis) - Anchored in medullary canal - Fixed via press fit or bone cement - Fenestration to facilitate bone growth and increase blood supply Thompson's - Rests on intertrochanteric line - No fenestration

Who gets the damn Monteggia fractures?

More common amongst children, especially them 4-10 year olds.

What are the factors and mechanisms that increase the risk of supracondylar fractures in pediatric patients?

Most common elbow fracture (55-75%) 1) Remodelling - Bone remodelling in 5-8 y/o results in decreased AP diameter in supracondylar region 2) Ligamentous laxity - common in this age group 3) Capsular irregularity - Anterior capsule stronger and thicker than posterior - In extension, anterior capsular fibres are taut and serve as fulcrum to firmly engage olecranon in it's fossa Mechanisms of inury: 1) Hyperextension (more common) - Fall on extended elbow causing hyperextension - If pronated = posteroMEDIAL displacement - If supinated = posterioLATERAL displacement 2) Flexion - Direct trauma or fall on flexed elbow

What is the Klatskin tumor?

Most common form of cholangiocarcinoma, where tumor arises above the junction of right and left hepatic ducts, AKA hilar cholangiocarcinoma

What are the mechanisms of injury of proximal humeral fractures?

Most common is falling onto an outstrecthed upper extemity from a standing height, typically in an older woman. As usual, younger patients get it from high energy trauma e.g RTA, fall from height. Less commonly, - Excessive shoulder ABduction in individual with osteoporosis (greater tuberosity prevents further ABduction), - Direct trauma - Electrocution/seizure - Pathological fractures

What are the types of melanoma?

Most common: Acral lentigious (15% in ASIANS) - Not a/w sun exposure Others: - Superficial spreading - Nodular - Lentigo maligna

What are the common presenting complaints of patients with abdominal aortic aneurysms?

Most commonly assymptomatic! Incidental finding of abdominal expansile mass. Possible symptoms: 1) Back pain 2) Tenderness on palpation Complications: 1) Compression - Ureter 2) Obstruction - Distal: distal thromboembolism -> trash feet - Branches: iliac/renal /mesenteric/ vertebral etc 3) Rupture - Intense abdominal pain radiation to back - Rapid hypotension and shock

What is the mechanism of injury in an elbow dislocation and what are the associated fractures?

Most commonly due to fall on outstretched hand or elbow, resulting in levering force to unlock the olecranon from the trochlea, with translation of the articular surfaces to result in the dislocation. Anterior dislocation: Direct force striking the posterior aspect of the flexed elbow. Posterior dislocation: Combination of elbow hyperextention, valgus stress, arm ABduction and forearm supination with 2' injuries to the capsule, collateral ligaments (esp medial) and musculature -> bascially like a Colles fracture position Associated fractures include: 1) Medial epicondyle 2) Coronoid process 3) Radial head and neck

What is the cause of lesser trochanter fractures

Most commonly in adolescence they occur due to forceful illiopsoas contraction. In elderly, it is pathognomonic for pathological lesions of the proximal femur.

In order of frequency of fracture, please arrange the carpal bones.

Most frequently fractured: 1) Scaphoid 2) Triquetrum 3) Trapezium 4) Lunate 5) and the rest: Capitum, hamate, pisiform

How do you treat SJS/TEN?

Mostly supportive treatment - Fluids - Skin care, barrier cream - Abx for infection prophylaxis - Stop drug - +/- ophthalmologist if eye involvement - Early (<10days) ciclosporin (?controversial) or prednisolone if early, no benefit if late - Burns unit referral if TEN

Patient with diabetes, signs of ketoacidosis and presenting with severe, ascending rhinosinusitis with potential intracranial involvement. What pathology are you suspecting?

Mucormycosis A fungal infection by a group of fungi under Mucorales. Extremely fast growing and thus characterised as 'lid lifters'

What is the mechanism of action for sorafenib?

Multi kinase inhibitor. Proven in SHARP trial to improve life expectancy ~ 3 months

What is the pathogenesis of referred pain?

Multiple mechanisms proposed 1) Convergent projection: where 1st order pain fibres from visceral organ project onto the same 2nd order neurones as the pain fibres from the referred pain site. BUT does not show why referred pain is delayed, and that it is unidirectional. 2) Convergence facilitation: where pain fibres from the visceral organ and the referred pain site have their own 2nd order neurones but the visceral organ pain fibres also projects onto the 2nd order of the referred site neurone. 3) Thalamic convergence theory: where the projection occurs subcortically.

What is a pepper pot skull and what is it associated with? What is the pathogenesis and it's cardinal features?

Multiple punched out lesions of the cranium Possibly multiple myeloma w DDx of metastatic disease Malignancy of plasma cells responsible for Ab production, resulting in CRAB 1) Calcium i.e (hypercalcemia from bone resorption) 2) Renal failure (bence jones proteinuria, nephrocalcinosis and amyloidosis) 3) Anemia (hematopoiesis failure) 4) Bone lesions (osteoclastic activation by RANKL)

What is the pathogenesis of sickle cell anemia?

Mutation in gene coding for B globin chain of hemoglobin, leading to formation of HbS. HBS polymerizes when deoxygenated, leading to sickle deformity of RBCs.

What are the signs of a myedemic coma?

Myxoedema - severe hypothyroidism characterised by 2 hallmark features of: 1) Decreased mental status 2) Hypothermia 3) Hypothyroidism signs and symptoms - Weight gain - Menorrhagia - Cold intolerance 4) Multi-organ dysfunction - Respi: respiratory depression with acidosis - CVS: bradycardia, heart failure, pericardial effusion and hypotension - Endocrine: hypoglycemia (due to decreased gluconeogenesis), adrenal insufficiency - Renal: hyponatremia from renal impairment or SIADH

What is the diagnostic criteria of Neurofibromatosis type 1?

NFC BEEF (2 or more of the following is diagnostic) - Neurofibroma (2 or more classic or 1 or more plexiform) - Freckling (inguinal or axillary) - Cafe au lait (6 or more, larger that 5 mm prepubertal or 15 mm post-pubertal) - Bone (cortical thinning or sphenoid dysplasia) - Eye #1 Optic glioma - Eye #2 Lisch nodules - Family history (1st degree relative w NF1)

How would you grade the severity of stroke? What are the components of the scoring system?

NIHSS score has 11 items Correlates well with size and severity of stroke 1) Level of consiousness - Patient alert/drowsy etc - Able to answer questions - Able to follow commands 2) CN deficit - Eye movement - Visual fields - Facial paresis 3) Motor function - Arms - Legs 4) Sensory function 5) Cerebellar - Limb ataxia 6) Cortical - Language/aphasia - Dysarthria - Neglect 0 -> no stroke symptoms 1-4 -> minor stroke 5-15 -> moderate stroke 16-20 -> moderate to severe stroke 21-42 -> severe stroke

What long term anticoagulant should be used in management of patients admitting for PE? How long before you review the patient and how long should the anticoagulant be prescribed?

NOACs: rivaroxaban See in 3/12 Continue anticoagulant FOREVER.

How may a neck of femur fracture be immobilised and splinted in the ED?

NOT thomas splint Traction NOT recommended due to reduction in capsular volume that may worsen vascular supply. No improvement in fracture reduction No change in analgesia use Risk of skin necrosis, pin tract infection Increased cost According to orthopedic trauma association http://ota.org/media/29309/L02_Femoral_Neck_Fx.ppt

What are the classes of drugs available for HIV treatment?

NRTIs: nucleoside and nucleotide reverse transcriptase inhibitors - Zidovudine, lamivudine, stavudine (lipoatrophy), emtricitabine - Tenofovir (renal tox and osteopenia), abacavir (risk of CVS effects + severe hypersensitivity reaction if HLA B5701 +ve) NNRTIs: Non-nucleoside/nucleotide RTs - Efavirenz (teratogenic and short term CNS side effects), Nevirapine (liver toxicity and hypersensitivity reactions, especially in high CD 4 levels, women > 250 and men > 400) - Etravirine Integrase inhibitors (not the TEG) - Raltegravir - Dolutegravir Protease inhibitors (Navir) - Lopinavir, fosaprenavir - Atazanicir, darunavir, saquinavir - Most can cause HLD CCR5 inhibitors (chemokine receptor that HIV uses to enter cells) - Maraviroc Fusion inhibitors - Enfuvirtide

What is the risk of contrast MRI in a patient with ESRF?

NSF - nephrogenic systemic fibrosis Caused by gadolinium exposure in patients with chronic renal insufficiency, manifesting as a fibrosing dermopathy similar but distinct from systemic sclerosis and scleromyxedema

What is amorolfine for?

Nail laquer for onychomycosis

Where do you biopsy for subungal melanomas?

Nail matrix

1) Central cord: Motor>Sensory, UL> LL, Distal>Proximal, bladder dysfunction 2) Anterior cord: Motor and crude/touch/pain/temp loss, but propioception maintained. 3) Brown sequard i.e hemisection: ipsilateral loss of motor and propioception with contralateral loss of pain and temperature at level of lesion 4) Posterior cord: propioception lost, but motor and crude touch/pain/temp milder deficit. Secondary gait abnormalities.

Name 4 incomplete spinal cord lesion syndromes and their respective neurological deficits.

1) Left lateral: emphasizes mitral stenosis/regurgitation 2) Holding breath in expiration: left sided murmurs (can combine w left lateral for mitral murmurs) 3) Holding breath in inspiration: right sided murmurs 4) Holding breath while listening to carotids: Aortic stenosis radiation 5) Leaning forward: Aortic regurgitation

Name the cardiac accentuation maneuvres and what they accentuate.

"1) Shy-drager's syndrome (i.e multiple system atrophy): depends on subtype: MSA-C (cerebellar: a/w cerebellar signs of ataxia, falls, dysdiadokinesia), MSA-A (autonomic: a/w postural hypotension, erectile and urinary dysfunction), MSA-P (parkinsonism dominant: parkinson sx more obvious), All with corticospinal signs i.e hyperreflexia and upgoing plantars 2) Progressive supranuclear palsy (frontal lobe): downward gaze disturbance (first downgaze, then upward gaze then horizontal gaze palsy), blepharospasm, frontal lobe signs (palmomental reflex: stroking thenar eminence results in twitch in chin and grasp reflex: putting anything in their hand results in them grasping it = contralateral frontal lobe damage) 3) Lewy body dementia: associated with early falls, early dementia w/ hallucinations and postural hypotension 4) Corticobasal degeneration (frontoparietal lobe): limb apraxia or alien hand syndrome (patient has no control over limb) and acute dystonia (abnormal, sustained, often painful muscular spasms producing twisted postures) 5) Vs Parkinson's: NO early falls, early dementia or postural hypotension and symptoms tend to be assymetrical (where all parkinson plus are symmterical (CBDG may be assymetrical)

Name the parkinson plus syndromes and their respective unique characteristics when compared to classical parkinson's disease

What is Breast Screen Singapore?

National breast screening program where - Distributed mammography services are provided with - Centralised reading and assessment - Coordinated by the Health Promotion board. Started because Singapore has one of highest age-adjusted breast cancer incidences in Asia, comparable to Australia, Canada and US.

What are the differences between pathogens responsible to native valve and prosthetic valve endocarditis?

Native valve 1) S.aureaus 2) Strep viridans 3) Coagulase negative staph ... Early prosthetic valve (within 2 months of implantation) 1) S.aureaus 2) Coagulase negative staph 3) Gram-negatives and fungi Intermediate and late prosthetic valve (2-12 and after 12 months respectively) 1) Streptoccocal spp 2) S.aureaus 3) CNstaph 4) Enterococci.

1) Imaging: Joint XR 2) Laboratory: - FBC (leucocytosis, may also be raised in inflammatory and septic arthritis) - ESR (raised in gout) - Uric acid levels (raised in acute, but non diagnostic. Repeat in 24 hrs to assess for non-flare hyperurecemia to guide pharmacological therapy) - Joint aspiration with microscopy, culture + sensitivity and cell count.

Necessary investigations for gout.

What is the Garden's classification and what injury does it classify?

Neck of femur fractures

How would you manage pelvic fractures?

Need to split into bleeding/haemodynamic stabilisation and bone fixation 1) Bleeding - Transfusion according to MTP protocol, transexamic acid - Pelvic binder - External fixation - Angioembolisation 2) Surgical fixation - ORIF Indicated in unstable fractures and open fractures - +/- diverting colostomy if perneal injuries severe Can allow weight bearing as tolerated in mechanically stable pelvic fractures

What is the classification of middle third clavicular fractures and how does it guide management?

Neer's classification Grade 1: < 100% displacement - Non operative management - Sling immobilisation with gentle RoM exercises at 2-4 weeks and strengthening at 6-10 weeks Grade 2: >100% displacement - Operative management indicated - ORIF w plates and screws

How would you classify a proximal humerus fracture and how does the classification guide management?

Neer's classification 5 types, depending on the 4 anatomical segments and extent of fracture 1) Greater tuberosity 2) Lesser tuberosity 3) Humeral shaft 4) Humeral head Type 1: One part - No displacement, irregardless of number of fracture lines - Sling immobilisation with passive RoM exercises at 7-10 days. - Active RoM exercise at 6 weeks, full RoM and recovery in 1 year. Type 2: Two parts - As long as 2 parts e.g fracture at anatomic neck, surgical neck, greater T, lesser T. - If anatomical neck, hard to reduce, do ORIF (if young) OR Replace (if old) - If surgical neck, usually operated (unless co-morbids, non-functional, no pain). Operation can be o Closed reduction w Pin insertion o ORIF (usually) w plates and screws or intramedullary device o Replacement if osteopenic w hemiarthroplasty/ total shoulder/ reverse shoulder. - If Greater T, do ORIF +/- rotator cuff repair if >5-10mm displacement (bec risk of non-union and subacromial impingement) - If Lesser T, rule out posterior dislocation and do closed reduction Type 3: Three parts - Surgical neck w Greater T broken off - Surgical neck w Lesser T broken off - Unstable so ORIF w o Plates and screws/ intermedullary nails o Prosthetic replacement in older individuals Type 4: 4 parts - ....you should know the trend by now, four parts hahah - Risk of osteonecrosis - ORIF or replace Type 5: Articular surface e.g HillSachs/Reverse HillSachs - Impression fracture (head collides w glenoid fossa, leaving an impact fracture) - Head split. - ORIF if young, good bone, otherwise replace.

What is the classification of distal thrid clavicular fractures and how does it guide management?

Neer's classification of lateral third clavicle fractures HAHHAHA 4 grades Grade 1 - fracture lateral to coracoclavicular ligaments (conoid and trapezoid) Stable, non operative mx Grade 2 - fracture medial to coracoclavicular ligaments (can be completely medial or between the ligaments etc) UNSTABLE, operative mx required Grade 3 - fracture involving acromioclavicular joint Stable, non operative mx Grade 4 - comminuted fracture in multiple parts Unstable, operative mx required Non operative mx: - SAME AS MIDDLE THIRD HAHAHAH - Sling immobilisation with gentle RoM exercises at 2-4 weeks and strengthening in 6-10 weeks Operative mx - ORIF with plates and screws

What is the classification of proximal humeral fractures and how does it guide management?

Neer's classification of prox hum # (greedy bastard hahah), 4 grades Grade 1: Un-displaced Grade 2: 2 part Grade 3: 3 part Grade 4: 4 part Grade 5?: articular involvement 1) Non-operative - Indicated for Grade 1 and 2 - Arm sling immobilisation with early RoM and strengthening 2) Operative - Indicated for higher grade, NV compromise, failed conservative treatment, unacceptable deformity, failed reduction - Open reduction and internal fixation - Reverse shoulder arthroplasty

What are the differentials of lytic tumors?

Neoplasm 1) Giant cell tumor Infection 1) OM Metabolic 1) Brown tumor of hyperparathyroidism 2) Metabolic bone disease 3) Fibrous dysplasia (scar tissue develops instead of bone 2' to congenital genetic mutation)

What are the components of a UC10

Nephrotic/nephritic (2) - Blood - Protein Infection (3) - Leukocytes - Nitrite - Specific gravity Metabolic (5) - Glucose - pH - Ketones - Urobilinogen - Bilirubin

What is the normal course of changes in leg alignment with age?

Newborn: mild physiological genu varus ~14 months: normal alignment ~3 years: mild genu valgus ~7 years: normal alignment

Would surgery for chronic venous insufficiency be offered in patients with active DVT? Why or why not?

No (Soft no) DVT means deep veins obstruction, where surgery removes the superficial veins. If both are obstructed/removed, you will worse the venous hypertension.

Is ciclosporine teratogenic? What are the indicated treatments of psoriasis for pregnant patients?

No.

Is a CT required before a lumbar puncture? Why or why not?

No. In cases of no focal neurological deficits and no papilledema on fundoscopic assessment, CT likely to be not have lesions with large mass effect. Hence can do lumbar puncture. Also look out for PMHx of CNS lesions, immunocompromisation, new onset seizure.

Will lactate in Hartman's solution increase blood lactate levels? Why or why not?

No. Lactate in Hartman's is anaerobically glycolysed to form bicarbonate, which helps buffer the blood pH.

Do all patients with symptoms of urinary tract infections require urine cultures?

No. Uncomplicated UTIs can be treated presumptively and empirically. Culture is indicated in 1) Patient factors - Allergic to first line tx - Pregnant patients - Hospitalized/ill patients 2) Disease factors - Pyelonphritis/complicated UTIs - Atypical symptoms - Failure of therapy

Can you use CHADSVASc for valvular AF? What other conditions affect the use of CHADSVASc?

No. Only for non-valvular AF. Prosthetic valves or previous valve repair or pre-existing LV clot.

Can penicillins be used for kleisiella?

No. They are intrinsically resistance to penicillins.

What types of BCC are there?

Nodular Cystic Pigmented (ddx melanoma) Sclerosing/ morpheaform, Superficial

What kind of alopecia does scalp psoriasis cause?

Non scarring Temporary hairloss in severe cases.

What are the risk factors for diverticulosis?

Non-modifiable: - Age Modifiable: - Low dietary fibre/high fat/high red meat - Low physical activity - Obesity

What are the risk factors of PUD?

Non-modifiable: - Old age > 75 - Family history Modifiable: - GERD - H.pylori infection - NSAID use - Smoking - Alcohol - Preserved food intake - Spicy food - Untreated stress

What is a Phlegmon?

Non-specific term for localized abscess

What is Grisel's disease?

Non-traumatic subluxation of the atlantoaxial joint due to inflammation but adjacent tissues. Usually affects children. Infection in nasopharynx or throat

What is the normal hallux valgus angle and the 1st intermetatarsal angle?

Normal HVA: < 15deg Normal IMA: < 9deg Mild: HVA < 30, IMA < 13 Moderate: HVA < 30-40, IMA 14-18 Severe: HVA >40, IMA >18 On WEIGHTBEARING, AP foot XR

What is the normal diameter of the bile duct?

Normal range is 3-7mm

What is the use of MRI in breast assessment?

Normally not indicated due to cost, low specificity, contraindication in metallic implants and risk of multiple, unnecessary biopsies. BUT MRI useful for assessment of 1) Implant intergrity 2) Mammogram/ultrasound occult lesions e.g invasive lobular carcinoma 3) Multifocal tumours 4) BRCA (+) MRI also: 1) More reproducible vs operator dependant U/S 2) Less radiation

What post-op analgesia is suitable after ORIF?

Normally post-reduction should have less pain, hence paracetamol is indicated. Not morphine or stronger because risk of missed compartment syndrome. Include nursing orders to assess for pressure 1-2 hrly.

Why do we care about pulse pressure? What are the adverse associated outcomes?

Normally, systolic and diastolic blood pressure are good indicators for CVS disease because both correlate with HTN. But in elderly, diastolic blood pressure begins to fall and loses correlation with HTN and CVS, while systolic BP continues to be relevant. Looking at pulse pressure, it incorporates both rise in systolic and fall in diastolic and occasionally demonstrates greater predictive ability vs SBP alone. High pulse pressure = high SBP and low DBP - High SBP = LV strain +/- hypertrophy and failure - Low DBP = limitation in coronary perfusion +/- ischemia

What are the parameters to report when reading an Xray?

OLDACID -O: Open vs Closed - L: Location - D: Degree - A: Articulation involvement - C: Comminution pattern - I: Intrinsic Bone quality - D: Displacement, angulation, rotation

What are the principles of occupational therapy?

OT is the use of purposeful activity or interventions designed to achieve functional outcomes which promote health, prevent injury or disability and which develop, sustain or restore the highest possible level of independance. Aims: 1) Intervention directed to developing, improving, sustaining or restoring ADLs 2) '' '' sensory-motor, perceptual or neuromusclar functioning or RoM 3) Education patient and caregivers on carrying out the appropriate interventions.

What are yellow flags?

Obstacles to healthcare that involve aspects of thoughts, feelings and behaviors. 1) Thoughts - Always thinking the worse - Unhelpful feelings about pain and work - Preoccupation with health - Fear of movement and re-injury 2) Feelings - Anxiety - DIstress - Low in mood 3) Behaviors - Passivity in health (expecting other people or interventions to solve problems) - Changes in behavior or recurring behaviours

What is the natural history of isthmic spondylolisthesis and what is the population it affects?

Occurs as a spectrum of events: 1) Pars stress reaction occurs first - Sclerosis with incomplete disruption, followed by 2) Pars defect results - termed as Spondylolysis (NON CONGENITAL anatomic defect that develops in 4-6% of population) - Most commonly at L5-S1 (90%) - Increased prevalence in sports that involve persistent hyperextention e.g gymnastics, weight lifters, football linemen - With progression, results in 3) Spondylolisthesis - forward translation of vertebral segment over the one beneath it - 15% of those w defect

When and how might an osteochondroma undergo malignant change?

Occurs in older patients ~ 50y/o Ask about aggressive growth. Malignant change arises from cartilaginous cap that results in mass effects and pain, visible on MRI.

What are the radiological findings of GIST?

On CT or EUS - large >5cm - lobulated - heterogenously enhancing Prognostic factors: - mesenteric fat infiltration - ulceration - regional lymphadenopathy - exophytic growth pattern - tumor rupture are more likely to metastasize.

What does a high riding prostate mean and indicate?

On DRE, prostate is not well felt, mobile and difficult to palpate. Due to urethral injury/rupture the results in a free-moving prostate. Non specific and may be difficult to assess in trauma setting

What is the pathogenesis of Rheumatoid arthritis?

On a baseline of genetic susceptiblity (HLA DR4 and DR1), immune reactions within synovial joints results in intraarticular inflammatory reactions with anti-IgG autoantibodies (i.e Rheumatoid factor or anti-citrullinated protein antibody) that cause chronic synovitis and articular damage. Possible triggers may be previous infections, responses to microorganisms and their possible molecular mimicry.

Where do you biopsy on a malignant lesion?

On the edge, the centre usually consists only of necrotic tissue.

How is sentinel node biopsy done?

Only done in early CA (< 5cm) T1 or 2, with no clinical/radiological signs of lymphnode spread: if sentinel node clear, rest of axillary nodes clear as well. 1) Blue dye: isosulphan blue/methylene blue OR Radioactive isotope (tc-99 sulfur colloid or colloidal albumin) injected in area of breast just before surgery 2) Marker concentrated in sentinel LN within 5 minutes 3) Intraop search for sentinel node via visual confirmation or geiger-muller counter (<5% wrong node) 4) Excise node and send for frozen section (~ 33% false negative) 5) Perform axillary clearance if +ve spread, reconfirm histology if unsure

What is Darier's sign?

Onset of erythema and pruritis after stroking skin -> indicates mastocytosis due to compression that causes degranulation of mast cells.

What are the cutoffs for precocious puberty in boys and girls?

Onset of puberty is precocious if it begins - Before 8 y/o in girls - Before 9 y/o in boys Manifesting initially as rapid growth of bones and muscles, changes in body shape/size and development of reproductive organs.

What are the types of abdominal drains?

Open Vs Closed 1) Open: corrugated rubber or plastic sheets, where drain fluid collects on gauze pad or stoma bag. e.g Penrose drain 2) Closed: tubes draining into a bag/bottle e.g Blake, JP

What is the main mode of treatment for croup?

Oral or nebulised steroids.

What is the pathogenesis of SCC?

Originates from squamous cells of epidermis 2-5% risk of distant metastasis (lungs, liver, brain, skin or bone) If LN involved, 5 year survival 35%

What kind of bone tumor is produced in multiple exostosis and why?

Osteochondroma (pedunculated lesion) Hereditary multiple exostosis is an AD mutation of the EXT gene that affects normal chonrdocyte proliferation and differentiation.

What is the definition and pathogenesis of osteoporosis and how is it different from osteomalacia?

Osteoporosis = Bone mineral density > 2.5SD belwo the mean peak bone mass of young (25-30) y/o subjects (T score </= 2.5SD) - Bone density is decreased due to gradual loss of bone due to hormonal changes. - With normal mineralization function - i.e we can build, but we just dont feel like building so much anymore Vs Osteomalacia = Soft bone due to poor mineralisation - i.e we cant build because we lack the materials

What are the differentials for dermatomyositis?

Other forms of myositis: - Polymyositis - Sarcoid myositis - Infectious myositis (lyme disease/toxoplasmosis) - Drug induced myositis (Elevated serum creatinine kinase: lovastatin, chloroquine and colchicine VS normal creatinine kinase: steroid induced proximal myopathy) Other autoimmune conditions: - SLE - Vasculitis - Systemic sclerosis - Mixed connective tissue disease (SLE + systemic sclerosis + polymyositis, look for raynauds)

Where else can cystic hygromas arise?

Other than the neck, they can arise in regions rich in lymphnodes e.g flexural regions: axilla, groin

How is the overnight dexamethasone suppression test done?

Overnight 1mg Dexamethasone Suppression test: at 2300hrs, measure cortisol before 0900 next day (normal <1.8ug.dl or 50nmol/L) Counsel patient to sleep and not do anything else (which can artificially elevate cortisol levels)

What is the malaria type in thailand/cambodia region? What are they resistant to and what chemoprophylaxis is possible?

P.falciparum (55%) P.vivax (35%) The are resistant to chloroquine and mefloquine, thus only doxycycline and atresunate combo is useful. Completely resistant strains also exist, thus only avoidance is an option. (in bangkok, phuket etc)

What enables pseodomonas aeruginosa to be multi-drug resistant and what can you use?

PAE tends to have multiple effluc pumps that prevent accumulation of abx within the bacterium before it can achieve effective concentration at site of aaction + a cell membrane with limited permeability. Hence PAE often resistant to - B lactams - Tetracyclines - Chloramphenicol - Macrolides - TMP - Aminoglycosides PIP tazo is first line

What is the mechanism of PCL tears? Hence what is the PCL responsible for stability in?

PCL ruptures from: 1) Direct blow to proximal tibia with a flexed knee (dashboard injury) 2) Noncontact hyperflexion with a plantarflexed foot 3) Hyperextension injury PCL prevents posterior translation of the tibia, prevents hyperflexion/ sliding

What are the pre-operative steps for management of open fractures?

PCR HADRON 1) Primary Survey: Initial trauma survey and resuscitation for life-threatening injuries (ATLS principles, need to rattle off) 2) Clinically assess the patient for severity of fracture (Gustillo-anderson for open and Tsherne's for closed) 3) Radiological evaluation, take picture of wound 4) Hemostasis via direct compression 5) Antibiotics IV (depending on classification) and tetanus prophylaxis 6) Dressing: Remove obvious foreign bodies and place moist sterile dressing over wound (avoid irrigation/ debridement in ED if surgery possible < 24 hours, do not remove bone fragments in ED) 7) Reduction: Perform provisional reduction of fracture and place in splint, brace or traction 8) Operative intervention is indicated within 24 hours for open # 9) N&V compromise or Compartment syndrome requires URGENT OP

What are the AIDS-defining illnesses?

PEST MILCH -Pneumocystis carinii pneumonia -Pneumonia (Recurrent) -Encephalopathy (HIV related) -Salmonella septicemia (Recurrent) -Toxoplasmosis of brain -Tuberculosis -Mycobacterium avium complex -Mycobacterium (extrapulmonary) -Isosporiasis (chronic intestinal >1/12, due to parasite isospora belli) -Lymphoma (non-hodgkin's) C6 -Candidiasis (esophageal, bronchial, tracheal) -Cervical CA -Coccidiodomycosis -Cryptococcosis (extrapulmonary) -Cryptosporidiosis -CMV -Herpes simplex -Histoplasmosis (disseminated/extra-pulmonary)

How is PF ratio calculated and what is it's clinical relevance in ARDS? Additionally, what is the FiO2s of the nasal cannulae, simple face mask and non-rebreather mask?

PF ratio = PaO2/FiO2 where FiO2 depends on the type of supplemental oxygen provided Non-rebreather mask on 15L = 100% theoretical FiO2 Simple face mask = 40-60% Nasal canula = 20 + (4x rate) Important in determining alveolar gaseous exchange Normally > 500 Think of ARDs e.g in sepsis or acute lung injury if < 300

What are the indications for a PICC and a Central line?

PICC: - Chemotherapy - PN/TPN - Repeated blood taking/blood product infusion - Frequent venous sampling in poor venous access - CVP monitoring Central line: - Volume resusciation - CVP monitoring - Emergent venous access - Nutritional support - Inotropes - Hemodialysis

What are the types of psoriatic skin lesions?

PIG.E 1) Plaque: dry, raised, erythematous plaques covered with silvery scales, extensor regions +/- pain with Koebner's phenomenon: appearance of new plaques at sites of cutaneous trauma 2) Guttate: multiple small scaly 'tear drop' plaques involving multiple areas, triggered by streptococcal sore throat. 3) Pustular: clearly defined, raised pustules with an erythematous base, associated with a systemic febrile illness. 4) Inverse psoriasis: non-scaly version of plaque psoriasis due to location in flexure regions that have more moisture aka flexure psoriasis. 5) Erythrodermic: severe diffuse erythema, may occur at first onset or flare of psoriasis with risk of fatality.

When does anastomotic leak typically occur?

POD 5

What exercises are required in physical therapy of isthmic spondylolisthesis?

PT for strengthening of abdominal and hamstring muscles, via 1) Pelvic tilts 2) Hamstring muscles 3) Abdominal strengthening

What is the definition of moderate-severe PCP?

PaO2 or A-A gradient PaO2 < 70mmHg on RA OR Alvoelar-arterial gradient >/= 35mmHg

What is the dosing regimen for paracetamol?

Paediatric: Paracetamol 15mg/kg = 450mg 6hrly PRN AKA - 19ml of 120mg/5ml syrup OR - Approx 1 tablet (500mg) Adult: Paracetamol 500mg-1g 6hrly PRN AKA - 1 to 2 tablets

What is the approach to a limping child?

Pain and age of presentation. Painful hip - <3 y/o: infection most common, trauma, distal tibial fracture - 3 to 7 y/o: transient synovitis (ddx of exclusion, commonest cause), inflammatory arthritis, growing pain (ddx of exclusion), leukemia - 7 to 10 y/o: Perthe's disease, transient synovitis - > 10 y/o: SCFE (15 y/o), growing pains Painless hip - 0 to 7 y/o: limb length discrepancies o Bone: femur vs tibial shortening o Joint: DDH (may be undiagnosed) o Nerves: NF 1 o Vessels: large hemangioma o Infection: polio infection (deformity 2' impaired growth, neurogenic deficit) o Developmental variants: femoral anteversion/ tibial torsion(pigeon toed), flat foot

What is the description of prostatic pain?

Pain felt deep within the groin or perineum, sometimes in the rectum or lower back. Often poorly localised.

What is the pain pattern of plantar faciiits and how would you treat it?

Pain in inferior heel of foot - Worse on waking or after resting - IMPROVES with walking - Pain along plantar fascia (at medial calcaneal tubercle and on dorsiflexion because of stretching) Treatment: 1) Conservative (90% resolves) - Rest and NSAIDs (4-6 months) - Stretching exercises (w golf/tennis ball on heel) - Supportive heel cup in shoes - Steroid injections 2) Surgical - If refractory to conservative treatment - Plantar fascia release w/o need for spur removal (50% effectiveness at pain relief)

What is dysuria?

Pain in the URETHRA during micturition.

What is Whipple's operation? When is it required?

Pancreaticoduodenectomy Can be pylorus preserving or involving. Pathologies: 1) Head of pancreas tumor (uncinate process) 2) Ampulla of vater adenoCA 3) Distal cholangiocarcinom 4) Duodenal CA 'Heavenly 4'

What is another name for warthin's tumor?

Papillary cystadenoma lymphomatosum or adenolymphoma Misnomer, is not a lymphoma.

What are the modes of spread for papillary and follicular thyroid cancer?

Papillary: lymphatic Follicular: hematogenous

What are the types of anal fistulas and what is Parks classification?

Parks classification of types of anal spinchters, based on the cryptoglandular theory. 1) Interspincteric (70%) - through internal sphincter to interspincteric space and then perineum WITHOUT passing external sphincter - Internal sphincter divided, but continence usually preserved - Treatment is fistulotomy 2) Transpinchteric (25%) - through internal, external and intersphincteric space - If low: Fistulotomy - If high or anterior fistulae in women: o Cutting seton or o Partial fistulectomy and endoanal flap o Injection of fibrin glue 3) Supraspincteric (5%) - via intersphincteric space up over puborectalis and back down to ischorectal fossa and perineum - a/w drainage of ischiorectal abscess - Tx w o Cutting setons o Endorectal advancement flap o Sphincter reconstruction 4) Extraspincteric (1%) - from perianal skin to levator ani muscles to rectal wall, completely not involving the spincters - a/w crohns, CA, recurrent fistulas - Tx w o Endorectal advancement flap o Laparotomy and resection of involved intestinal segment and curettage of fistula tract (for fistulas more proximal)

What is the pathogenesis of pulmonary hypertension?

Passive backwards transmission of raised left atrial pressure leads to raised pulmonary venous and capillary pressures. Raised venous pressures result in interstitial edema and pressure damage, with fibrosis and obliteration of vascular bed. Hence pulmonary arteriolar pressure increases.

What are some clinical tests for fluid responsiveness?

Passive leg raise Tredelenberg positioning Observe for increase in BP.

Gout: • Inflammatory arthritis caused by cellular reaction to uric acid crystal deposition. • Nucleic acid and purine metabolism forms xanthine and hypoxanthine which is catalysed by xanthine oxidase to uric acid for renal and gut excretion Excess deposited in joints, resulting in local acute inflammatory reaction

Pathogenesis of Gout?

Pseudogout: Inflammatory joint condition comprising of 3 factors: 1) Chondrocalcinosis: calcific material in articular cartilage and menisci 2) Acute synovitis due to crystal deposition 3) Degenerative joint disease due to chronic pyrophosphate arthropathy Due to Pyrophosphate production by abnormal cartilage via enzyme activity, which combines with calcium ions in matrix causing crystal nucleation within. Thus pronounced in fibrocartilaginous structures (menisci, TFCC, pubic symphysis, intervertebral discs, hyaline cartilage, tendons, peri-articular soft tissues. Inflammatory reaction occurs in occasional extrusion of crystals into joint spaces.

Pathogenesis of Pseudo-gout?

Produced by late diastolic atrial contraction and atrial blood flow hitting the walls of a poorly compliant ventricular wall i.e hypertrophic

Pathophysiology of 4th heart sound

What factors are considered in admission of patients to the ICU/HDU?

Patient factors: - Poor premorbids with major organ dysfunction Surgical factors: - Major or prolonged surgery with significant physiological disturbance and fluid shifts expected Anesthesia factors: - Difficult airway: obstruction, respiratory muscle weakness requiring ventilatory support - Haemodynamic instability: requiring inotropic support

How do patients with inter-trochanteric fractures present? How is it different from femoral neck fractures?

Patients with IT fractures present with - Shortening (hip flexors, extensors and adductors pull distal fragment proximally) - External rotation - Coxa vara (illopsoas displaces the lesser trochanter medially)

Which patients are at high risk for HCC and how are they followed-up?

Patients with: - Cirrhosis - Hep B/C - Porphyria 6/12 to 1 yrly w U/S liver and AFP levels (1 in 5 HCC not raised)

What are the test to assess for sacroiliac joint pathology?

Patrick/Faber's test - Flex, Abduct and Externally rotate the hip (to form number 4) - Place hand on contralateral ASIS and also on flexed knee - Pressing down to illcit pain in sacroiliac region -> +ve if painful.

How do you spell PEW DE ORANGE?

Peau D' Orange

What is the dosing regiment for ibuprofen?

Pediatric: Ibuprofen 7.5mg/kg = 225mg 8hrly AKA - 11ml of 100mg/5ml syrup - Approx 1 tablet (200 mg) Adult: 300mg 6 to 8 holy PRN AKA - 1 to 2 tablets

What do you call an ingrown toenail affecting both sides of the nail?

Pincer nail

What is the difference between tinea capitis and pityriasis capitis?

Pityriasis capitis: dandruff due to eczema Tinea capitis: dandruff due to fungal infection - Well cricumscribed, hai

What are the principles to placement and care of feeding tubes?

Place as high along GIT tract as possible to increase absoprtion surface area Ensure tube: 1) Patency (regular flushing, checking) 2) Placement (prevent migration) 3) Check gastric residual volume (for adequacy of peristalsis/food mvoement) 4) Determine choice of diet

How is hemolytic uremic syndrome treated?

Plasma exchange

What do questions want when they ask you 'how would you manage?' ?

Please provide requested investigations and referrals plus possible initial management.

Why is RT not usually done in BCC?

Poor cosmetic results: permanent areas of hypo/hyper pigmentation Potential long term: chronic radiation dematitis, alopecia, radiation induced skin malignancies

What is the pathogenesis of ankylosing spondylitis? Who is more likely to get it?

Possible an autoimmune reaction to an external pathogen in a genetically susceptible individual (HLA B27 positive) Manifesting as 1) Enthesitis (more common in juvenile form <17y/o) - Inflammation where ligaments/tendons insert into bone - Results on bony erosion, ossification of surrounding soft tissue and eventual joint ankylosis 2) Disc-space involvement - Inflammation of the annulus leading to formation of marginal syndesmophytes Risk factors: - Males:Female 4:1 - <10% of HLA B27 individuals have AS

What are the possible bones/joints that require orthopedic intervention in the setting of spastic cerebral palsy?

Possible areas: UL Spine: scoliosis +/- paralysis Hip: paralytic subluxation/dislocations Knee: Contractures, gait issues Foot/ankle: equinus, varus/valgus

What are the possible causes of acute HTN?

Possible causes - Progression of essential HTN - Sympathetic: pain, anxiety - Intracranial: ICH, raised ICP (cushings triad) - Cardiac: congestive cardiac failure, tachycardia - Renal: acute renal failure - Endocrine e.g Phae, cushings - Drugs, sympathomimetics, serotonin syndrome, malignant hyperthermia

Lower limb: 1) Big toe (podagra) 2) Lesser toes 3) Ankle joint 4) Achillies tendon Upper limb: 1) Finger joints 2) Olecranon bursa Others: 1) Pinna of ears

Possible sites of Gout flares and tophi

Usually larger joints - Knee - Hip and Unusual joints - Shoulder - Elbow - Ankles - Wrists Usually polyarticular

Possible sites of Pseudo-gout flares

What vessels may be affected in hemispheric stroke/cerebral stroke and what signs do they manifest?

Possible vessels involved: - Anterior cerebral artery - Middle cerebral artery - Posterior cerebral artery 1) Anterior cerebral artery (frontal cortex) - Contralateral motor and sensory deficit (leg > face > arm) - Grasp and sucking reflexes - Abulia (inability to act decisively) - Gait apraxia (forgot how to walk, but coordination normal in seated/lying position) - Paratonic rigidity (intermittent abnormal increase in resistance to passive movement) 2) Middle cerebral artery - Dependant on dominant hemisphere and which lobe (frontal, parietal) o Dominant (usually left) hemisphere - Aphasia (broca if frontal, wernickes if temporal) - Parietal lobe i.e Gerstmann's syndrome - Motor and sensory deficit (face, arm > leg > foot) +/- hemiplegia if internal capsule involved - Homonymous hemianopia o Non-dominant hemisphere - Neglect (parietal lobe) - Anosognosia (lack of insight/understanding) - Contralateral motor/sensory deficit +/- hemiplegia and +/- homonymous hemianopia 3) Posterior cerebral artery o Parietal - Alexia without agraphia o Occipital - Visual hallucinations/perserverations o Thalamic - Sensory loss, choreoathetosis, spontaneous pain o CN III - Palsy, opthalmoplegia o Cerebal peduncal/midbrain - Contralateral motor deficit

What is the pathogenesis of a Baker's cyst?

Posterior herniation of the knee joint capsule forming an outpouching synovial fluid-filled sac. Can be primary: idiopathic or Secondary: often associated with degenerative joint disease: OA, RA, charcot's joints

Which nerve is commonly injured in Monteggia fractures and why?

Posterior interosseus nerve Found along interosseus membrane and compression when radial head anterior dislocates.

Please describe the dermatomes of the posterior leg?

Posterior thigh - S2 Posterior knee - S2 Posterior leg - Medially: L4 - Middle posterior: S1 - Lateral posterior: L5 Heel: S1

What are PLC injuries?

Posterolateral corner injuries - contains primary stabilizers of external tibial rotation at all knee flexion angles and the secondary restrains to anterior and posterior translation. - Injury results in increased instability to the knee.

What electrolytes are intracellularly driven by insulin?

Potassium and phosphate

Why do we bother testing for CMV viral load?

Predictive role. 1) Initial viral load - e.g in transplant patients, viral load of >5000copies/ml was 86% sensitive and 87% specific for active CMV disease 2) Rate of increase - a higher rate of increase seen in patients with CMV disease

What is the Spilled teacup sign?

Presence of a volar dislocation and tilt of a dislocated lunate on lateral wrist X-ray Convexity of lunate is no longer articulating with distal radius and Concavity is no longer in articulation with capitate VS perilunate dislocation - Lunate convexity is still in articulation with distal radius

What is the double duct sign?

Presence of simultaneous dilation of the common bile duct and pancreatic duct. Most commonly seen in HOP and periampullary tumors, but can occur w impacted CBD stone w edema causing pancreatic duct obstruction Can be seen on MRI, CT, U/S, ERCP etc.

What are the signs of tarsal coalition?

Presents in adolescence as coalitions become increasingly ossified. Patients may present with sprains and other minor injuries Look: 1) Hind foot with subtalar joint in valgus 2) Mid foot is in planus 3) Forefoot is in pronation and 'too many toes sign' 4) Whole foot is abducted 5) +/- peroneal spasms due to compensatory hyperactivity of peroneus longus and brevis Move: 1) Tip toe/dorsiflex to to observe for reformation of medial arch 2) Jack's test (passive extension of great toe)

How do patients with hip dislocations present? What mechanisms of injury may be responsible?

Presents with 1) Pain, inability to bear weight 2) Deformed resting posture: If posterior dislocation (more common): hip and leg in flexion, aDDuction and internal rotation. If anterior dislocation: hip in aBDuction and external rotation, but hip extension or flexion depends on superior(pelvic) or inferior(obturator) dislocation respectively. Posterior dislocation occurs with axial load on femur, typically while it is flexed and adducted Anterior dislocations occurs while hip in aBDuction and external rotation

What are the indications for liver transplant?

Primary - Biliary atresia - Glycogen storage disease Secondary - Chronic liver disease +/- decompensation - HCC - Acute fulminant hepatitis e.g Hep B flare, alcoholic hepatitis, panadol, TB, antifungal medications

What is the pathogenesis of Addison's disease?

Primary adrenal insufficiency -> upregulation of ACTH in attempt to increase production -> ACTH stimulates melanocytes that cause hyperpigmentation.

What are the types of wound healing?

Primary intention: healing of clean wound without tissue loss - Wound edges are brought togther and re-approximated - Closed with sutures, staples, adhesives etc 2) Secondary intention: healing with soft tissue loss - Heals slowly with fibrosis - Leads to a wide scar, often hypertrophied and contracted 3) Tertiary: initially treated by repeated debridement, abx, negative pressure wound therapy etc - Surgical intervention via suturing, skin graft placement or flap design is performed - AKA delayed primary intention

What are the causes of abdominal compartment syndrome?

Primary/secondary 1) Primary: arising from abdominal conditions e.g trauma, hemoperitoneum, pancreatitis 2) Secondary: arising from non-abdominal causes e.g sepsis, burns, fluid resuscitation

1) Initiation of pharmacologic therapy within 24 hours (optimally) 2) Pharmacologic therapy depends on severity of symptoms and number of joints involved. 3) Continue Urate Lowering Therapy

Principles of acute gout flare treatment

What are the treatment principles of C.diff and how is success of treatment determined?

Principles: 1) Antibiotic management - Stop inciting abx - if cannot stop, switch to those less implicated in C.dff 2) Infection control - Isolate patient, place on contact precaution - Strict handwashing with soap 3) Diarrhoea management - Watch fluid losses and electrolyte imbalances - Encourage regular diet - Antimotility agents controversial (keeps bacteria in) Assessing success of treatment - Stool assays are NOT WARRANTED because 50% remain +ve six weeks after therapy - Watch for resolution of symptoms (which may last 12 weeks or may stop but recur within the 12 weeks)

What is the pathogenesis of Duputryen's contracture?

Progressive disorder of the palmar fascia resulting from proliferation of fibroblasts and deposition of collagen. Etiology is unknown, associated with: 1) DM 2) CLD 3) Epilepsy 4) Occupation with vibratory stimulation 5) Being a middle aged scandinavian man

What is the natural history of DRESS syndrome? What are the possible complications?

Prolonged duration > 2 weeks up to 1 year, hence need to follow up A/w sequelae: Thyroiditis Lupus DM Alopecia areata

What are the possible clinical signs of previous stroke that you can screen for in short cases?

Pronator drift Walking aids Abnormal resting posture/facial assymetry

When is antimicrobial prophylaxis indicated for infectious endocarditis and what regimens are available?

Prophylaxis is indicated for patients with valvular defects or those with previous history of infective endocarditis 1) Prosthetic valve or prosthetic repair of valve 2) Congenital heart disease that is - Not repaired - Recently repaired (<3/12) - Persistent/newly found defect in repair 3) Previous IE 4) Cardiac transplant patients who develope valvulopathy Recommended regimens depends on patient factors 1) Can take orally - Amoxicillin 2) Cannot take orally - Ampicillin or cefazolin or ceftriaxone 3) Penicillin-allergic but can take orally - Cephalexin or clindamycin or azithromycin or clarithromycin 4) Cannot take orally - Cephalexin or ceftriaxone or clindamycin

What is PCSK9?

Proprotein convertase subtilisin Kexin 9 Degradation of hepatocyte LDL receptos and decreases low density lipoprotein cholesterol levels.

What is the PIRADS scoring?

Prostate Imaging - Reporting And Data System

What are the landmarks dividing the ureter on XR KUB?

Proximal = above SI joint Mid = across SI joint Distal = below SI joint

What are the attachement points of the posterior cruciate ligament?

Proximally: attached to anterolateral medial femoral condyle Distally: attached to the posterior tibial sulcus

What are the attachment points of the anterior cruciate ligament?

Proximally: attached to the posteromedial lateral femoral condyle Gives rise to 2 bundles that attach differently on the tibia Anteromedial bundle: attaches to the anteromedial portion of the anterior tibial sulcus Posterolateral: attaches to the posterolateral portion of the anterior tibial sulcus

What are the anatomical borders of the humeral shaft?

Proximally: insertion of the pectoralis major Distally: supracondylar ridge.

What is the pathogenesis of Psoriasis and psoriatic arthritis?

Psoriasis: An unknown trigger mechanism (trauma, stress, hypothermia, infections, idiopathic) that results in increase d leukocytes capable of inducing inflammation and triggering keratinocyte hyperproliferation recruitment to the dermis and epidermis that result in plaque formation. Psoriatic arthritis is when the recruited lymphocytes trigger inflammtory cascades resulting in synoviocyte, macrophage and neutrophil increase that result in inflammatory joint disease.

What are the radiographic features of an unstable pelvis?

Pubic symphysis joint widenening > 2.5cm SI fracture > 1cm Sacral fracture with displacement > 1cm Rotation or displacement of hemipelvis

How would you treat vacsular malformations?

Pulsed dye laser therapy

What is the difference between pyelonephritis and pyonephrosis? and renal abscess?

Pyelonephritis: inflammation and infection of renal parenchyma (medical condition, requiring abx) Pyonephrosis: pus within renal calyces (surgical emergency, requiring drainage: PCN or J-stent) Abscess: pus within renal parenchyma, possibly 2' to pyelonephritis

What is QTc, how is it measure/calculated and what are the normal ranges?

QTc is derived from the QT interval, defined as Q wave to end of T wave (demarcated by tangent of descending wave +/- a closely related U wave) and the RR interval. It is calculated by Bazett's formula: QTC = QT / √ RR (other formulae available) QTc is prolonged if > 440mm in men > 450mm in women > 500mm is at risk of toursades des pointes A useful rule of thumb is that a normal QT is less than half the preceding RR interval

What is the diagnostic criteria of toxic megacolon? Compared to ogilvie syndrome?

R31 FART HEAD 1) Radiological findings of distended colon AND 2) 3 or more of the following (FART) - Fever > 38c - Anemia - Raised neutrophils > 10,500/ul - Tachycardia > 120bpm AND 3) 1 or more of the following (HEAD) - Hypotension - Electrolyte imbalances - Altered mental state - Dehydration If no mechanical cause of dilation and patient does not qualify for toxic megacolon - think ogilvie (diagnosis of exclusion)

What is the treatment for familial medullary thyroid carcinoma?

RE: MTC - Surgery is most effective, doesn't respond well to other forms of Treatment - Prophylactic thyroidectomies in proven familial cases - Total thyroidectomy w bilateral central compartment LN clearance - Take central compartment histo o If LN in one side -> do ipsilateral lateral neck dissection (70% will also be positive) o If LN both, do bilateral lateral neck dissection o Need more aggressive surgery since non responsive to other treatment - Take FmHx and offer genetic testing o If positive for MEN/FMTC (familial medullary TC), screen 1st degree family members o If family members have, offer thyroidectomy since it's almost 100% chance of getting MTC o Age can be 6 months old depending on how aggressive the genetic sub-type is

What are the drugs in TB quadruple therapy? What complications can they cause?

RHEZ 1) Rifampicin - drug drug interactions - 'liver and blood' - hepatitis, anemia and thrombocytopenia 2) Isoniazid - liver and nerves - hepatitis and peripheral neuropathy - give piridozine (B6) to prevent neuropathy 3) Ethambutol - retrobulbar neuritis (loss of color vision) - risk increases after 2 months hence E usually stopped then 4) Pyrazinamide - liver and joints - hepatitis (usually most important cause) and hyperuricemia (may precipitate gout/cause arthralgia)

When is radiotherapy indicated in breast CA?

RT may be indicated as adjuvant therapy or palliative therapy. 1) Adjuvant therapy - After breast conserving therapy: to reduce recurrance rate - After non-breast conserving therapy as well if: tumor size >/= 5cm or involvement of >/=4 axillary LN 2) Palliative therapy - Brain metastases - Bone metastases to painful areas or impending fractures

What are the risk factors for papillary thyroid carcinoma?

Radiation exposure Polyposis syndromes (FAP, Gardner's) Positive family history

What is the Gallavardin phenomenon?

Radiation of systolic murmur toward apex, which may be confused as an MR murmur.

What lymph nodes should be cleared in gastric cancer?

Radical lymphadenectomy of LN within 3 cm of margins + LN of all major arteries except para-aortic +/- spleen

What does the open mouth view in XR consist of and what are the indications?

Radiograph comprising for assessmenf of the patient's C1 and C2 articular surfaces. Provides cleared view of C1 and C2 without obstruction/superimposition by other bony structures e.g mandible etc Able to see: 1) C1 and C2 bodies 2) Inferior articular surface (C1) and superior articular surface (C2) 3) Zygapophyseal joint space (Between the articulating surfaces) 4) Odontoid process (of C2) 5) Bifid spinous process (of C2) 6) Lateral masses (of C1)

How would a hip dislocation be investigated and managed?

Radiographic assessment of hip and sites of possible associated injury (should be clinically assessed as well). XR - can diagnose posterior dislocation on AP pelvis, assess for #, disruption of lines of hip CT - should be done pre and post reduction to check for direction, loose bodies and associated fractures. Management 1) Conservative - Emergent closed reduction under anesthesia within 6 hours - For all unless associated femoral neck fractures 2) Surgical o Open reduction - +/- removal of incarcerated fragments - for irreducible dislocations, radiolographic confirmation of incarcerated fragment, delayed presentation, non-concentric reductions - Should be performed on urgent basis o Open reduction + internal fixation - indicated when there are associated fractures: acetabular wall, femoral head/neck - Possible to remove fractures/repair intra-articular injuries to cartilage, capsule and labrum.

Where are the possible atypical sites of zoster reactivation?

Ramsay hunt: geniculate ganglion C4 zoster: diaphragmatic zoster Sacral zoster (L5) Brain: meningoencephalitis Lungs: pneumonitis GIT

How can you classify cervical myelopathy?

Ranawat classification (3 classes) Class 1: pain only, no neurological deficit Class 2: subjective weakness w hyperreflexia and dysthesias Class 3A: objective weakness w long tract signs, but still ambulatory Class 3B: objective weakness w long tract signs but NOT ambulatory OR According to the Japanese orthopedic association classification: which is a point system of 17 points based on function in the categories of 1) Upper extremity motor function 2) Lower extremity motor function 3) Sensory function 4) Bladder function

What are the complications of correcting hyponatremia and hypernatremia too quickly?

Rapid correction of Hyponatremia - cerebral pontine myelinosis Hypernatremia - cerebral edema

When is gardasil recommended for females and how many doses?

Recommended for those 9 to 26 years old 2 doses if 9-13 y/o, 3 if older.

What do patients with zollinger-elison syndrome present with?

Recurrent, intractable gastric ulcers refractory to medical therapy, occuring in atypical or multiple sites. +/- diarrhoea and LOW, due to gastrin stimulating gastric acid production and secretion. They may present with MEN1 symptoms as well: parathyroid and pituitary adenomas. Confirm with secretin stimulation test, where in normal patients, secretin INHIBITS gastrin production and release. In Zollinger ellison, gastrin levels INCREASE with secretin instead due to abnormal cell function.

What is the definition of spinal stenosis and what are the causes? How might they be classified?

Reduction in dimensions of central canal or lateral lumbar spinal canal by (radiological diagnosis) 1) Bony structures - Facet osteophytes - Uncinate spur posterior vertebral body osteophyte) - Spondylolithesis - Degenerative/spondylotic changes - Short pedicals with medially placed facets (achondroplasia) 2) Soft tissue structures: - Herniated/bulging discs - Hypertrophy/buckling of ligamentum flavum - Synovial facet cysts Can be classified anatomically: 1) Central stenosis 2) Lateral canal recess stenosis 3) Foraminal stenosis 4) Extraforaminal stenosis (e.g lateral disc herniation causing impingement of the existing nerve root)

What is metaphyseal rarefraction?

Reduction of metaphyseal bone density.

What is the treatment of lunate and perilunate dislocations?

Reduction: Closed vs open 1) Closed reduction: - pull hand with wrist in extension and apply thumb to reduced displaced bones - plaster slab with wrist in neutral - +/- K wires 2) Open reduction: - failure of closed reduction - anterior approach to expose carpus with reduction and K wire placement +/- carpal tunnel release

How is drug desensitization done? How about drug challenge?

Referral to an allergist who will - Administer the drug in increasing doses until the full therapeutic dose is reached Drug challenge - Drug is given in a safe setting with CPR and meds ready - Patient given drug and watched for reaction.

What are the differences between referred pain and radicular pain?

Referred pain Vs Radicular pain Looks at location, nature, associated signs, depth, persistance.

What is the inoculumn effect?

Refers to laboratory phenomenon of increase in minimal inhibitory concentration when numbers of a bacteria increase e.g in betalactamase producing bacteria, concentration of abx required to achieve effect increases.

What is the bulbocavernosus reflex and what is it used for?

Reflexive contraction of anal sphincter when glan penis is squeezed. A polysynpatic reflex useful for assessment of spinal shock/cauda equina.

What is the cremasteric reflex?

Reflexive elevation of testes when inner thigh stroked/pinched by at least 0.5 cm.

What is the treatment of hypoglycemic unawareness?

Relax the target sugar levels. Increase pre-meal and post-meal glucose levels to eliminate any subsequent episodes of hypoglycemia for the next few months.

How is excision surgery done in BCC?

Remove tumor with 3-4mm margins. If tumor < 2cm can achieve 1' skin closure If cannot achieve skin closure, consider reconstruction surgery or Moh's micrographic surgery (layer by layer frozen section intraoperatively)

What is the management on diverticular fistula?

Resection of diseased colon + closure of fistula

What is the natural history of biceps tendon rupture?

Resolution of pain in about 4-6 weeks Gradual improvement of flexion, function and RoM to almost baseline level.

What is the most common cause of death in congenital diaphragmatic hernia?

Respiratory failure secondary to persistent pulmonary hypertension and pulmonary hypoplasia.

What are the possible locations of the appendicitis?

Retrocaecal Antecaecal Ileal Pelvic

What investigation should be ordered for a suspected urethral injury?

Retrograde urethrogram

In a CT scan showing rhombencephalitis, what does that mean, what pathogen are you suspecting and how would you treat?

Rhombencephalitis AKA brainstem encephalitis Seen as high intensity in brainstem on MRI. Think of listeria monocytogenes (most common infective cause, followed by HSV) but also - Autoimmune (Behcet's) - Paraneoplastic syndrome Given causes of infection, treat empirically with Ampicillin and acyclovir While waiting for blood and CSF PCR and culture.

What risk factors and pathogens are the most common causes of epididymoorchitis?

Risk factors: - PmHx: UTI, urethritis, urethral instrumentation, dysfunctional voiding, bladder outlet obstruction (urine reflux causes infection) - Lifestyle: sexual activity months before Causative pathogens: 1) Sexually active men < 35 - C.trachomatis - N.gonorrhoeae - Coliform bacteria in homosexuals 2) Men > 35 - E.coli - Pseudomonas - C. trachomatis and N gonorrhoeae less common 3) Non bacterial causes - Viral: mumps - Myoobacterial: TB

Why is it important to assess for the rest of the spine when patient suffers from ossification disease?

Risk of DISH Diffuse idiopathic skeletal hyperosteosis.

What are the possible complications from transrectal biopsy?

Risk of hematuria, hemospermia, PR bleeding Risk of sepsis 3% requiring admission for IV ABX, hence prophylactic abx given before op Risk of ARU, return to AnE for cath insertion

How would you classify AC joint dislocations and how does it guide management?

Rockwood classification: 6 grades Grade 1: AC ligament and capsule SPRAIN/partial tear Grade 2: Complete tear of AC ligament, CC ligaments intact For Grade 1 and 2: Convervative treatment - Ice - Rest/activity modification - Arm sling for 3/52 - Early shoulder RoM exercises Grade 3: AC and CC ligaments torn, clavicle displacement 25-100% Grade 4: posterior displacement of clavicle Grade 5: 100-300% displacement of clavicle Grade 6: INFERIOR displacement of clavicle Grades 3 and above need surgery - ORIF w endobutton or hook plate w ligament repair/recontruction

What are the normal radiographic parameters of the distal radius?

Rule of 11s (Eleven Lips) 1) Length: 11mm 2) Inclination: 22 degrees (13-30) 3) Palmar tilt: 11 degrees (0-28 degrees)

What is the epidiomological distribution of umbilical hernia?

Rule of 3s 1) 3 years old, usually closes by then, repair if not closed 2) 3% of population will have it 3) if larger than 3 cm, must operate because it will not heal by itself

What are the complications of thyroglossal cyst? How is it treated?

Rupture Infection Malignant change (papillary thyroid cancer) Sistrunk operation - foramen caecum, midline at junction and anterior 2/3 and 1/3 posterior of tongue - tract is removed up to foramen caecum with anterior portion of hyoid

How would you classify a subtrochanteric fracture?

Russell-Taylor classification 2 types, with 2 subtypes 1) Type I, piriformis fossa intact - 1A: lesser trocahnter is attached to proximal fragment - 1B: lesser trochanter is attached to distal fragment 2) Type II, primiformis fossa involved in fracture - 2A: Stable medial construct - 2B: Cominution of the piriformis fossa and lesser trochanter, a/w varying degrees of femoral shaft comminution Previously used to guide the choice of implants, 1st vs 2nd gen nails (centromedullary which required both trochanters to be intact and cephalomedullary which can do everything)

Which oral hypoglycemic agent is associated with improved cardiovascular outcomes? What are some adverse drug effects?

SGLT 2 Inhibitors (e.g dapaglifozin) But associated with increased risk of euglycemic DKA and UTI.

What is Nicholsky's sign?

SIgn in SJS/TEN where blisters form after rubbing skin

Whats the big deal of scaphoid fractures?

SNAC (scaphoid nonunion advanced collapse) Progressive wrist - Stiffness - Weakness - Osteoarthritis

What is the difference between SSS and SJS/TEN?

SSS comes with fever and does not involve the mucosa.

What do the parts of the scotty dog show? Ear, nose, eye, neck, front foot.

STIPPI Ear: superior articulating facet Nose: transverse process Eye: pedicle Neck: pars interarticularis Front foot: inferior articulating process Tail: opposite superior articulating process Hind foot: opposite inferior articulating process Body: lamina and posterior spinous process

What ECG finding is characteristic of pericarditis?

Saddle-backed ST elevations

How would you classify medial collateral ligament injuries and how does it guide management?

Same classification as ATFL ligament, 3 grades Grade 1: sprain Grade 2: partial tear Grade 3: Complete tear, no end point with valgus stress test at 30 degrees flexion (to remove ACL and PCL stabilisation) Management 1) Non-operative - Indicated for grade 1 and 2 as well as mechanically stable 3s - NSAIDs - Brace immobilisation - Rest and activity limitation 2) Operative - Indicated in multi-ligamentous injuries, avulsion fractures, mechanical instability - Surgical ligament repair/reconstruction

What is benzyl benzoate for?

Scabies

What is the classification of tibial plateau fractures and how might they guide management?

Schatzker classification 6 types Type 1: lateral split fracture Type 2: lateral split with depression (commonly a/w lateral meniscus tear, oblique XR/CT better to view depression) - If condylar widening > 5mm, ORIF Type 3: lateral pure depression fracture - If depression > 3mm, ORIF Type 4: Medial plateau fracture (commonly a/w medial meniscus tears, more impt because bears 60% of patient knee load) - All will need ORIF Type 5: Bicondylar fracture - All will need ORIF Type 6: Metaphyseal-disphyseal dissociation - All will need ORIF

What are the screening recommendations for cervical cancer?

Screening begins for individuals 25 and above who are sexually active. It consists of a 3 yearly PAP smear. May consider spacing the initial smears closer together before spacing out to 3 yearly.

What is the match box sign?

Seen in delusions of parasitosis - Patient presents a matchbox or similar container, reporting that it contains parasites responsible for reported pruritis - Container sample consists only of skin cells or irrelevant organisms e.g ants, flies etc.

What is Pucker's sign/dimple sign in the knee? How is it formed and what does it indicate?

Seen occasionally in posterolateral rotational knee dislocations, seen as skin puckering due to impingement between the medial femoral condyle and the tibia condyle joint space. Indicates a rotational knee dislocation that is likely irreducible and associated with increased risk of skin necrosis.

What is the mechanism of action of metoclopramide and an important side effect?

Selective D2 receptor antagonist for antiemetic purposes. Acute dystonic reactions that may occur as contractions of the muscles, opisthotonus, torticollis, dysarthria, trismus, and oculogyric crisis

In patient with hip pain after fall but no fracture on Xray, what is the appropriate management?

Send to hospital for MRI TRO occult fractures (of pelvis as well)

What is tenesmus?

Sensation of incomplete defecation/voiding. Pressure within lumen stimulates stretch receptors in rectum causing pain.

How might non-union be classified?

Septic vs aseptic 1) Septic - Infection ongoing 2) Asceptic - Hypertrophic - Oligotrophic - Atrophic

What are the signs and symptoms of compartment syndrome? How would you investigate?

Severe pain, pain on passive stretching of fingers/toes, tense extremity Distal pulses may remain PRESENT long after ischemic damage to nerves and muscles are irreversible. STRKYER (catheter measurement of intracompartment pressure) > 30mmHg

What criteria might you use to differentiate seizures from syncope?

Sheldon's criteria, based on pre-syncopal symptoms 'Fear the man who faints without knowing how he did.' 'The Jerk DJ Turned the disc with his Tongue, making everyone Confused and Emotional' Seizure if point score >/= 1 Syncope if point score < 1

What are the short and long term complications of poor pain management?

Short term: 1) CVS: tachycardia, hypertension 2) Respiratory: atelectasis, retention of secretions, pneumonia, increased O2 consumption 3) Psychosocial: sleep disturbance, physical and emotional suffering 4) Delayed mobilization: DVT risk, nosocomial infections Long term: 1) Chronic pain 2) Psychological and behavioural changes: poor future compliance, reluctance to seek health/treatment

Rigler's sign vs Rigler's triad

Sign - severe pneumoperitoneum showing as air Triad - ileus, gallstone ileus w large gallstone causing obstruction

1) Inability to bury sclera on lateral gaze (scleral show) 2) Ipsilateral esotropia on corneal light reflex (Hirschberg reflex)

Signs of CN 6 palsy.

1) Loss of resting asymmetry 2) Loss of angular retraction 3) Loss of frontalis action 4) Incomplete eye closure/partial-loss of eyelash burial

Signs of CN 7 palsy.

After long term use of levadopa, patients may exhibit dyskinesia in the form of 1) Dystonia (gradual tight painful contraction of muscles) 2) Chorea (sudden jerking of hands, neck etc) 3) Athetosis (writing snake like movements)

Signs of Levadopa induced dyskinesia (3)

1) Pulse - parvus et tardus 2) Apex beat - heaving 3) Palpation - systolic thrill 4) Heart sounds - reversed split, lack of split, soft S2, 4th heart sound 5) Clinical complications - pulmonary hypertension and cardiac failure

Signs of Severe Aortic Stenosis (5 categories)

What are the signs of normal pressure hydrocephalus and how is it diagnosed?

Signs: 1) Bladder dysfunction 2) Frontal gait disorder (magnetic gait and frontal disequilibrium) 3) Cognitive impairment Diagnosis: Via Neuroimaging showing ventricular enlargement without evidence of hippocampal atrophy or dilation of peri-hippocampal fissures, calculate via Evan's index >0.3 where A/B. A = maximal width of frontal horns and B = maximal internal diameter of skull

What are the signs of a complete supraspinatus tear? How would you differentiate it from supraspinatus tendinitis?

Signs: 1) Drop arm sign: when arm is lowered to a certain angle it falls 2) ABduction paradox: unable to initiate ABduction when able to ABduct when passively moved to 90degrees To differentiate: - Inject lignocaine into subacromial space to relief pain - If patient is able to Initiate ABduction now = not complete tear.

What is the classification of osteogenesis imperfecta?

Sillence classification: Types I to IV: Collagen disorders of COL1A1/2 I, III, IV = mild, moderate, severe deformity II = lethal in utero Blue sclera in I and II (either mild or you die) AD for I and IV AR for II and III Types V to VIII: Non-collagen

What are the similarities between CMV and EBV mononucleosis and how can you differentiate them?

Similarities - Mononucleosis o Symptoms: significant, protracted fevers o Laboratory: >50% mononuclear cells and >10% atypical lymphocytes. o Hematologic: anemia, reduced haptoglobin, raised RF and ANA, thrombocytopenia and DIC. Differences - CMV: more 'typhoidal' where fever and systemic symptoms are more common and happens more in older patients - EBV: more commonly presents with tonsilitis (Exudative), cervical lymphnodes (posterior), splenomegaly and happens more in younger patients

How would you classify posterior elbow dislocations?

Simple vs Complex 1) Simple - Isolated dislocation - No associated fractures 2) Complex - Associated fractures present

What are the extra-intestinal manifestations of inflammatory bowel disease?

Skin - Erythema nodosum - Pyoderma gangrenosum Joints - Arthralgia - Sacroillitis - Ankylosing spondylitis Eyes - Anterior uveitis - Episcleritis - Conjunctivitis HPB - Acute fatty liver - Primary sclerosing cholangitis - Cholangiocarcinoma - Chronic hepatitis - Gallstones Renal - Nephrolithiasis Haematologic -Thomboembolism Inflammatory - Amyloidosis

What are the 3 layers to cut for release of thyroid hematoma?

Skin (follow crease) Platysma (follow crease) Strap muscles (cut vertically) Cover the area w sterile dressing, give abx, rush to OT for hemostasis

What is fox fordyce disease and how would you manage it?

Skin coloured/slight erythematous papular eruptions in axillar/genitofemoral region due to plugging of follicular infundibula leading to subsequent rupture and inflammation. AKA folliculofundibulitis Treat with topical clindamycin, electrocoagulation, liposuction/curretage.

What is the gold standard for food allergy testing?

Skin prick testing.

What is the diagnostic test for leprosy? Can you culture it?

Skin scrapings of earlobe (NOT SKIN) and microscopy with Ziehl neelsen staining. Culture in artificial medium has never been successful, it can only be grown in mouse footpads and armadillos.

What is the diagnostic test for leprosy?

Skin scrapings of earlobe and microscopy with zieehl neelsen stainin

What is atrophoderma?

Skin thinning

What is Gallow's traction and when is it indicated?

Skin traction of legs with buttocks lifted off of bed. Only for children < 12 kg or risk of skin necrosis. Indications: 1) Femoral shaft fracture, (not neck of femur because can fix) 2) DDH (to loosen before reduction)

What is the risk of rupture in an AAA?

Small (<5cm) - 2-3% per year Larger (>5.5cm) - 10% per year 75% of >7cm will rupture in 5 years

What do you see on histology of anaplastic thyroid carcinoma?

Small blue round cells, highly anaplastic.

What skin biopsy findings would you expect to find in each of the vasculities?

Small vessels - Lymphocytes: viral infection - Neutrophils: cutaneous small vessel vasculitis, HSP, urticarial vasculitis Small +/- medium vessels - Lymphocytes: connective tissue disease (SLE, RA, Sjogrens) - Neutrophils: ANCA vasculitis - Eosinophils: Eosinophillic granulomatosis with polyangitis - Granulomatous: Granulomatous polyangitis Medium vessels - Lymphocytes: lymphocytic thrombophillic arteritis or macular arteritis - Neutrophils: Polyarteritis nodosa

What kinds of melanoma are there?

Speading/superficial Nodular Lentigo maligna Acral lentiginous

What is retinitis pigmentosa?

Spiculated hyperpigmentations along periphery of retina. A/w 1) Polydactyly (laurence moon biedl) 2) ESRF and hearing loss (alports) 3) Unsteadiness (frederich's ataxia) 4) Heart block (kearnes sayre)

What is the difference between spironolactone and eplerenone?

Spironolactone antagonizes both aldosterone and testosterone inhibitor, which may result in unintended effects of erectile dysfunction and gynacomastia. Eplerenone is a pure aldosterone receptor antagonist, which eliminates the side effects of hypotestosteronism, but it is more expensive. Both are options in treatment of primary hyperaldosteronism due to bilateral adrenal hyperplasia.

How would you grade ankle ligamentous injuries and how does this guide management?

Sprain grades (1 to 3) Grade 1: microscopic tear of collagen fibres (minimal pain/swelling w/o give on stress tests) Grade 2: macroscopic incomplete tear (moderate tenderness and swelling w present but limited give on stress tests) Grade 1-2: Non operative tx - RICE - Immobilize (aim for contact between the ligament in question, e.g dorsiflexion and eversion in ATFL) untill swelling gone - PT w crutches until can walk -> return to activity in 2-3 weeks Grade 3: macroscopic, complete tear (significant swelling and instability w less pain, large talar tilt on mortise XR) - Non operative tx: RICE w brace in hinged knee orthosis and crutches for 1-2 weeks w PT -> return to activity in >/= 6 weeks - Operative tx: only indicated in athletes w ATFL and CFC tears or complicated entrapped ligaments, repair of ligament: semitendinosus tendon

What is the physiological progression of the tibiofemoral angle? How do you measure it?

Spread along 0, 2, 4, 6 years of age 0 years old: maximum varus 2 years old: varus corrects, leg becomes straight 3 years old: physiological valgus 4-6 years old: legs straight Legs straight = 0-5 deg of tibiofemoral angle (measured from hip joint to middle of foot)

How does disseminated gonococcal infection happen and what are the risk factors?

Spread of the sexually transmitted N.gonorrhoea via bacteremia to joints, tendons, skin etc. Increased risk with certain types of N.gonorrhea and patients with 1) Increase in bacterial growth - Recent menstruation - Drugs e.g Eculizumab 2) Immunological abnormalities - Complement deficiency (acquired or congenital) - HIV/immunocompromisation - SLE - Pregnancy

What does fracture stabilisation comprise of in traumatic fractures? Why is it necessary?

Stabilization via: 1) External fixation 2) Internal fixation 3) Intramedullary nails Important in: 1) Preventing additional soft tissue injury 2) Enabling better patient mobilization and mobility 3) Enabling greater wound access for management

What are the components of the Hinchey Classification and what managment do they suggest?

Stage 1 - Pericolic/mesenteric abscess - Treat with IV Abx Stage 2 - Pelvic/retroperitoneal abscess - Treat with drainage and IV Abx Stage 3 and 4 - Purulent peritonitis and feculent peritonitis respectively - Treat with Hartman's procedure + resection of perforated bowel + peritoneal lavage - KIV stoma reversal 3/12 later

What are the stages of rheumatoid arthritis, their signs and Xray findings??

Stage 1: Synovitis - Inflammation, cell rich exudate and angiogenesis - Acute pain, warmth, erythema - Chronic thickened synvoium around joints - Soft tissue swelling +/- osteoporosis of affected joints. Stage 2: Destruction of joint - Joint/ tendon/ ligament and articular surface destruction with pannus formation due to proteolytic enzyme action and osteoclastic bone resorption - Chronic pain - Joint instability, stiffness/RoM restriction - Tendon ruptures and muscle wasting - OA changes with periarticular erosions Stage 3: Deformity of joints - Progressive destruction with pannus fibrosis/calcification and joint ankylosis - Loss of function (Class 1 to 4) - Classic RA deformities - Marked articular destruction with joint deformity and dislocations

What is the treatment for RCC?

Stage I to III 1) Surgery - Partial nephrectomy if T1N0M0 and renal function impaired OR bilateral small lesions - Radical nephrectomy for T2 and above - Open or laparoscopic 2) Adjuvant chemo/immunotherapy not well proven 3) Post-op surveillance - for locoregional recurrence (CT/US) - for Mets to lung (CT), bone (bone scan), liver (CT) - F/u period depends on stage - KIV metastasectomy Stage 4 - Systemic therapy via immunotherapy and anti-angiogenic therapy (2nd line) for clear cell RCC (non clear cell not well established) - RT good for metastatic symptom reduction e.g painful bone, brain mets - KIV metastasectomy/radical nephrectomy OR debulking/cytoreductive nephrectomy to correct compressive/invasion effects

How would you classify Staghorn stones?

Staghorn classification (pelvic component extending into calyces) - Partial: 1 or 2 calyces - Complete: involving upper, middle and lower calyces

What radiological investigations might you order and how would you classify the AC injury? How might classification guide treatment?

Standard shoulder trauma series: AP, lateral, Y scapular +/- Stress views (5-10lb) weight strapped to arm and XR taken of both shoulders to compare Classification is according to the Dameron and Rockwood classification 5 types Type 1: No radiographic abnormalities, sprain of ligaments w/o periosteal tube disruption Type 2: Slight widening (>7mm in Men, >6mm in women) of AC joint space, minimal superior displacement with partial disruption of the periosteal tube and mild distal clavicle instability Type 3: Superior displacement of 25-100% on radiographs compared to normal contraleral shoulder. longitudinal split of periosteal sleeve. Type 4: Superior displacement with posterior displacement through trapezius Type 5: >100% superior displacement, palpable subcutaneous clavicle with +/- disruption of deltoid and trapezius attachments +/- open fracture Type 6: Infracoracoid displacement of the distal calvicle due to inferior force vector. Types 1 to 2: Non operative treatment is indicated with sling immobilisation and early RoM exercises, healing in about 4-6 weeks Types 4 to 6: Open reduction required with repair of periosteal sleeve +/- internal fixation: endobutton, hook plate (need to be removed eventually, risk of supraspinatus impingement and osteolysis) +/- ligament/tendon repair (allograft/autograft)

What is the treatment for kaposi's sarcoma?

Start HAART - In most cases it is sufficient to induce regression - Otherwise, local/systemic chemo or RT.

What are the components of the WHO pain ladder?

Step 1: Mild pain (1-3) - Non-opiods (aspirin/paracetamol/NSAIDs) +/- Adjuvants (muscle relaxants, antidepressants, antiseizure, sedatives etc e.g gabapentin) *NSAID + PPI *Opiods + antiemetics/anticonstipation Step 2: Moderate pain (4-6) - Weak opiods (codeine, tramadol) - Non-opiods +/- adjuvants Step 3: Severe pain (7-10) - Strong opiods (morphine/fentanyl) - Non-opiods +/- adjuvants RE: opiod strength - >CTMHOF> 'Cant Touch My Holy Orifices Frequently'

What do we need to look for in patients with Strep bovis endocarditis and how do we treat?

Strep bovis spreads hematogenously from the gut and is highly associated with colonic neoplasm. Hence need to do a colonoscopy TRO colorectal cancer and redo in 4-6 months if nothing is found. Also look for liver and extracolonic malignancy. Treatment: 1) Non-endocarditis patients: - Ceftriaxone (if penicilin 'S') 2) Endocarditis patients: - Ceft + Gentamicin

How do stress fractures present? What is their pathogenesis?

Stress fractures present as joint or bone pain, with a history of recent abrupt increases in intensity and volume of activity rather than acute singular trauma. High numbers of occurrences of cyclical overloading of intensity lower than the maximum bone strength on non-pathological bone tissue. Whereby repetition and combination lead to fatigue and imbalance between osteoblast and osteoclast activity, thus favouring bone breakage. (Astur DC, Zanatta F, Arliani GG, Moraes ER, Pochini A de C, Ejnisman B. Stress fractures: definition, diagnosis and treatment. Revista Brasileira de Ortopedia. 2016;51(1):3-10. doi:10.1016/j.rboe.2015.12.008.)

What is the pathogenesis of subchondral cysts?

Stress induced bone resoprtion

What is lynch syndrome?

Subset of Hereditary Non-polyposis Colectecal CA, of which there is lynch I and lynch II. Lynch I - increased risk of cancer at - Splenic flexure (70%) - Arising from polyps (commonly flat with villous architecture) - Forming poorly differentiated tumors Lynch II - increased risk of cancer elsewhere (RUG) o Reproductive: - endometrial - ovarian o GIT: - gastric - small bowel - hepatobilliary o Urological: - renal pelvis - ureter o Others: - Brain (turcot syndrome) - Sebaceous gland or keratoacanthomas (Muir-torre syndrome) - hence offer a THBSO with colectomy if detected Diagnosed via Amsterdam Criteria.

What are the differences between a total and subtotal gastrectomy?

Subtotal leaves a small portion of proximal stomach: - Easier to anastomose than oesophagus because latter does not have serosa (higher risk of leak) - Less morbidity and better functional outsome due to preservation of some reservoir function However - May be insufficient for proper clearance, especially for proximal or diffuse CA.

Where are the common areas of BCC?

Sun exposed regions: - Face, head and neck, helix of ear, periorbital

What are the risk factors of BCC?

Sun exposure Light skin Immunosuprersion Genetic syndromes ( nevoid BCC, xeroderma pigmentosum) Radiation therapy

How is activated vitamin D synthesized?

Sunlight exposure results in production of Vitamin D3 in skin. Liver converts to calcidiol. Kidney's concert

Which nerve and nerve roots innervate the gluteus medius and minimus?

Superior gluteal nerve (L5)

Which nerve is responsible for voice projection?

Superior laryngeal nerve: responsible for straightening the cricothyroid muscle for purpose of making explosive speech.

Describe the blood supply of the colon.

Superior mesenteric: ileocolic, right colic and middle colic (becoming the marginal artery of drummond) Inferior mesenteric: left colic (joining drummond), sigmoid branches, superior rectal

What is a complication of chronic supraspinatus tear?

Superior migration of shoulder due to lack of rotator cuff stablisation and deltoid pull. Seen as disruption of Maloney's line Results in Cuff tear arthropathy e.g 2' OA

What is the treatment of a Gunstock deformity?

Surgery is indicated if symptomatic or patient wishes. Supracondylar corrective osteotomy.

How can you treat implant infection?

Surgery is required. 1) Debridement and retention - Indicated in implants <3 months old, stable, mild symptoms with no abscess or sinus tract and no S.aureas or MDR organisms 2) Removal and replacement o 1 Stage - Implant removed and replaced in single op o 2 stage - Implant removed with spaced placed, Abx treatment - Subsequent 2nd op to replace implant 3) Girdle stone - Implant removed and joint fused 4) Amputation If patient NOT FIT FOR OP! 1) Long term antimicrobial suppression - Pathogen specific abx + rifampicin to disrupt biofilm. - 3 months for hip joint, 6 months for knee joint.

How is a remplissage procedure done and what is the indication?

Suturing of the subscapularis into the Hill Sach's defect to prevent it's engagement of the glenoid rim. Important in Hill sach's leisions causing engagement and locking.

How long may C.diff symptoms persist for after treatment and what are the criteria for recurrence? How do you treat the recurrence?

Sx can last for 12 weeks after treatment 1st recurrance - Appropriate resolution while on treatment but recurrence within 2-8 weeks after treatment has been stopped. Treatment is oral vancomycin (if treated w metronidazole) and oral vancomycin in a pulse-tapered fashion (if previously treated w oral vancomycin)

What counts as complicated UTI?

Symptomatic urinary infection in individuals with functional or structural abnormalities of the urinary tract, or co-morbid conditions

How would patients with osteochondroitis dissecans of the knee present?

Symptoms: - Intermittent ache/swelling - Giving way - Locking Signs: - Tenderness (localised to one femoral condyle) - Quadriceps wasting - Joint effusion - Wilson's sign o Pain felt when knee flexed to 90deg, medially rotated and gradually straightened (assuming medial condyle is the issue), with no pain when repeated on the other condyle.

What are the possible manifestations of giant cell arteritis?

Symptoms: headache, visual symptoms, jaw ache/dysphagia Signs - Arteries: o tenderness, bruit, pulsation, erythema - Ocular: o Transient vs permanent - Constitutional: o Fever, fatigue, weight loss - Associated: o Legs: limb claudication o Hands: Raynauds o Body: poly myalgia rheumatica

What is Steal syndrome? How do you diagnose and treat it?

Syndrome caused by ischemia resulting from a vascula access device e.g AVF or synthetic AVF. Symptoms: Limb ischemia signs (6Ps) Decreased wrist-brachial index < 0.6 Pain distal to ischemia Invx w arteriography Treat with revascularisation techniques: - DRIL - distal revascularisation and interval ligation procedure - PAI: proximalisation of the arterial inflow - RUDI: Revision using distal inflow

What is Felty's syndrome

Syndrome comprising of: - Chronic rheumatoid arthritis - Splenomegaly - Neutropenia Risk of opportunistic infections and associated with Sjogren's syndrome.

How would you treat a PVNS?

Synovectomy - high recurrence in diffuse type due to remnant synovium (>50%) If refractory, consider low dose RT

What is the definition and normal range of T scores and Z scores?

T score = bone density in terms of standard deviations from that of a young (25-30y/o) man/woman Osteoporosis: </= -2.5 SD Osteopenia: -1 to -2.5 Z score = bone density in terms of SD from that of a person of the patient's age, sex, weight and ethnicity.

What is the pathogenesis of psoriasis?

T-lymphocyte mediated disease with possible genetic predisposition, where environmental factors trigger the chronic, inflammatory and proliferative disease of the skin.

What is the definition of Early gastric cancer and how can it be treated?

T1 stage w/o Nodal or metastatic spread Treat with Endoscopic submucosal resection

When is adjuvant chemoTR indicated in rectal cancer?

T2 and above

What is the half life of T3 and T4 respectively?

T3 is 1 day T4 is 1 week Hence T3 is lower risk of hyperthyroidism but more costly.

What are the conditions commonly associated with congenital talipies equinovarus?

TADA 1) Tibial hemmelia/anterior tibial artery hypoplasia/absence 2) Arthrogryposis 3) Diastrophic dysplasia - Congenital disorder of bone and cartilage - Manifests as dwarfism 4) Amniotic band syndrome - Caused by entrapment of fetal parts in fibrous amniotic bands in-utero - Look for constriction rings around digits, arms, legs with distal swelling or even amputation.

When are steroids indicated in TB treatment?

TB meningitis and TB pericarditis.

What is considered a small abscess in Acute appendicitis?

TBC hahah

What is the use of TEG in trauma?

TEG: Thromboelastography Clot properties tested and parameters used to guide transfusion strategy. R time - reaction time (how fast fibrin forms) = give FFP is slow reaction time Alpha angle - speed of fibrin build up and cross link formation = give cryoprecipitate if decreased MA - maximum amplitude (ultimate strength of the fibrin clot/overall stabilty) = platelet transfusion Clot lysis time - measures rate of fibrinolysis = tranexamic acid

Why is fundoscopy required for newly diagnosed HIV patients?

TRO CMV retinitis, which will result in blindness 2's IRIS if ART started.

What is the definition of tandem stenosis and double crush syndrome?

Tandem stenosis: 2 concomittant stenosis e.g cervical and thoracic spinal stenosis Double crush syndrome: 2 concomittant compressions of peripheral nerves/nerve roots e.g carpal tunnel and radiculopathy

What are the target INR's for prosthetic valves?

Targets INR for warfarin - For mitral valve prosthetic only: 2.5-3.5 - For the rest: 2-3

What is the difference between telogen effluvium and alopecia areata? How would you treat telogen effluvium?

Telogen - resting hair (club), thus telogen effluvium is loss of resting hair due to sudden systemic stress/illness Alopecia areata - loss of anagen hair (growing hair: exclamation mark) Tx of telogen effluvium is 1) Nothing really - Treat underlying illness - Ensure adequate nutrition (B12, folate, thyroid also) - Avoid vigourous combing, brushing or scalp massages

What is the surgical treatment of biceps tendon rupture?

Tenodesis (reattachment of tendon to either bicipital groove or coracoid process) Typically ~ 10% improvement in flexion function only

What is the law of LaPlace?

Tension is proportional to the radius of the lumen (to the power of 4) Hence with each increase in tension, radius increases within the lumen.

How is Gaenslen's test done and what does it test for?

Test for sacroilitis via hyperextension of the hip. Side to be tested has the leg placed off the table and held in place. 'Hyperextension' is done by flexing the contralateral hip by getting the patient to pull on his/her leg while the examiner stabilises the extended leg and holds it in place.

What are the MEN syndromes and the organ systems they affect?

The 3, 2, 1 Ps rule 1) Men I (3 Ps) - Pituitary adenoma - Parathyroid hyperplasia - Pancreatic tumours 2) Men IIa (2 Ps, 1 M) - Parathyroid hyperplasia - Phaeochromocytoma - Medullary thyroid carcinoma 3) Men IIB (1 P, 2M) - Phaeochromocytoma - Medullary thyroid carcinoma - Marfanoid habitus

Which ethnic group is predisposed to ligamentum flavum ossification.

The Japanese.

What is the lisfranc joint and what ligaments stabilise it?

The Lisfranc joint consists of: 1) tarsometatarsal articulation 2) Intermetatarsal articulation 3) Intertarsal articulations Ligaments involved in it's stability: 1) Lisfrac ligament - Holds the 2nd metatarsal to the medial cuneiform - Important in stabilising the 2nd metatarsal and the medial arch 2) Plantar tarsometatarsal ligaments - Holds the 2nd and 3rd metatarsal to the medial cuneiform 3) Dorsal tarsometatarsal ligament - weakest and prone to injury 4) Intermetatarsal ligament

What is CRITOE?

The age at which the physeal closes in the elbow. 1) Capitulum - 1 year 2) Radial head - 3 years 3) Internal epicondyle - 5 years 4) Trochlear -7 years 5) Olecranon - 9 years 6) External epicondyle - 11 years

Which bin do you throw your used N95 mask into?

The biohazard (yellow) bin

Which bone is the most common site for pathological fractures?

The femur

What are the borders of a subtrochanteric fracture?

The fracture occurs between the lesser trochanter and a point 5 cm distal to the lesser trochanter.

What is the use of the fenestrations in Moore's unipolar hemiarthroplasty?

The implant may be press-fitted or fitted with cement. The fenestrations are for bone growth (+/- bone graft) and increased blood supply.

What is the difference between thyroid mass and brachial cyst/lipoma/other skin lesions?

The latter does not move with swallowing, thus if the examined mass moves, do not offer them as differentials.

What is the OGD finding of a hiatal hernia?

The longitudinal gastric folds are visible below the esophago-gastric junction, with a distal dilation.

What is the most common cause of rhinosinusitis and how do you tell if it is bacterial?

The most common cause is viral, despite the presence of yellow purulent discharge or fever. Signs that indicate a bacterial infection are - Double sickening phase (initial improvement in symptoms after 5 days before worsening) - Persistence of symptoms (>10 days without improvement) - Risk factors (immunocompromisation, swimmers, anatomical defects, GERD, atopic history)

What are the complications of AvF?

The most important complications of fistulae for HD are lymphedema, infection, aneurysm, stenosis, congestive heart failure, steal syndrome, ischemic neuropathy and thrombosis.

What is the blood supply of the humeral shaft?

The perforating branches of the brachial artery.

Why is ABPI measurement less reliable for diabetic patients?

The stiffening and calcification of vessels result in an overestimation. However, toe pressures are unaffected, thus more useful to measure TBPI.

Where on the spine is the most common site of metastasis?

The thoracic spine, possible T10.

What is the bunion in hallux valgus formed from?

The unexposed head of the 1st metatarsal head due to subluxation of the metatarsal-phalangeal joint. +/- osteophytes +/- Callus formation from chronic friction +/- metatarsalgia +/- Hallux rigidus (OA with hallux valgus)

What are the symptoms and signs of malaria? Which parasites tend to cause which?

There are no pathognomonic signs of malaria, hence diagnosis mainly depends on laboratory confirmation of parasite infection 1) Uncomplicated malaria - Fever, malaise, myalgia, arthralgia, headache - Fever tends to be cyclic, intermittent, occuring at irregular intervals in the day and may be > 40deg in children - +/- tachycardia, AMS and febrile seizures. - Fever every other day: F.O.V (falciparum, ovale, vivax) - Fever every third day: Malariae - +/- Superinfection (risk of concomittant LRTI/GIT/bacteremia/viral infection increases with lower levels of parasitemia) 2) Severe malaria (commonly due to falciparum but also can be vivax) - AMS +/- seizures - Respiratory distress/ARDS - Haemodynamic instability - Metabolic acidosis - Renal/hepatic failure - Hemoglobinuria (blackwater fever) - Severe anemia, hemolysis - Coagulopathy +/- DIVC - Hypoglycemia.

What makes a lymphocyte atypical?

They are 1) Larger (more cytoplasm) 2) Have nucleoli 3) Cytoplasm 'indented' by surrounding RBCs

What is the pathogenesis of Dequervain's tenosynovitis?

Thickening of extensor retinaculum of 1st dorsal extensor compartment (1 of 6, which contains abductor pollicis longus and extensor pollicis brevis), leading to increase in friction and thus pain, associated with overuse (golfer/raquet), previous trauma, post partum state.

How would you classify pelvic fractures?

Tile classification with grades and sub-grades Grade A = stable A1: Not involving pelvic ring A2: Stable or minimally displaced fracture of the ring A3: transverse fracture of the sacrum Grade B = rotationally unstable but vertically stable B1: open book fracture (externally rotated) B2: lateral compression (internally rotated) B3: bilateral rotation Grade C = rotationally and vertically unstable C1: unilateral fracture dislocation e.g sacroiliac or iliac C2: bilateral with one type C and the other type B C3: both sides type C

What is the pathogen responsible for pityriasis versicolor and tinea versicolor?

Tinea: Malaseezia furfur Pityriasis: Malaseezia globosa But generally interchangable.

What are the factors in the extrinsic coagulation pathway?

Tissue factor Factor 7 Factor 10

What are the management goals of DDH?

To achieve concentrically reduced and stable hips with good acetabular cover (to avoid early OA).

What is the management of primary hyperaldosteronism?

To confirm Primary hyperaldosteronism (in situation of high aldo, low renin) 1) Normal saline suppression test a. Over 4 hours, 2L of normal saline given i. Renin expected to fall, leading to fall in aldosterone ii. But in 1' hyperaldosteronism, fall in renin does NOT result in fall in aldosteronism b. Can be bilateral adrenal hyperplasia or aldosterone producing adenoma (conn's syndrome) i. CT scan can be done but not diagnostic 1) Confirm with AVS (adrenal venous sampling) a) Catheter inserted via femoral vein up to ipsilateral adrenal vein b) Level of aldosterone measured, compared to contralateral side c) To determine side of problem or if bilateral adrenal hyperplasia ii. If adenoma = surgery iii. If bilateral adrenal hyperplasia = medical therapy 1) Aldosterone receptor antagonist: spironolactone a) Warn about erectile dysfunction and gynaecomastia b) Aldosterone AND testosterone receptors will be blocked Hence alternative is eplerenone = pure aldosterone blocker but expensive

What is Prehn's sign?

To differentiate between epididymitis and torsion, whereby the former experiences relief when the testicle is lifted.

What is the purpose of the stoma key?

To prevent premature retraction of stoma in either loop or stitched double barrel stoma. Usually in large bowel due to weight of feces.

What is the purpose of defunctioning stoma?

To reduce the mortality of anatomic leak. Need to do colon washout before (takes 1-2hrs), or else distal leak will still lead to feculent peritonitis. Be sure to discuss w anesthetist on patient status

What happens if you apply ESWL to a staghorn stone?

Too large, results in fragmentation that obstructs the ureters. 'SteinStrasse' the German word for "stone street", describing a possible complication of extracorporeal shock wave lithotripsy (ESWL) for urinary tract calculi, wherein a column of stone fragments forms that blocks the ureter.

What is the treatment of lichen simplex chornicus?

Topical steroids - Betametasone valerate is good (moderate potency) - Very potent steroids should be avoided on genital areas

What are the differences between a total colectomy, sub-total colectomy and a proctocolectomy?

Total colectomy: entire colon removed without the rectum Sub-total colectomy: part of colon removed Proctocolectomy: entire colon removed along with rectum

How would you treat medullar thyroid carcinoma?

Total thyroidectomy No TSH suppression therapy because it arises from parafollicular C cells. Serum calcitonin and CEA 6/12 after surgery -> cured if not raised (5% recurrence in 5 years) Treat recurrence surgically with RT or chemo as well

What is the pathogenesis of Osgood-Shlatter disease?

Traction apophysitis of the tibial tubercle due to overuse stress from the extensor mechanism that results in avulsion fracture of the tibial tuberosity.

What is malarone?

Trade name of combo drug: Atovaquone-proguanil Use this in patients with history of photosensitivty or contraindications of doxycycline (pregnancy, children, esophagitis)

What is the dosing regimen of tramadol?

Tramadol For acute pain: - Immediate release: 50-100mg PO 4-6hrly PRN (max dose 400mg/day, 300mg for >75 y/o) For chronic pain: - Extended release: 100mg PO OD (max dose 300mg/day) Watch for vomiting Tramadol is NOT indicated for those under 17 years old. Start at lower dose for elderly

What is TAVI?

Transcatheter Aortic Valve Implantation. Can be done through trans-femoral route.

1) Transfuse until stable, bleeding slow/stopped 2) If patient no longer bleeding, aim Hb 7g/dL 3) If patient no longer bleeding but has hx of cardiac surgery, aim Hb 9g/dL

Transfusion targets according to The New England Journal of Medicine.

What is talar shift? What does it imply?

Translation of the talus relative to the tibia and fibula. Indicates disruption of stabilising ligaments.

What chromosomal abnormality is associated with Ewing's sarcoma? What other tumor is also associated?

Translocation of 11 and 22 Primitive neuroectodermal tumors and Askin tumors

What is the pathogenesis of vocal resonance and its implications?

Transmission of sounds across lung parenchyme. Increased vocal resonance = improved transmission of high pitched sounds - Pneumonia w consolidation ('ninety-nine sounds clearer than normal lung which is muffled)

What are some surgical options for large bladder stones?

Transurethral vs suprapubic vs open Cystolitholapaxy (fragmentation of stone w removal) vs Cystolithotomy (just removal, for very alrge stones

What investigations would you order for a traumatic shoulder dislocation?

Trauma series: 1) AP 2) Y-scapular view 3) Axillary views or Velpeau axillary (if too painful to do axillary, patient leaning backward over Xray with arm in sling) Pre-reduction and post-reduction must be taken +/- Hill Sachs view: AP w shoulder in maximal internal rotation to visualise the posteriolateral defect (Resulting from impact with antero-inferior glenoid rim)

How do you treat prurigo nodularis?

Treat w tissue destructive methods 1) Cryotherapy Anti-inflammatory methods 1) Intralesional and topical steroids 2) Phototherapy w NBUVB NOT immunotherapy

What are the management principles of Rheumatoid Arthritis?

Treatment depends on 1) Disease activity 2) Previous DMARD use 3) Early vs established RA (>/=6/12 duration or meeting ACR classification criteria) According to the American College of Rheumatology guidelines for RA. Pharmacologic options: 1) DMARDS: - Methotrexate (PO/SC) - Hydroxychloroquine - Leflunomide - Sulfazalazine 2) Biologic DMARDS - TNF inhibitors (adalimumab, eternacept, infliximab etc) 3) Immunomodulators - Glucocorticoids: - Predisone - Immunosuppressants - Azathioprine - Cyclosporine - Others - Minocycline (anti-inflammatory tetracycline antibiotic) - Gold (IM)

What is hemolytic uremic syndrome? How does it happen?

Triad of 1) Anemia 2' hemolysis 2) Thrombocytopenia 3) AKI with uremia Classically: 1) Shiga toxin absorbed and reaches kidneys -> binds to endothelial cells of glomerulus 2) Activates apoptosis and leukocyte binding to endothelial cells -> AKI + activates inflammatory cascade + deactivates metalloproteinases (ADAMST13) -> thrombogenic -> thrombotic thrombocytopenic purpura - THROMBOCYTOPENIA 3) Microthrombi form and obstruct small capillaries -> microangiopathic hemolysis as RBCs burst while trying to squeeze through -> HEMOLYSIS 4) Shiga toxin also binds to complement H and activates complement pathway, causing inflammation, hemolysis and thrombosis to spread systemically. Seen in Shiga producing bugs - Shigella dysentriae - EHEC - And others which is hard to remember

What is yellow nail syndrome?

Triad of 1) Yellowish, opalescent, slow growing nails 2) lymphedema 3) Pleural effusion Postulated to be due to lymphatic insufficiency, thus, also often a/w LL swelling

What symptoms do patients with disseminated gonoccocal infections present with?

Triad of: 1) Tenosynovitis 2) Dermatitis (pustules on an erythematous base) 3) Polyarthralgia/synovitis +/- purulent arthritis

What is the distribution of death in trauma?

Trimodal distribution 1) Immediate - Apnoea 2' brain/high spinal injury or catastrophic haemorrhage from great vessel transection. - Prevent onset of trauma 2) Early - Haemorrhage related - Stabilise patient according to ATLS 3) Late death - MDOF or sepsis - Observe for complications

What are the components to a full diagnosis of glomerular disease?

Tripartite description: 1) Clinical manifestation 2) Histological findings 3) Underlying etiology e.g Patient presents with nephrotic syndrome from membranous nephropathy secondary to lupus nephritis.

What is the treatment for pelvic fractures?

Triple therapy: 1) Preperitoneal packing 2) External fixation 3) Angiographic stabilisation

What is the difference between true limb length and apparent limb length?

True LL: measured from ASIS to medial malleolus/heel of foot Apparent: measured from umbilicus to medial malleolus/heel of foot.

How can you classify closed fractures in trauma?

Tsherne's classification 0, I, II, III 0: Indirect injury, no visible soft tissue injury/swelling I: direct injury, mild soft tissue injury with superficial abrasions and shallow lacerations II: Severe soft tissue injury with deep abrasions and lacerations and contusions III : Degloving or avulsion of skin, arterial disruption, compartment syndrome

1) Peak dose dyskinesia: reduce dose and frequency, add on COMT inhibitors (entacapone) that increase Ldopa half life, initiate therapy w dopamine agonist first before L dopa. 2) Wearing-off/off period dyskinesia: increase dose and frequency, adding on DA, MOAI, COMTI. 3) Diphasic dyskinesia

Types and management of L-dopa dyskinesia

1) Peak dose dyskinesia 2)

Types and management of levadopa dyskinesia

Acute immune-mediated: 1) Acute hemolytic transfusion reaction 2) Febrile non-hemolytic reaction 3) Urticarial/anaphylactic 4) Transfusion related acute lung injury: neutrophil activation and 2' alveolar capillary endothelial damage (higher risk in plasma transfusion) Acute non-immune mediated: 1) Bacterial contamination: febrile reaction/septic/endotoxin release w sepsis 2) Circulatory overload: presents w fluid overload 3) Non-immune hemolytic reaction (ddx of exclusion) Delayed immune-mediated: 1) Delayed transfusion reaction (1-14 days, due to pre-formed ab/rh incompatibility, mild, self-resolving) 2) Transfusion associated GvHD (7-10 days, due to donor T-lymphocytic attack on recipient cells) Delayed non-immune mediated: 1) Transfusion hemosiderosis (cx of long term transfusion patients e.g in Thal) 2) Disease transmission (HCV common, HIV, HBV, Westnile etc) = 1 in 10000

Types of transfusion reactions

What are the types of and possible causes of urticaria?

Types: 1) Spontaneous - no identifiable cause 2) Physical urticaria - physical factor identified e.g heat, cold, UV, exercise, vibration, water 3) Contact urticaria - contact w substances that cause if e.g latex 4) Autoimmune urticaria - abnormal TFT, need to assess for autoAb and do - Autologous serum skin test - Basophil histamine release assay

What are the differences the typhoid and paratyphoid presentation, pathogen and pathogenesis?

Typhoid vs paratyphoid 1) Pathogen - Salmonella typhi/paratyphi (A, B, C) 2) Incubation period - 8-14 days/1-10 days 3) Symptoms - All organisms are ingested and survive exposure to gastric acid before reaching small bowel to penetrate epithelium and invade lymphatic tissue, disseminating via lymphatic and hematogenous route. - Severe/less severe - Abdominal pain - Fever (stepwise rise, from bacteremia) w chills (rigors rare) 4) Signs - Faget's sign - Rose spots - Hepatospleenomegaly, intestinal bleeding (peyer's patches), perforation w peritonitis

When is ART delayed in HIV patients? For how long?

Typically infections in closed spaces with risk of damage in IRIS. 1) CMV retinitis 2) TB meningitis or TB with CD4 <50 (IRIS) 3) Cryptococcal meningitis Delayed for about 3-8 weeks.

How would you follow up on a patient with a 4-4.9cm fusiform AAA?

U/S aorta at 1 year intervals

What is the diagnostic criteria of eczema?

UK diagnostic criteria: clinical diagnosis 1 Mandatory criteria: - History of itchy skin 5 Major criteria: - Involvement of flexural regions - Clinical examination shows flexural involvement - Hx of atophy - Hx of dry skin - Started before 2 years old (unless the child is 4)

What might recalcitrant seborrhoeic dermatitis indicate?

Underlying HIV infection.

What is the pathogenesis of hiradenitis suppurativa? How would you manage it?

Unknown etiology, a/w obesity, smoking, genetic predisposition to acne One of the follicular occlusion syndromes Keratinous plugging of hair follicle w dilation, inflammation, bacterial growth, duct rupture and extension of suppuration, tissue destruction and sinus tract formation Not a simple infection, mx includes 1) Systemic Abx 2) Intralesional triamcinolone 3) Oral isotretinoin +/- infliximab 4) Surgical drainage +/- split skin grafting

What is the pathogenesis of Congenital muscular torticollis?

Unknown, but postulated to be compartment syndrome of SCM in utero due to impaired venous outflow.

What is the pathogenesis of tertiary hyperparathyroidism and how is it treated?

Unregulated parathyroid secretion by parathyroid glands after chronic secondary hyperparathyroidism in renal failure; when secondary hyperparathyroidism is treated by PTH levels remain high = 3' hyperPTH. Treated conservatively by preventing chronic renal failure or control of secondary hyperparathyroidism. Surgical treatment involves removal of 3 and a half (of 4) parathyroid glands.

What kind of urinary dysfunction do patients with cauda equina present with? What mechanism does it occur by?

Urinary retention -> overflow incontinence Detrusor weakness w inability to relax the internal urethral sphincter (both innervated by S1-S2)

What are the clinical signs of urethral injury?

Urine - Anuria - Gross haematuria - Blood at urethral meatus Clinical exam: - High riding prostate - Butterfly haematoma (bleeding + confinement by Buck's fascia) - Inability to insert cath/no urine aspirated

What is the indicated treatment for patients with urticaria/angioedema refractory to antihistamine treatment?

Use of 2nd line agents - Mast cell stabiliser e.g montelukast - Immunocuppressants e.g prednisolone/ciclosporine * Must look at PmHx for contraindications to treatment e.g Giving steroids to a patient with closed angle glaucoma (steroids raise intraocular pressure)

What is the Sgarbossa Criteria and what does it imply?

Used to diagnoses infarctions in patients with Left Bundle Branch Block OR a ventricular paced rhythm 1) ≥ 1 lead w positive QRS with ≥1 mm of concordant ST elevation (5pts) i.e STE in a QRS positive lead 2) ≥ 1 lead of V1-V3 with ≥ 1 mm of concordant ST depression (3pts) i.e ST depression in anterior lead 3) ≥ 1 lead anywhere with ≥ 1 mm STE and proportionally excessive discordant STE, as defined by ≥ 25% of the depth of the preceding S-wave. (2pts) Score of >/= 3 pts is 90% Specific for infarction

What is the Swimmer's view and when is it indicated?

Useful for assessing full c-spine, as normally shoulders obstruct the view of C7.

What is the use of a functional thyroid scan?

Uses technetium 99 to determine the function of the thyroid gland, to see if the lesion is Hot or Cold. Hot nodules are virtually non-malignant. Cold nodules have 10-15% chance of malignancy Only done if patient has low TSH (since if high TSH, will all be cold)

What is the management of intraparenchymal bleeds?

Usually conservative it no significant neurological deficit. SBP to less than 140-150 within 24-48 hrs Consider surgical clot evacuation (not for those w dominant left sided bleeds, as it will not change the outcome)

What is the cause of greater trochanteric fractures and how are they treated?

Usually due to eccentric muscle contraction in elderly or less commonly due to direct trauma. Treatment is usually nonoperative. Unless, young patient with widely displaced greater trochanter: - ORIF with tension band wiring or plate and screw fixation with a 'hook plate'

How and where does a Branchial cyst arise? How would you treat it?

Usually found in the 1/3, 2/3 junction of anterior border of SCM. Originates from cleft that failed to fuse (pouches give rise to organs e.g tympanic membrane/ inferior parathyroid (3rd)) Hence when needle used to draw fluid to see cholesterol crystals (classical diagnostic factor) Excision requires tracing of the tract up to it's orgin (which crosses the bifurcation of the carotid) up to posterior wall of the anterior tonsillar pillar

How would you manage a ganglion cyst?

Usually no management indicated, self resolving after some months Surgery indicated if nerve compression - Drainage (10-15% recurr in 6-12/12) - Excision (after drainage, recurs in 20-30%)

What are the typical sites of distribution of psoriatic skin changes?

Usually on extensor surfaces e.g - Knee - Elbow Also on other exposed regions: - Scalp - Natal cleft - Navel Plus folds of skin: - Intragluteal folds - Submamary folds

What is the natural history of erb's palsy and what is the management?

Usually recovers as is neurpraxia in nature 1) Non-operative - Observation 2) Operative - Indicated in persistent palsy, aim to correct deformities and restore as much function as possible - Tendon transfers, soft tissue release.

When is surgery indicated in Crohns?

Usully not indicated, only occurs in emergency Fistuloplasty - high rate of recurrance Colon cancer (increased risk), can come in w obstruction or screening

What are the risk factors of AVN? Which are the most common?

VITAMIN C 1) Vascular - hemoglobinopathies e.g sickle cell anemia - Vasculitis 2) Infective - Septic arthritis 3) Trauma - Previous trauma 4) Autoimmune - SLE -RA 5) Metabolic - alcohol abuse - obesity - smoking - Caisson's disease (Deep sea diving) - Gaucher's disease - Cushing's disease 6) Iatrogenic - Steroid usage (non dose related) 7) Neoplastic - Malignant invasion - Paget's disease 8) Congenital - Perthes disease - SCFE - DDH - Coagulopathies Most common are steroids and alcohol, especially in view of bilateral AVN, which is less likely to be traumatic.

What is the difference between Valganciclovir and ganciclovir?

Valganciclovir is the oral prodrug of ganciclovir, it is converted to ganciclovir on absorption.

What are the causes of mitral regurgitation?

Valvular: -Mitral valve prolapse (most common form of myxomatous valve disease: abnormally stretchy valves) -Rheumatic heart disease -Infective endocarditis/endocarditis -Annular calcification (degeneration of mitral valve ring) Myocardial: -Left ventricular dilatation -Papillary muscle dysfunction/acute rupture -Cardiomyopathy -Coronary artery disease -Trauma - Congenital (ostium primum atrial defect, partial atrioventricular canal and repaired transposition of great arteries) Systemic: -Connective tissue disorder (e.g marfan's)

What is the mechanism of injury in tibial plateau fractures?

Varus/valgus load +/- axial load A/w soft tissue injury if high energy A/w insufficiency fractures if low energy

What are the causes of hypertension in patients with neurofibromatosis?

Vascular malformations: - Coarctation of aorta - Renal artery stenosis Intracranial tumors: - Phaechromocytoma Spinal tumors: - Noradrenaline producing paraspinal tumor Idiopathic/baseline: - Essential hypertension

What is the VAD?

Ventricular assistive device - Bypasses the ventricles, taking blood from the atrium to the exiting artery to compensate for poor ventricular contraction - Can do bilateral if both ventricles are dysfunctional

What is VACTERL?

Vertebral defects Anal atresia Cardiac defects Tracheoesophageal fistula Renal anomalies and Limb abnormalies If at least 3 = VACTERL

What are the risk levels in hypercholesterolemia?

Very high risk: establised CAD, athersclerotic CVA, aortic aneurysm, PAD High risk: Moderate to severe CKD, DM w/o CAD/PAD etc High to low risk: estimate 10 year CAD risk via CPG tables.

What are the grades of oesophagitis?

Via Savary-miller classification, 5 grades 1) Reddish spots: - One or more supravestibular non-confluent reddish spots +/- exudates 2) Erosive/exudative lesions - May be confluent but NON circumferential 3) Circumferential erosions - covered by haemorrhagic and psuedomembranous exudate 4) Presence of chronic complications such as deep ulcers, stenosis, strictures 5) Barrett's metaplasia

How are anal fistula's investigated?

Via imaging 1) Endoanal U/S w H2O2 for hypoechoic effect 2) MRI - Can visualise entire pelvis, beyond sphincter complex 3) CT/Fistulography - Contrast needed - For complex fistula's/unusual anatomy

How is closed reduction for a posterior hip dislocation done?

Via the Bigelow Maneuver Usually done under General Anesthesia in the OT.

What is VATS?

Video-assisted thoracic surgery

What is the J sign in humeral X rays?

Visible glenohumeral ligament in the joint space due to HAGL (humeral avulsion of the glenohumeral ligament)

What are the boundaries and contents of the carpal tunnel?

Volar: Flexor retinaculumn Radial: scaphoid tuberosity, trapezium Ulnar: Pisiform, hook of hamate Contents: 1) FDP, FDS 2) FPL 3) Median nerve

How is diabetic foot disease classified and how does it guide management?

Wagner's classification based on degree of infection 0 to 5 0: No ulcer, just high risk - Education, disease control - Protective footwear - Regular podiatry/clinical exam 1: Ulcer, superficial, no involvement of underlying tissue - Offloading via total contact cast/walking brace/special footwear 2: Ulcer, deep w muscle/tendon but no bone, no abscess - Debridement - Wound care - Culture specific Abx - Offloading 3: Ulcer, deep w bone (OM) or abscess - Debridement/partial amputation - Wound care - C/S Abx - Offloading 4) Forefoot gangrene - Lower extremity amputation - Vascular consultation 5) Whole foot gangrene - Major extremity amputation - Vascular consultation

If protein C and S are used for anticoagulation, why does warfarin (that blocks them) not cause thrombosis?

Warfarin does cause initial hypercoagulability. Protein C and S has shorter half life than 9, 10 and 2 respectively, and they are involved in anticoagulation. Hence they break down sooner and no more is made (because of warfarin). So thombosis increases initially... Until factors 2, 9 and 10 breakdown later, so you bridge w heparin in the first few days ~ 3 days (protein S half life 42.5 hrs, factor 60-70 hrs)

What are the differences between Warfarin and NOACs?

Warfarin is: - Inhibits vitamin K dependent synthesis of factor 2, 7, 9, 10, protein C, S and Z. - More potent - More side effects/drug interactions - Require close follow up for INR maintanance - Hepatically metabolised and excreted - Can be used in ESRF, CKD, Prosthetic valves - Reversed with FFP or cryoprecipitate with Vit K1 while NOACs are: - reversible/non-reversible antagonists of clotting factors - Less potent - Fewer side effects/drug interactions - Require less follow up for INR - Renally excreted thus contraindicated in Renal failure - Only expensive, specific antidotes available e.g for dabigatran is idarucizumab @ $3500/dose

Which HPV virus sub types cause warts and which cause cervical cancer?

Warts : 6 and 11 Cancer: 16 and 18

How would you classify fibular fractures?

Weber's classification of FIBULAR fractures (based on level of fibular fractures relative to syndesmosis, 3 types) Type A: Infrasyndesmotic - low risk of syndesmotic injury and instabiltiy Type B: Trans-syndesmotic - 50% risk of syndesmotic injury and ankle instability Type A and B can do closed reduction w internal fixation via screws w cast for 6-8 weeks Type C: Suprasyndesmotic - 100% risk of syndesmotic injury and ankle instability Type C requires ORIF, w cast for 6-8 weeks

What are the types of ANCA associated vasculities?

Wegner's: Granulomatosis with polyangitis Churg-strauss: Eosinophillic granulomatosis with polyangitis Microscopic polyangitis

What are the US findings of liver hemangioma? What are the possible differentials?

Well defined Lobulated Homogenous hyperchoic (unless fatty liver makes it look relatively hypoechoic) Doppler shows peripherally feeding vessels Compressibility is pathognomonic Differentials: 1) Benign: - Focal nodular hyperplasia - Hepatic adenoma w high fat - Focal fatty change - Inflammatory pseudotumour of liver 2) Malignancy - Hepatic mets e.g colorectal, breast, pancreas, RCC, thyroid, melanoma) - HCC - Cholangiocarcinoma (intrahepatic)

- Rash Psoriasis SLE - malar rash Infection - Rheumatic fever, lyme disease, viral exanthem Drug rxn - NSAIDS, Sulfsalazine Raynauds Vasculitis - Orogenital ulceration SLE Behcet's disease (assymptomatic, non erosive arthritis) Reactive arthritis - Alopecia SLE - Eyes: red eyes, gritty/dry eyes - Recent infection: rheumatic fever (enteritis, urethritis, gonnoccocus) - Other systemic: fever/weightloss/anorexia/night sweats

What are extra-articular features important in assessing joint pain and what are their implications?

• Pain • Stiffness/limited RoM • Wasting . Metabolic disturbances (Hypo/hyperK etc)

What are possible causes of pseudo-weakness?

Myasthenia Gravis Foundation of America and Osserman's grading 1) MGFA I - ocular muscle only II - mild weakness in non-ocular muscles IIA - limb and axial muscles affected IIB - Respiratory and bulbar muscles affected III - moderate weakness IV - Severe weakness V - Intubation required 2) Osserman's grading I - ocular IIA - mild generalised w slow progression IIB - moderate generalised III - acute fulminant MG IV - late severe MG

What are the 2 grading systems for weakness in Myasthenia Gravis?

1) Microscopic polyangitis 2) Wegner's granulomatosis (i.e granulomatosis with polyangitis) 3) Churg strauss syndrome (i.e eosinophillic granulomatosis with polyangitis) 4) Drug induced (hydralazine, propylthiouracil, minocycline) A Microscopic Weiner Choked on Drugs

What are the ANCA associated vasculitides?

Electrical alternans: - QRS complexes of alternating amplitudes - Due to swinging of heart in a fluid filled pericardium + Tachycardia and small QRS complexes

What are the ECG findings in severe cardiac tamponade and what is the pathophysiology?

1) Severe cutaneous adverse drug reactions - Steven-johnson syndrome - Toxic epidermal necrolysis - DRESS syndrome - Consider testing for HLA B 5801 2) Can precipitate gout flares - Hence prescribe with colchicine - Start at lowest dose and assess for uric acid levels + symptoms on follow up 3) Can interact with mercaptopurine (allo prevents its metab, causing excessive plasma levels), azathioprine and theophylline - Reduce dose by ~ 75%

What are the adverse effects and drug-drug interactions of Allopurinol?

1) Hydroxychloroquine 2) Methotrexate 3) Anakira (biological DMARD) 4) NSAIDs +/- PPI 5) Intra-articular steroids

What are the agents for treatment of acute synovitis in pseudogout?

1) No treatment for crystal deposits 2) Consider surgery to replace damaged joints

What are the agents for treatment of chronic pseudogout (CPPD - calcium pyrophosphate deposition)?

1) NSAIDS (indomethacin and naproxen) +/- PPI 2) Cox 2 inhibitors (etoricoxib) 3) Oral steroids (prednisone) 4) Colchicine

What are the agents recommended in monotherapy of acute gouty flares?

1st line: Low dose colchicine OR low dose NSAIDs w PPI 2nd line: Low dose oral steroids

What are the agents used in prophylaxis of gout arthropathy?

1) Valvular - Rheumatic heart disease - Infectious endocarditis - Congenital bicuspid valve (Associated with coarctation of aorta) 2) Aortic - Aortic root dilation (syphillis, RA, Ankylosing spondylitis, Marfan's, severe hypertension) 3) Mechanical causes - Trauma - Aortic dissection - Rupture of sinus of valsalva/aneurysmal rupture

What are the causes of Aortic Regurgitation?

1) Traumatic injury 2) Ischemia - Occlusion of vertebral arteries 3) Inflammatory: - TB - MS

What are the causes of Brown Sequard Syndrome?

Most common: Diabetic kidney disease Also: - Hypertension - Glomerulonephritis - Gout nephropathy - Medications e.g NSAIDs, traditional medications

What are the causes of Chronic Kidney Disease?

DAD MIC 1) Dehydration - Acute (due to diuretics, hyperglycemia, haemorrhage, vomitting, diarrhoea) - Chronic (seen in autonomic (sympathetic) dysfunction) 2) Cardiac - Cardiac pump failure (Aortic stenosis, pericarditis/myocarditis, arrthymias) 3) Autonomic - Neurodegenerative disease (parkinson's, parkinson+ syndromes, 'Pure autonomic failure - comes with impotence, urinary and GIT symptoms) - Neuropathy (peripheral neuropathy - can affect postganglionic autonomic nerves) -- DM -- Inflammatory (Amyloidosis, Sjogren's, Sarcoidosis, autoimmine autonomic impairment via anti-nAhR ab) -- Infections (syphillis, lyme, HIV, Chagas, Guillian Barre) -- Congenital/genetic (porphyria, Riley Day syndrome aka familial dysautonomia) -- Neoplastic (Paraneoplastic autonomic neuropathy) 4) Drugs - Alcohol - Alpha blockers e.g terazosin - Antidepressants e.g SSRIs, Trazodone, MOAI, TCA - Antihypertensives - Antiparkinson (Ldopa) - Antipsychotic (Olanzaine, risperidone) - Diuretics (hydrochlorothiazide, furosemide) - Muscle relaxant - Analgesics (Morphine) - Phosphodiesterase inhibitors (Sildenafil, tadalafil) 5) Idiopathic - Aging (decrease in baroreceptor-mediated response 6) Metabolic - Adrenal insufficiency - Post-prandial hypotension (? splanchnic pooling w reduced venous return)

What are the causes of Postural hypotension?

1) Pre-renal - Systemic causes of renal hypoperfusion - e.g shock, dehydration 2) Renal - Acute tubular necrosis: most common, caused by prolonged renal hypoperfusion and nephrotoxic agents - Acute interstitial nephritis: allergic reaction due to drugs - Acute glomerulonephritis: e.g goodpasture, ANCA vasculities, post infectious GN

What are the causes of acute renal failure?

1) Flexion injury 2) Prolapsed intervertebral disc 3) Anterior vertebral artery occlusion 4) Post abdominal aortic aneurysm surgery (infrarenal approach has risk of anterior vertebral artery damage = 'anterior spinak artery syndrome')

What are the causes of anterior cord syndrome? (4)

1) Calcific stenosis (most common, occuring in old age) 2) Congenital: Bicuspid aortic valve 3) Rheumatic heart disease

What are the causes of aortic stenosis?

1) Neck hyperextension 2) Spinal stenosis 3) Tumors/syringomyelia

What are the causes of central cord syndrome? (3)

DELIRIUM Drugs e.g opiods Electrolye imbalances Lack of sleep Infections Restraints/reduced sensory input Intracranial e.g CVA, SAH Urinary retention/constipation Myocardial/pulmonary

What are the causes of delirium?

1) Infection - Bacterial: Brucella, typhoid fever, septicaemic plague - Viral: Acute viral hepatitis, CMV, IM, Rubella, Malaria - Parasites: Leishmaniasis, schisotosomiasis, filiriasis 2) Hematologic diseases: - Myeloproliferative disease - Leukemia (chronic myeloid leukemia) - Lymphoma (Hodgkin's/non-hodgkin's) - Anemia: Pernicious, sickle cell - Thalassemia - Myelofibrosis 3) Metabolic disease - Chronic liver disease w portal hypertension/chronic active hepatitis - Amyloidosis - Sarcoidosis - Acromegaly - Glycogen storage disease 4) Vascular - Congestive cardiac failure - Budd-chiari syndrome

What are the causes of hepato-splenomegaly?

1) Intra-abdominal - Viseral inflamamtion/infection: -Pancreatitis/hepatitis/gall-stones/appendicitis -Look for localising symptoms 2) Neurological - Meningitis/encephalitis - Vertigo (central/peripheral) - Raised intracranial pressure - Migraines (cyclical vomiting) 3) Endocrine - Uremia - DKA - Adrenal insufficiency - Electrolyte imbalances (hypercalcemia) - Thyroid disease 4) Other infections - Pneumonia and UTI 5) Cardiac - AMI - CCF 5) Drugs and toxins - Chemotherapy - Antibiotics - Cardiac medications - Analgesics - Alcohol - Toxins etc 6) Pregnancy/hyperemesis gravidarum - (Early morning vomiting in young reproductive age females) 7) Psychiatric - Anorexia/bulimia - Anxiety - Depression

What are the causes of nausea/vomitting without diarrhoea and abdominal distension?

1) Neck trauma 2) Prolapsed intervertebral disc (esp cervical, where pressure affects posterior more than anterior despite the location of the lesion) 3) Posterior spinal artery occulusion 4) B12 deficiency 5) Syphillis 6) Multiple sclerosis

What are the causes of posterior cord syndrome? (6)

Different types of protein, due to different causes. 1) Glomerular proteinuria - Albumin loss - Intermittent: postural, fever, exercise, dehydration -> Primary: - Idiopathic (minimal change disease, FSGS, Mesangial proliferative) - Glomerulonephritis (membranoproliferative GN and membranous GN) -> Secondary ( HI CADE) - Infective (post strep GN, IE, malaria, hepB/C, HIV) - Congenital (Alport's, denys drash syndrome: gonadal dysgenesis, nephropathy and neuroblastoma) - Autoimmune (SLE nephritis, HSP, ANCA vasculitis) - Drugs (Penicillamine, gold, mercury) - Endocrine (DM nephropathy) - Haematologic (ALL, CML) 2) Tubular proteinuria - Low molecular weight protein - Can be Inherited - Can be Acquired: pylonephritis, interstitial nephritis, AKI and drugs-NSAIDs 3) Overload proteinuria - Hemoglobinuria/myoglobinuria? Bence Jones proteinuria (seen in multiple myeloma) 4) Post-renal proteinuria - Any protein from urinary tract distal to kidney

What are the causes of proteinuria?

6 causes of Pseudo 6: 1) Muscle: - Thyroid ophthalmopathy with muscle fibrosis - Ocular myositis (rare) 2) Neuromuscular Junction - Myasthenia Gravis 3) Duane's retraction syndrome 4) Convergence spasm of Cogan 5) Decompensated squint 6) Thalamic/mescencephalic esotropia + 2 rare 1) Cerebellar esotropia 2) Heavy eye syndrome in severe myopia

What are the causes of pseudo 6th nerve palsy?

Organised by course of 6th nerve 1) Brainstem - Vascular - Neoplastic (tumors, CPA etc) - Demyelinating diseases (Multiple sclerosis) - Metabolic (Wernicke's and Central Pontine Myelinolysis - CPM) 2) S/A Meninges - Vascular - Intracranial HTN (raised ICP) - Inflammatory (menigitis, Miller-fischer syndrome) - Siderosis (deposition of iron) 3) Base of skull - Neoplasm - Chordoma (rare skull base and spine bone sacroma) - Meningioma - NPC (esp w conductive hearing loss via ET compression in fossa of rosenmuller) - Vascular - Cerebral vein thrombosis - Gradenigo syndrome (petrositis/inferior petrosal sinus thrombosis: comes w palsy plus facial pain +/- deafness and facial palsy) 4) Cavernous sinus - Inflammatory - Immune: Tolsa hunt (idiopathic granulomatous inflammation of cavernous sinus w painful ophthalmoplgia) - Infective: aspergillosis/abscess etc - Vascular - Caroticocavernous fistula (triad of conjuntivitis, infection and proptosis) - Aneurysms - ICA dissection - Neoplastic - Meningioma - NPC

What are the causes of true 6th nerve palsy?

• Hypoactive reflexes • Downgoing plantars • Reduced tone, flaccid paralysis • Severe atrophy . Fasciculation common

What are the characteristics of LMN lesions?

• Hyperactive reflexes • Upgoing plantars • Increased tone • No fasciculation • Usually no atrophy unless chronic . Pyrimidial/regional distribution of weakness

What are the characteristics of UMN lesions?

• Proximal weakness • Reflexes equivocal • Downgoing plantars • Mild atrophy . Tone equivocal

What are the characteristics of myopathic lesions?

High lesion: Elbow/shoulder etc - Carpal tunnel, above carpal tunnel and AIN deficits AND: All forearm muscles except FCU and FDP of F3,4 -> cannot radial deviate or flex wrist

What are the clinical signs of a high median nerve lesion?

High lesion: Shoulder etc - PIN and humeral shaft deficits AND: - Axillary nerve loss causing triceps deficit -> elbow drop - Consider Saturday night palsy

What are the clinical signs of a high radial nerve lesion (Shoulder)?

High lesion: Shoulder - Guyon canal deficits and Ulnar canal deficits AND: Flexor carpi ulnaris deficit -> cannot ulnar deviate wrist

What are the clinical signs of a high ulnar nerve lesion (Shoulder)?

Low lesion: Carpal tunnel - Flexor pollicis brevis of LOAF affected -> Cannot flex thumb 'point to ceiling, don't let me push it down' - Palmar cutaneous nerve given off before carpal tunnel -> Thenar sensation preserved Consider Tinels test and Phalen's test

What are the clinical signs of a low median nerve lesion (Carpal tunnel)?

Low lesion: Posterior interosseus nerve All extensor muscles (except ERCL) -> cannot retropulse thumb or extend fingers

What are the clinical signs of a low radial nerve lesion (PIN)?

Low lesion: Guyon's canal @ wrist - 1st Dorsal interossei deficit -> cannot abduct index finger - 1st Dorsal interossei deficit -> cannot adduct thumb AKA Froment's sign where flexor pollicis brevis is used to grip paper instead - 2nd Dorsal interossei deficit -> cannot cross F1 and F1 4th Dorsal interossei deficit -> cannot adduct F4 AKA Wartenberg's sign where extensor digiti minimi overpowers dorsal interossei to cause abduction

What are the clinical signs of a low ulnar nerve lesion (Guyon's canal)?

1) Medium lesion: Anterior interosseus nerve - Carpal tunnel deficits MAY NOT BE PRESENT: Flexor pollicis longus + FDP of F1 (+F2) affected -> cannot do OK sign

What are the clinical signs of a middle median nerve lesion (AIN)?

2) Medium lesion: Median nerve above carpal tunnel - Carpal tunnel deficits AND: Palmar cutaneous nerve deficit -> Loss of thenar sensation + splitting of ring finger sensation

What are the clinical signs of a middle median nerve lesion (Median nerve above carpal tunnel)?

Middle lesion: Humeral shaft # - PIN deficits AND: - Extensor carpi radialis longus loss -> loss of wrist extension i.e wrist drop when combined with other extensor deficits - Superficial branch of radial nerve deficit -> loss of sensation over dorsal 1st web space - TRO C6 radiculopathy

What are the clinical signs of a middle radial nerve lesion (Humeral shaft)?

Medium lesion: Ulnar canal at elbow - Guyon canal deficits AND: - FDP of F3,4 deficit -> cannot flex DIPJ of ring finger - Palmar branch of ulnar nerve deficit -> loss of hypothenar and little finger sensation Check ulnar nerve subluxation

What are the clinical signs of a middle ulnar nerve lesion (Ulnar canal)?

1) Murmur - An Early Diastolic Murmur Accentuated by: Sitting forwards (aortic pathology i.e posterior) Holding expiration (left sided pathology) 2) Peripheral/pulse - Quinke's sign (visible pulsation in nail bed) - Waterhammer pulse/collapsing pulse - Brachial dance - Corrigan's sign (visible carotid pulsation in the neck) - De Musset's sign (head bobbing in time w heart beat) - Muller's sign (pulsation of the uvula) - Traube's sign (audible pistol shots of systolic/diastolic over femoral) - Duroziez sign (audible diastolic murmur on compression auscultation of femoral artery)

What are the clinical signs of an Aortic Regurgitation?

1) ESM heard loudest at right upper sternal edge (aortic region) 2) Pulse: pulsus parvus et tardus (slow rising and slow falling, low volume) 3) Soft second heart sound - poorly mobile and stenotic valve 4) Reversed splitting of S2 (late closure of aortic v relative to pulmonary v due to mechanical or electrical prolongation of systole) 5) Single second heart sound implies fibrosis and fusion of leaflets 5) Normal second heart sound implies mild stenosis

What are the clinical signs of aortic stenosis?

Split into complications of disease and complications of treatment (HHHIII) 1) Disease - Hypovolemia (bec of 3rd spacing, may lead to organ ischemia) - Hypercoagulability (loss of antithrombin 3 + increase liver clotting factor synthesis) - Hypercholesterolemia (increased lipoprotein synthesis by liver, accelerated atherosclerosis) - Immunodeficiency (loss of immunoglobulins + steroid toxicity) - Impaired growth (loss of proteins) 2) Iatrigenic (treatment related) Iatrogenic Cushings (steroids) Iatrogenic renal toxicity (ciclosprin) Iatrogenic neutropenia

What are the complications of nephrotic syndrome?

4 Time related 1) Age 2) DOB 3) Time of day 4) Year 3 Place related 1) Where are we 2) What is your home address 3) Can you remember '37 bukit timah road', I will ask you later 2 People related 1) Recognize doctor and nurse 2) Who is the current prime minister? 1 Counting 1) Count backwards from 20 to 1 7-8 = mild cognitive impairment at time of test 4-6 = moderate <3 = severe

What are the components of the abbrieviated mental test?

2 peaks 1) 20-30 years old, female predominance 2) >50 years old, male predominance

What are the demographic risk factors for Myasthenia Gravis?

RTA results from either impaired distal acidification (type 1), impaired proximal bicarb reabsoprtion (type 2) or impaired secretion or effect of aldosterone (type 4) Think of RTA when there is HYPERchloremic metabolic acidosis, look at urine pH to determine type. Primary causes: - Idiopathic/sporadic - Hereditary Secondary causes: - Autoimmune disease: Sjogren's, SLE, Primary biliary cirrhosis, Thyroiditis - Drugs: toulene (HAGMA can also) , lithium, amphotericin B, ibuprofen - Hematologic: hypergammaglobinaemias, sickle-cell anemia - Renal: obstructive uropathy (hx of urolithiasis) and chronic pyelonephritis

What are the diagnosis, pathogenesis and causes of Renal Tubular Acidosis?

1) Pathophysiology: LES has antibodies against pre-synaptic nerve that affects calcium channels which reduces Ach release while MG is against post-synaptic AchR. 2) Clinical course: LES occurs proximally and moves out (cannot stnad from sitting etc) , while MG occurs most commonly from eyes and moves down to mouth and body 3) Improvement with activity: LES improves with activity while MG is fatigable and worsens with activity. 4) Associations: LES is a/w small cell lung cancer while MG is a/w thymomas (75% of cases, the rest is genetic or congenital or idiopathic) 5) Clinical signs: LES presents with arreflexia while MG usually has preserved reflexes

What are the differences between Lambert Eaton syndrome and Myasthenic Gravis?

1) Pathophysiology: MFS is a variant of Guillian-Barre syndrome that occurs after infection by campylobacter jejuni, CMV, EBV, Strep.pyogenes etc 2) Clinical course: occurring about weeks after initial infection, manifesting as ocular weakness that descends vs guillian barre that presents as symmetrical muscle weakness that usually begins in legs and ascends, while MG occurs in eyes first before spreading to mouth and then rest of body, so they are q similar. 3) Clinical signs: classical triad of MFS is very similar to MG - Opthalmoplegia - Areflexia (normal reflexes in MG) - Ataxia

What are the differences between Miller-fischer and Myasthenia Gravis?

1) Both sudden onset but Bell's occurs progressively over hours to days, while stroke is an immediate significant deterioration. 2) Stroke is not associated with hyperacusis or post-auricular pain

What are the differences between facial palsy in Bell's palsy and Stroke?

Small vessel rashes tend to be - Palpable purpura with petechiae Vs Plaques and tender nodules in medium vessel - May be vesiculobulbous - Present as superficial ulcerations Vs deeper ulcers in medium vessel and medium vessel vasculitis is associated with - Digital gangrene and - Livedo racemosa (reticular net-like pattern of reddish-blue)

What are the differences in Rash presentation between small vessel and medium vessel vasculitides?

1) Vesciular rash (may or may not be there due to possbility on Zoster Sine Herpete) 2) Location of ear pain (Ramsay hunt is within ear, Bell's is behind ear) 3) Hearing (Imparied in Ramsay-hunt, Hyperacusis in Bell's) 4) Ear appearance (usually reddened in Ramsay-hunt)

What are the differences in clinical presentation of patients with Ramsay-hunt syndrome and Bells palsy? (4)

SPED (Stroke, dementia, epilepsy, psychiatric) Differentials: 1) Neurodegenerative disease - Sundowning in dementia 2) Neurovascular - Strokes - Temporal-parietal: wernicke's aphasia - Bitemporal dysfunction: may present with transient global amnesia/visual agnosia and cortical deafness - Occipital: anton's syndrome (cortical blindness w confabulation) - Frontal: mutism, judgement/spontaneity/memory lapses. 3) Non-convulsive status epilepticus 4) 1' Psychiatric illness - Depression - Mania vs hyperactive delirium - Schizophrenia

What are the differential diagnoses of delirium?

1) Heart failure (causing fluid overload) 2) Hypoalbuminemia (protein losing enteropathy/liver disease/protein malnutrition) 3) Increased capillary permeability (angioedema, anaphylaxis etc)

What are the differential diagnoses of limb swelling?

Depends on presentation of MG 1) Ocular MG (30%): - Oculopharyngeal dystrophy(OPMD is characterized by slowly progressive muscle disease (myopathy) affecting the muscles of the upper eyelids and the throat) - Chronic external ophthalmoplegia (Mitochondrial cytopathy) - CN palsies (cavernous sinus syndromes, Horner's syndrome) - Thyroid eye disease 2) Bulbar myasthenia (15%) - Brainstem lesions (stokre, syndromes, gliomas) - Motor neurone disease (e.g ALS) - Obstructive/malignant lesions of the nasal and oropharynx (e.g NPC) 3) Generalized weakness/neurological deficit (<5%) - Lambert-Eaton Myasthenic syndrome - Miller-Fischer syndrome - Demyelinating disease (e.g multiple sclerosis) - Myopathies (e.g dermatomyositis) - Motor neurone disease e.g ALS)

What are the differentials to Myasthenia Gravis?

Aim serum urate < 360 umol/L For chronic tophaceous gout, aim < 300 umol/L

What are the goals of Urate Lowering Therapy?

1) Steroid resistance 2) Lack of C3/other etiological clues 3) Hypertension Bleeding (severe) - Intractable, requiring angioembolisation or even partial nephrectomy

What are the indications and complications of renal biopsy?

AEIOU 1) Acidosis - pH < 7.2, refractory to medical therapy 2) Electrolyte imbalances - Hyperkalemia etc - Intractable with conservative measures 3) Intoxications - Lithium, toxic alcohols and salicylates 4) Overload (fluid) - Pulmonary edema resistant to diuretics 5) Uremia (symptomatic) - Uremic pericarditis - Uremic encephalopathy

What are the indications for starting renal replacement therapy?

Turbid fluid colour White cell count > 20 x 10^3 Microbiology with gram stain/culture +/- AFB smear and culture

What are the joint aspiration findings in Septic arthritis?

1) Continue acute gout prophylaxis 2) Continue monitoring of uric acid levels and for adverse effects of Urate Lowering Therapy (ULT) 3) Maintain uric acid levels < 6mg/dL indefinitely once tophi + signs/symptoms/flares resolve.

What are the long term management goals of gout?

1) Conservative - Advise patient to get up slower, wait longer before walking - Abdominal binder - Compression stockings (watch for limb ischemia) - Hydration (increase drip etc) 2) Pharmacological - Fludrocortisone (corticosteroid with strong mineralocorticoid effect. but ironically weak glucocorticoid effect) - Midodrine (prodrug of a1 receptor agonist - vasopressor)

What are the management options for postural hypotension?

CN 5, 6, 7, 8 and sometimes 9. - Space occupying lesions e.g Acoustic neuroma Schwanoma Lipomas AVM Hemangiomas

What are the nerves affected in cerebello-pontine angle syndrome and what are the causes?

Lifestyles (2): 1) Risk factor/co-morbid control - e.g Gout, polypharmacy 2) Weight management - obesity compounds baseline CVS risk of CKD + obesity increases hyperfiltrate and accelerates eGFR fall Intake (3): 3) Nutrition - refer dietician to ensure adequate nutrition 4) Volume - risk of fluid retention after CKD 2-3, use diuretics, fluid restriction 5) Proteinuria - ACEI useful in diabetics, spironolactone if potassium levels acceptable and low protein diet (helps increase AA synthesis and reduce metabolic stress that accelerates eGFR fall) Bloods (3): 6) Diabetic control - keep HbA1c < 7% 7) Lipids - no fixed numerical target, stratify according to risk and give statins 9) Hypertension - aim SBP <130/80

What are the points of management in chronic kidney disease?

Muscle -> Myalgia (50%) Gastrointestinal (30-50%) -> Mesenteric vasculitis -> Intestinal angina Joints (50%) -> Arthralgias -> Arthritis Nerves (peripheral neuropathy 50-70%) -> Foot drop -> Mononeuritis multiplex (painful, asymmetrical, asynchronous sensory and motor peripheral neuropathy involving isolated damage to at least 2 separate nerve areas) Renal (70-80%) -> Hypertension -> Renal artery vasculitis Skin (40%) -> Liverdo reticularis -> Palpable purpura -> Ulcers Cardiac and testicular (Rare)

What are the possible areas of involvement in Polyarteritis nodosa (PAN)?

Cardiac causes - Arrhythmias (AF, SVT, VT, check narrow or broad complex) - Pulmonary embolism Shock - Hypovolemia (from bleeding or dehydration) - Septic (septic shock or sepsis induced AF) Metabolic - Hypoglycemia - Hyperthyroidism (can also be thyroid storm with AF) Psychogenic - Anxiety - Pain (common)

What are the possible causes of Tachycardia?

Respiratory: - Obstructive: muscous plugging, bronchospasm, foreign body aspiration - Parenchymal: lobar collpse, pneumo/haemothorax - Alveolar: pneumonia consoldiation, cardiac failure edema, emphysema disease. - Vascular: systemic hypoperfusion, pulmonary embolism, heart failure Thoracoabdominal: - MSK injuries/lesions, pain - Diaphragmatic splinting/fatigue - Diaphragmatic hernia Neurological: - CNS: strokes, brainstem lesions, coning in raised ICP, drugs e.g BZD and opiates, C-spine lesions - PNS: NMJ (MG, botulism, aminoglycosides), myositis, peripheral neuropathy (GBS), muscular dystrophies

What are the possible causes of acute desaturation?

Physiological - Pre-existing conditions/low baseline/athletes Endocrine - Hypothyroidism Cardiac - Heart blocks - Sick sinus syndrome (hx of palpitations, intermittent skipped beats, see on ECG) Intracranial - Cushings triad from late raise ICP Drugs - Beta-blockers

What are the possible causes of bradycardia?

D.E.L.I.R.I.U.M - Drugs - Electrolyte imbalances (to assess for underlying cause) - Lack of sleep - Infections (go systemically) - Restraints/reduced sensory input - Intracranial (structural: tumor, vascular, TBI or non-structual: infection, seizures: absent seizure or postictal, hypertensive encephalopathy, SLE encephalopathy) - Urinary retention/constipation/malnutrition/dehydration - Myocardial (AMI, arrythmia)

What are the possible causes of delirium?

1) NSAIDs + colchicine 2) Oral steroids + colchicine 3) Intra-articular steroids + NSAIDS + Colchicine Oral steroids not to be used with NSAIDS due to potential of gastric toxicity.

What are the possible combinations in combination therapy for acute gout flares?

1) Degenerative - Chronic tophaceous gout: 2' joint deformity - Chronic pain - Functional impairment 2) Infective - Skin ulceration and cellulitis: 2' to tophi 3) Hyperuricemia - Urolithiasis 4) Complications of treatment - Drug reactions

What are the possible complications of gout?

1) DKA 2) HHNK 3) 'Impending Crisis' 4) Euglycemic DKA LET SENIORS KNOW

What are the possible crises of hyperglycemia and their clinical signs?

What are the possible sites of LMN lesions, their clinical presentations and their causes?

What are the possible sites of LMN lesions, their clinical presentations and their causes?

What are the possible sites of UMN lesions, their clinical presentations and their causes?

What are the possible sites of UMN lesions, their clinical presentations and their causes?

Cholinergic crisis occurs due to overtreatment of severe myasthenia gravis with acetylcholinesterase inhibitors. Symptoms = Flaccid S.L.U.D.G.E - Flaccid paralysis - S: salivation - L: lacrimation - U: urinary incontinence - D: diarrhoea - E: emesis Manage with - Conservative: taking with food - Pharmacologic: for symptomatic treatment via glycopyrrolate, propantheline or hyoscyamine sulfate.

What are the signs of a cholinergic crisis and how does one manage it?

1) Cardiac failure - S3 - Congestive cardiac failure - Apex beat displaced and thrusting - Austin flint murmur (functional mid diastolic murmur due to regurgitant jet hitting mitral valve leaflet and obstructing LA flow into LV) 2) Valve stiffening - Soft S2 - Widened pulse pressure 3) Impaired systemic circulation - Hill's sign (popliteal SBP abnormally much higher than brachial SBP, 20 - mild AR, 40 - moderate, 60 severe)

What are the signs of a severe aortic regurgitation?

1) Calcium stones - Most common (75%) - Due to hypercalcemia - Hyperparathyroidism - Increased gut absorption of calcium - Renal calcium/phosphate leak 2) Struvite (15%) - Magnesium, ammonium, phosphate stones (MAP) - Chronic UTI w g(-), urease (+) organisms that split urea into ammonia that combines w Mg and PO4 to crystalise - e.g Proteus, pseudomonas, klebsiella - Urine pH usually >7 - Investigate TRO anatomical tract abnormalities 3) Uric acid stones (6%) - a/w high purine intake, urine pH < 5.5 and malignancy (high turnover) 4) Cysteine stones (2%) - Intrinsic metabolic defect preventing renal tubular reabsorption of cysteine, ornithine, lysine and arginine (COLA) - Saturation of urine w cysteine leads to crystal formation 5) Drug induced stones - Stones high in respective metabolites - HIV Antiproteases e.g indinavir - Sulfadiazine in cerebral toxoplasmosis

What are the types of renal stones (5) and their causes?

Blood tests for HLA B 1502, which has shown to be highly associated with severe cutaneous adverse drug reactions e.g SJS and TEN

What blood test is required before prescribing Carbamazepine?

1) Infection 2) Emotions 3) Drugs - Non-compliance to medication - Abx: Aminoclycosides, tetracyclines, macrolides and fluoroquinolones (FAMT) - CVS: BB, CCB (verapamil) - Others: choloquine, qunidine, procainamide, Li, Mg, prednisolone, quinine, penicillamine (for Wilson's disease)

What can precipitate myasthenia gravis or a myasthenic crisis?

1) Endocrine: - Thyroid disease (Grave's, thyroiditis etc) - Diabetes mellitus - Pernicious anemia 2) Autoimmune/inflammatory: - RA - SLE - Polymyositis

What diseases are associated with Myasthenia Gravis?

They are useful in localisation of lesions. Voluntary deficits imply cortical lesions while mimetic imply sub-cortical lesions. Mixed or equivocal patterns may indicate lower brainstem lesions where the pathways intersect.

What do differences in voluntary and mimetic facial palsy imply?

1) Neuroleptics - flupenthixol 2) Anti-emetics: metoclopramide 3) Others: cinnarazine (a CCB)

What drugs commonly cause parkinsonism?

1) ECG - LVH + TRO AMI 2) CXR - Cardiomegaly - Widened aorta - Pulmonary congestion - Valve calcification 3) 2D Echo - Assess severity, EF

What investigations should be done for Aortic Regurgitation and what to look out for?

1) Laboratory investigations - AchR antibody (+ in 80% of generalized, 90% of penicillamine induced and 50% of ocular MG) - Anti-striated muscle Ab - Anti-muscle specific kinase antibody (NOT found in AchR ab +ve MG and found in 50% of seronegative MG) 2) Imaging studies - CXR to assess thymus or 2' aspiration pneumonia - CT can also be used to assess thymus 3) Bedside - Tensilon test (1-10mg test dose of edrophonium, an easiky reversible AchE inhibitor, to assess for improvement of symptoms - be sure to standby atropine for 2' brady, also positive for ALS) - Ice pack test (applied for 2 minutes, improvement is diagnostic of MG, 80% positive) 4) Electrodiagnostic studies - Repetitive nerve stimulation test (decrease in muscle action potential after 4-5th response to stimulation) - Single fibre nerve electromyography - increased jitter is evidence of neuromuscular blockade (sensitive but not specific)

What investigations should be done for Myasthenia Gravis?

Objective-based approach: 1) Confirming diagnosis of proteinuria - UFEME/dipstick - 24 hr UTP/first morning void Urine P/C ratio - LFT (hypoalbuminemia) - Lipid panel (hyperlipidemia) 2) Baseline bloods - Renal panel (hypovolemia w AKI, electrolyte imbalances, hypercalcemia in multiple myeloma) - FBC (infection, thrombocytosis for thrombotic risk, anemia in multiple myeloma) 3) Etiology of nephrotic syndrome - C3 (decreased in SLE) - C4 (decreased in SLE, increased in post strep GN) - ANA, anti-dsDNA (TRO lupus) - Hep B (TRO membranous nephritis) - Hep C (TRO mesangioncapillary GN) - ANCA, IgG, IgM, IgA if atypical - Anti GBM if lung involved (CXR may be useful for Goodpasture's) - Renal biospy only if: - hypertension - low C3 - steroid resistant

What investigations should be ordered in Nephrotic syndrome?

1) Brain imaging (MRI) to determine cause/rule out disorders - Normal pressure hydrocephalus - Multi-infarct syndromes - Frontal menigiomas - Parkinson plus (atrophy of respective parts) - MSA: cerebellum and brainstem - PSP: atrophy and hypersensitivity of midbrain and red nucleaus - CBD: Frontoparietal cortical atrophy 2) If young parkinsonism, need TRO Wilson's disease - Slit lamp exam for KF rings - Serum ceruloplasmin (copper carrier protein made in liver, decreased in Wilson's disease) - 24 hr urinary copper

What investigations should be ordered in parkinsonism?

Drug Reaction with Eosinophillia and Systemic Symptoms - Can result from Allopurinol use in gouty arthritis 2-6 weeks or even longer after initiation of therapy - May cause a rash, fever, inflammation of internal organs, lymphadenopathy and characteristic hematologic abnormalities (eosinophillia, thrombocytopenia and atypical lymphocytosis) - Patient at risk of mortality from multi-organ failure

What is DRESS syndrome?

Lesion of the pons, AKA ventral pontine syndrome VS cerebello pontine angle which is at the dorsal side. Affects: - CN6 - CN7 - Corticospinal tract: contralateral hemiplegia

What is Millard-Gubler syndrome?

Lesion at level of midbrain, commonly due to occlusion of the paramedian branches of posterior cerebral artery AKA superior alternating hemiparesis Manifests as: - Ipsilateral CN3 palsy: diplopia and ophthalmoplegia - Edinger-westphal nuclei may be involved: impaired pupillary dilatation in accomodation (parasympathetic deficit) - Contralateral muscle weakness: due to being above the level of corticospinal (body) and corticobulbar (face + hypoglossal) deccusation - Contralateral parkinsonism: due to substantia nigra

What is Weber's syndrome?

A biphasic T wave in anterior leads indicative of critical LAD stenosis.

What is Wellen's syndrome?

For use in expected PCO2 in metabolic acidosis: ePaCO2 = [(1.5 x HCO3) +8)] +/- 2

What is Winter's formula?

Zoster disease of dermatomal distribution of pain without antecedent rash, diagnosed by clinical symptoms and virologic confirmation.

What is Zoster Sine Herpete?

Chronic progressive external ophthalmoplegia (CPEO) is a disorder characterized by slowly progressive paralysis of the extraocular muscles via a form of mitochondrial cytopathy. Patients usually experience bilateral, symmetrical, progressive ptosis, followed by ophthalmoparesis months to years later. Ciliary and iris muscles are not involved.

What is chronic progressive ophthalmoplegia?

ADDORA 1) Acute and fluctuating course AND 2) Distractibility with either: 3) Disorganized thinking OR Altered level of consiousness

What is the CAMS criteria for diagnosis of delirum?

Used to assessment of prognosis in chronic liver disease and cirrhosis. A.I B.A.E - Albumin - INR - Bilirubin - Ascites - Encephalopathy Childs A = 5-6 points, least severe, < 5% operative mortality Childs B - 7-9 points, moderately severe, 25% operative mortality Childs C = 10-15 points, most severe, >50% operative mortality

What is the Child-Pugh scoring?

Radiation of systolic murmur toward apex, which may be confused as an MR murmur.

What is the Gallavardin phenomenon?

Score for risk stratification of patients with UGIT bleeding for outpatient or inpatient requiring transfusion, endoscopy or surgery. (Heart Liver MASHUP) - Cardiac failure present - Liver disease history M - melena presentation A - anemia S - syncope presentation H - hypotension U - uremia P - pulse rate >/= 100 Any score of > 0 is high risk (so please admit and investigate any pts w UGIT bleed and any of these sx)

What is the Glasgow-Blatchford Bleeding Score?

I - No dyspnoea/palpitation/fatigue at rest or with normal activity II - No dyspnoea/palpitation/fatigue at rest, mild with normal activity III - No dyspnoea/palpitation/fatigue at rest, mild with reduced activity IV - Dyspnoea/palpitation/fatigue at rest.

What is the New York Heart Association grading criteria for functional impairment?

1) Pre-treatment: Lidocaine 2) Induction: Ketamine 3) Mechanical ventilation with controlled hypoventilation where Inspiratory phase : expiratory phase is 1 : 3to4.

What is the RSI regimen for asthmatic patients?

Deep S wave in Lead I Q wave in Lead III T wave inversion in Lead III A rare but pathognomonic ECG finding in pulmonary embolism.

What is the S1Q3T3 pattern and what does it imply?

Baseline: Hydroxychloroquine (watch for retinal damage) Acute organ damage: - Pulsed methylprednisolone - Cyclophosphamide * for 3 days, before long term therapy of: - Maintanance steroids - Mycophenolate mofetil or azathioprine

What is the baseline treatment of uncomplicated Systemic lupus, acute treatment of organ manifestations and subsequent treatment?

Use of SGLT (sodium-glucose CoTransporter) inhibitors e.g empaglofozin, canaglifozin, dapaglifozin etc. They reduce blood glucose levels via reducing renal reabsoprtion of glucose. But are associated with lipolysis and subsequent ketosis. Look for pH acidosis and raised Betahydroxybutyrate.

What is the cause of Euglycemic DKA?

- It is a uricosuric agent (that increase uric acid excretion in kidney) - Works by inhibiting the OAT (organic anion transporter) of proximal renal tubules that normally reabsorb uric acid into systemic circulation.

What is the class and mechanism of action of probenecid?

Monotherapy: indicated for mild-moderate pain with only one or few small joints or 1-2 large joint involvement. Combined therapy: indicated for severe pain, especially with poly-articular involvement/multiple large joints

What is the criteria for monotherapy and combined therapy in Acute Gout flares?

1) Nephrotic range proteinuria 2) Hypoalbuminemia < 25g/L 3) Hyperlipidemia 4) Edema

What is the diagnostic criteria of nephrotic syndrome?

1) Cause: -> Wallenberg's - posterior inferior cerebellar artery -> Lateral pontine - anterior inferior cerebellar artery 2) Presentation/deficit -> Wallenberg's - includes nucelus ambiguus: glossphargyngeal nerve and vagus nerve (CNX), causing **dysphagia/diminished gag reflex as well as dysautonomia, dynsphonia, bradycardia etc -> Lateral pontine - lesions in pons, thus includes facial nerve CN7, presenting with ipsilateral LMN facial palsies

What is the difference between Wallenberg's syndrome and lateral pontine syndrome?

1) ABCs first (10% require intubation) 2) Remove precipitants - Drugs (remove non-essential medication) - Underlying infections (treat fever symptomatically as well as medication for offending pathogen) 5 lines of symptomatic treatment 3) Acetylcholinesterase inhibitors - Pyridostigmine or neostigmine to increase levels of Ach (watch for cholinergic crisis) 4) Immuno-modulators - Steroids: Prednisone - Immunosuppressors: Azathioprine, cyclosporine 5) Plasmapheresis to remove AchR Ab 6) IVIG (mechanism unclear) 7) Thymomectomy

What is the management of a myasthenic crisis or severe myasthenic gravis?

1) Ability to maintain gaze on object despite rapid head movements, via eye movements through input from vestibular system as brainstem reflexes are still maintained. Possible outcomes: - Excessive correction (eyes turn to contralateral side of direction of head turn) = loss/weakening of cortical influence of brainstem reflexes - Absent reflex = loss of brainstem reflexes - Nystagmus after head turning = asymmetrical vestibular input

What is the occulocephalic reflex, what are the possible outcomes and their implications?

An autoimmune condition characterised by production of anti-acetylcholine receptor antibodies that bind to and destroy post-synaptic AchR. Leading to impaired electrical impulse conduction across neuromuscular junctions, leading to clinical sign of fatigable, neurological deficit that recovers with rest.

What is the pathogenesis of Myasthenia Gravis?

Mainly imbalance is bone formation vs bone reasbsorption via 1,25 Dihydroxycholecalciferol vs parathyroid hormone levels 1) Bone resorption increases due to increase PTH - In response to phosphate retention due to falling eGFR - In response to acidosis from falling eGFR 2) Bone formation decreases due to fall in 1,25(OH2) vit D3 - In response to renal scarring hence less functional parenchyme - In response to increase in PTH

What is the pathogenesis of mineral bone disease in chronic kidney disease?

1) Normal: plantar flexion of all toes, curling downwards 2) Abnormal/infant: dorsiflexion of big toe with fanning out of snall toes. Indicates upper motor neurone lesion in the corticospinal tract.

What is the plantar reflex (Babinski) pathway and what do the possible outcomes imply?

1) 4 initial features - Presenting history of patient + correct lead placement - Rate - Rhythm - Axis deviation 2) 4 Waves to look at - P wave (mitrale - 2 humped, pulmonale - peaked + tall, biatrial abnormality - terminal negativity in V1) - QRS complexes (amplitude, progression, presence of Q waves) - T waves - U waves 3) 4 Intervals to look at - PR interval (~ 3-5 small squares, prolonged in heart block) - QRS width (~ 3 small squares) - ST segment - QT interval (long in toursade's de pointes i.e > half of RR interval)

What is the rule of 4s for ECG reading?

> 5.5 mEq/L - T wave peaked >6.5 mEq/L - Tall tented T waves > 7.5 mEq/L - Flattening and broadening of P waves +/- sine form QRS, +/- 2' complete heart block, VF, death

What is the sequence of ECG changes in hyperkalemia?

1) 10ml of 10% IV Calcium Gluconate 2) 10 units of IV insulin with 50ml of dextrose 50 3) Salbutamol (5mg/3ml saline) over 10 minutes +/- Sodium bicarbonate (only if concomittent metabolic acidosis) 4) PO 15g Resonium A 4-6 hrly KIV hemodialysis if hyperkalemia severe + refractory to previous treatment or patient already has ESRF Review medications (Span K, ACEI, BBlockers) and diet.

What is the sequence of treatment of hyperkalemia and its doses.

The increased thickness of the visceral with visible fluid collection

What is the split pleura sign and what does it imply?

Ipsilateral CN 2, 3, 4, 5(1) and 6

What nerves are affected in Superior orbital syndrome?

AKA Lateral medullary syndrome/posterior inferior cerebellar artery syndrome Ipsilateral CN 5 (spinotrigeminal tract), 8, 9, 10 nuclei and sympathetic tract (Horner's, bec deccusation above lesion) with Contralateral loss of spinothalamic (due to deccusation below lesion) Caused by posterior inferior cerebellar artery supplying the medulla oblongata

What nerves are affected in Wallenberg's syndrome and what is the cause?

Ipsilateral CN 3, 4, 5(1) and 5(2), 6

What nerves are affected in cavernous sinus syndrome?

CN 5, 6, 7, 8

What nerves are affected in cerebello-pontine angle syndrome

30%

What percentage of receptors must be affected before weakness sets in for Myasthenia Gravis?

1) Eye movements (Doll's eyes if vertical gaze impaired) 2) Blepharospam when eyes closed (seen in SNP) 3) KF rings in eyes (wilson's disease) 4) Feel for seborrhea (dryness of skin around nose and mouth, dandruff from scalp, due to autonomic dysfunction) 5) Ask patient to count from 1-20 (rough cognitive function assessment)

What to look out for in face in Parkinsonism. (5)

1) Resting tremors 2) Bradykinesia (slowness of movement, test via finger-to-thumb test, wrist rotation and twinkle stars) 3) Lead pipe rigidity and cogwheeling 4) Acute dystonia (abnormal twisting posture) and alien hand syndrome (inability to control movement of hand: ask patient to attempt to comb hair) 5) Pronator drift and cerebellar signs 6) Palmomental reflex (stroking of thenar eminence results in chin twitch, indicative of contralateral frontal lobe damage), grasp reflex

What to look out for on upper limbs in Parkinsonism and how they look like (6)

When does one start HIV treatment? How much does it cost?

When CD4 counts < 350 cells/uL - Too early = intolerable side effects, drug complications - Too late = disease complications are significant. Or when AIDs defining illnesses have presented Whichever is earlier. It costs about 12000/year

When does cerebral toxoplasmosis risk increase in HIV infections and how can you prevent it or treat it?

When CD4 counts fall below 100, around the advanced stage. Prevent by starting anti-HIV medications to raise CD4 count If diagnosed, treat with Pyremethamine Sulfadiazine and Folinic acid

What is abdominal compartment syndrome and what are it's signs?

When IAP > abdominal perfusion pressure. Sustained intra-abdominal pressure > 20mmHg that is associated with new-onset organ dysfunction. Signs: - Tensely distended and tender abdomen - Progressive oligouria - Increased ventilatory requirements - Vitals: hypotension, tachycardia, elevated JVP - Peripheral edema - Hypoperfusion (cool skin, obtundation, restlessness and lactic acidosis)

1) Unclear etiology of hyperuricemia 2) Difficulty reaching ULT goals especially in face of renal failure and/or post-trial of Xanthine Oxidase Inhibitors 3) Refractory symptoms/signs/flares of gout 4) Multiple/severe adverse effects of ULT.

When is a specialist referral for gout indicated?

What is a Greenstick fracture?

When softer pediatric bone bends and partially breaks.

When is neoadjuvant chemoRT indicated in rectal cancer?

When there is involvement or threat to the mesorectal fascia (<2mm distance).

When is MTX indicated in psoriasis?

When there is joint involvement?

What is the painful arc and what does it indicate?

Where pain sets in when arm is in ABDuction of about 60-120degrees. For identification of possible subacromial impingment syndrome.

What is the progression of acne?

White comedome > black comedome > papules > pustules > cysts > nodules > nodule cystic > acne conglobata (deep interconnecting abscesses with b/g of hypertrophic and atrophic scars)

Why cant you give aminoglycosides and fluoroquinolones to ESBLs even though they are not beta lactams?

Why, that is a great question. It is very true that they are not beta lactams but... blah blah blah its because ESBLs still show greater than average resistance to these drugs, often complete resistance, so best to stick with the carbapenems or the 5th generation cephalosporins with beta lactamase inhibitors.

What are the radiological findings of SCFE?

Widening of the epiphyseal growth plate. Trethowan's sign: Klein's line does not transect the femoral head

What are the most common causes of the Winking Owl sign?

Winking Owl sign - crushed pedicle - Pedicle is the strongest part of the vertebrae - Strongly indicative of pathological bone fractures: Osteolytic process of metastatic disease Most common causes: 1) Spinal metastasis (breast and lung commonly) 2) Intraspinal malignancies (e.g astrocytoma) 3) Tuberculosis and other infections 4) Primary bone lesions: lymphoma DDx - Congenital abscence of a pedicle - Neurofibromatosis - Poorly visualised - Radiation therapy

How can you classify femoral shaft fractures?

Winquist and Hansen classification (5 types) Type 0: No comminuition Type 1: Insignificant amount of comminution Type 2: Greater than 50% cortical contact Type 3: Less than 50% cortical contact Type 4: Segmental fractures with no contact between proximal and distal fragment

How is anion gap corrected in relation to albumin?

With each fall in albumin from the normal level (40g/L) by 10g/L, add 2.5 to the anion gap.

What is the mechanism of action of incretin based therapy?

With timed GLP-1 secretion from small intestine with relation to meals. The GLP-1 is broken down by dipeptidepeptidase 4 (DPP-4). Hence DPP-4 inhibitors allow longer duration of action of secreted GLP-1 AND combined it with GLP-1 analogues to supplement the effect. DPP-4 inhibitors are orally taken while GLP-1 is injected but DPP-4 reduce HbA1c by 0.5-1% while GLP-1 is more effective at 1-1.5%, patients still prevent eating than injecting.

What is the limit in increase in serum sodium for treatment of chronic hyponatremia?

Within 24 hours, Increase CANNOT go beyond 10-12mmol/L Within 48 hours, Increase CANNOT go beyond 18mmol/L Exceeding this results in risk of central pontine myelinosis, manifesting as neurological deficit and brain stem damage.

When should valaciclovir be given by in zoster infections?

Within 72 hours, because...

What are the recommended limits of alcohol intake?

Women 14 units/week Men 21 units/week 1 can 330ml of beer is 1 unit Typical beer 5%alc

What are the principles of wound care?

Wound should be: - Well vascularised - Clear of devitalized tissue and infection - Packed and dressed to remove dead space, control exudate and prevent bacterial overgrowth - Closure/coverage if granulation tissue and epithelialization taking place

What are the radiological findings of septic arthritis and what complications may be seen?

XR: - Effusion (Lateral displacement of joint) - Necrosis (hyperdensity/ fragmentation/ deformity of bone) - Degeneration (erosion of articular surfaces) Which can progress to complications - Effusion -> joint dislocation - Necrosis -> loss of adjacent growth plate - Degeneration -> complete destruction of articular cartilage and underlying epiphysis)

What would you see on radiological (XR and U/S) investigations for transient synovitis?

XR: no changes U/S: may see joint effusion

1) Acute: soft tissue swelling 2) Chronic: - Tophi, radiolucent lesions of para-articular surfaces manifesting as punched-out erosions/rat-bitten lytic lesions with overhanging edges - Erosion with preservation of joint space is typical - May have secondary osteoarthritic changes

Xray findings of Gout

1) Chrondrocalcinosis: opacification within joint spaces/thin opaque line parallel to joint or irregular opacities within fibrocartilaginous menisci and discs 2) Chronic pyrophosphate arthropathy: similar to OA changes but in non-weight bearing joints

Xray findings of Pseudogout

How can you estimate bone age of a patient?

Xray of the left hand, fingers and wrist usually taken - Minimal radiation - Shows many bones in a single view Compared with standardised atlas e.g Greulich and Pyle Or use of computer programs e.g BoneXpert

What is the mechanism of injury for IT fractures and what is the anatomical boundary?

Younger individuals: high velocity injury e.g RTA or fall from height Older individuals: simple fall or direct impact (10% to greater trochanteric region) The IT region is between the greater and lesser trochanter +/- extension to the subtrochanteric region

What are the grades of performance status?

Zubrod/ECOG Performance Status Test 0 - no symptoms, normal activity 1 - symptomatic, but can carry out ADLs 2 - symptomatic, limited in ADLs, in bed < half the day 3 - symptomatic, in bed > half the day 4 - bedridden

What is the prognosis of fat embolism?

~ 1-15% mortality with supportive care.

How long does pain from zoster last and when does it become post-herpetic neuralgia?

~ 2 weeks If lasting beyond 3 months: post-herpetic neuralgia

What are the average bowel habits of patients with ileal pouch?

~ 6 bowel movements per day with 90% continence in day and 60% continence at night Risk of gross incontinence is 5%

What are the complications of TIPPS?

○ Complications would include § Procedure (bleeding, infection, perforation) § Liver bypass (liver failure/encephalopathy, cerebral edema due to toxins) § 50% chance of thrombosis within 1st year


Ensembles d'études connexes

Module 7: The History Of The Computer

View Set

principles of management midterm

View Set

Data Structures & Algorithms - Chapter 7 & 11 QUIZ

View Set

Chapter 4: Firewall Technologies and Administration

View Set

Chapter 40 Professional Roles and Leadership

View Set

EASA Part 66 : Electrical Question17

View Set